Sei sulla pagina 1di 219

FCPS PART 1

PAST PAPERS
PUBLISHED AT:

www.MedicosRepublic.com

KEEP VISITING OUR BLOG FOR MORE USEFUL STUFF ON FCPS,


USMLE, MRCP AND PLAB! ☺
BEST REGARDS,
DR. AURANGZAIB BALOCH
FOUNDER/ADMINISTRATOR OF MEDICOSREPUBLIC
1

GOLDEN FILE# 3 INCLYDES SEPTEMBER, OCTOBER AND


NOVEMBER PAPERS- by Dr. Rafi ullah

Author Notes about the controversy regarding his & Dr Salah-ud-din


Kamal book(s) is removed as the aim of file sharing is to help juniors.
Although I appreciate the author and all those complied these tests for up-
coming juniors.
2

SEPTEMBER 6, 2016 EVENING


1. Regarding Aphasia; Lesion in =Temporal lobe
2. Fascia enclosing masseter muscle =Deep Cervical Fascia
3. There was a scenario female with webbed neck and short stature = turner syndrome
4. A Scenario of Male with XXY =klinfilter syndrome
5. Decrease in core body temperature results in = Shivering
6. Virus cause disease by = Alteration in protein synthesis
7. Regarding node of Ranvier = no Myelin
8. Highest TGs concentration = Chylomicrons
9. Regarding hemorrhagic infarct = Lungs
10. Type of collagen present DURING wound healing = Type 3
11. Regarding reticular connective tissue = Spleen
12. Smoking working in tyre factory, cause of trAns:itional cell carcinoma = Smoking >
Aromatic.(ref Goljan)
13. Supernumerary ribs = compression of Brachial plexus and subclavian
14. Diseased vs non diseased = Case control
15. Gardener with linear lesion = Cutaneous larva migrAns:
16. Repeat question of a diabetic who died despite aggressive measure = Mucormycosis
17. Insulin inhibited by = B blockers
18. Vitamin in fatty acid oxidation = biotin
19. Aggressive sympathetic drive = Bronchodilation
20. Most important response in shock as a whole = CNS ischemic response
21. Increased BT is observed in = vWF deficiency
22. Regarding Renal vein = Anterior to Aorta
23. Left adrenal vein drainage = Left Renal Vein
24. Retromandibular vein = Maxillary with superficial temporal
25. Chromatids align at equatorial plain = Metaphase
26. Child presented with proteinuria of >3gm, cause on edema? = Decrease colloid osmotic
pressure
27. Natural Defense Against Cancer = Apoptosis
28. Step in Apoptosis = Activation of caspases
29. Child with macrocytic type CBC = B12 level Ans:
30. Elastic cartilage present in = Epiglottis
31. Regarding Hyaline cartilage = Articular surfaces
32. Mother having measles, child at risk of =Cataract
33. Mother delivered a baby with cataract, cause? = Measles
3

34. Sound produced in Ventricular filling = S3


35. Which of the following decreases serum osmolality?= antidiuretic hormone
36. Thiazide diuretic = Hypokalemia
37. Fracture of Lumbar vertebra = Conus medularis syndrome
38. True about osmosis? = its osmolality depends on particles number
39. Regarding nucleus? = Euchromatin is trAns:criptionally active
40. In severe dehydration = decrease in total body water
41. A patient with severe dehydration drinks 2L tap water: inc ICF (reference ganong)
42. Global cause on infective blindness: = Chlamydia > bacteria
43. Internal carotid artery: anterior choroidal branch to cavernous sinus
44. Leukocyte Adhesion = LFA1
45. True about Leukocytes = Phagocytic in blood
46. Repeat case of= EXTRAHEPATIC CHOLESTASIS
47. Preganglionic autonomic Fibers = B type
48. T cell Receptor bind with = MHC 2
49. Secondary ossification center present in? = lower end of femur
50. Associated with intestinal malignancy = H. Pylori
51. Decrease ESR = Increase Albumin
52. Patient aspired on lying down = Right lower lobe
53. Irreversible step of glucose metabolism? = G6P to fructose 1,6 po4.
54. In second degree heart block = Progressive lenghthening of Pr interval until a beat is
dropped(type1)
55. 41% hematocrit = 41% RBC WBC Platelets
56. Structure embedded in cerebral cortex? = Insula
57. SA node location = upper part of sulcus terminalis
58. Ureteric bud development = Mesonephric duct
59. Patient presented with chest pain of 2-4 hours’ duration, all baseline was normal next
investigation = CK-MB
60. Mechanoreceptor of Warm = Long receptive fields
61. Gracilis and cuneatus damage = Astereognosia
62. PICA supply = dorsolateral medulla
63. Cells having most telomerase activity = Germ Cells
64. Most drugs are metabolized in = Liver
65. Pseudomembrane colitis = C. difficile
66. Reason of tuberculin test negative = taking immunosuppressant
67. Case of PARKISON with all associated symptoms like tremor, bradykinesia, rigidity gait
problem, lesion in = Substantia Nigra
68. Heart Rate of 120 = RR interval 0.5sec
69. 60 years old farmer with Cushing symptoms = Adrenal Adenoma
70. Malignant tumor = Metastasis
71. 2*2 table = Chi Square
72. Which augments the effect of phagocytes? = Complement
73. Metastasis occurs because = loss of E cadherin
74. Allele = Non identical genes
75. Hypovolemia results in = Decrease carotid sinus nerve firing
76. Typhoid investigation in first week = Blood Culture
4

77. Thiamine deficiency causes = Peripheral neuropathy


78. Increase in GFR and RPF= afferent arteriolar dilatation
79. Cimetidine effect = Inhibit hepatic enzymes
80. Inducers of liver enzymes = Phenobarbital
81. 50 years’ post-menopausal lady = Increase FSH and LH
82. Severe trAns:fusion reaction occurs = A+ to O+
83. Which drug increases LES tone and gastric motility = Metoclopramide
84. NK Cells = Kills virally infected cells without sensitization
85. Cushing features with Raise ACTH = Cushing disease
86. Mediators of Pain = Bradykinin and PG
87. Young patient dyspnea on lying down = Retrosternal goiter
88. Breaking bad news = Information in crisp and formal sessionin exclusivity .
89. QRS complex occur = Prior to ventricular systole
90. Alpha and gamma motor neuron = UMN supply
91. Atheroma development major factor = HTN
92. Response to bleeding due to dermal abrasion = vasoconstriction
93. Increase thyroid hormone cause = increased in gluconeogenesis
94. There was a female with malar rash = Anti DSDNA
95. Regarding somite’s = Arise from paraxial mesoderm
96. Early sign of Vitamin A deficient= Night blindness
97. Partial mole = 69XXY
98. Lady presented with uterine bleed due to coagulation failure secondary to
miscarriage=Evacuation .
99. Regarding sensory fibers = May pass or may not pass reticular depending upon
modality
100. Motor Fibers =Supply alpha and beta simultaneously
101. Young female with decrease Hb, Dec mch, hbA 94% hbA2 was 4% = Thalassemia trait
102. A smoker patient, was having interstitial lung disease and pleural plaque=
Asbestosis
103. Compression of sciatic nerve level was asked = L4,L5
104. Best measure of GFR estimation = Creatinine( if best measure asked:inulin).
105. A patient with BP200/120 and glucose >200, what to give = Lisinopril
106. FEV1/FVC = 0.8
107. Bronchial asthma = decrease FEV1
108. 4th heart sound correlates with = Atrial contraction against stiff ventricles
109. Rapid acting hypoglycemic drug = second generation sulfonylurias.
110. Autonomic supply to mid gut = T8-t12
111. 100cm of ileum resected = decrease absorption of bile salts
112. How to monitor a patient on warfarin = PT, APtt, INr
113. Increase Bt and Aptt normal PT = vWF defect
114. Ganglion found at c7 vertebra = Cervicothoracic
115. Patient having constricted pupil, unable to dilate it = cervical sympathetic
116. Regarding esophagus = upper 1/3rd striated
117. Color Blindness due to = Cones defect
118. In severely dehydrated child, venesection was performed; most likely nerve to be
damaged = Saphenous
5

119. Osmotic diuretics act upon = PCT


120. A known case of HCV, histology of liver shows eosinophilic cytoplasm, nuclear
clumping = Councilman bodies.
121. A patient presented with fatigue, mcv104, alt 74 ast 80 = B12
122. A patient presented with irregularly irregular pulse, p wave was replaced by irregular
waves = Atrial Fibrillation
123. Digoxin given in =Atrial Fibrillation
124. Atonic bladder =Damage to parasympathetic below sacral
125. Most radiosensitive tumor =Lymph node
126. A patient presented with acute exacerbation of COPD, Po2 45, Pco2 51, pH 7.31,
Bicarbcarbonates =Type II respiratory failure.
127. A patient doing exercise, increased respiration. Which of the following will activate, that
remains inactivated at normal respiration = Ventral respiratory group
128. Mechanism of action of Aspirin = inhibit COX( irreversibily both cox1 n 2)
129. OndAns:etron control vomiting by = antagonism pf 5HT3
130. Itching is carried by = Type C
131. Viral infection and purpura = antibodies to Platelets (ITp)
132. Abduction and medial rotation or arm = Teres Major
133. Microscopic feature of malignancy = Invasion> pleomorphism
134. Brain perfusion at rest = 50ml/100gm
135. Brain auto regulatory pressure? = 50-150mm Hg
136. Coccygeal ligament = L1
137. On right lung bronchoscopy first section to be visualized? = inferior bronchus
138. Narrowest point of esophagus = Cricopharynx
139. Point mutation occurs in = Sickle cell disease
140. A patient diagnosed as amyloidosis having plasma cell disease, amyloidosis is due to
deposition of = Amyloid light chain
141. Which of the following function through cAMP = Glucagon
142. Which of the following passes through aortic hiatus? = Azygous vein
143. Norepinephrine secreted at = post ganglionic sympathetic
144. Patient from Baluchistan presented with fever, hepatosplenomegaly, and has history of
fly bite = Kala Azar
145. Intracranial veins connected to extracranial= Emissary veins
146. A child with congenitally missing dorsalis pedis artery, blood supply to dorsum of foot
maintained through = peroneal artery
147. A child got blow at elbow, elbow joint displaced, now he is having loss of sensation in
little finger = ulnar collateral ligament.
148. 35 years old male patient died suddenly, autopsy finding nil, tox screen shows cocaine =
contraction band necrosis
149. Edrophoneum most common side effect =Nausea
150. In females, neck of bladder lies = above urogenital diaphragm
151. Perineal innervation = S2,3,4
152. Death due to C. Botulinum = Respiratory Muscles Paralysis
153. A female having blurred vision, decreased muscle activity, other neurologic nerve
involvements = demyelination of nerves
154. Antibodies to post synaptic Ach receptors = Myasthenia Gravis
6

155. Loss of sensation of lower teeth and chin = Inferior Alveolar Nerve
156. Alpha 2 agonist used in ICU for sedation= Dexmedetomidine
157. Decrease release of Oxygen from Hb = decrease in Temp
158. Hyperuricemia is side effect of ATT = Pyrazinamide
159. AIHA diagnosed by = Anti Globulin Antibody Test (coomb ‘s test).
160. One question was on = OCCULOMOTOR Loop
161. C. Diphtheriae = secretes Exotoxin
162. Benign tumor is = Warthin
163. On biopsy, endocervix show squamous cells = Metaplasia
164. Which structure loops around arch or aorta = Left recurrent laryngeal nerve
165. Sound heard in left 3rd Intercostal space = Aortic A2
166. Regarding kidney trAns:plant = HLa1 more important than HLA2
167. Scenario was given 5 years’ arthritis history with congestive cardiac failure features
cardiomegaly and glucose was 167 what lab finding in this patient you will find = raised
Ferritin
168. Benzodiazepine cause =Dependence
169. Highest pH is seen in = Pancreatic Secretion
170. A child presents with perianal pruritus; which cells will be elevated = Eosinophils
171. Gastric HCL is necessary for = Converting pepsinogen to pepsin
172. In pleural fluid aspiration in mid axillary line which structure will not be pierced =
Levator Costrum
173. Left Ventricular Failure cause = Aortic Valve pathologies
174. A patient having a lot of pizza and burgers = Hypertrophy of Steatocytes
175. There was one question on coronary circulation = run in respective grooves
176. sciatic Nerve root = L4 L5 S1,2,3
177. Which of the following cause paraneoplastic syndrome = Small Cell CA
178. A patient was presented with Malar Rash and other symptoms of SLE what other finding
can be found? = Pericarditis
179. Blood flow to muscle during exercise = Inc. local metabolite
180. Exophthalmos occurs due to = TSI
7
8

Ans:: aa
9 . after tooth extraction cause of sub
acute endo carditis
7 SEPTEMBER MORNING a.viridins goup of streptococci
SHIFT…….. MED AND ALLIed b staph aurus
1.theophyline interaction with which c.fecalis
causes decREASE its clearance Ans:: aa
a.adenosine 10.Most common cause of subacute
b.cimitidine endocartis
c.beta bloker a.viridiAns: streptococi
d. litium b. staphloccus aureus
Ans:: aa Ans:: aa
2.most common premalignnt condiotion… 11. Blood born infection
a.leukoplakia A. staphlococus
b.erythroplakia b.hepatitis B
c.lichen planus Ans:: bb
d.apthos ulcer 12.Child with sore throat and fever you
Ans:: cc gave antibiotics...after 3 weeks hematuria
3.painful vesicular eruption at chin and occurs
dermatomes involve diagnosis a. good pasture syndrome
a.trigeminal neuralgia b.poststreptococal glomerulonephritis
b.herpes Ans:: bb
Ans:: bb 13.Increase LES tone and speeds up
4.Immunoglobulins produced by: gastric emptying ?
a. plasma cells a. Metoclopramide.
b. hepatocytes b.OndAns:etron.
Ans:: aa Ans:: aa
5. following produce by plasma cells 14.there is achalasia of lower esophagus
a. albumin a. no myentric plexus in lower esophagus
b. firinogen b.food rapidly goes through esophagus
c. gama globulin Ans:: aa
Ans:: cc 15. Dignosis for Hashimotos
6. an enzyme which combine two and a.antimitochondraial antibody
make a single compound b.Antimicrosomal and anti thyroglobulin
a. synthetase Ans:: bb
b. isomerase 16. thyrocervical trunk
c.trAns:ferase a. suprascapular branch
Ans:: aa b. arise from second part of subclavian
7.Intercoastal nerves c.superior intercostal is branch
a.only supply skin Ans:: aa
b.12 in number 17. High carbohydrate diet taking should
c.Supply only intercostal muscles have in diet or deficientin
d.Associated with sympathetic chain a.thiamine
Ans:: cc b.riboflavin
8.percentage of lichen planus chance to Ans:: aa
covert in malignancy 18. Muscle of back supplied?
a.1-10% a. dorsal rami of cervical spinal.
b10-15 b. ventral rami of spinal nerves.
c.15-20 Ans:: aa
d.20-25 19.Cavernous sinus contain
e.25-30 A. abducent nerve
9

b. trochlear 29. Man has non lethal MI what happens


c. oculomotor to him in few minutes?
Ans:: aa A. complete recovery without any
20.premalingnat nevus is complication
a.compound nevus b. ventricular arrhythmia
b.intradermal nevus. Ans:: bb
c.junctional nevus. 30 What decreases penicillin excretion
Ans:: cc……as There is no option of from kidney
dysplastic or congenital giant nevus a.Probenecid
21.hyperthyroid ,goiter and exophthalmos b.Erythromycin
a.toxic goiter c.Gentamicin
b. graves disease d.Benzathin penicillin
Ans:: bb e.Penicillin G
22.Alchohol cause deficiency of which Ans:: aa
vitamin 31.Dead space decrease in
A, vitamin b1 A.standing
b.vitamin a b.tracheostomy
c.vitamin d c. deep inspiration
Ans:: aa d. shallow breathing
23.Sle diagnostic test Ans:: bb
a.anti double strand antibody( anti DsDNA 32. SA node action potential due to
Ab) a.Na influx
b. Anti Ach Ab. b.Ca influx
c. AMA. Ans:: bb.
Ans:: aa 33. Large intestine most accurate
24.Sjogren associated with? a.Caecum may have a diameter of 9cm
a.sle b.TrAns:verse colon supplied by right colic
b. RA c.Appendix is rarely retro caecal
Ans:: bb. d.20% have mesentry attached to ascending
25.lumber puncture site colon
a. above l4 e. Descending colon lies medial to left
b. below l5 kidney Cecum
Ans:: aa Ans:: dd
26. baby has abdominal pain and anemia 34. Mechanism of action of methyldopa
diagnosis.....repeated scenario of sickle a.Binds adrenergic receptors in cns
a.hbss b.Acts directly on arteriolar smooth muscles
b.hb a Ans:: aa
c.hb cs 35. Left axis deviation in
Ans:: aa A .Right sided myocardial damage
27. Hyperbaric oxygen use in treatment of b.RBBB
sea divers.. Ans:: aa
a.barotrauma 36. while taking blood sample at cubital
b.decompresion sickness fossa which nerve injured?
Ans:: bb a. median
28. optic chiasma damage with also b.ulner
hypothalamus damage will result in c.radial
a.dibetes inspidus Ans:: aa
b. hyperphgia 37. Common peroneal nerve lesion
c.hypertension. a. unable to evert
Ans:: bb b .unable to invert
c. unable to planter flex ankle and big toe
10

d Loss of sensation on medial side of leg. b.hypotension


Ans:: aa Ans:: aa
38.A child has bald patch on scalp since 46.bar body in xxx
birth. Which is leathery. On microscopic a. 1
exam there is papillomatous projections b. 2
in epidermis. Dermis main abundance of c. 3
sebaceous glands What is the serious Ans:: bb
complication 47.Short arms /legs . Lordosis . With
a.Basal cell carcinoma normal mental health . Disease mode of
b.Squamous cell carcinoma trAns:mission.. (achondroplasia)
c.Keratocanthoma a. autosomal dominant
d.scaling alopecia b. autosomal recessive
Ans:: cc c. x-linked
39. Mgso4 action inhibited by Ans:: aa
a. Na 48. adult Polycystic kidney disease mode
b. Cacl2 of trAns:mission
c. K a. autosomal dominant
Ans:: bb b. autosomal recessive
40.Thiazides decreses clearence of Ans:: aa
a.Class 2 antidepressants 49. L dopa given in Parkinsonism
b.Phenothiazide antipsychotics a. decrease drug inactivation in periphery
c.TCA b. increase peripheral utilization
d. lithum c. increased upake into plasma
Ans:: dd Ans:: aa.
41. patient cant flex index finger and loss 50.Vomiting present and absent
sensation of lateral palmer surface .which a.Ratio
nerve damage b.Nominal
a. median c.Ordinal
b. radial d.Pametrics
c. ulnar Ans: : bb
Ans:: aa 51.hormone increase in response of meal
42. Heparin primary action achieved by in duodenum
binding to a.cck
a. Antithrombin 3 b.secretin
b. Xa c. lipase
c. Thrombin Ans:: aa
Ans:: aa 52.increase gall bladder contractility
43. Skin incision with sharp object a.cck
redness due to b.gastrin
a. Axonal reflex Ans:: aa
b. Histamine 53. Heparin acts by binding to
Ans:: aa a.Factor xa and thrombin
44. Angina may be worsened by b.Factor xa only
A. Atropine c.. factor 8
B. Salbutamol d.Factor 7
C. Theophyline Ans:: aa
D. Vassopressin 54. In pancreatic fistula action of
E. Verapamil somatostatin???
Ans:: cc a. Inhibit enzymes
45. Most important feature of shock b Inhibit secretion
a.tissue hypoxia Ans:: bb
11

55. Inversion and eversion of foot on a.Megaloblastic anemia


which joint b.IDA
a.subtalar joint c. AOCD
b. calcaneocuboid. Ans:: aa
c.talonavicular 64. severe steatorrhea
Ans:: aa a.pancreatic resection
56.Phrenic nerve relation to ant to which b. stomach resection
muscle c.duodenal resection
a.scalnues anterior Ans:: aa
b.scalneus medius 65.nutmeg liver.
Ans:: aa a.chronic passive congestion
57.Phrenic nerve root value b.active hyperemia
a.C3c4c5 c. reactive hyperemia
b.C2,c3,c4 Ans:: aa
Ans:: aa 66.3rd heart sound murmr occur due to
58. Pregnant lady go into obstructed a. rapid inflow in ventricles
labour and reach hospital 50 miles away b. slow inflow in ventricles.
when reached hospital she died....cause? Ans:: aa
A.Fat emboli 67. esophgeal ca and middle 3rd is
b.amniotic emboli squamous diagnose
Ans:: bb a. alcoholism
59. Aspirin should be stopped before b. smoking
surgery. How many days the effect of Ans:: aa
aspirin remains? 68.EEG change
A. 7-9 days a.alpha waves disappear on eyes open
B. 3-4 days b.beta waves when eyes are closed.
C. 10-12 days Ans:: aa
D. 14-16 days 69.hyoid bone level
Ans:: aa a. C3
60. Heart chamber related to sternum b. c2
(costochondral surface) c. c1
a. right ventricle Ans:: aa
b. right atrium 70.human intermediate host of
c. left ventricle A.Hydatid disease .
Ans:: aa b. ameobic dysentery
61. Regarding blood supply of bones c.liver fluke
a. Small bones supplied by nutrient artery Ans:: aa
b. Irregular bone supplied by nutrient artery 71. patient died on brain biopsy which
c. Flat bones bone supplied by nutrient type of necrosis…
d. Shaft of long bone supplied by Nutrient a.liqufactive necrosis
artery only. b.coagulative necrosis
e. ends of long bones supplied by epiphyseal c. fat necrosis
arteries. Ans:: aa
Ans:: ee. 72.about pesudomAns:
62.urinary bladder mass there and a.endotoxin
features towards squamous cell b. exotoxin.
ca….cause Ans:: aa
a.smoking 73.aplastic anemia…
b. cotton industry worker a.hypocelluar tap on trephine biopsy
Ans:: aa b. bloody tap
63. mcv raised 107 and gut problem….. ? Ans:: aa
12

74.prophylaxis agaist tetnus to population 85.creatinine clearance Values 144 s


a.toxoid creatinine 1.4 And plasma creatinine =1.2
b. antitoxin a.1.68
Ans:: aa b.168
75.thamalic stoke or hamrohge like c.163
scnerio … d.cannot be calculated
a. hperasthesia Ans:: dd.
b.Synesthesia 86.true hermaphrodite,…
Ans: : aa a.xy
76.red infarct b.xyy
a.organ have dual blood supply c.xxy
b. organ having single blood flow Ans:: cc
Ans:: aa 87.xerostomia , and keratitis patient is
77.yellow fever in non endemic area having?
a. epidemic a.sjorjen syndrome.
b. sporadic b.Mukilicks syndrome
c.endemic Ans:: aa
Ans:: aa 88 .. patient with mydriasis and ptosis.
78.ventricles completely depolarized at Whats the cause
a.qrs a.occulomotor damage
b. st b.horner syndrome
Ans:: bb Ans:: aaa as in horner there is miosis ,ptosis
79. patient cant speak or utter words n enaphthalmos
damage to 89.In G6PD what is present?
a. wernickes a.comb test ,
b.brocas b.osmotic fragility
Ans:: bb. c. heinz body
80.. hemartoma… d.microcytosis
A. mass of mature but disorganised tissue Ans:: cc
indiginous to a location. 90.In venous blood rbc have?
b. mass of well differentiated tissue outside a.Inc cl conc
a particular location. b.Inc hco3
Ans:: aa Ans:: aa
81.left precentral gyri damage 91. In deep sleep hormone secreted is
a.right hemiplegia a. serotonin
b. left hemiplegia b.melatonin
Ans:: aa Ans: : bb ( melatonin secreted in deep sleep
82.adenoma while serotonin mediates deep sleep)
a.Glandular pattern. 92.diabetic patient wound healing
b. histologic variation impaired due to
Ans:: aaa a.neuropathy and angiopathy.
83.phledelphia chromosomes b. angiopathy
A.cml c.neuropathy.
B. AML Ans: : aa
Ans:: aa. 93.Mg store in
84. Atrial flutter a.bone
a.Treated with defibrillation shock therapy b.muscle
b. rate is so rapid above 400beats/min Ans:: aa
c.Heart beat between 200 and 350beats /min 94. Diabetic patient after gram negative
Ans: : cc infection present with shock after 6 days
sugar level normal
13

a. acute tubular necrosis b. chronic ulcer


b. increase permeability Ans:: aa
Ans:: aa 105.hypophysis cerebri related with
95. Patient stands from supine position a.sphenoid sinus
what will happen? b.ethmoid sinus
a. increase heart rate Ans:: aa
b. decrease venous return 106.soft palate lined by
Ans:: bb. a. stratified squamous non cornified
96.boat shaped organism in case of b. stratified squamous cornified.
pneumonia… Ans:: aa.
a. pneumocystitis jervoci 107.pt died after 6 days of MI on autopsy
b. cmv large amount of fluid seen in pericadia
Ans:: aa cavity
97.Patient smoker psychotic with a.cardiac tamponade
vomiting abdominal pain n visible b. arrythmias
perstalsis in upper portion of stomach Ans:: aa
a.Pyloric stenosis 108.gfr increase by
b.Psychogenic. a. afferent arteriolar dilation
c.achlasia b. afferent arteriolar constriction.
Ans:: aa Ans:: aa
98.fisherman with gums hypertrophy and 109.post 1/3 interventrcular sptum
bleeding …deficiency of which supplied by
a.vitamin c a. marginal
b.vitamin k b. right coronary artery
Ans:: aa c. circumflex
99. Bilateral ptosis at the end of day is Ans: : bb .
conformatory test? 110.thirst is stimulated by
A. Tensilon a .increase osmolarty and decrease volume
B.antibodies (Ach Receptor Ab) b. decrease osmolarty and increase volume
c.emg Ans:: aa
Ans:: bb 111.prolactin regulation by
100.isthamus of thyroid… a.dopamine
a.2,3,4 rings b. serotonin
b. 3,4 rings Ans:: aa
c. 1,2 rings 112.about pulse pressure depends.
Ans:: aa a. Stroke volume
101.corticospinal tract decussate at… b. venous return
a.pyramid Ans:: aa
b.medulla 113.heparin ,histamine and protease
c.pons release from
Ans:: aa a. mast cells
102.pus contain b. eosionphils
a.dead bacteria c. basophils
b.dead neutrophil Ans:: aa
Ans:: bb 114.glucocorticoid causes dec which cell..
103.which cause malignancy in git … a. lymphocyte
a.h.pylori b. neutrophil
b. cmv Ans:: aa
Ans:: aa 115.gluconeogenesis is caused by
104.which condition predispose to cancer a. insulin
a. barretes esophagitis b. glucagon
14

Ans:: bb Ans:: aa
116.resection of ilium 127.in myasthenia gravis treatment
a. decreased bile salt absorption. affected by
b.increased water content of feaces. a. Choline acetyl trAns:ferase
Ans:: aa b. acetyl coA
117.ventral corticospinal end at c, acetylcholine
a.mid thoracic d. acetylcholinesterase
b.upper thoracic e. cholinestrase.
c.all spinal cord Ans:: dd ( should be anti Ach esterase but
Ans:: aaa no option there).
118.muscle stretch then relaxation by 128. definitive diagnosis of a patient with
a. muscle spindle Tb asked
b. gto a.caseous necrosis
Ans:: bb b.identification of acid fast bacili
119. muscle stretch to extreme which Ans: : bb
prevent from damage or tear like that 129. length of right bronchus
a. muscle spindle a.2.5 cm
b. GTO b.5 cm
Ans:: bb c.10 cm
120. Hyaline cartilage Ans::aa
A. present in nasal septum 130. in mountain the po2 and nitrogen
B pinna from 700 was asked..
C TMJ articulation a.pO2 144 and pNo2 546
D epiglottis b.po2 110 and pno2 590
E.two menisci c. po2 100 pno2 600
Ans:: aa d. po2 98 and Pno2 602.
121.antimetabolite use in cancer Ans:: cc
treatment 131.in copd patient
a.methotrexate a.Vt 350.fev .20000. fvc 36000
b.azathioprine 132. univeral recipient
c.flourouracil a. AB -ve
Ans:: aa B.AB +ve
122.left shift of oxygen hb curve C. O+ve
a.increase temp D.O-ve
b.co poisononing Ans:: bb
Ans:: bb 133.about cerebellum
123. about left renal a. lies below tentorum cerebelli
a.ant to both aorta and left renal artery b.middle cerebellar connect with medulla or
b.post. to aorta spinal was given….
Ans:: aa c. only supply by basilar artery
124.one of delayed wound healing Ans:: aa
a.vitamin c deficiency 134.glucagon secretion increase by.
b. vitamin A exc ess. a.exercise
Ans:: aa b.insulin
125.intermediate filament for cancer is Ans:: aa.
a.cytokeratin 135 Patient have malar rash and anti ds
b.vimentin DNA positive now having proteinuria and
Ans::aa hematuria
126.diffusion depends upon like stem… a.lupus nephritis
a.total surface area . b.sore throat .
b.saturation kinetics, Ans:: aa
15

136 . edema of kidney origin 147.The most common changes which


AAlbuminuria and Na retention occur in a case of uterine prolapse
B.Only Na retention . a.chronic cervicitis with squamous
C.Decrease Ca. metaplasia
Ans:: aa b. dysplasia of cervix
137 A patient has injury red flare was c. hypertrophy of cervix
seen due to d. squamous metaplasia , endocervix
a. Histamine e.vaginal discharge
b. bradykinin Ans:: aa.
Ans:: aa 148.a 2 year boy having mcv raised .. and
138 Decrease Na secretion due to .?? choice of investigation
a. maximum Na reasbsorption a.vitamin b12
b. maximum Cl reabsorption. b. intrinsic factor
Ans:: aa c. anti intrinsic factor antibody.
139. One was asked about N REM Ans:: aa
A. night mares 149.heparin given affect on
b. on EEg delata waves a.factor X
Ans:: bb B.fctor x and thrombin c
140. hormone of stress c.factor vii
a. cortisol Ans::aa
b. norepinephrine. 150.Christmas disease
Ans:: aa a.Factor 9 deficient
141. gluocorticoid. b.Factor 8 deficient.
a. decrease utilization of glucose Ans:: aa.
b. increase utilization of glucose. 151. By the End of 5th week
Ans:: aa. a.heart definite form
142. thyroid hormone increases what? b.stomach rotates
a. free fatty acid c.limbs bid appear
b.cholesterol Ans:: bb
Ans:: aa 152. Internal carotid artery
143. micturation reflex. a.Lateral to external carotid at origion then
a.once start then self regenerative goes POST to come medialy
b.control by sacral segments of spinal cord. b. Give ophthalmic artery that enter the eye
Ans:: bb. through superior orbital fissure
144.post inter ventricular septum1/3 Ans:: aa.
a.right coronary artery 153.renin decrease by…. ??
b.circumflex a.antidiuretic hormone
c. left anterior descending artery. b. angiotension 2
Ans:: aa. Ans:: bb
145. Essential fatty acid 154.fascial nerve
a.linolenic acid a. supplies stapedius
b. lithocholic b. posterior belly of diagastric
c.cholic acid Ans: :bb
Ans:: aa 155.Visual Accomodation
146.Coarctation of aorta is associated a.Contraction of ciliary muscle
with b.Sphincter pupillae relax
A.Constriction just after the subclavian c. Tension of ligaments to lens
artery Ans:: aa
B.Dilatation below the coarctation 156 A protein that binds oxygen in
Ans:: aa. skeletal muscle
a.Myoglobin
16

b. Actin 165.Somatostatin helps in pancreatic


c. Myosin fistula by
Ans:: aa a.decreasing secretions
157 Germ cells arise in yolk sac at 3rd b.Inhibiting enzymes
week of Intrauterine life ..they originate Ans::aa
from ? 166At upper part of lung /zone 1
a. Endoderm a.Shunting is increased
b. Mesoderm b.Dead space is increased
c. Ectoderm c.Circulation highest
Ans:: aa Ans:: bb
158. Melanocytes derived from 167Which part of brain receives major
a. Dermis excitatory input to from cerebral cortex
b. Neural crest cells but it itself doesn't send axons to cortex
c. Mesoderm a.Substantia nigra
d. Endoderm b.Globes pallidus
Ans:: b. c.Caudate
159. Which is absorbed passively by d.VL thalamus
kidney e.VA thalamus
a. Urea Ans:: aa
b. Cl SEPTEMBER 7 EVENING
c. K
Ans:: bb MEDICINE 2016
160.Epitheliod cells of tubercle are 1. Which part of brain receives
a.Mononuclear phagocytes major excitatory input to from cerebral
b.LangerhAns: giant cells cortex bt it itself doesn't send axons to
c.PMN leukocytes cortex
Ans::aa a.Substantia nigra
161.Diffusion of a solute across cell b.Globus pallidus
membrane depends on c.Caudate
a.Saturation kinetics with nernst equation d.Ventrolateral thalamus
b.Carrier protein e.Ventro anterior thalamus
c.Surface area of membrane Ans::a.
Ans:: cc 2. MgSO4 effects can b overcome by ?
162.Most abundant in serum ? a.Calcium chloride
a.IgG b. Labetalol
b.IgM c.Atropine
Ans:: aa Ans::a.
163.A normal n healthy individual is 3.Most severe steatorrhoea in
a.Who eats sleeps n exercises regularly n a. Gall bladder resection
lead a disciplined life b...Pancrease resection
b. Economically Occupationally socially c..Terminal ileum resection
acceptable Free o f discomfort d..Colon resection
.c-Who is famous in his society Ans::b.
Ans:: aa 4.At upper part of lung /zone
164. A pt with hypoxia ..with po2 :60 a Shunting is increased
mmhg n HB :15g/dl...reason of hypoxia b.Dead space is increased
a.Histotoxic c.Circulation highest
b. Anemia Ans::b.
c. Hypoxia 5.Somatostatin helps in pancreatic fistula
Ans:: cc by
17

a.decreasing secretions a.-Who eats sleeps n exercises regularly n


b.Inhibiting enzymes lead a disciplined life
Ans::a. b.-Economically Occupationally socially
6. A patient with hypoxia ..with po2 :60 acceptable Free of discomfort
mmhg n HB :15g/dl...reason of hypoxia c.-Who is famous in his society
a.Histotoxic Ans::a.
b.Anemia 15. Most common premalignant condition
c.Hypoxia of mouth
Ans::c. a.Leukoplakia
7. True about blood supply : b.Erythroplakia
a.Flat bones supplied by nutrient Artery c.Lichen planus
b.Short bones supplied by nutrient Artery d.Apthous ulcer
c.End of long bones in Adult by epiphyseal Ans::c
Artery 16.Epithelium of oral surface of soft
d.Shaft by metaphysial artery palate
e.Vertebrae supplied by nutrient Artery a.Stratified columnar epithelium
Ans: : cc b.Stratified squamous non cornfield
8. Pin stroke on skin red flare cause ? epithelium
a.Histamine c Stratified cuboidal epithelium
b.Kinins d.Pseudo stratified columnar epithelium
Ans::a. Ans: bb
9. compensation on standing from sitting 17.Hyaline cartilage
a.Decrease stroke volume a.Precursor of endochondral ossification
b.Decrease Venous return b.Epiglottis
c.Increase Heart rate Ans::a.
d.Decrease Cardiac output 18.Cortico spinal tract crosses at
Ans:: bb a.Base of pons
10. Which drug increases LES of b.Pyramids
esophagus n speeds up gastric emptying c.Midbrain peduncle
a.Metochlopramide d.Ant horn cells
b.ondensaron e.Internal capsule
Ans::a Ans::b.
11. Effect of sympathetic system by Alpha 19.Intercostal nerve
discharge a.12 pairs
a.Contraction of radial muscles of iris b.Connection with sympathetic nerves
b.Glycogenolysis c.Only supply skin
Ans::a. d.Only supply intercostal muscles
12. A patient with a bald patch on scalp Ans::d
.which on examination is yellow velvety .. 20.Beginning of 5th week
It can lead to which dangerous condition a.Limb bud appear
a.Squamous cell carcinoma b.gut rotation begins
b.Basal cell carcinoma Ans:: bb
c.Keratoacanthoma 21.Hormone decreasing stomach
Ans::c. contractility upon fatty food in duodenum
13. Left deviation of oxygen dissociation a.Secretin
curve b.Cck
a.Carbon monooxide poisoning Ans::b.
b.Increased ph 22.Pt with chronic asthama for 5 yrs ..vt?
Ans::a. Fev1 ? Fvc?
14. A normal and healthy individual is a.Vt 300 fev 1 : 2000. Fvc:3200
b.Vt 450. Fev1 : 2100. Fvc:2600
18

c.Vt :350 fev1 : 3000. Fvc. 3700 b.Water diuresis


23.A man with short limbs but normal c.Increased absorption of Na
trunk n lumbar lordosis ..which type of Ans:.c
inheritance? 34.Melanocytes derived from
a.Autosomal recessive a.Dermis
b.Autosomal dominant b.Neural crest cells
Ans::b c.Mesoderm
24.Pulse pressure : d.Endoderm
a.Is more in peripheral artries than aorta Ans::b.
b.Depends on venous compliance 35.Germ cells arise in yolk sac at 3rd
Ans::a. week of Intrauterine life.they originate
25.Most abundant in serum ? from ?
a.IgG a.Endoderm
b.IgM b.Mesoderm
Ans::a c.Ectoderm
26.Diffusion of a solute across cell Ans::a
membrane depends on 36.Protein that binds oxygen in skeletal
a.Saturation kinetics with nernst equation muscle ?
b.Surface area of membrane a.Myoglobin
Ans::b b.Actin
27.True about large intestine c.Myosin
a.Descending colon is medial to left kidney Ans::a.
b.Right colic supplies trAns:verse colon 37.Hemorrhage in thalamus ..sensory
c.Ascending colon has mesentery in 20% changes ?
cases a.Numbness
Ans: cc b.Hyperesthesia
28.Lichen planus malignant Ans::b.
trAns:formation in ? 38.Patient with fatigue which worsens
a.10-15% towards evening ..most appropriate
b.20-25% investigation
Ans::a. a.Acetylcholine receptor antibody
29.When to stop aspirin before surgery b.Tensilon test
a.10-12 days c.Emg
b.7-9 days d.Ct brain
c.1-3 days Ans::a.
Ans::b. 39.Posterosuperior relation of pituitary
30.Atmospheric pressure 700 ..humidity a.Sphenoid air cells
zero ..partial pressure of nitrogen and b.Ethmoid air cells
oxygen ? c.Maxillary sinus
31.Epitheliod cells of tubercle are d.Mastoid air cells
a.Mononuclear phagocytes Ans::a.
b.LangerhAns: giant cells 40.True about cerebellum
c.PMN leukocytes a.Lies below tentorium selle
Ans::a b.Connected to midbrain via middle
32.Which is absorbed passively by kidney peduncle
a.Urea Ans:: aa
b.Cl 41.True about internal carotid
c.K a.Lies lateral to external carotid at origin and
Ans:.a latter turns to medial side
33.Decreased clearance of Na in ... b.Enters skull through squamotympanic
a.Osmotic diuresis fissure
19

c.Lies lateral to external carotid at origin n Ans::a


latter turns to medial side 51.. Beta endorphins
Ans::a. a. hypothalamus
42.Visual Accomodation b.pituatary
a.Contraction of ciliary muscle Ans::a.
b.Sphincter pupillary relax 52.. Typical scenario of sickle cell anemia
c.Tension of ligaments to lens with abdominal pain jaundice etc...
Ans::a. A. HB ss
43.Angina worsened by B. HB sc
a.Virapamil Ans:.a
b.Theophylline 53. Difference b/w sarcoma and
Ans:.b carcinoma
44.Tubular excretion of penicillin a.. pleomorphism
decreased by ? b. increase vascularity
a.Probenecid Ans::b.
b.Gentamycin 54. ITP scenario was given splenectomy
Ans::a done which organism will cause
45.Theophyllin action slowed down by postsplenectomy recurrent infections
concurrent use of a.. hemophilis influenza
a.Atropine b. streptococus pneumonia
b.Beta blocker Ans: :b ( Shin : strept pneumonia >
c.Adenosine h.influenza type b > Nisessria meningitidis )
Ans::c 55. Hepatitis scenario viral markers
46.Mechanism of action of methyldopa negative ALT slightly raised on
a.Directly causes relaxation of arteriolar sm examination there were some eye findings
muscles what is the suitable diagnostic test
b.Stimulation of adrenergic system in CNS a. serum ceruloplasmin
c.inhibition of adrenergic system in CNS b. urinary copper
Ans:: cc c. HbsAg
47.Post 1/3 of Interventricular septum d. anti mitochondrial antibody
damaged ..artery ? Ans::a.
a.posterior interventricular artery 56. Patient comes with tender right
b.Right coronary artery hypochondrium stool with occult blood on
Ans::a colonoscopy, ascending colon was
48.Patient had non lethal MI .what can involved, showing ulcers while other parts
develop in few minutes ? of colon were spared on histology/
a.Cardiac arrythmias microscopy what will be seen
b.Left ventricular failure a. necrotizing vasculitis
c.Nothing b. crypt abscess
Ans::c c.entamoeba histolytica
49.linear growth of bone is affected if d. carcinoma
following structure is fractured Ans:: bb
a. metaphysis 57.microscopic findings were epithelium
b. epiphyseal line and rete ridges
c. epiphyseal plate a.verrocus carcinoma
d. diaphysis b.squamous cell carcinoma
Ans: c Ans:: bb
50..Abscess anterior to pretracheal 58. Child with history of bed wetting
fascia.infection can spread to though toilet trained investigations done
a. anterior mediastinum at one side duplication of ureter was
b.post.mediastinum
20

found and one of them was opening in b. decrease neutrophil migration


vagina. what is the cause c.infection
a. early division of ureteric bud. Ans:: aa
b. Inability of mesonephric duct to join to 66. Slowest growing malignant thyroid
ureteric bud Carcinoma
Ans::a. a. papillary
59.Sand blaster b. follicular
a. asbestosis c.lymphoma
b. silicosis Ans:: aa.
C. anthracosis 67.Which infusion will increase
Ans::b Extracellular fluid to maximum extent
60.Anxiety is decrease by the activation of a. hypertonic fluid
which receptors b. isotonic fluid
a.GABA c.hypotonic
b. glutamate Ans:: aa
c. glucocorticoid 68.. Steady pressure is detected by
d.nicotinic cholinergic a. meissners
e. dopamine b. ruffini
Ans::a. c.pacinian
61. Turner syndrome Ans:: bb.
a. autosomal recessive 69.Patient with myocardial infarction
b. autosomal dominant with typical sign and symptoms.st
c.gynaecomastia elevation in leads II, III and AVF
d.short stature a. anterior wall MI
Ans::d b.inferior wall MI
62. Chronic renal failure patient died on c.anerioseptalMI
autopsy which organ will show Ans::b.
hypertrophy 70.Patient with AML undergone bone
a. thyroid marrow trAns:plant got pneumonia
b. parathyroid histology showing large cells with
c.adrenal intracellular inclusions cause is
Ans::b. a. CMV
63. Which cell organelle contain double b.candida
membrane c. pneumocystic
a. nucleolus Ans: : aa
b. golgi apparatus 71. Child brought by mother suffering
c. ribosomes from diarrhea on examination rectal
d. Rough endoplasmic reticulum prolapsed was found. worm with small
Ans:: dd anterior end
64. Fisherman with gingival hyperplasia a. entrobius vermicularis
and ecchymosis which one is deficient b. trichuria trichuris
a. vitamin B12 c. whip worm
b.vitamin K Ans:: bb.
c.vitamin c 72.Best indicator of venous return
Ans:: cc a. end diastolic volume
65 Patient on immunosuppressant’s b.end systolic volume
abscess is formed on upper outer half of c.capillary filling pressure
arm after drainage healing process is very Ans: : aa
slow though a month has been passed 73. myocardial infarction patient after 48
reason hour found collapsed no pulse. on ECG
a. decrease collagen formation
21

irregular waves were seen. whats the 81. Middle aged man presented with
pathophysiology? meningitis after having lung abscess
a.decrease cardiac output a.Staph. Aureus
b. ventilation perfusion defect b. Streptococcus fecalis
c.decrease capillary filling . c.Streptococus viridiAns:
d.venous pooling Ans: : aa.
Ans:: bb 82. Which toxin is responsible for scarlet
74.Subcapsular afferent fever
a. lymph nodes a. exotoxin
b. spleen b. erythrogenic
c. thymus c. endotoxin
Ans: : aa. Ans: : bb
75. Alveoli are kept dry because of 83. Serum sodium is regulated by
a. alveolar macrophages a. osmoreceptors
b. tight junction between capillaries b.Volume of ECF
c. surfactants c.ICF volume
d.negative interstitial pressure Ans: : aa
Ans:: dd 84. Highest sodium channel
76.Dead space does not change in concentration
a .shallow breathing a. initial segment
B deep inspiration b. node of ranvir
c.standing c. dendrites
d. re-entery current Ans: : bb
Ans:: aa 85. in chronic liver disease which
77.Least amount of minerals are found in histological finding suggests chronicity
a. roots a. fibrosis
b. Tubers b. councilman bodies
c. cereal c. lymphocytes
d. pulses Ans: :aa
e. vegetables(leaves) 86. FFPs used for the acute management
Ans: : bb of
78. Erythropoietin secretion is inhibited a. factor 8 deficiency
by b. warfarin overdose
a. cobalt c. PLT function defects.
b. hypoxia Ans: : bb
c.theophylline 87. intense trAns:fusion reaction occurs if
Ans: : cc we trAns:fuse
79. Which is low in csf as compare to a. A- to A+
plasma b. A+ to O+
a. Na c. A to A-
b. Osmolarity Ans: : bb
c. Cl 88. which prevents muscle from tearing
d. Mg under pressure
e. protein a. GTO
Ans: : e b. muscle spindle
80.Cells of chronic inflammation c. Ligaments
a.macrophages Ans: : aa
b.lymphocytes 90. flexor withdrawal reflex
c.eiosionphils a.multisynaptic
d.basophils b.monosynaptic
Ans: : aa c.disynaptic
22

Ans: : aa b. nor epinephrine


91. right atrial pressure is increased c. dopamine
a. increases cardiac output Ans: : aa
b. increased venous return 99. saliva prevents iron utilization by
c. increases intrathoracic pressure microbes via
Ans: : bb a. Lactoferrin
92. After adrenalectomy which is b. lysozymes
preferred c. Lactase.
a. glucose Ans: : aa
b. NaCl 100. benign tumor
c. Mg cl a.Leiomyoma
Ans: : bb b.Hamartoma
93. in cell membrane of RBCs CL- and c.Basal cell carcinoma
HCO3- exchange occurs through Ans: : aa
a. ankyrin 14 OCTOBER MED N ALIED
b. band3
c. spectrin EVENING PAPER
Ans: : bb 1. A patient undergoes kidney transplant
94. patient with discoid rash arthritis and
hemolytic anemia whats most specific on first day he was unable to produce
investigation urine there is also tenderness at transplant
a. ANA site
b. anti ds DNA antibdy A} Graft versus host rejection
c. RNA studies {b} hyper acute graft rejection
Ans: : bb {c} blockage of ureter
95. patient with malaise and raised BP {d} renal artery stenosis
died because of hemorrhagic stroke in {E} thrombus at the side of transplant
basal ganglia. on autopsy bilateral small ANS.D
kidneys with petechial hemorrhages,
hyperplastic arteriosclerosis and fibrinoid 2. Blood supply interrupted to a tissue
necrosis. whats the diagnosis which type of necrosis will occur
a. fibromuscular dysplasia {a} coagulative necrosis
b. DM type II {b} fibrinoid necrosis
c. systemic sclerosis {c} caseous necrosis
d. NSAID induced ANS.A
Ans: : aa 3. what are present at thin loop of henle
96. which one is most pre malignant {a} brush border
a. compound nevus {b} mitochondria
b. intradermal nevus
c. seborrhaic keratosis {C} flat epithelium
d. dysplastic nevus ANS.C
Ans: : dd Flat epithelium is correct as there are no
97. Old male patient has history of brush borders in thin descending loop but
headache in temporal region thick cord few mitochondria are present
like vessels in that area on biopsy giant 4. one scenario about dislocation of
cells are seen acetabulum posterio superior which bone
a. ESR more than 110 . dislocate
b. Diagnosis is made on biopsy . {a} sacrum ilium
Ans: : bb
{b} pubis sacrum
98. presynaptic sympathetic fibers release
a. ACH {c} ischium sacrum
23

{D} ilium ischium {D} increase venous compliance


ANS.D ANS.C
5. At which point in ecg more Ca ions 11. vagal trunk cut what happens
enter in ventricular muscle cell {A} incomplete gastric emptying
{a} p wave {B} augmented gastrin secretion
{b} r wave {C} augmented pepsin
{c} s-t segment {D} augmented enterogastric reflex
{D} p-r interval ANS.A
ANS.C 12. most abundant IG in adult body
6. even in the absence of ADH 65% of {A} igG
water is absorbed where this absorption {B} igA
occur {C}igM
{a] pct ANS.A
{B} thick loop of henle 13. in 3rd degree block what is true
{C} thin loop of henle {A} atrium beat independently of ventricles
{d} collecting ducts {B} there fixed ratio of between atria and
ANS.A( yaad rakhen bcqs me absence ho ventricles
ya presence of ADH max water PCT me Ans:A
he absorb hoga ref ganong) 14. ECG of patient showed absent p wave
7. A diabetic patient has unhealing ulcer normal QRS complex n T wave , where
on the foot what is correct should b the pacemaker?
{A} angiopathy {A} SA node
{B} neuropathy {B} AV node
{C} poor sugar control {C} purkinji fibers
{D} neuropathy and angiopathy {D}ventricular muscle
ANS.D ANS.B( ideally rt ventricle is ans but it was
8. spinal part of accessory nerve not in options)
{A} passes below transverse process of atlas 15. Gravida 2 first pregnancy normal ,
{B} passes through the substance of 2nd boy delivered at 32 weeks with
sternocledomastoid hydrops fetalis , icteric phase , exchange
ANS.B transfusion done what is the type of
9. patient had severe dehydration what is immunity?
the best treatment {A} type 4 hypersensitivity
{A} 5% dextrose {B} complement mediated hypersensitivity
{B} 10% dextrose {C} antireceptor antibody
{C} albumin {D} self tolerance
{D} normal saline {E} autoimmune complex formation
ANS.A ( ganong 22nd edition bcq key is A ANS.B
there) 16. which one is essential amino acid?
10. a patient presented with hypotension {A} serine
he is cool and tachycardiac he was asked {B} tyrosine
to stand up on standing his heart rate is {C} therionine
increases why {D} proline
{A} decrease in total peripheral resistance ANS.C
{B} decrease after load 17. regarding cemitidine effect ?
{C} decrease venous return {Ans } inhibit the hepatic enzyme
24

18. A diabetic patient smoker with {C} hypovolumic shock


myxedema which drug u will give for {D} due to trauma
hypertension? Ans:C
{A} nifidipine 26.. diagnostic test for diabetic
{b} b bloker nephropathy?
{C} captopril {A} cretinine clearance rate
{D} varapamil {B} serum cretinine
ANS.C {C} blood urea nitrogen
19. commn cause of thromboembolism? {D} microalbuminurea
{Ans} prolonged immobilization ANS.D
20. female with infection pap smear 27. A patient with thyrotoxicosis heart
shows prominent nucleoli n increased rate 145 per mint BP 160/90 which drug
nuclear size irregular architecture? should be given
{A} dysplasia {A} B-blocker
{B} metaplasia {B} ca channel blocker
ANS.A {C} digoxin
21. a group of people studied for effect of ANS.A
a drug and compared to a control group 28. EBV cause which carcinoma
they are followed longitudinally over a {A} bronchoalveolar
period of time what type of study is? {B} nasopharryngial
{A} case control ANS.B
{B} cohort study 29. difference between skeletal and
{C} randomized control trials smooth muscle?
ANS.C {A} scattered myofibrils
22. edema due to renal cause can be due {B} ca calmodulin present in smooth muscle
to? {C} specialized cells for forceful contraction
{A} albuminurea ANS.B
{B} glomerulonephropathies 30. which hormone is responsible for
{C} Na retension multiple fracture in pt on dialysis?
{D} Na retension and albuminurea {A} PTH
ANS.D {B} cortisol
23. Which organelle inside the cell can {C} low vitamin D
replicate? {D} epiandristenedione
{a} mitochondria ANS.A
{B} centriole 31. which enzyme responsible for
{C} nucleolus degradation of norepinephrine and
ANS.A serotonin?
24. defecation carried out by? {A} COMT
{A} sacral parasympathetic nerves {B} MOA
{B} myentric plexes ANS.B
{C} mass movements 32. on chest p/A view sternocostal surface
ANS.C is formed by?
25. A patient undergoes tonsillectomy he {A} right atrium
became hypotensive after 2 hours why? {B} right ventricle
{A} blood loss {C} left ventricle
{B} vasovagal shock ANS.B
25

33. true about blood? 40. Organophosphate poisoning


{A} wbc most abundant cells treatment?
{B} pletelets are more then rbcs {Ans} atropine
{C} iron in blood is mainly in hemoglobin {pralidoxime was not in option
ANS.C 41. true about left renal vein?
34. which of the following donot causes Ans. anterior to aorta
acid secretion 42. in carbon monoxide poisoning
{A}stretching of antrum hyperpnoea does not occur why?
{B} alcohol {A} paO2 is normal
{C} acidic antrum {B} paCO2 is normal
{D} peptides {C} blood flow increased
ANS.C ANS.A
35. Ansa cervicalis supplies to? 43. systemic arterioles vasoconstriction
occurs if local concentration rises?
{A} omohyoid {A} ANP
{B} strnohyoid {B} NO
ANS.A {C} angiotensin2
36. CLD with upper GI bleeding best ANS.C
treatment? 44. 4 n 5 brachial arch give rise to?
{A} transamine {A} hyoid
{B} vit k {B} thyroid
{C} octriotide {C} cricoid cartilage
ANS.C (if terlipressin is in option then click ANS.C
45. which drug if given intravenous
it hre its c) causes 60% thrombophlebitis
37. one question was about that there was {A} oxazepam
no elevation of neural tissue below {B} i/v diazepam
lumbosacral region what the condition {C} midalozam
called {D} triazolam
{A} meningomylocele ANS.B
{B} Rachishesis with myloschisis 46. which drug can be given
{C} occulta preoperatively to a patient with liver
ANS.C decompensation?
{A} lorazepam
38. A patient after accident talks {B} oxazepam
excessively but talk irrelevant and {C} diazepam
purpose less which area damaged? {D} Phenobarbital
{A} brochas area {E} fentanyl
{B} wernick ANS.A
{C} motor sensory 47. A boy with recurrent chest infection ,
ANS.B lymphadenopathy, klebsiella at each
39. posterior surface of heart is supplied occasion diagnosis?
by how many arteries? {A} chronic granulomatous disease
{B} x-linked agammaglobulinemia
{A} usually two in numbers ANS.A
{B} usually three in numbers 48. hyperthyroid patient with pulse 180,
{C} ant descending supply anterior surface syncope, palpitation treatment ?
{D} interventricular artery {A} digoxin
{E} right coronary artery {B} varapamil
ANS.A {C} lignocaine
26

{D} propranolol ANS.B


ANS.D 56. Again scenario on hemophilia,
49. A patient developed atrial fibrillation pattern of inheritance?
he was treated accordingly but same {A} autosomal dominant
patient died due to occlusion of vessel in
brain why? {B} autosomal recessive
{A} arrhythmia {C} x linked recessive
{B} thrombus {D} x linked dominant
{C} embolism {E} multifactorial
{D} hypotension ANS.C
ANS.C 57. which vitamin take part in the
50. one question was there is a coin lesion formation of clotting factors?
in lung , how to differentiate between {A} vit A
granuloma and tumor? {B} VIT K
{A} autonomous growth
{B} growth at increased rate {C} Vit E
ANS.A {D} vit C
51. A multipara women presented to ANS.B
doctor with pulsatile mass below inguinal 58. A farmer visited a clinic for some
ligament what is it ? chest related problem on x-ray a round
{A} psoas abscess ball was seen in chest?
{B} femoral hernia {A} aspergilloma
{C} femoral artery aneurysm {B} bronchogenic carcinoma
{D} saphenous varix ANS.A
{E} lymphadenopathy
ANS.C 59. vital capacity 5L, inspiratory capacity
52. again question was about 3.5,tidal volume 500 expiratory reserve
nasopharyngeal Ca caused by? volume was asked?
{A} EBV {Ans} 1.5
{B} HPV 60. IN tabes dorsalis urinary bladder?
ANS.A {A} atonic
53. A rash first started on a child near {B} hypersensitive
ears then spread to face neck and trunk {C} hyposensitive
diagnosis? {D} spastic neurogenic
{A} herpes virus
{B} measles ANS.A
{C} mumps 61. A patient comes to u with chest pain
{D} varicella what question will give u maximum
ANS.B information?
54. one scenario about hemophilia {A} tell me about pain
platelets 350000,family history of {B} any history of heart disease in your
increased bleeding ,what investigation family
first to performed? {C} point to the location of pain in your
{A} coagulation studies chest
{B} platelet function test
{C} iron studies {D} have you ever visited doctor for heart
ANS.A complain in past 6 months
55. second heart sound produced by? ANS.C
{A} closure of mitral and tricuspid 62. in complete heart block pace maker is
{B} closure of aortic and pulmonary placed?
{C} rapid filling of ventricles {A} left atrium
27

{B} Right ventricle {A}sensory


{C} right atrium {B} due to ear wax
{D} SA node {C} conductive
{E} AV groove {D} mixed
ANS.C
ANS.B 70. patient presented to dentist with tooth
63. confidentiality can be breached? problem the dentist referred him back to
{A} when patient authorizes you the physician coz pt had resting tremors
{B} insurance claim where is problem?
ANS.A {A} hippocampus
64. A male complains of pain at penis, {B} substancia nigra
urine culture negative for N.gonorrhea ANS.B
but leukocyte are positive in urine D/R 71. A patient presented with mental
which organism can cause this infection? retardation, protruded tongue , umbilical
hernia and he is hypermobile and
{A} E.coli
dystonic which trisomy this is?
{B} Chlamydia {A} 13
{C} HPV {B} 21 DOWN SYNDRO
ANS.A Ans B
65. organophosphorus poisoning , 72. saesamoid bone is present where?
characteristic feature? {A} ala of nose
{A} dry skin {B} epiglottis
{B} flaccid paralysis ANS.A
{C} pinpoint pupil 73. oneconfusing scenario about vertebral
fracture?
ANS.C {A} axis
66. lesion at the median region in optic {B} atlas
chiasma causes which lesion? {c} c7 vertebra fracture
{A} bitemporal hetronomous hemianopia ANS.B
{b} bitemporal homonomous hemianopia 74. bifurcation of trachea?
ANS.B {Ans} t4,5 in normal supine person
67. a pregnant lady visited gynecologist 75. ventricular filling in which phase?
for dilatation n curettage on microscopic {A} atrial systole
examination which type of precancerous {B} rapid inflow
{C} slow ejection phase
change will be indication for ANS.B
hysterectomy? 76. which substance in earth crust causes
{A} squamous pulmonary fibrosis?
{B} complex hyperplasia {A} silicon
{C} simple hyperplasia 77. one question was about middle
{D} metaplasia meningeal artery ?
ANS.B {A} it passes through foramen spinosum
68. lesion at left optic tract causes which {B} causes subdural hematoma
lesion? ANS.A
78. what arises from intermediate
{A} right homonomous hemianopia mesoderm
{b} left homonomous hemianopia {A} mesonephrone and mesonephric duct
ANS.A {B} epithelium of gut
69. A young girl finds it hard to hear at ANS.A
low frequencies of sound which type of 79. true hermaphrodite ?
conduction ? {Ans} XXy
28

80. A patient visit Pakistan first time {C} hypertrophy and hyperplasia
develop fever ,headache n then coma ANS.B
{A} plasmodium falciparum 91. which one is more sensitive to pain
{B} dengue something like that scenario
ANS.A {A} visceral
81. most widest part of Dural space? {B} parietal
{A} L2 ANS.B
{B} s2 92. which intercellular junction is like a
ANS.A spot weld between cell and the base?
82. jaw trauma cant open mouth {A} hemi desmosome
something like that scenario {B} adhering junction
{A} orbicularis oris {C} desmosome
{B} lateral pterygoid ANS.A
ANS.B 93. In a person with PH 7.49, hco3 level
83. agammaglobulinemia person is more 30, pCO2 is 50, pO2 95
prone to? {A} compensated respiratory acidosis
{A} pyogenic infection {B} partially compensated respiratory
{B} viral infection acidosis
ANS.A {C} partially compensated metabolic
84. young sexually active male presented acidosis
with painful micturation urine showed 20 {D} partially compensated metabolic
to 30 wbc with no penile lesion ,N alkalosis
gonorrhea negative most likely organism ANS.D
is 94. A male patient having unilateral
{A} HPV undescended testis if not treated what will
{B} CMV occur?
{C} herpes {Ans} neoplastic change
{D} Chlamydia 95. Regarding cimetidine?
ANS.D {Ans} inhibit hepatic enzymes
85. which drug does not cause 96. a big scenario patient having aplastic
gynaecomestia ? anemia which type of hypersensitivity?
{A} griseofulvin {A} type 4
{B} Androgen {B} type 3
ANS.B {C} type 2
86. hydroxlation of proline and lysine {D} type 1
done by? ANS.C
{Ans} vit C 97. first sacral spine level is at
87. pregnant lady with jaundice went to {A} c1
remote village to see her parents. {B} T1
{A} hep E {c} l2
{B} hep A {d} s1
{c} hep D ANS.D
ANS.A 98. the histology of thin ascending limb of
89. salicylate toxicity causes what? loop of henle?
{A} alkalosis {A} brush border
{B} acidosis {B} flat epithelium
{C} hyperventilation {c} mitochondria
ANS.A ANS.B
90. which one occur on breast? 99. one question was about CSF finding
{A} hypertrophy meningitis?
{B} hyperplasia 100. high fiber diet benefit?
29

{A} causes diarrhea MEDICINE 7 NOVEMBER


{B} decrease GI motility
{C} causes constipation MORNING
{D} Lowers cholesterol 1) Articular cartilage most commonly?
ANS.D A. Belongs to category of elastic cartilage
101. Neural crest cell arise from? B. Can easily repair after injury
{A} neural tube C. Cells orderly arranged in column
{b} neural plate D. Covered by perichondrium
{C} ectoderm E. On epiphysis of long bones in synovial
ANS.C joints
102. coccygeal segment of spinal cord lie Ans is E
at the level of? 2)Most imp in irreversible cell injury ???
{A} L1 A. Damage to cell memb
{B} c1 B. Loss of co enzymes
{c} s2 C. Massive Ca ion influx
ANS.A D. Lysosome swelling
103. maximum clearance in kidneys E. Mitochondrial swelling
{A} creatnine Ans is C
{B} PAH 3)Acute MI,post 1/3rd Interventricular
{C} inulin septum,artery involved
ANS.B A. Ant descending
104. inversion and eversion at a joint? B. Circumflex
{A} ankle joint C. Left coronary
{B} subtalar joint D. Marginal
ANS .B E. RCA
105. which structure passes through ANS IS E
esophageal hiatus? 4)Lesion at rt precentral gyrus
{A} vagus nerve A. Exaggerated knee jerk on left
{B} azygous vein B. Rr hemiplegia
{C} thoracic duct C. Loss of voluntary control on left
ANS .A D. Normal plantars on rt
106. function of fibroblast ? E. Rt side sensory loss
{A} secrete collagen only Ans:A
{B} secrete elastin only 5)Multinucleated giant cells least likely
{C} amorphous extracellular matrix present in
ANS.C A. Blastomycosis
107. child eat coin Its location on x-ray? B. Cat scratch disease
(a) Right lower segment C. Sarcoidosis
(b) right Lower bronchus. D. Leprosy
ANS. A E. Strep pneumoniae
108. One bcq related with golgi tendon??? Ans is E
Ans. causes relaxation 6)Urine flow rate 1ml/min. Lowest
109.. Patient awake eyes closed waves on osmolarity in which part
ECG? A. Beginning or pct
Ans: Alpha waves B. Dct
110. A climatize patient increase HB due C. End of cortical collecting tubule
to ?? D. End of cortical papillary collecting tubule
Ans: Secondary polycythemia E. Tip of loop of henle
111. branches of right coronary artery Ans B
Ans: Sa nodal artery 7)At the end of ventricular
30

contraction,what happens D. Variety of Hydrolytic enzymes


A. Aortic valve opens E. Oxidative enzyme
B. Av valve close Ans:D
C. Pulm valve close 14)Long scenario regarding rectal
D. Av valve open carcinoma. Or end pe pucha tha most imp
E. Aortic opens microscopic finding for malignant ca ??
Ans C A. Pleomorphism
8)BT increased in B. Inc N/C ratio
A. Vwb C. Invasion
B. Hemophilia Ans:C
Ans:aaaaaaaaaaaa 15)Oral (buccopharyngeal) membrane
9)Drug tolerance due to ??? separates???
A. Dec degradation of enzymes in liver a. Amniotic sac from yolk sac
B. Inc degradation of enzymes in blood B. Larynx from pharynx
C. Failure to take med on time C. Nasal cavity frim oral cavity
D. Dec effect of drug upon repeated D. Proctoderm from oral hindgut
administration E. Stomodeum from foregut
Ans D Ans is E
10)What stimulates thirst? 16)Thoracolumbar outflow composed of
A. Dec plasma osmolarity and inc plasma A. ANS
vol B. CNs
B. Inc plasma osmolarity n dec plasma vol C. Parasympathetic
C. Dec plasma osm n vol D. PNS
D. Inc plasma osm n vol E. Sympathetic
E. Injecting vasopressin into hypothalamus Ans is E
Ans:B 17)Difference of primary n secondary
11)55 yrs old female,non insulin healing
dependant diabetes. Had Ans. Contraction Of wound
cholecystectomy. 3rd post op day she 18)Corpus callosum?
developed sudden dyspnea, chest pain n A. Connected to fornix by septum
hemoptysis. Cause pallucidum
A. hiatus hernia B. Most fibers interconnect symmetrical
B. MI areas of cerebral cortex
C. pneumonia C. Related superiorly to falx cerebi
D. Pulmonary embolism D. Fibers of genu curve into frontal lobe as
E. Spontaneous pneumothorax forceps minor
Ans:D E. Rostrum connects genu to lamina
12)Horners syndrome. If sympathetic terminalis
fibers to eyes are involved. What happens Ans:B
?? 19)Achondroplasia ka scenario tha with
A. Absent sweating on same side short limbs waghera . Genetics puchi thi
B. Eye lid drooping same side >?
C face flushing same side Ans:Autosomal dominant is
D. Same side xerosis n anhydrosis 20)Dorsal ramus of 2nd cervial nerve?
E. Common in younger age group A. Greater auriculad
Ans:A B. Greater occipital
13)Lysosomes?? C. Lesser occipital
A. Oxidation of long chain fatty acids D. Supraclavicular
B. Formed by smooth endoplasmic reti E. Tranverse cervical
C. Cant digest worn out cell components Ans:B
21)Which receptors undergo rapid
31

adaptation? C. Mode
A. Baroreceptors D. Standard deviation
B. Cold receptors Aaaaa or bbbb stll to confirm
C. Muscle spindles 31)Duchene muscular dystrophy ki
D. Pain rec defective protein???
E. Touch Ans:dystrophin
Ans:A 32)Exposure of what causes demylination
22)Baroreceptors ka function tha and injury ti peripheral nerves ??
A. Respond to inc b.p A. CO
B. Respond to.dec b.p Baki kuch respond to B. Lead
heart rate C. Benzene
Ans:B D. Silica
23)Parasympathetic ka effect on heart E. Asbestosis
A. Inc CO Ans: B
B. Inc SV 33)Ek inorganic dust wala scenario men.
C. Inc PR interval Old baba. No cough. B/L lung men kuch.
Ans:ccccccccccccc ?
24)PTH ka effect on kidneys Ans :was inorganic duct (From past papers)
Ans: Increased ca reabsorption i marked 34)Post MI death. On 4th day findings on
Dont remember the site biopsy of lefr myocardium ???
25) Ek tha EEG records electrical activity A. Edema, loss of striation
of brain during sleep area of brain?? B. Transmural collagen deposit
Ans:Reticular formation C. Lymhocytes
25) Max speed on conduction in heart D. Necrosis, neutophils,macrophages
tissue----??? E. Perivascular n interstitial amyloid
Ans Purkinje Fiber deposits
26)Ek tha pt on total parenteral nutrition. Ans:D (macrophages replace neutrophils)
Develops alopecia and dermatitis. Likely 35)Most common cause of death in
cause ? rheumatic fever???
A. Riboflavin Ans:myocarditis
B. Zinc 37)Increase BT and PTT………??
Ans:B Ans:vWD
27)Spinal se parasympathetic outflow 36)Iron deficiency anemia ………???
A. L1,L2 Ans: hypochromic microcytic can occur
B. S2 3 4 38)Pregnant female. Anemic. MCV n
C. S3 MCH low. HbA 96% or HbA2 4%??
Ans:B A. Alpha thalassemia
28)Clically GFR is calculated by?? B. Beta thalassemia trait
Asn:creatinine Clearance C. Sickle cell waghera
29)Shift to left due to hypocarbia. Which Ans:B
effect 39)Wohi hypochromic wala scenario.
A. Bohr effect Elder bro has blood transfusion hx
B. Haldane effect Best investigation ??
C. Ficks law Ans:Hb electrophoresis
Ans:B 40)Hemophilia ka characteristic bleeding
30)Ek tha us men bht sari values thi. pattern
Meds ki prices thi like 2.5, 4, 4.5,9. Is A. Ecchymosis
tarah ker k. Us men pucha tha best B. Petechiae
measure of central tendency C. Bleeding in tissuesAns
A. Mean Ans: C
B. Median 41)16 yrs old girl. Epistaxis.
32

Menorrhagia. Raised BT. Bht zeada C. Supplied by inferior gluteal nerve


raised aptt tha. Normal pt. Platelets 280. Ans:C
Dx 51)Duodenum??
A. Vwb A. Ampula of vater opens in posterolateral
B. Glanzman thrombasthenia B. 25 cm long
C. Factor 9 deficiency C. Feathery something (which cant be...ye
Ans:A jejunum men hota shayad)
42) . Old age man. Epigastric pain for 3 D. Retroperitoneal parts are 1st,3rd n ye wo
yrs not relieved by antacid. Gastric Ans: B
antrum ki biopsy men kuch erythema 52)Filum terminale??
waghera tha. Non specific gastritis. Cause A. Ends at 2nd sacral vertebra
??? B. Composed of pia
A. Adenocarcinoma C. Composed of arachnoid
B. Zollinger ellison D. Has some central canal thing
C. H.pylori Ans:A It is also a modification of pia
D. Duodenal ulcer 53)Hemisection spinal cord rt side???
ANs Cccccc (hint h pylori is present in A. Same side ki proprioception lost
antrum) B. Same side ki pain temp lost
43) Drug which alters thyroid hormone Ans:A
levels in pregnancy , crosses placenta? 54) Man has atrophy of left leg.
Ans:. PTU for sure Atrophied calf muscles and lateral side of
44)Child with rectal prolapse. Worm with foot.??
some chota ant end. ?? Sensations ka kuch nai mentioned tha.
Ans:Trichuris Lesion
45)Chronic cough for more than 5 yrs. A. LMN
Copious amounts of sputum. ?? B. UMN
Ans:Chronic bronchitis C. LMN
46)Characteristic feature of sarcoidosis Ans:C
?? 55)70 yrs female diabetic. Difficulty in
A. Diabetes insipidus seeing distant objects. VA reduced in both
B. Erhythema nodosum eyes ????
C. Hepatic granuloma A. Cataract
D. Hypercalcemia B. Hypermetropia
E. Pleural effusiom Ans: Aaaaaaaaaa
Ans:D 56)PDA ???
47)12 yrs old afghan boy. Fever. Night A. Inc arterial pco2
sweats,On rt side neck discharging sinus. B. Dec arterial po2
No growth revealed on routine culture. C. Peripheral cyanosis
Most appropriate initial single step D. Inc systemic pressure
investigation ?? E. Inc pulmonary pressure
A. Biopsy Ans:C
B. ZN staining 57)Regarding split of S2. Split can be
C. Mycobacterium culture enhanced by ??
Ans: B A. Early closure of aortic valve
48)Drug for actinobactee boumanii ?? B. Delayed closure of aortic valve
Ans: imipenem C. Early closure of pulm valve( also delayed
49)Captopril. Can cause ?? closure of pulmonary valve but it was not in
Ans:hyperkalemia options both r rite)
50)Gluteus maximus ?? Ans:A
A. Covers pyriformis 58)40 yrs ka obese male with sedentary
B. Arises from gluteal tubercle lifestyle. Bp 140/100. Pulse 80. Cause???
33

A. Increased diastolic pressure dilute urine. Lesion will be at????


B. Inc SV A. Thalamus
C. Inc CO B. Hypothalamus
D. Inc vascular tone C. Caudate
Ans:D D. Basal ganglia
59). Child with itching. Worms. Raised Ans:B
cells-??? 67)Boy with fracture arm after bout of
Ans:eosinophilis fight with sister ( kuch is tarah tha)
60)3 yrs old child with repeated bacterial withBlue sclera ??
infections since infancy. At 32 yrs he A. Osteogenesis imperfecta
developed SLE. Wbc 7850. IgG ki koi B. Marfans
values theen. Lymph node biopsy ki ajeeb Ans:aaaaaaaaaaaaaaa
si findings thi i dont remember those alien 68)Basic electrical rhythm of brain is
terms. Cause???? located in????
A. Something NADPH A. Reticular formation
B. Dec of adenine deaminase may be. Dont B. Thalamus
remember C. Cerebellum
C. Decreased cd4 lymphocytes Ans: A
D. Failure of b cells to become plasma cells 69)Deep cervical artery arises from which
Ans:D ( past paper mcq ) Ans:Costocervical trunk
61)Ek classical scenario abt cerebellar 70)Right brachiocephalic is a branch
lesion.??? of???
A Gait disturbance. Ans Right aortic arch
B .Past pointing. 70)toxoplasmosis pregnant female. She
C.Rapid movements delivered a baby. To check acute infection
Ans:A in neonate, which Ig to be done On elisa
62)Degradation of deoxynucleic acid ???
releases? A. IgG
A. Phosphoric acid B. IgG4
B. Ribose C. C. IgM
Uracil Ans:C
Ans:A 71)Minutes after bee sting a woman
63). Damage to tibial nerve. But some develops wheeze, sob n ye wo. It is due to
flexion still present at knee joint. due to which???
which muscle ??? A. Histamine
A. Long head of biceps femoris B. Bradykinin
B. Short head of biceps femoris Ans:A
Ans: B 72) Typical scenario post streptococcal. 12
64)Macule of utricle ka function. ?? yrs old girl,pehle sore throat. Uske baad
Ans:Acceleration Linear / horizontal ans. post streptococcal walay sign n symptoms.
65)Breast atrophy in old age due to This is due to???
A. Dec estrogen,progest A. Antibody to immune complexes
B. Dec estrogen B. Antigen-antibody complexes deposit in
C. Dec progesterone tissues
Ans: B Ans:B
66) The scenario was 40 yrs old female. 73). Neutropenic patient. Admitted to
Pregnancy. Inc vol of urine (most oncology ward. C/o ear pain n fever.
probably). Inc thirst. Relatives also Causative organism pseudomonas. Which
compaint of her Behavioural changes (jo of the following shows why these
k unka kaam hy))...Fasting glucose was symptoms have developed???
108 (normal). Passing large amounts of A IL1
34

B TNF 84)Goodpasteurs. ??
Asn :A Ans:IgG k depositions in basement
74) Aeobaric vertigo in divers is due to?? membranes
A.BAro trauma 85) Normal fev1/fvc ki value ??
B. Perforated tympanic membrane Ans:80%
C.Some ear plugging thing 86)Actinomyces?
D. Unequal pressures in middle ear during Ans:Talcum granules
ascent 87)Female with vaginal discharge.
Ans D Metaplasia of cervix due to ??
75)Haan ek wo cricoid cartilage wala ?? A.HPV
A. Incomplete ring B HSV
B. At c4 Ans: Aaaaaaa
C. Vocal cords attached to it 88.Irreversible step of glucose
D. Inferior pharyngeal constrictor attached metabolism???
to it Ans:G6p to f6p
Ans:D 89) Benzodiazepenes …… ??
76 trachea ki bifurcation Ans:Lead to dependence
A. At t2 90) which drug shud be avoided in
B. Trachea in is midline but bifurcates a angina???
little to the right A. Beta blockers
ans: B B.CCBs
80) right bronchiole ka size pucha tha?? C. Theophylline
ANs: 2.5 cm Ans:C
77) Goblet cells change to clara cells in?? 91)Increased t3 and t4 in pregnancy due
Ans: terminal bronchiole to ??
78) Giving large amounts of normal saline A. Inc TBG
will cause?? B. Inc TSH C.
A. Dec urinary Na excretion Inc TRH
B. Inc plasma vol Ans:A
Ans: Bbbbbbbbbb 92)Regarding CO2 ???
A. High solubility in plasma
79)ADH suppressed by.?? B. High diffusion coefficient
Ans:Alcohol (imp Bcq) Ans:B
80)Morphine ?? 93) Muscle membrane is relatively
A Increased GIT secretions permeable to???
B.Inc gut peristalsis A. Water
C.Constipation B. Electrolytes
Ans: cccccccccccccc (All opiods causes C. Amino acids
constipation) Ans: C
81)Endotoxin?? 94)Ek tabes dorsalis ka typical scenario
A. Heat labile tha. Dribbling urine. Unable to see in
B. Released when pathogen dies dark. Something with light reflex and
Ans:bbbbbbbbb pupils Cause??
82)In pus. Ans: syphilis
Ans: Dead Neutrophils 95)Hypercalcemia effects the formation
83)In acute renal transplant rejection. On of?
microscopy which of the following will A. 1,25 dihydroxycholecalciferol
show something worse? B. 24,25 dihyroxycholecalciferol
A. Interstitial lymphocytes C. 25 cholecalciferol
B. Vasculitis and thrombosis Ans:B
Ans B
35

96)Tumor metastasis ?? 107)Ectopic Thyroid hormone produced


Ans:loss of e cathedrins . by ??
97 which of them causes adhesion Ans:Sq cell CA larynx (ref MRCP notes)
AnsA. Integrins 108)Pre malignant lesion
98)Dietary fibers contain what?? Ans. Leukoplakia
A. Starch
B. Glycoprotein 109)Medial lamniscus men decussation ??
Ans:AA
99.Organelle which has cisterns n concave Ans:Post limb of internal capsules
surface faces nucleus????
A. Golgi 110)Patient on ATT pins and niddle
B. Smooth endoplasmic reticulum sensation??
C. Mitochondria Ans:isoniazid
Ans:A 124)ADH n oxytocin??
100)Regarding mitochondrial mutations Ans: originated from hypothalamus
??? 111)Decreased intestinal motility ?
A. Can cause fragile x syndrome A. Secretin
B. Only transmitted from mother B. CCK
C. From paternal side C. Gastrin
D. Prader wili syndrome D. Somatostasin
Ans:B Ans:D
101)Neuron ……..?? 112)Vagus nerve leaves skull through ??
Ans: Axonal hillocks where axons leave A. Foramen spinosum
from cell body B. Jugular foramen
102)Spermatogenesis ?? Ans:BBBB
Ans:Spermatids into spermatozoa 129)Severe transfusion reaction ???
103). Most imp and acute effect on Ans: A+ to O+
testosterone production is of?? 113)Secretin relased in response to???
A. LH A. Hcl in stomach
B. FSH B. Hcl in duodenum
C. Prolactin C. Fat in duodenum
Asn:A Ans:BBBBB
104)About ovaries ?? 114)Effect of hyperactivity of sympathetic
A. Lymphatic drainage into para aortic system ???
lymph nodes A. Inc gut motility
B. Koi 6 something million ova at the time B. Inc gut secretions
of birth C. Bronchodilation
C. Supplied by ovarian artery D. Construction of pupils
Asn:A Ans:CCCCCCCCCCC
105)Atrial systole 115)Hormone increased in stress???
A. During “a” wave Ans: Cortisol
B. PR segment 116)Middle thyroid vein drains
C. dicrotic notch into……….. ???
Ans:A Ans:internal jugular vein
106)About thymus ?? 133)Pellegra treatment ??
A. Regresses shortly after birth Ans: niacin vit b3
B. Contains lymphocytes 117)Superexcitability (hyperexcitability)
C. Derived from thyroid of heart is ????
Ans:bbbbbbbbbbbb A. Longer than relative refractory period
B. Ventricles are hyperexcitable
C. Ventricle are less excitable
36

Stem not cleared … aaaa may be right Ans: A


118.)Within 12 hrs of MI best 128)Which drug/antipsychotic enhances
marker?????. the efficiency/activity (dont remember
A. ALT exact word ) of others A. Valproate
B. ALP B. Phenytoin
C. AST C. Phenobarbital
D. LDH Ans: Controversial
E. CK 129) Warfarin effect increased.?
Asn:E Ans: Cimetidine
119:Creatinine released from……………..
Asn:skeletal muscles 130)Vasodilation with hypotension ??
120:Insulin inhibited by ??? Ans:Septic shock
Ans:Beta receptor antagonist 131:Drug tolerance due to ???
Ans:decrease effect of drug upon repeated
121)Best about kidneys and regarding administration
structures?? 132:Metabolic alkalosis is feauture of??
A. Perinephric fat is outside renal fascia a.pancreatic fistula
B. Adrenals not covered in fascia b.hyperaldosteronism
C. Left renal vein is anterior to aorta and Ans:B
renal artery 132:Anterior cerebral artery supply??
Ans:CCCCCCCCCCC A.brocas
122)Pain referred from diaphragm to b.wernickes
shoulder c.paracentral lobule
ANs: C3 4 5 Ans:C
138.Pt Aptt raised by antibiotic use 133:Blood supply of thalamus ??
treatment a.anterior crebral artery
a..FFP b.middle cerebral artery
b.Vitamin k c.posterior communicating n
Ans:B Ans: cc
123:Medullary thyroid cancer marker 134:Patient on antihypertensive drug ..ER
Ans:Calcitonin collapse.. Potassium 6 cause is 1 cause??
124)Dead space is decreased in??? A.Diuretic intake
A Emphysema B.Acute renal failure
B. Tracheostomy Ans:B
C. Pleural effusion 135: years old boy..generalized
Ans:BBBBBBB edema..first line of investigation????
125)Red infarcts occur in?? a.albumin globulin ratio
A. Where venous blockage alongwith b.serum creatinine
arterial c.urinary protein
B. Only arterial blockage Ans:C (exclude nephrotic syndrome)
C. Supplied by more than one arteries 9-NOV-2016 MED & ALLIED
Ans:C
126)Gangrene in diabetes ?? (MORNING SHIFT)
A. Dry 1.Taenia solium pork worm doc tx?
B. Wet a Niclosamide
Ans: A (if asked complication then b) b praziquental
127) Parasitic infection spread by c albendazole
mosquito bite?? d mebendazole
A. Filariasis Ans B (ref Katzung)
B. Onchocerca
C. Tapeworm
37

2. Hyaline cartilage b.hydatid disease


A present in nasal septum Ans is B
B pinna 15. Dopamine n ach imbalance in corpus
C TMJ articulation striatm leads???
D epiglottis a.Chorea
e. menisci of knee b.Hemibellismus
Ans A c,Parkimson
3. Propranol contraindication d.athetosis
Ans. Asthma ans is c
4. MAP 16.Minimun amount of caloric required
Ans .Diastolic plus 1\3rd of Pulse pressure for man??
5. Blood suply of head of humerus Ans 25_30
A.Arcuate 17. Heparin acts by binding to??
B.anterior circumflex c post circumflex a.anti thrombin3 factor 10
ANS B>A b.Factor xa only
6. Aluminium Hydrooxide c.. V|||
Ans.Dec Gastric Emptying ans a
7. A structure that is formed by 18.Tumour in brain compressing
aponeurosis of External oblique Muscle oculomotor nerve that area venous
Extending from Anterior superior iliac drainage into
spine upto pubic tubercle is called a.straight sinus
Ans. Inguinal Ligament b superior sagital sinus
8. Common in CRF +Conns + c inferior sagital sinus
pheochromocytoma d cavernous sinus
Ans: HTN Ans is D
9. Patient having Plt count 1 lac 8000 with 19.Internal jugular vein when it passes
increased bleeding time??? from jugular foramen its imediate
Ans ws Platelet Functional Defect ( relation with
Qualitative Disorder ) A. Sternocledomastoid
10. .A drug that in given in B.Trocher nerve
imunocompromised patients causes C.Acessory nerve
peripheral Neuropathy and Pancreatitis ans C
Ans.Diadinosine 20.Dec/shortest ventricular muscle fiber
11. Epithelium of small intestine length in which one of the following ??
a simple columner a.Inc venous return
b simple coloumner non ciliated b.contracetion of priphral muscles
ans is b c.Dec intra thoracic pressure
12. Utricle detects?? d.Inc intra pericardia pressure
A.horizontal acceleration Ans D
B.velocity of horizontal acceleration 21. Submandibular gland location?
C linear accelaration Ans mendible above n ant post belly of
Ans is C digastric muscle
13. Regarding Active transport 22. A 30 year old woman in azad kashmir
A.require carrier proteins that are enzymatic was injured mildly in an earthquakethat
in nature killed
B.uphill transport many people apart from her distant
Ans is B uncle. She would most probably be in
14. Imunologic test diagnostic for which A. Social crisis
....???? B. Personal crisis
a.amebic liver abcess ... C. Situational crisis
38

D. Developmental crisis A.Bronchogenic carcinoid CA


ans is c B.Pnemonia
23. 4x4 table? C.Tb
a.student t test, D. Sle.
b.chi square tes ans is b
Ans B 35. S1 loud
24.Shivering???? Ans. Lower limit of pr interval
Ans Post hypothalamus 36.”V” wave on cardiac cycle?
25. Patients whose small gut is resected A .diastole
due to any disease, they present B.isovolumic contraction
e "hyperoxaluria". The cause of this cond C.isovolumic relaxtion
is...? ans is c
A. Dec blood calcium to form calcium 37. Old patient Excretion of drug is
oxalate decreased bcz of ?
B. There is increaed abs of oxalate from gut Ans. dec renal threshhold of drugs excretion
C. There is increased phosphate in blood 38. Cardiac muscle cant be teranized
Ans C Ans long refrectry period
26.CNS myelination by which cells... 39..Stress hormone
Ans . oligodendrocyte Ans acth
27.Prostatic CA metastize to skull (no cortisol in option)
through. 40.Thyroxine causes
Ans.Vertebral plexus Ans. Fetal brain development
28.Internal jugular vein just coming out 41.Insulin
of skull whats anterior to it. Ans. anabolic for cho, lipid nd protein
a.ICA 42.Councilmen bodies
b. phrenic nerve Ans apoptosis
c.. sternocleidomastoid 43. Microscopic feature of compact bone
d accessory nerve A haversian canal arranged obliquely
Ans is c B lacunae have osteoblast
29.Cause of edema in renal dysfunction. C haversian canal are not interconnected
A. Salt retention collagen
B. Hypoalbunemia nd salt retention D lamellae are regularly arranged
ans is b Ans is D
30.Which is high amount in tear ? 44.Mask like faces
a.Na Ans. Parkinson disease
b.K 45. Scalenius ant grooves over …..
ans a ans 1st rib
31.Tall T wave 46. Patient serum glucose 100 mg/dl but
Ans Hyperkalemia inspite of that glucose apears in urine
32. long term bp control by Ans.low renal threshold
Ans.renin angiotensin system 47,. C3a and C5a both
33.Pt had cut on dorsal surface of wrist A..opsonins
and now failure to abduct n adduct B..anaphylotaxins
fingers with intact sensations?? C..chemotactic factors
Ans deep branch of ulnar ans is B
34. 5 yr old girl wid Sob no cough..for 48. Tb patient pleural effusion what to
three months..presente with temp 38.7 F do.drain from??
..R/R 25.sputum clear negativ culture A. 4 to 6th intercostal upper border mid
with ordinary media..Ana neg..On Cxr axial line
hilar Lymphadenopathy and right lowr B. 9th intercostal lower border anter axilary
lobe infiltrates. line
39

C.9th upr border mid clavicular B.24-72 Hr


D.9th uwpr border ant axillary ans is A
Ans aa ref oxford med 64.Cerebral blood flow regulated by
a.pco2
49.Denticulate ligament………??? b.ph
Ans.Lateral extension of pia matter btw Ans Is A
ventral n dorsal roots 65. Hypoglosal nerve damage……… ??
50.Subacute endocarditis Ans. Ttongue deviate same side
organism……………?? 66. Adrenal gland..supply…… ??
Ans..Strep viridans Ans.Thoraxic sympathetic plexus
51.Transitional cell ca risk factor of 67.Tyre workshop worker and smoker,
bladder ………..??? has squamous cell carcinoma cause???
a.smoking a.aromatic amine
b. schistosomiasis b.smoking
Both r right go wid bbb Ans is B ref Goljan pathology
53.Diabtes inspidius treatment. ……??? 68.One regarding atlas vertrbra
Ans.Desmopressin a.Have no spinous process
54. Hep B and C caused by ……….. ?? b.Have no ant arch
Ans.Contaminated needles c.Hve no lateral mass
55. Platelets formed from ………..?? Ans A
Ans.Megakaryocytes 69.Regarding arithematic average. .
56.Erythropoietin secretion is inhibited Ans mean
by 70. Pt have Headache Vomiting Stiff
a. cobalt Neck On Lp Csf contain Blood
b. hypoxia A.Epidural
c. theophylline B.SubDural
ans is C C.Subarachniod
57. Erythropoeuin secretion increased by ANs is C
Ans hypoxia 71. Hemophilia
58.True hermaphrodite Ans. X linked reccesive
A.xx/xy 72. Inc ejection fraction what dec….. ??
B.xxy Ans.esv
C.xx 73. Stomach circular muscle become more
ans B thicker
59. On complete removal of colon wat a fundus
happens death b antrum
A..malnutrition C cardiac
B.electrolyte imbalance d Pylorus
Ans is B Ans D
60. Patient having zollision ellisin 74. Down Syndrome…. ?
syndrome for that gastrectomy done , ans.Trisomy 21
which will occur? 75. Man had Fasting Lipid Profile After
Ans B12 deficiency eating junk food, after centrifugation
61.Btructure lying with LAD having risk plasma becomes milky white??
of damage during surgery……… ?? A.Chylomicron
Ans.Great cardiac vein. B.VLDL
62. Sodium nitropruside mechanism of C.HDL
action D.LDL
Ans.dilation of vessels E.IDL
63. Life span of sperm at body temp Ans A
A.24-48 hr
40

76. Preganglionic sympathetic A. plasmodium Falciparum


secrete…??. B.plasmodium vivax
Ans.Ach c.plasmodium malaria
77.Golgi tendon decreases ….?? d.pasmodium ovale
Ans.Tension Ans is A
78. Ankylsing spondylitis ……..? 88. K level 3.0 n ph 7.54 due to whch
Ans.HLA B27 hormone effct
79. About cholera ?? A.cortisol
A..Treated by parentral antibiotics B.aldo
B..treated by fluids only C.testo
C..ransmitted by water n food D.progstrn
d.Cl- ans CAMP has role Ans is b (aldo causes decrease in
Ans is D potassium dat leads to alkalosis)
80.SA node located ………… 89. Factry worker..neuropathy …… ??
Ans.Epicardium sulcus terminalis Ans lead
81 Strontium 90 is associated with which 90. HIV drug causing pancreatitis???
ca……. Ans …..Didanosine
Ans.ousteosarcoma 91. 29 yr old went remot area develop
82. Chronic rhinitis mild watry diarrhoea for 1 wk.no accult
a.mast cells blood.r resolved after 2 wks couses??
b. eosinophill A.Amoeba
c.lymphocytes B.Vibrio
ans A C.Rota
82. Histamine type 2 receptor antagonist.. D.Shigella
…… E.Giardia
ansCimetidine Ans A
83. Macrophage...... 92. Hyaline cartilage..
ans.scavenger cells Ans endochondrial ossification
84. Homonomous hemianopis 93. Viral neoplasm…...
A.gwniculate body Ans.T cell leukemia
B.Optc tract 94. Pain , temperature, from face
c.Optc chaisma a anterlateral spinal
Ans b b trigeminothalamic
85. A child wid reccurnt respiraty tract c dorsal culumn medial leminscs
infctn n etc etc also having kartnegr d trigeminal leminuscus
syndrome?? ans is A
A.dynin arm 95. Dignostic creteria for sof tissu
B.actin tumour ??
C.mysoain a.Inc vascularity
D.cell microtube b.Local invasion
ans is A Ans B
86.A surgicaly removed ovary showed 96. ACE inhibitor produce bradykinin
aggregate of epitheloid with lympho these by??
aggregate called a Dec angiotensin2
A.Graffian follicle b increase/activate kallikrein
b.Granulation tissue Ans b
c.Granuloma pyogenicum 97. Mast cell
D. Granulomata formation a produce heparin,histamin,protease
E.Granulosa cell tumor b function like eosinophils
Ans is d C Have IgA surface receptor
87. Malignant Quarten Malaria Ans A
41

98. Local anesthetic…. A.Contact dermatitis


Ans.A delta fibre B.Focal Sle
99. In carcinoma which filaments are C.Discoid sle
most likely present d. Dermatomyositis..
a. keratin filaments Ans D (controversial bcq)
b.vimentin filaments 110. Brush border in which part of
c.desmin filaments nephron
Ans A A.Pct
100 Gastric emptying is observed thru B.Thck loop
A.Radioisotope scan C.Thin loop
B.Repeated xray after barium enema D.Dct
Ans b Ans is A
101. Child had generalized edema due to 111. Nalbuphine
A..Dec hydrostatic pressure A.Effect comparable to morphine
B>Dec plasma colloid oncotic pressure B.1/7 of morphine
C.Lymphatic obstruction Ans A
Ans B 112. Pulmonary emboli of left lung.what
102. Parietal cell secrete finding????
A.Hcl+ intrinsic factor A. Ventilation perfusion ratio infinity
B.Hco3+ intrinsic factor B. Vent pro ratio dec
ans A C. Alveolar o2 equal to atmospheric o2
103. Iron deficiency D. Alveolar o2 equal venous o2
A.Persistent blood loss ans A
B.Defected globin chain 113. Dec/shortest ventricular muscle fiber
CTreated by iv iron length in which one of the following
DMegaloblast A.Inc venous return
Ans IS A B.Contraction of priphral muscles
104. 70 kg male 30 yr old hypertensive. C.Dec intra thoracic pressure
Na 140.k 2.5.urae normal .s creatinine D.Inc intra pericardia pressure
normal. Total cholesterol 200. ans is A
A.Essential hypertension 114. Vastus medialis?
B.Primary hyperaldosteronism A. Supplied by femoral branch
Ans is B (Conns syndrome) B. Attacdfhed to intertrichantetic line
105. Pt had accidental damage to neck of C.attached to petala
fibula and now presnt with foot drop? D.attached to medial tubrcl with vasts
Ans common peroneal nerve intermedius and adductr magns
106. Patient having blood group A Ans is A
Ans:.B Antibody 115. A young girl during exercise
107. Defect of thalasemia? develops dyspnea,she has developed the
Ans defect in chain left sided heart failure n havnig
108. Sjgorens sydrome patient..ANA pulmonary edema wat is the cause of
positive..most serious long term developing dyspnea during exercise
complication? a.ryt heart failure
A.Glomerulonephritis b.left ventricles pump failure
B.Pulmonary interstitial fibrosis Ans b
Ans B 116. What could be damaged in surgery
109. .A lady who got rash on face after of superficial submandibular gland
going in sun.the biopsy of lesion show igG A.Hypogastric nerve
with surrounding area clear. No other B.Mandibular nerve
abnormality. Ana negative.what is the CTrigeminal nerve
cause?
42

D.Marginal Brach of facial given diagnosis


Ans is D A. Septic Arthritis
117. Cytokeratin pigment is used to B. Acute Gout
diagnose?? C. chronic gout
A.Diff sarcoma and carcinoma D. pseudogout
B.Diff be metaplasia and dysplasia Ans B (Hint is inc CRP which is always in
ans is B acute condition)
118. Fat necrosis due to ….. 126..Peroxisome contain ………….???
ans .breast trauma Ans.oxidases
119.Coagulatie necrosis ..... 127 Regarding Beta thalassemia
ans due to ischemia… A.mutation in promoter region of beta gene
120. In brain type of necrosis ………… B.Point mutation in beta gene
ans.liquefactive necrosis C.Large deletion in beta gene
121.Hairy leukoplakia D. Small deletion in beta gene
A.fungal growth Ans A reference Robins
B. smoking 128. . Patient of renal transplant on
c associated with hiv immunosuppressive drug.cns symptoms
D leukemia culture negative.lympho increased sugar
Ans is c normal.oval cell with halo found
122.The most common cause of infarction A.Cryptococus
in vicera???? B.Tb
A.Air embolism C.Influenza
B.amebic ulcer Ans A
C.Anemia 129. A pregnant diabetic lady gave a
D.Atheroma birth to a child apgar score 4 and 6 in 1
E.Thrombosis and 5 min. Child with respiratory
Ans is D distress. What will you give patient before
123. . What is time period of absolute delivery to prevent the condition?
refractory resting period of heart A.Hydricostisone
A.100milli sec b.Testosterone
B. 150 msec CProstaglandin
C. 250 msec D.Estrogen
D.10-20 msec Ans A
ans c 130. Survey of pts having melanoma,
124. Scenario of pt with completely blood sample taken which of the following
blocked blood flow to left lung, which of cells would be present initially(First)
following is true A. NK Cells
A. v/Q ratio in left lung will be zero B. Machrophages
B. systemic arterial Po2 will be elevated C. Lyfmpho
C. V/Q ratio in left lung will be lower than D. Eosinophills
Rt lung Ans A
D. Alveolar Po2 in left lung will be approx. 131. Dec unloading of o2 in peripharl
equal to Po2 in inspired air tissue duet
E. Alveolar Po2 in rt lung will be equal to A.pH
Po2 in venous blood BBmr
Ans is d CTemp
125. An old man playing with his D2,3bpg
grandson in garden got trauma, pt fibrile, Ans A
redness in the toe, C- reative protein 132. Complete trasaction of brain stem
positive, culture negative, negative above pons would
biferengence, some values of uric acid A. Result in cessation of all breathing
43

movements B.MCA
B. Prevent any voluntary holding of breath Ans B
C. Prevent control of central chemoreceptors 140. .local factor causing delayed wound
from exerting any control over ventilation healing
D. Prevent peripheral chemoreceptors from a infection
exerting any control over ventilation b vitmin c
E. Abolish hering breuer reflex Ans A
Ans B 141. lymphatic nodule not present in
133. Which of following is not related to Ans. Thymus
pain transmission??? 142.Pyogenic bacterial
A. Thalamus peritonitis………..???
B. Substantia gelatinosa ANS .Bacteroid
Ans A 143.lipoma on thigh ,local anesthetic
134. Hypothalamic hypophysial portal xylocaine is given which will be 1
syestem connects hypothalamus to?? affected??
A. Adenohypophysis a.B fibers
B. Neurohypophysis B.c fibers
Ans A c.A delta fibers
135. Scenario of pt with congestive Ans C (for details of general anesthesia
cardiac failure with pulmonary edema, see section in Gynae obs theory)
now in acute distress appropriate 144. Presence of st squamous epithelium
treatement at this stage would be?? in endocervix is called?????
A. High flow O A..neoplasia
B. Fuoseamide B..metaplasia
C. Nitroglycerin C..dysplasia
D. Dopamine D..anaplasia
E. Adrenaline Ans C
Ans B 145.Patient having hemoptysis and
136. Movement of body in response to developed heamaturia cause….. ???
visual stimulus...structure involved? Ans .Goodpasture syndrome
A.Sup colliculus 146.Skeletal muscle protein
BInf colliculus synthesis………… ??
CSup olivary Ans CPK
D.Inf olivary 153.Pus contain…
Ans A a. Dead Neutrophil
137. Chlorpromazine exhibits anti b.active bacteria
pyschotic effects by??? Ans A
A..alpha agonist action 147.Vit c causes….. ???
B..act on dopamine receptors ans.hydroxiltion of proline
ans B 148. Supra renal gland relation ..
138. Which doesnt cause esoniphilia a.rt suprarenal is one centrla tendon
A...igM b.Left one is realted anteriorly to pancreas
B.igE c.Left is on central tendon.(poor recall)
C.parastic infestations Ans is Bb
D.asthma 149. Protrusion of mandible ………….?
Ans A ans .lateral ptyrgoid
139. One qst k person while fighting gets 150. Which one dec blood supply to brain
a blow on temporal side of the A.Inta ventricular epinephrine
head,internal bleeding occurs due to B.Inhalation off 17% co2
damage to which vessels?? C.Siezures
A.middle meningeal atery
44

D.Hyeperbaric O2 164. Pt undrgone tonsillectomy and


Ans is B presented with signs of shock its due
151. Which harmone is NORMALLY to(cant recall exact but it was also easy
secreted in pregnanc one)
A.Pth Ans Hypovolemia..
BGH 165. Acetylcholine is formed by
CThyroxin A acetyl co A nd choline
D.Cortiosl B preganlianic nerve end
asn is D Ans A
152 Enterococcus fecls tx 166. Cricoid level?
A.Ampicilin A. At division of common carotid
B.Metronidazole B. Cervical 6 vertebrae
C.Co trimezol C.Junction of pharax and oesophagus
D.Aztreonem D. Isthumus of thyroid
E.Imepenem Ans B
Ans A 167. Patient after RTA losses 2 liters of
153. In cardiac cycle atrial fibrillation blood, which of the following will
occurs occur???
A at start of QRS A. Increase venous tone
B at end of QRS B. slight stimulation of sympathetic syestem
C at peak of QRS c. Increase heart rate
D.at the lower end of PR interval d. Negative feedback compensation
Ans D All are right Best is AAAA
154. Hypoxic pulmonary vasoconstriction 168. phrenic nerve damage resulting in:
is accentuate by??/?? (RABIA ALI BCQ)
A. Increase Po2 A.loss of sensations from central part of
B. Decrease Po2 diaphragm..
C. Increase pco2 B.loss of respiratory reflex
D. Decreased pco2 Ans A
Both B and C r right choice is yours 169 linear deposit……….. ??
155. Ascaries causes………….?? Ans.good pasture syndrome
Ans. respiratory system(causes pneumonia) Medicine 9th November, 2016
156. Transplant best donor….???
Ans.Identicle twin Evening Shift (194 MCQs)
157. Steroid mechanism of action……….. 1. HLA-B27 associated with
?? A. Type 1 DM
Ansexpression on genes B. Juvenile rheumatoid arthritis
158. Gfr clinically measured by,…… ? C. Addison disease
Anscreatinine clearance D. Graves disease
159. Cholestrol synthesis …………. ?? E. Multiple sclerosis
Ans:Liver ANS. B
160. Tractus solitarius …… ?? 2. Longest acting insulin
Ans. Taste sesation A. Human lente
161. Secondary ossification B. Procine NPH
center………?? C. Procine lente
AnsEpiphysis D. Bovine ultralente
162. Recurent larngeal nerve hooks E. Bovine semilente
around………. ??? ANS. D
AnsLigamentum arteriosum 3. Information care session is for:
163. Benign tumour ………..?? A. Take informed consent for the procedure
Ans WarthinTumor the patient will undergo
45

B. Clear the myths and misconceptions of D. Con's disease


the patient and relatives about the disease E. Cushing disease
C. Tell the patient and family about new ANS. D
advances in treatment of the disease 9. A 50 years patient who used to drinks
D. Take informations from the family and from 20 years. His labs are Bilirubin 5
friends of the patient about the disease times raised than normal, ALT 2 times
E. Tell the patients about the facilities and ALP 3 times. His histopathology of
available to in the hospital for the treatment liver will show?
of the disease A. Piecemeal necrosis
ANS. B B. Bridging necrosis
4. A boy while playing was hit to right C. Councilman bodies
elbow. He developed swelling and bruise. D. Periportal Lymphocytosis
What is the cause of swelling? ANS. A
A. Carbohydrates 10. Antipsychotic effect of phenothiazine
B. Proteins due to:
C. Fats A.Block muscarinic receptors in corpus
D. LDL striatum
E. Water and electrolytes B. Block dopamine receptors in corpus
ANS. B striatum
5. For a substance X GFR 125mg/ml, Tm C. Block dopamine receptors in limbic
125mg/mint, plasma concentration is system
200mg/100ml, What will be the filtration, D. Block adrenrgic receptors
reabsorption and excretion of substance E. Block Serotonin receptors in limbic
X? system
A. 250 filtration, 125 reabsorption and 125 ANS.C
excretions 11. Rexed lamina 3, 4 present in
B. 125 filtration, 125 reabsorption and 0 A. Lateral Gray horn
excreted B. Posterior Gray horn
C. 200 filtration, 125 reabsorption and 75 C. Anterior Gray horn
excreted D. Gray commissure of spinal cord
ANS. A ANS. B
6. Patient with backpacks, diarrhea, 12. A patient had non-productive cough
fatigue. Stool culture shows binucleate for 2 months. Now presented with blood
organism with 5 flagella. Which drug streaked sputum. Left Lung has 5 cm
you'll give? hilar mass. Sputum shows hyper
A. Stibogluconate chromatic, pleomorphic cells with scanty
B. Metronidazole cytoplasm. Cause of this condition?
C. Sulphadoxine A. Silicosis
D. Quanidine B. Asbestosis
ANS. B C. Smoking
7. Fenestration in choriocapillaries D. Passive smoking
numerous and large in E. Radon exposure
A. Equatorial plate ANS. C
B. Around optic disc 13. A person visited Thailand, now
C. Ora serata presented with fever, chills, body aches
D. Sub macular kuch tha and postural hypotension. What is the
ANS. C most probable diagnosis?
8. Latent tetany seen in A. Syphilis
A. Hyperparathyroidism B. Malaria
B. Graves disease C. Dengue
C. Addison disease D. Chagas disease
46

ANS. B A. Carry impulses toward the cell bodies


14. A tourist 2 weeks after returning from B. Carry impulses away from the cell bodies
another country presented with fever, C. Carry pain impulses
generalized lymphadenopathy, and rash. D. Are several in numbers for multipolar
Atypical Lymphocytes on peripheral neurons?
smear. Most probable diagnosis? E. Are found primarily in the gray matter
A. HIV (seroconversion) ANS. D
B. Falciparum Malaria 21. Basement membrane of lung alveoli:
C. Chagas disease A. Comprises of type 2 collagen
D. Lyme disease B. Contributes in the formation of surfactant
ANS. A C. Has chondrotin sulphate as a constituent
15. Fibres arises from main sensory component
nucleus of spinal cord D. It is a mucopolysaccharide
A. Fasciculus gracilis E. Separates epithelium from endothelium in
B. Lateral spinothalamic tract alveolar wall
C. Posterior spinothalamic tract ANS. E
D. Anterior spinothalamic tract 22. Cardiovascular change in geriatric
ANS. A A. Systolic hypotension
16. Regarding HPV: B. Diastolic hypotension
A. Serotype 16,18, 31 associated with C. Decrease elasticity of myocardial
dysplasia and cervical carcinoma ANS. C
B. Serotype 11 associated with ca cervix 23. Blood vessel valve present in
C. More than 100 genotypes A. Large size veins
D. Vaccination effective only after infection B. Medium size veins
E. Vaccination at 20-23 years age is C. Muscular arteries
recommended D. Fenestrated capillaries
ANS. A ABS. B
17. Nifedipine causes refectory 24. Highest oxygen tension in:
hypotension when given with halothane A. Pulmonary capillaries
due to B. Arterial blood
A. Causes tachycardia C. Venous blood
B. Causes vasoconstriction D. Left ventricle
C. Has long action when given orally E. Right Atrium
D. Has synergetic effect on calcium ANS. A
channels 25. Circumduction is:
ANS. C A. Abduction/adduction & flexion/extension
18. Normal ECG do not record electrical B. Medial rotation/lateral rotation &
activity of flexion/extension
A. AV node C. Flexion and extension
B. SA node D. One axis movement
C. Ventricles ANS. A
D. Left atrium 26. Plateau occurs due to
E. Right atrium A. Na influx
ANS. B B. Na and calcium influx
19. Counter part of mesonephric duct C. Potassium channels opens
A. Appendix D. Chloride channels
B. Ejaculatory duct ANS. B
C. Round ligament 27. Ischio-pubic ramus fuse at the age of
D. Seminal vesicle A. 7 years
ANS. B B. 1 years
20. Axons: C. 24 years
47

D. 21 years 34. A patient after CVA has loss of


E. 14 years appetite to food and drink and tendency
ANS. E to throw. Cause of his decrease thirst?
28. During isovolumetric contraction A. Vasopressin
what will happen: B. Stimulation of lateral hypothalamus
A. Decrease coronary blood supply ANS. A
B. First heart sound splits 35. Muscle tone of axial ans paraxial
C. Semilunar valve open antigravity muscles control by
ANS. A A. Rubrospinal tract
29. True about Venous return: B. Pontine reticulospnal
A. Will become zero when all circulatory C. Medullory reticulospinal
responses abolish ANS. A
B. Will increase when mean systolic 36. A 12-year-old boy with hemophilia A
pressure falls has had episodes of pain about his knees
C. Is decrease when right atrium pressure for the past 6 years. Over time, there has
decreases? been an increase in size of his knee joints,
ANS. A with deformity. Laboratory studies show
30. The axillary artery: decreased levels of coagulation factor
A. Lies posterior to pectoralis minor VIII activity. Which of the following
B. Lies lateral to the lateral cord of brachial materials is most likely to be seen within
plexus the joint space following episodes of pain?
C. Lies lateral to the short head of bicep A. Lipofuscin
D. Extends to the lowest border of pectoralis B. Russell bodies
major C. Neutrophils
ANS. A D. Cholesterol crystals
31. Obese person working as a Clerk with E. Anthracotic pigment
sedentary life style, was routinely ANS. D
checking his blood 37. Enterococcus faecium:
pressure and blood pressure was raised. A. Resistant to ampicilin
He was investigated but no cause was B. Resistant to carbapenams
found, his raised blood pressure is due to? C. Usually Sensitivity to aminoglycosides
A. Increase Vessels tone D. Catalase positive
B. Decrease diastolic blood pressure E. Frequent cause of pyogenic infections
C. Increase heart rate ANS. A
ANS. A 38. Vertigo with nausea and vomiting
32. Lesion of caudate, putamen and loss associated with?
of GABA neuron in substantia nigra and A. Dorsal vagus nucleus juxtaposted to
gollibus pallidus. Patient is suffering from vestibular nuclei
which disease B. Vertebral artery ischemia
A. Parkinsonism C. Vagus and glossopharyngeal nucleus
B. Hemiballismus ANS. A
C. Chorea 39. Secretry Granules of neurohypophysis
D. Athetosis are present at
E. Ballismus A. Nerve endings
ANS. C B. Pituocytes
33. Retic count decrease in C. Sinusoids
A. Chronic renal disease ANS. A
B. Hereditary spherocytosis 40. A case of Androgen insensitivity
C. Bleeding from gut syndrome was referred to your clinical.
D. Polycystic kidney disease What investigation you Wil NOT do in
ANS. A this case?
48

A. Serum 17 OH progesterone C. Hyponatremia


B. Testosterone level after stimulation with D. Hypernatremia
HCG ANS. A
C. Dihydrotestosterone after stimulation 47. An isolated complete rupture of
with HCG anterior cruciate ligament will result in
D. FSH and LH instability of tibia over femur. The
E. Ultrasound abdomen direction of instability will be
ANS.C A. Anterior
41. Polysaccharide antigen conjugated to B. Antero medial
increased immunity with vaccine C. Antero lateral
A. Meningococcal D. Postero lateral
B. Pertusis E. Postero medial
C. Chlamydia ANS. A
D. Vibrio 48. Which of following tumor in children
ANS. A is due to gene amplification?
42. A neutropenia, hypotensive patient A. Neuroblastoma
was admitted in oncology ward with B. Retinoblastoma
history of dysuria and high grade fever. C. Wilms tumor
His blood culture yielded growth of D. Nasopyryngeal carcinoma
pseudomonas aeruginosa. The disease as E. Burkitt lymphoma
a result of excessive release of the most ANS. A
likely. Bacterial product in the blood is? 49. From a population 2 groups were
(Asim Shoaib Medicine BCQ) selected on age basis and then
A. Retiulocyte macrophage stimulating subgrouped are assigned randomly for
factor study. Type of sampling is:
B. Granulocyte stimulating factor A. Random stratified sampling
C. Tumor necrosis factor(TNF) B. Simple random sampling
D. IL 1 ANS. A
ANS. C 50. Most effective in preventing DVT?
43. Proto oncogene converted to oncogene A. Intermittent Pneumatic compressions
A. Point mutation B. Low dose of LMWH
B. Translation C. Low dose of unfractioned heparin
C. DNA transcription ANS. A
ANS. A 51. Vitamin A deficiency most important
44. Viruses cause malignant lesion by A. Xerophthamia
alternation in: B. Increase predisposition to skin cancer
A. Oncogene C. Increase predisposition to lung cancer
B. Proto oncogene D. Infertility
C. DNA ANS. A
ANS. B 52. Ribosomes
45. Which fibres sensation will be lost A. Protein synthesis
first after administration of local B. In human composed of two equal
anesthetic agent? subunits of S50 and S40
A. A delta C. Attach to endoplasmic reticulum and
B. B fibers synthesize cytoplasmic protein
C. C fibers ANS. C
D. A beta 53. Characteristic feature of infarction:
ANS. A A. Frequently occurs due to pressure
46. Alkalosis can cause: B. In kidney is associated with red infarct
A. Hypokalemia C. In liver it produces suppuration
B. Hyperkalemia D. Is a usual consequence of TB
49

E. Usually seen with the arterial occlusion A. TSH


ANS. E B. Insulin
54. Basillar artery give rise to? C. Thyroxine
A. Posterior cerebral arteries D. GnRH
B. Anteriorly cerebral arteries E. Aldosterone
C. Posterior communicating arteries ANS.A
D. Anterior communicating arteries 62. In Normal physiological conditions
ANS.A Thyroxine causes?
55. Most abundant glial cells in gray A. Dec fatty acid oxidation
matter B. Dec gluconeogenisis
A. Fibrous astrocytes C. Inc fatty acid oxidation
B. Protoplasmic astrocytes D. Ca deposition to bone
C. Microglial E. Inc protein synthesis
D. Oligodendrocyte ANS.A
E. Ependymal 63. Most common pattern of inherited
ANS.B diseases?
56. Which test you'll do for diagnosis of A. X linked recessive
Typhoid fever during the second week? B. Autosomal dominant
A. Blood culture only C. Multifactorial
B. Blood culture plus Widal D. Autosomal recessive
C. Stool culture E. X linked dominant
D. Urine culture ANS.C
ANS.B 64. Child with generalized edema and
57. Test for definitive diagnosis of TB? proteinurea 6g per day. Which part of the
A. Demonstration of AFB kidney involved?
B. Caseous granuloma A. Interstitium
C. Epitheliod cells B. Basement membrane
ANS.A C. Collecting tubules
58. TB is important because it is: D. Proximal convulated tubules
A. Type 4 Hypersensitivity reaction E. Distal tubules
B. Type 3 HSR ANS.B
C. Type 1 HSR 65. True about Alpha motor neuron
D. Type 2 HSR A. Myelinated
ANS.A B. Unmyelinated
59. Which vessel is involved in portal C. Ends at motor end plate
hypertension? D. Terminate in intrafusal fibre of
A. Portal vein neuromuscular spindle
B. SMA ANS.A
C. IMA 66. If root of left lung is injured which
D. IVC structure will be spared?
ANS.A A. Vagus nerve
60. Symphysis pubis is which type of B. Azygous vein
joint? C. Phrenic nerve
A. Secondary cartilaginous joint D. Pulmonary ligament
B. Primary cartilaginous joint ANS.A
C. Ghamphosis 67. The structure which arches over the
D. Syndesmosis root of left lung.
E. Plane synovial joint A. Azygous vein
ANS.A B. Left recurrent l nerve
61. Which of the following acts through C. Phrenic nerve
cAMP? D. Arch of aorta
50

ANS.D blood transfusion and episodes of chest


68. Vagal stimulation of the heart causes: and abdominal pain..presented with non
A. Increased heart rate healing leg ulcer, What will her blood
B. Increase PR interval shows?
C .Increase force of heart contraction A. Sickle cells
D. Increased cardiac output B. Schistocytes
ANS.B C. Heinz bodies
69. Only nerve affected in Multiple ANS.A
Sclerosis is : 76. Jejunum is recognised by?
A. Optic A. Single or double arcade arteries
B. Trigeminal B. Narrow luman
C. Olfactory C. More Fat in the mesentery
D. Occulomotor D. Thin mucus with less blood supply
E. Abducent ANS.A
ANS.A 77. Which of the following shows an
70. Most important event in initial Left to Right shunt before
IRREVERSIBLE cell injury reversal?
A. Damage to cell membrane A. Eisenmenger complex
B. Loss of co enzyme B. Transposition of great vessels
C. Massive calcium influx C. Persistent Truncus arteriosis
D. Swelling of lysosomes D. Ebstein anomaly
E. Swelling of mitochondria E. TOF
ANS.C ANS.A
71. Child with recureent knee joint 78. During which phase of the cardiac
swelling and pain. Factor 8 was 90% and cycle 2/3 of ventricular filling occurs and
factor 9 was 8%. What is the disease child 3rd heart sound is produced?
suffering from? A. Rapid inflow
A. Christmas disease B. Atrial contraction
B. Hemophilia A C. Slow ejection
C. VWb disease D. Isovolumetric relaxation
ANS.A ANS.A
72. A person working in coal mine. His 79. Which heart sound is produced due
work is stone polishing and sand blasting. vibration of the ventricles during rapid
He might can develop which of the inflow?
following disease in the future? A. 3rd heart sound
A. Asbestosis B. 4th heart sound
B. Silicosis C. Ist heart sound
C. Anthracosis D. 2nd heart sound
ANS.B ANS.A
73. Transient adhesion is caused by? 80. Most abundant spingolipids in Gray
A. Integrin matter of the brain?
B. Selectin A. Spingomyelin
ANS.B B. Cerebroside
74. A person goes from standing to supine C. Globoside
position. What will happen? ANS.A
A. Increase in stroke volume 81. Parasympthetic flow to heart via????
B. Increase leg muscles pressure A. vagus nerve
C. Dec firing of baroreceptors B. Superficial cardiac plexus
ANS.A C. Deep cardiac plexus
75. An African girl who has history of D. T1-T5
anemia since birth and history of multiple E. T1-T4
51

ANS.A B. H2 blockers
82. Drug given in HOCM C. Sucralfate
A. Verapamil D. H1 blockers
B. Diltiazem ANS.A
C. Amlodipine 91. Which of the following disease with
D. Nifedipine raised ALT most commonly?
ANS.A A. Acute viral hepatitis
83. Axillary Fascia derived from? B. Hepatic malignancy
A. Prevertebral fascia C. Sclerosing cholangitis
B. Pretracheal fascia ANS.A
C. Superficial layer of Cervical fascia 92. A child having purpura in all over the
ANS.A body, otherwise normal. Cause of
84. Most potent pain mediator? pupura?
A. Prostaglandin A. Decrease platelate count by 20,000
B. Histamine B. Decrease platelets count by 90,000
C. Bradykinin C. Factor 8 deficiency
ANS.C D. VwB disease
85. Denticulate ligament: ANS.A
A. Lateral extension of pia matter 93. Ventral spinothalmic tract carry?
B. Extension of filum terminale A. Itch and tickle
ANS.A B. Pain
86. Superficial inguinal ring present in? C. Temperature
A. External oblique aponeurosis D. Vibration
B. External and internal oblique muscles E. Proprioception
C. Fascia transversalis ANS.A
D. Conjoined tendon 94. QRS complex produced prior to?
ANS.A A. Ventricular contraction
87. Paraneoplastic syndrome is caused by: B. Atrial contraction
A. Small cell carcinoma of lung C. Isovolumetric relaxation
B. Renal cell carcinoma ANS.A
Other options were irrelevant 95. Married man had no children for few
ANS.A years and was diagnosed as sterile with
88. Most important cause of death in azospermia. what is investigation of
Rheumatic fever? choice?
A. Myocarditis A. Serum FSH and serum LH
B. Pericarditis B. Serum prolactin
C. Mitral stenosis C. Serum testosterone
ANS.A D. Semen analysis
89. Thyroidectomy was done in left lower ANS.A
lobe. Artery was ligated which supply this 96. Which of the following drains into
area. Which nerve have more chances to jugulodiagastric nodes?
be damaged during the operation? A. Palatine tonsils
A. Recurrent laryngeal nerve B. Pharyngeal tonsils
B. External branch of superior laryngeal C. Submandibular gland
nerve D. Sublingual glands
C. Internal branch of superior laryngeal ANS.A
nerve 97. A person lying naked in a room at
ANS.A temperature of 25°C. His heat loss will be
90. Prolong use of which of the following through?
will cause increase in gastrin level A. Radiation plus conduction
A. Omeprazole/PPIs B. Convection
52

C. Conduction (CK-MB, Trop were not in the options)


D. Evaporation ANS.E
E. Sweating 103. Organism associated with bladder
ANS.A ref Ganong squamous cell carcinoma
98. Formation of Interstitial fluid A. Schistosoma hematobium
increases with decreased in? B. Schistosoma Mansoni
A. Capillary hydrostatic pressure C. Schistosoma Japanicom
B. Capillary permeability ANS.A
C. Interstitial fluid colloid osmotic pressure 104. Not a part of natural immunity
D. Lymphatic flow A. Plasminogen
E. Plasma colloid osmotic pressure B. Interferon
ANS.E ANS.A
99. A patient has nodule on vocal cord. 105. Smoking affect organogenesis during
The least likely mechanism? which period of gestation?
A. Hyperplasia A. 3-8 weeks
B. Atrophy B. 4-10 weeks
C. Hypertrophy C. 12-15 weeks
ANS.B D. 1-2 weeks
100. 30 yrs old person has stab wound on ANS.A
right side of chest. what will happen? 106. Open wound healing includes:
A. Ipsilateral lung collapse and ipsilateral A. Contraction of myofibroblast
chest wall spring out B. Fibroblast
B. Ipsilateral lung collapse and contralateral C. Macrophages
chest wall collapse D. Lymphocytes
C. No change in chest wall ANS.A
D. Contralateral lung collapse and ipsilateral 107. Basal rhythm of small intestine?
chest wall collapse A. Synchronized with stomach
E. Ipsilateral lung collapse and contralateral B. Decrease when mechanical activity start
spring out C. Variable effect by hormone
ANS.A D. Constant in duodenum variable else
101. Deficiency of pulmonary surfactant: where
A. increases surface tension, increases ANS.D
compliance 108. Exotoxin:
B. increases surface tension, decreases A. Corynebacterium diphtheria
compliance B. Staph aureus
C. decreases surface tension, decreases C. Strep pyogen
compliance D. Clostridium tetani
D. decreases surface tension, increases ANS.A
compliance 109. Which among following is a chemical
E. increases surface tension, no effect on carcinogen?
compliance A. Alpha 1 antitripsin
ANS.B B. Benzidine
102. A patient presents to the emergency C. Ethyl alchohol
department with 6 hours history of severe D. Alfatoxin
central chest pain and dyspnea. Which of ANS.B
the enzymes would be elevated? 110. Regarding Glycolysis which one is
A. SGOT correct:
B. SGPT A. Reversible
C. LDH B. Hexose phosphate converted to lactate
D. Alkaline phosphatase and pyruvate
E. CPK C. Same as Hexose monophosphate shunt
53

D. Includes hexose sugars to pyruvate A. Vocal cords strongly approximate


ans>D B. Larynx moves downward
111. 55-year-old woman with insulin C. Esophageal sphincter is contracted
dependent diabetes of 40 years’ duration D. Palatopharyngeal fold moves downward
complains of severe bloating and ANS.A
abdominal distress, especially after meals. 118. The effect which a drug causes on
Evaluation is consistent with diabetic body is called?
gastroparesis. The drug you would be A. Pharmacodynamics
most likely to recommend is B. Pharmacokinetics
A. Docusate C. Bioavailability
B. Dopamine ANS.A
C. Loperamide 119. Parasympathetic origin in spinal
D. Metoclopramide cord
E. Sucralfate A. S2 S3 S4
ANS.D B. S1 S2 S3
112. Most specific for diagnosis of SLE C. S3 S4 S5
A. Anti dsDNA D. L1-L2
B. ANA ANS.A
C. Anti SSA 120. True about Ulnar nerve
D. Anti histone antibodies A. Loss of abduction and adduction of
ANS.A fingers
113. Patient presented with dryness of B. Loss of opposition of thumb
eyes and mouth. Which of the following C. Wrist drop
would be most helpful in the diagnosis? D. Thenar muscles wasting
A. Anti SSA ANS.A
B. Anti drNA 121. Epileptic drug which increases the
C. ANA effect of other Antiepileptics when given
ANS.A together is
114. Frequency distribution A. Valproic
A. Tells distribution of value appear in data B. Phenytoin
B. Tells distribution of value which is C. Carbamazepine
missing ANS.A
C. Gives percentage of values 122.Drain is inserted in chest wall at the
D. Can’t be used for data from different lower border of rib. Which structure will
categories. be damaged first ?
ANS.A A. Intercostal nerve
115. Most potent response in massive B. Intercostal vein
hemorrhage? C. Intercostal artery
A. CNS ischemic response ANS.A
B. Baroreceptors 123. Which vein drains into the left
C. Peripheral chemoreceptors brachiocephalic vein?
D. Brain bridge reflex A. Inferior thyroid vein
ANS.A B. Superior thyroid vein
116. Difference between septic and C. Middle thyroid vein
hypovolumic shock? ANS.A
A. Temperature 124. Preganglionic autonomic fibres are?
B. Cardiac output A. B fibres
C. Heart rate B. C fibers
ANS.A C. A beta
117. Regarding pharyngeal phase of D. A delta
deglutition which is appropriate? ANS.A
54

125. Avidin binds with B. Decrease body temperature


A. Biotin ANS A
B. Vit B12 133. A 65 Year old mate patient Presented
C. Vit C with chest pain for last 30 mins. And has
D. Vit D ST segment elevation in leads V1 to V4.
E. Riboflavin He is most likely to have
ANS.A A. Anterior infarction
126. In pancreatic fistula somatostatin B. Anterolateral infraction
helps by C. Lateral infarction
A. Decreasing output of secretions D. Inferior infarction
B. Increasing ph of secretions E. Posterior Infarction
C. Increasing blood flow of gland ANS.A
D. Inhibiting enzymes 134. Which of the following is not directly
ANS.A caused by cortisol in Cushing's
127. Which is present just lateral to the syndrome?
trachea? A. Buffalo hump
A. Vagus nerve B. Thin lower limb
B. Phrenic nerve C. Hirsutism
C. RLN D. Pendulous Abdomen
ANS.A ANS.C
128. Usual cause of atrophy of breast in 135. In bone new matrix and fibres are
adult female is deficiency of? form by?
A. Gonadotropin A. Fibroblast
B. Growth hormone B. Osteoclast
C. Estrogen C. Osteoblast
D. Estrogen nd progesterone D. Osteocytes
E. Progesterone ANS.C
ANS.C 136. Blood Antigens:
129. Women using OCPs are most A. Present in saliva
common at risk for B. Called agglutinins
A. Thromboembolism ANS.A
B. Osteoporosis 137. Anemia of chronic disease
C. Osteomalesia A. Serum transferin saturation is low
ANS.A B. Iron is mirror for Bonemarrow ferritin
130.One of the following is related to the C. Iron not coming out of monocytic
diagnosis of asthma in 20 yr old male? macrophage
A. FEV1/FVC <65 Cant recall other 2 options
B. PO2 =40 mmHg ANS.C
C. Respiratory rate=12 138. Q fever is caused by which
D. Tidal volume 500 ml organism?
E. Vital capacity= 3.0 A. Rickettsiae
ANS.A B. Rick
131. Male infertility due to testicular C. Virus
cause? D. Parasite
A. Cryptorchidism ANS.A
B. Hypospadiasis 139. Primary Center of Ossification at
C. Kliinefilter syndrome Epiphysis of Long bone at Birth?
D. Varicocele A. Lower end of Femur
ANS.A B. Lower End of Humerus
132. Metabolic rate is increased by: C. Upper End of Fibula
A. Thyroxine D. Upper End of Tib
55

ANS.A D. Glomerulus has single arteriole


140. Patient with Hypotension, shouldnot Ans>B
give benzodiazepine in which case? 147. Aphasia due to lesion in
A. Hypovolumic A. Temporal lobe
B. Obese B. Parietal
C. Age > 65 C. Occipital
D.Pregnancy ANS.A
ANS.A 148. Plasma half life of a drug depends
141. Autoimmune hemolytic anemia is upon:
diagnosed by A. Rate of clearance
A. Positive antiglobin B. Volume of distribution
B. Splenomegaly C. Dose
C. Spherocytosis D. Rate of absorption
D. Reticulocystosis E. Route of administration
E. Erythroid hyperplasia ANS.A
ANS.A 149. Prozone phenomenon?
142. Right iliac fossa pain reffered to A. Antigen excess
umbilicus??? B. Antibody excess
A. T 10 C. Antigen depletion
B. T 8 D. Antibody depletion
C. L 1 ANS.B
ANS.A 150. Gonorrhea DOC?
143. Regarding Thymus A. Penicillin G
A. Supplied by internal thoracic and inferior B. Ceftriaxone
thyroid artery C. Gentamycin
B. T Lymphocytes enter into the blood ANS.A
stream through post capillary venules 151. Dose of which of the following drug
C. Is a lobulated organ should be reduced in renal impairment:
D. One option was about something A. Gentamicin
collagen (Can’t recall) B. Rifampicin
ANS.B C. Penicillamine
144. Small emboli from veins is most D. Phenytoin
commonly lodged in ANS.A
A. Lungs 152. Primary brain vesicle
B. Heart A. Mesencephalon
C. Brain B. Telencephalon
D. Kidneys C. Diencephalon
E. Liver D. Metencephalon
ANS.A E. Mylencephalon
145. Lobar pneumonia diagnosis ANS.A
A. Sputum culture 153. Diabetic gangrene:
B. Sputum staining A. Atheromas in arteries
C. Blood culture B. Always start as moist necrosis
ANS.A C. A/V fistula develop
146.True about Kidney: D. Large arteries are always responsible
A. Cuboidal epithelium in parietal layer of ANS.A
bowman capsule 154. Most common cause of sex
B. Podocytes on visceral layer of bowman chromosome abnormality????
capsule A. Hermaphrodite
C. DCT longer than PCT B. Klinefelter syndrome
C. Downs Syndome
56

D. Multi X female E. Ketogenesis


Ans>C ANS.A
155. Which of the following veins drains 162. The most usual complication of
into coronary sinus and itself becomes wound healing in black population
part of coronary sinus? is?????
A.Great cardiac vein A. Hemorrhage
B. Middle cardiac vein B. Keloid formation
C. Anterior cardiac vein C. Malignant change
D. Small cardiac veins D. Metaplasia
ANS.A E. Premalignant change
156. A 24 years old female with 12 weeks ANS.B
of amenorrhea presents with frequent 163. In a healthy alert adult sitting with
burning micturation and high grade eyes closed, the dominant EEG pattern
fever. Diagnosis? with electrodes over the occipital lobe?
A. DM A. Alpha
B. Early pregnancy B. Theta
C. Urinary bladder stone C. Delta
D. UTI D. Beta
E. Urethral stricture E. Fast irregular low voltage activity
ANS.D ANS.A
157. Sodium reabsorption in baso lateral 164. Paracetamol
membrane occurs through: A. Weak anti inflammatory property
A. Gucose co transport B. Nephrotoxic
B. Na K pump ANS.A
ANS.B 165. Cerebellar lesion
158. Parasympathetic effect on heart A. Ataxia
increases permeability in SA node to B. Rigidty
A. Sodium C. Difficulty in initiating movement
B. Potassium D. Resting tremors
C. Calcium ANS.A
D. Potasium and sodium 166. Cause of cyanosis:
E. Sodium and calcium A. Decrease O2 in the blood
ANS.B B. Decrease Hb
159. A hematocrit of 41% means: C. Increase concentration of deoxy
A. 41% formed elements in blood are RBCs hemoglobin
B. 41% blood in plasma ANS.C
C. 41% of blood in serum 167. Secondary oocyte is surrounded by
D. 41% of formed elements in blood corona radiate, Which of the following
compromises of RBCs, WBCs, platelets layer contribute to the formation of
ANS.D ref ganong corona radiate?
160. GFR is decreased by A. Granulosa cells
A. stone in ureter B. Theca interna cells
B. Angiotensin 2 C. Theca externa cells
C. Afferent arterioles dilation ANS.A
ANS.A 168. Gas gangrene. Cause of death?
161. Ziinc is important essential element A. Septecemia
in one of the following: B. Bacteremia
A. Carbonic anhydrase C. Toxemia
B. Cellular oxidation ANS.C
C. Vitamin B12 169. Decreased ESR is due to increase in
D. Glycolysis A. Fibrinogen
57

B. Plasma albumin ANS. Nucleus gracilis and cuneatus


ANS.B 178. One question about Autosomal
170. Which of the following protein binds dominant
free hemoglobin in plasma ??? A. Incomplete penetrance
A. Alpha 2 macroglobulin B. Both genes are mutated
B. Transferin C. Early presentation
C. Heptoglobin ANS. A
ANS.C 179. Patient with eyelid dropping, muscle
171. Difference between malignant and weakness, anterior mediastinal mass.
normal cells such that malignant cells What is seen?
repair the shortening of chromosome so ANS. Antibodies against acetylcholine
that they can continue dividing. Enzyme receptors
responsible? 180. After hemorrhage which will return
A. Telomerase last to normal?
B. Reverse transcriptase ANS. Red blood cells in peripheral smear
C. Isomerase 181.Hearing is best perceived at?
ANS.A Ans> Superior temporal gyrus
172. Cimetidine is not used frequently 182.Stress Hormone?
because it causes??? Ans>Cortisol
A. Decrease hepatic metabolism 183. Most common cause of gastric
B. Nausea and vomiting malignancies?
ANS.A Ans>H pylori
173. Gonorrhea can be easily diagnosed 184.After feeding the baby, diarrhea
by: develops. Cause?
A. Gram stain Ans>Gastrocolic reflex
B. ZN stain 185. Ectoparasite?
C. Culture Ans?Lice*
D. PCR 186. Pituitary mass?
ANS.A Ans>Bitemporal hemonopia
174. Glucagon secretion is increased by? 187.One about Graft rejection.. ??
A. Exercise Ans>Associated with HLA
B. secretin 188. One was about knee jerk/When hit
C. Somatostatin patellar tendon??
D. Fatty acid Ans>Contraction of quadricep muscles
E. Increase Glucose 189. Highest renal clearance?
ANS.A Ans>PAH
175. Most common premalignant lesion of 190. Potassium gradient inside and
the skin? outside the cell is maintained by?
A.Sebhorric keratosis Ans> Na/K pump
B.Intrdermal Nevus 191. Which cells form collagen and
C.Junctional Nevus amorphous substance?
Ans>A Ans> Fibroblasts
176. Inhibitory factor released by
hypothalamus against which one of the 192. Fatal disease of a patient
following? A. Should not be told to the patient and his
A. Prolactin family
B.Oxytocin B. Tell the patient and his family as soon as
C. ADH possible
D. Growth Hormone C. Crisp clear information should b provide
ANS.A to the patient according to his need
177. Medial lemiscus formed by D. Should be told the relatives of that patient
58

ANS.C infections. The most likely organism


193. One was about meningitis..MCC? involved is:
Ans>Streptococcus (No age etc were a. Streptococcus pneumonia
given) b. HemophilusInfluenzae
194. Sternum ossify at the age of? c. Staphylococcus aureus
Ans>21 Years d. Streptococcus Pyogenes
Regards Ans: A
Dr. Irshad Hussain 6. Dermatome distribution around
Remember me in your prayers. umbilicus is at the spinal level of
November 12, 2016 a. T7
10TH NOVEMBER, 2016 b. T12
c. T10
MORNING SHIFT MEDICINE & d. L1
Ans: C
ALLIED (PAPER I & II) 7. Thoracolumbar outflow is:
1. Amount of urine excreted by a normal a. Sympathetic
adult in a day b. Parasympathetic
a. 500 ml c. Cranial nerves
b. 1000 ml Ans: A
c. 3 litre 8. Pudendal nerve block can be given by
d. 2200 ml feeling which of the following landmarks?
e. 5 litre a. Ischial tuberosity
Ans: B here ( Normal range is 800-2000ml b. Ischial spine
per 24 hrs) Ans: B
2. Narcotic with the lowest pKa is: 9. True about facial nerve
a. Sufentanil a. Arises from upper part of pons
b. Morphine b. Lesion compresses vestibular part of
c. Fentanyl cranial nerve 8
d. Meperidine c. exits skull through foramen rotundum
Ans: B (Alfentanil > Remifentanil > d. receives upper motor fibers from
Morphine > Sufentanil > Fentanil > contralateral cortex only
Meperidine) Ref. Lippincott Clinical e. contains secretomotor fibers.
Anesthesia, 6th ed, chp 19, page 469 Ans: E
3. 19 year old girl presents with history of 10. levato rani muscle takes motor supply
weakness and lethargy. On examination from
she is pale. Hb is 6.2g/dl MCV 62 MCHC a. Pudendal nerve
(---reduced---). Diagnosis? b. Iliohypogastric nerve
a. Anemia of chronic disease Ans: A
b. Iron deficiency anemia 11. Cervical rib
c. Thalassemia a. compresses brachial plexus and
Ans: B Subclavian artery
4. Pregnant lady with yellow sclera and ans A
dark colored urine… Obstructive 12. True about descending thoracic aorta
jaundice… What is the appropriate a. supplies lower 8 intercostal spaces
investigation in this patient? b. supplies pericardium, lungs and
a. Serum glutamyltransferase esophagus
b. alkaline phosphatase c. pierces diaphragm at L1 and unites with
c. Gamma glutamyl transferase abdominal aorta
Ans: C Ans: B
5. A young boy underwent spleenectomy. 13. Progesterone works in contraceptive
After the procedure he gets repeated pills by
59

a. spermicidal action? c. Hepatoma


b. induces uterine contraction d. Warthin tumor
c. inhibits ovulation Ans: D
d. inhibits maturation of ovarian follicles 20. Secretion of gastrin is inhibited by
Ans: C here (The progesterone only pills a. Acetylcholine
work by thickening the mucus in the neck of b. Somatostatin
the womb, so it is harder for sperm to Ans: B
penetrate into the womb and reach an egg 21. A man after long marathon race will
and sometimes it may also prevent show blood level changes as follows
ovulation-depends upon the type of a. Increase Insulin and decrease glucagon
progesterone pills) b. increase Insulin and increase glucagon
14. Regularly placed lamella are seen on a c. decrease insulin and decrease glucagon
histopathological slide. Such an d. decrease insulin and increase glucagon
arrangement is present in e. no change
a. Cancellous bone Ans: D
b. Compact bone 22. Vitamin required for the dietary fatty
c. Dentine acids to undergo beta oxidation in cells
Ans: B a. Thiamine
15. When activated, C3a and C5a cause?? b. Biotin
a. Vasodilation and anaphylactogenic Ans: B
response. 23. Hormone which uncouples oxidative
b. cytolysis by MAC phosphorylation is:
c. PhagocytoseIgG coated bacteria a. Growth hormone ?
Ans: A b. Cortisol
16. True about neutrophils: c. Glucagon
a. Produced in spleen and lymph nodes d. thyroxine
b. Migrate in and out of blood e. epinephrine
c. Actively phagocytose microbes in plasma. Ans: D
Ans: C 24. A female patient has history of
17. History of abdominal pain, bloating, thromboembolism/DVT. She is started on
fatigue, weakness, pallor. Stool oral anticoagulant therapy. Which test
examination reveals some eggs. Most will be helpful in monitoring the effects of
likely organism is:?? oral therapy?
a. Ascaris lumricoides a. Platelet count
b. Ankylostoma duodenale b. Prothrombin time
Ans: A (Their eggs are visible in feces c. clotting time
under microscope) d. activated partial thromboplastin time
18. An 8 month old baby boy has history e. bleeding time
of repeated attacks of sinusitis, rhinitis, Ans: B
diarrhea and 2 episodes of pneumonia. 25. Patient has a lesion of the L4 nerve
On investigations his B lymphocytes are root. He will feel pain at:
reduced with deficiency of all types of a. Anterior upper part of thigh
immunoglobulins. Diagnosis? b. Back of thigh
a. Isolated IgA deficiency c. Front of knee
b. Common variable immunodefeciency d. Upper part of calf
c. X linked agammaglobulinemia e. lower calf
Ans: C Ans: C
19. Out of these, a benign tumor is 26. Only nerve involved in multiple
a. Wilm’s tumor sclerosis
b. Neuroblastoma a. Optic nerve
b. occulomotor nerve
60

Ans: A Ans: C
27. Different conditions are associated 34. Structure affected by a mass in the
with different HLA types. For instance a posterior mediastinum will compress
disease of the spine which leads to fusion a. Arch of aorta
of vertebrae i.e. Ankylosing spondylitis is b. ascending aorta
associated with which HLA type? c. descending aorta
a. HLA DR3 Ans: C
b. HLA B8 35. A neonate was brought to the
c. HLA B27 emergency with history of fever, neck
d. HLA DR2 rigidity. Organism likely to be the
e. HLA DR4 causative agent:
Ans: C a. Hemophilus influenza
28. A malignant tumor is seen on b. Streptococcus Pyogenes
microscopy. It will show following c. E. coli
features: d. Staph aureus
a. Invasion e. Streptococcus pneumonae
b. Increase nuclear cytoplasmic ratio Ans: C (MCC of meningitis in new
c. Pleomorphism born/neonates is Group B strep and 2nd MC
Ans: A is E.coli)
29. Scanty Barr bodies will be present in: 36. Left shift of the hemoglobin
a. Testicular feminization syndrome dissociation curve occurs due to
b. Klinefelter syndrome a. hypothermia
c. Turner syndrome b. increase in temperature
Ans: C c. acidosis
30. A 20 year old gardener with Ans: A
symptoms of respiratory tract. Chest x- 37. In a person with meningococcal
ray shows a 4 x 3 cm mass in Right lobe of meningitis and in the event of no known
the lung. Most likely diagnosis is: drug allergy the medicine preferred will
ans. Aspergillosis be
31. A student carries a study in a ans. Penicillin G
population comparing symptoms in 38. cAMP is formed in the cell by the
diseased individuals against lack of those action of
symptoms in non-diseased people. This a. Enzymes
study is called b. Receptors
a. Cohort c. Cholesterol
b. Cross sectional Ans: A
c. Interventional Cohort 39. Substance which induces the flow of
d. Case control saliva
Ans: D a. Motilin
32. A young farmer was working b.VIP
continuously on a sunny day for several c. Substance P
hours. He collapsed suddenly, labs show DAcetylcholine
sodium 120 mEq/L. Reason? Ans: d
ans. Excessive sweating 40. Low pressure in the carotid sinus will
33. High pitch frequency in relation to cause?
basilar membrane strikes ans. increase in heart rate and contractility
a. round window 41. Bleeding time is prolonged in??
b. intermittently throughout the length of a. Hemophilia
basilar membrane b. von Willibrand disease
c. Base of cochlea Ans: B
d. Apex of cochlea
61

42. Atrial fibrillation shows following The swelling moved up & down with
clinical signs: swallowing. This is because the structure
ans. Pulse deficit that caused this swelling is in:
43. True about Rh blood group system, ans. Pre-tracheal fascia
Rh agglutinins 50. Histamine release causes:
a. Are absent in Rh negative mother in first a. Venodilation
pregnancy with Rh positive baby b. Vasoconstriction
b. All agglutinins are IgM variety c. Muscarinic receptor activation
c. All Rh negative people have anti Rh d. Arteriolar dilation
antibodies. Ans: A
Ans: A 51. Left recurrent laryngeal nerve is
44. Cardiac output is mainly determined related to:
by which of the following? a. Brachiocephalic trunk
a. Venous Return. b. Arch of Aorta
b. Stroke Volume. c. Azygous vein
c. Heart Rate d. Subclavian
d. EDV in the left ventricle e. Esophagus
e. ESV in the left ventricle Ans: D
Ans: A 52. Acrosome cap of the sperm is formed
45. At what point in the cardiac cycle does by which of the following organelles?
T wave appear on ECG? a. Peroxisome
a. Beginning of the refractory period. b. Golgi Bodies
b. Ventricular repolarization. c. Lysosomes
c. End of an action potential. Ans: B
d. Ventricular depolarization 53. A patient for --- underwent liver
Ans: B transplant surgery. After 5 years this
46. Which of the following statements is patient would be at a risk of developing
correct regarding middle meningeal a. Skin Cancer
artery? b. Lympho proliferative disorders
a. can be felt at pterion Ans: A (In the first yr after transplant 
b. Enters the skull through foramen Lymphoproliferative disorders and After 5
spinosum yrs  Skin cancer common. Ref Bailay &
Ans: B Love)
47. Heart failure is commonly (associated 54. One of the following fungus acts by
with/due to?) which of the following involvement of the Reticulo endothelial
conditions? system
a. Fibrinous pericarditis a. Histoplasma
b. Hemopericardium b. Blastomycosis
c. Pericardial effusion c. Cryptococcus
d. Chylopericarditis d. Candida
Ans: A Ans: A
48. Main arterial supply of the 55. Characteristic feature of the
Interventricular septum is: cerebellar disease is:
a. Anterior Interventricular artery a. Shuffling gait
b. Posterior Interventricular artery b. Ataxia
c. left coronary artery Ans: B
d. left circumflex artery 56. Propranolol acts on:
e. Right coronary artery a. Muscarinic receptors
Ans: A b. Beta 1 receptors
49. A young boy presented with a c. Beta 2 receptors
globular swelling in the midline of neck. d. Alpha receptors
62

Ans: B (Although it is a non-selective beta a. Lung hypoplasia


blocker and acts on both B1 and B2) b. Respiratory distress syndrome
57. Notochord gives strength to the post c. Asthma
natal adult body as it becomes one of the Ans: B
following structures 64. Superficial temporal artery is related
a. Vertebral bodies to which of the following nerves?
b. Annulus fibrosis ans. Auriculotemporal nerve
c. Nucleus pulposus 65. Nerve that supplies Trapezius muscle
Ans: C a. Radial Nerve
58. A 2 years old child has the history of b. Cranial part of Accessory nerve
premature birth. He is most likely to have c. Spinal part of Accessory nerve
the followingcongenital anomaly except Ans: C
a. Patent ductus venosus 66. Which nerve supplies the parietal &
b. Patent ductus arteriosus outer layer of serous pericardium?
c. Patent foramen ovale a. Phrenic
Ans: A b.Vagus nerve
59. Potassium level in blood is most likely Ans: A
to be increased due to 67. Which of the following enzymes is
a. Diarrhea involved in the transfer of radicals from
b. Spironolactone therapy one compound to another?
c. Metabolic Acidosis a. isomerases
d. Metabolic Alkalosis b. oxidoreductases
Ans: C > B c. transferases
60. Class III anti-arrythmics include: Ans: C
a. Procainamide 68. Which of the following is a must for
b. Disopyramide the diagnosis of sepsis?
c. Flecainide a. TLC <2000 or >15000
d. Quinidine b. SBP <70 mmHg
e. Dofetilide c. Blood culture positive for bacteria
Ans: E (Other drugs of this group are d. HR >120/min
Amiodarone, Ibutilide and Sotalol) e. HR >100/min
61. Why is cimetidine given before Ans: C ( SIRS criteria is absolete now so
surgery? ans is CC)
a. to decrease gastric motility 69. The most common cause for
b.to block H2 receptors formation of deep venous thrombosis is:
c.to decrease gastric acid secretion a. Prolonged immobilization
Ans: C b. Protein C deficiency
62. A patient was taking cimetidine Ans: A
&sucralfate simultaneously for GERD 70. Which of the following statements is
but he was advised to stop taking them unlikely to be true about H+ in the renal
both together. Why was he advised this? tubules?
a. Cimetidine decreases absorption of a. it can bind with NH4+
Sucralfate b. It can bind with HPO4-
b.Cimetidine increases the potency of c. It can bind with HCO3-
sucralfate d. It is secreted by H+ ATPase pump
C. Cimetidine increases the levels of e. It can exist as free H+
Sucralfate because of decrease degradation Ans: A ( H+ can bind with NH3+ not
Ans: C NH4+)
63. A neonate born prematurely has a 71. Granulocyte cell is:
deficiency of surfactant. She is at a risk of a. Neutrophil
developing b. Platelet
63

c. Lymphocyte can be breached in which scenario?


d. Monocyte a. after the death of the patient
e. Red blood cell b. for an insurance claim
Ans: A c. when the patient authorizes you
72. What will be the labs in primary Ans: C..( if u r taking USMLE then prefer B
hypothyroidism? coz insurance has got more importance in
a. Low TSH USA)
b. High TSH 79. A doctor can ensure better
c. High TSH & high T3, T4 communication with his patient by
Ans: B a. Prescribing many medications
73. Which of the following statements is b. Decorating his office beautifully
correct regarding CSF? c. Asking many questions
a. it is reabsorbed by ependymal cells d. Active listening
b. same pH & constitution as plasma Ans: D
c. Flows above dura mater 80. Which is true regarding the
d. Reabsorbed by arachnoidal villi withdrawal reflex?
Ans: D a. it is a monosynaptic reflex
74. Which of the following is the b. includes input from higher centres
characteristic feature of irreversible cell c. Multisynaptic
injury? Ans: C
a. Hydropic degeneration 81. Role of lidocaine as an anti-arrythmic
b. Clumping of nuclear chromatin drug is due to its action on?
c. Cytoplasmic contraction bands a. prolonging action potential
d. Cell swelling b. Inhibition of the generation of a new
Ans: C potential
75. Which of these is a malignant brain Ans: B
tumor? 82. Opioids cause vomiting by?
a. Meningioma a. Direct activation of vomiting center
b. Ependymoma b. Activating the chemoreceptor trigger zone
c. Pseudolymphoma Ans: B
d. Glioma 83. A chronic smoker is working in tire
Ans: D factory for (>10 years) develops
76. When a stimulus reaches an excitable hematuria. A diagnosis of transitional cell
membrane, it causes: carcinoma is made on histology. Which
a. Always generates action potential carcinogen led to the development of this
b. reverses the resting membrane potential cancer in him?
c. Chloride influx a. Nicotine
d. Potassium influx b. Nitrates & Nitrites
Ans: B c. Aromatic Amines
77. True regarding a normal adult heart d. Aniline dyes
is that purkinje fibers have fastest speed e. Smoking
of conduction. This is because these fibers Ans: E (Tar in cigarette smoke causes
have cancer while nicotine doesn’t cause cancer,
a. high no. of gap junctions it is an addictive agent which may keep you
b. threshold near resting membrane potential smoking.)
c. highest no. of intercalated discs 84. A patient underwent surgery for
d. largest diameter mitral valve replacement. After 8-12
e. maximum number of sodium channels months of the surgery, she develops
Ans: C complains of pallor & dark colored urine.
78. Regarding doctor – patient What is the likely cause?
relationship, confidentiality of the patient a. Anemia of chronic disease
64

b. Iron deficiency anemia b. Right ventricle


c. Infective endocarditis c. Left ventricle
d. Microangiopathic hemolytic anemia d. Left atrium
Ans: D Ans: B
85. Regarding black water fever, the 92. An old man developed severe chest
correct statement is: pain in an event & died in a few minutes.
ansz. associated with falciparum malaria Cause of death was established to be
86. A young patient presents to you with dissecting aortic aneurysm. The autopsy
fever, neck rigidity. Diagnosis of finding in this man would be:
meningitis is made. Gram stain of CSF ans.Medial necrosis
reveals pus cells & cocci. Likely organism 93. Which structure passes in front of the
is: root of Right lung?
a. Streptococcus pneumonia a. Azygous vein
b. Streptococcus Pyogenes b. Hemiazygous
c. Neisseria meningitides c. Phrenic Nerve
d. Staphylococcus Aureus Ans: A
Ans: A 94. A 12 year old child has dwarfism due
87. The strongest inhibitory signals in to short limbs & normal height of the
central nervous system come from which trunk. There are no signs or symptoms
neurons? suggestive of mental retardation. Pattern
a. Substantia nigra of development of dwarfism in this child
b. Pyramidal cells of cerebral cortex would have been:
c. Purkinje cells of cerebellar cortex a. Autosomal dominant inheritance
Ans: C b. Autosomal recessive inheritance
88. Malaria is associated with? c. X linked dominant
a. Lymphocytosis d. X linked recessive
b. Hemolytic anemia e. Not inherited
c. Thrombocytosis Ans: A (Achondrioplasia)
Ans: B 95. Linear growth of the body will be
89. An old lady has 5 years history of affected by a fracture of the bone that
fatigue. Investigations reveal Hb = 12 passes through:
g/dl, MCV = 120. Bilirubin = 12, AST = a. Epiphyseal plate
70, ALT = 80.Haptoglobin = 80. Cause of b. Metaphysis
her lethargy is? c. Diaphysis
a. Iron deficiency anemia d. Epiphyseal line
b. Megaloblastic anemia Ans: A
c. Hepatitis C 96. A diagnosis of malignant melanoma is
Ans: B made on a patient. The first line cells
90. A 9 year old boy presents to you with which fight this tumor are:
the history of yellow sclera & dark colored a. Monocytes
urine. Most appropriate investigation would b. B cells
be? c. Macrophages
a. SGPT d. Cytotoxic Lymphocytes
b. SGOT e. NK cells
c. Bilirubin Ans: E
d. SGPT & bilirubin 97. Athetosis is
Ans: D a. due to defect in chromosome 4
91. On antero-posterior view of chest B. slow continuous writhing movements
radiograph, the sternocostal surface of c. Jerky movements
heart is made of: Ans: B
a. Right atrium
65

98. Thrombocytopenia develops in a child c. Leaves impression on skull through its


who has suffered an acute viral infection course
recently. The cause of low platelet count d. Enters skull through foramen spinosum
would be: Ans: D
a.Platelets were consumed during the 105. After suffering head trauma, patient
infection is being examined. Pulsations appear in
b. antibodies are formed against platelets the orbit with heartbeat, this
Ans: B phenomenon is due to relation of:
99. Which of the following amino acids is a. Cavernous sinus & ICA
not formed in the body & has to be taken b. cavernous sinus & ECA
in diet? c. ophthalmic nerve & ophthalmic artery
a. Phenylalanine d. cavernous sinus & optic artery
b. alanine e. ICA & ophthalmic nerve
c. tyrosine Ans: A ( Ref. Snell Neuroanatomy)
d. glycine 106. Old man has a history of stroke.
Ans: A Later he develops reduced motivation &
100. Shaft of hair is mainly composed of depression. He has a lesion in
a. Stratum corneum a. Basal Ganglia
b. Stratum basale b. temporal lobe
c. Corneum &basale c. parietal lobe
d. Cortex of hair d. occipital lobe
Ans: D e. frontal lobe
101. The distinctive arrangement of Ans: E
collagen fibers in the skin is 107. Which of the following tracts is
predominantly found in involved in carrying out finely controlled
a. Reticular layer point to pint movements by a miniature
b. Dermis artist?
c. Hypodermis a. Cortico-spinal tract
d. Papillary Layer b. Cortico-rubro-spinal tract
e. Epidermis c. Vestibulo-spinal tract
Ans: B d. Cerebello-spinal tract
102. Which vaccine shall be given to the e.olivo-pontine tract
patient who has undergone Ans: A
spleenectomy? 108. Left half of pons is related to
a. H. influenzae a. left olivary nucleus &
b. Hepatitis B b. left cerebellar hemisphere & right basal
c. Streptococcal ganglia
d. Pneumococcal c. right cerebellar hemisphere & left basal
Ans: D ganglia
103. During quite breathing, the d. anterior vestibular nuclei
expenditure out of total body energy is: e. right cortical tract & left olivary nucleus
a. 1% Ans: E
b. 5% 109. Anterior & posterior spinal arteries
c. 10% are the branches of:
d. 15% a. PICA & vertebral arteries respectively
e. 20% b. AICA & vertebral arteries respectively
Ans: B c. ICA & vertebral arteries respectively
104. Most appropriate statement d. vertebral & posterior cerebral arteries
regarding middle meningeal artery respectively
a. can be located at the pterion Ans: A ( Ref. RJ Last)
b. Is commonly involved in the extradural 110. Maximum percentage of carbon
hematoma dioxide in the blood is found as
66

a. Bicarbonate 118. Right suprarenal vein drains into:


b. Carbaminohemoglobin a. IVC
c. Carboxyhemoglobin b. Right renal vein
Ans: A c. Right hepatic vein
111. Inhibitors of carbonic anhydrase will Ans: A
cause: 119. Which of the following is absorbed in
a. Reduced urine ileum? (bile wasn’t in the options)
b. Increased formation & reabsorption of a. fatty acids
HCO3- b. vitamin B12
c. Acidosis c. amino acids
d. Decreased Potassium d. water
e. Increase pH Ans: B
Ans: C 120. After successfully performing two
112. Surfactant deficiency causes adrenalectomies (removal of the adrenal
respiratory distress. It was either “due gland), the surgical resident was
to” or “because normal action of disappointed to learn that he would be
surfactant is” merely assisting at the next one. The chief
a. Decrease alveolar pressure of surgery told him: "I'm doing this one,
b. Role in immunity since the one on the right side may be a
c.. Reduce surface tension over alveoli little too difficult for you." The difficulty
Ans: C he envisioned stems from the fact that the
113. One of the following is a right suprarenal gland is partly overlain
premalignant condition: anteriorly by the:
a. Solar keratosis of skin A. aorta
b. Nodular hyperplasia of liver B. inferior vena cava
c. Pneumonia C. left hepatic vein
d. Bronchial asthma D. right crus of the diaphragm
Ans: A ( aka actinic keratosis) E. right renal artery
114. Characteristic feature of Ans: B
malignancy? 121. True statement regarding kidneys is:
a. Invasion a. is wrapped in a fascial sheath alongwith
b. Vascularity its adrenal gland
c. Pleomorphism b. Hilum is at the level of L4
d. Increase nuclear-cytoplasmic ratio c. pelvis of ureter lies between the renal
Ans: A artery & renal vein at the hilum.
115. Diagnostic feature of a dead cell is: d. related posteriorly to ribs
a. Pyknosis of nucleus Ans: A (Ref RJ Last)
b. Karyorrhexis 122. Diameter of bronchi & bronchioles
c. Karyolysis is:
d. Cell shrinkage a. mainly under the control of sympathetic
e. Hydropic change nervous system
Ans: A b. under muscarinic control
116. GFR in a normal adult male is: c. narrowed due to local stimuli
a. 90 Ans: A
b. 125 123. Which of the following tumors of
c. 300 CNS is caused by radiation?
Ans: GFR over a. Glioma
90mls/min/1.73m2 is normal. b. Meningioma
117. “foot drop” is due to injury of: c. Lymphoma
a. common peroneal nerve d. Ependymoma
b. deep peroneal nerve Ans: B
Ans: A
67

124. Weil felix test is done for the 131. First part of duodenum is posteriorly
identification of? related to:
a. Klebsiella a. Gastroduodenal artery
b. Rickettsia b. Hepatic artery
c. Pseudomonas c. Gastric artery
ANs: B Ans: A
125. A patient has irregularly irregular 132. During pregnancy, obliteration of
pulse on examination. ECG shows many uterine cavity occurs due to fusion of:
abrupt P waves & the QRS complexes A. Amnion & decidua capsularis
occur irregularly. What is the diagnosis? b. Decidua Parietalis& decidua capsularis
a. Ventricular fibrillation c. Chorion& amnion
b. WPW syndrome d. Decidua basale& Capsularis
c. Atrial fibrillation Ans: B
d. Atrial flutter 133. A drug is given by error that causes
e. Ventricular tachycardia further increase in calcium level in blood.
Ans: C The drug is:
126. Best index of preload is: a. Loop diuretic
a. Pulmonary artery wedge pressure b. Thiazide diuretic
b. Central venous (pressure/volume) c. Calcitonin
c. EDV in left ventricle d. Etodine
d. ESV in left ventricle Ans: B (Thiazide diuretics cause
Ans: C (LVEDV) Hypercalcemia while Loop diuretics are
127. Lymphatic drainage of the lower lip used in Hyperclacemia)
is into 134. Which of the following substance is
a. Jugulo digastric nodes more concentrated at the end of proximal
b. Submental& submandibular nodes convulated tubule as compared to
c. Submandibular &Jugulo-digastric nodes beginning of proximal tubules?
d. Parotid nodes a. bicarbonate
e.Submandibular nodes b. glucose
Ans: B ( Central part of the lower lip drains c. calcium
into submental while remaining into d. sodium
submandibular nodes) e. creatinine
128. True about the sensation of taste: Ans: E (because no reabsorption of
a. salty taste is perceived at the lateral aspect creatinine occur in PCT but others occur
of tongue along with water so creat conc increases)
b. adapts within seconds 135. Eosinophilia is likely to occur in:
c. is a chemical sensation a. Leishmaniasis
Ans: A b. Atopy
a. person ascends > 1500 feet c. Hodgkin’s lymphoma
b. expiration to residual volume Ans: C
c. moderate exercise 136. True about laxatives
d. inspiration to total lung capacity a. Lactulose is digested by gut bacteria
Ans: C (Ref. Guyton) b. Ispaghol husk is bulk forming laxative
c. Senna stimulates GI within 30 minutes
130. True statement about the d. Lactulose is a sugar
physiological dead space is: e. Excessive use can lead to metabolic
a. Is not diffusion dependent. alkalosis & colonic atony
b. is equal to anatomical dead space in a Ans: B
normal adult 137. A young boy has complaint of
c. does not include anatomical dead space diarrhea & abdominal bloating. Stool
d. Decreases with exercise examination shows rod shaped bacteria
Ans: B that are slightly curved. Organism
68

involved is: diagnosis is:


a. Salmonella a. Discoid lupus
b. Shigella b. SLE
c. E coli c. Progressive systemic sclerosis
d. Campylobacter jejuni d. Contact dermatitis
Ans: D Ans: A
138. 50 years old diabetic woman 143. 45 years old lady developed
complains of abdominal bloating after pericardial effusion & pleural effusion.
meals. A diagnosis of diabetic Effusion fluid was examined & it turned
gastroparesis is made. What is the best out to be Transudative. (Normal baseline
treatment for her symptoms? investigations)… Anti-DS titer was 1:560.
a. Antacids Urine nitrogen was > 55. (Renal) biopsy
b. Loperamide will show:
c. Omeprazole a. Amyloidosis
d. Metochlopramide b. anti-GBM antibodies
Ans: D c. Glomerular deposits
139. Anterior tibial artery runs inferiorly d. Immune complex deposition
down the leg along with e. Chronic Vasculitis
a. Medial plantar nerve Ans: D
b. Lateral plantar nerve 144. When a person encounters an
c. Sural nerve obnoxious stimulus, he reflexively
d. Deep peroneal nerve withdraws his limbs & body away from
e. Tibial nerve the stimulus. Most appropriate statement
Ans: D regarding this reflex is:
140. A 50 years old man who has previous a. is monosynaptic
history of diabetic retinopathy & MI b. is absent in lower animals
presented with chest pain… coronary c. Remains for many months after complete
occlusion occurs commonly due to: cord transection
a. Vasospasm d. weak stimulus prolongs it
b. Embolism e. is a superficial reflex
c. Stasis of blood Ans: C
d.Hypercoagulable state 145. 70 years old man presents with
e. Thrombosis myalgia, rash & arthralgia. Joints
Ans: E examination reveal no abnormality.
141. A 25 year old man presented with the Motor power is 5/5 . ANA is positive with
complaint of post prandial bloating & speckeled appearance. If anti-
abdominal cramping. He has history of ribonucleoprotein RNP is also positive
passing foul smelling stool. Which then the diagnosis is
investigation will give us multiple a. Mixed connective tissue disorder
information regarding the problem? b. Dermatomyositis
a. Stool C & S c. SLE
b. Stool DR d. Polymyositis
c. Serum lipid profile Ans: A
d. Ultrasound abdomen 146. During last seven months of
Ans: B (Stool R/E) pregnancy, estrogen and progesterone are
142. A young girl (15-25 years) develops secreted by 2.?
rash on cheeks & nose when she goes in ans. placenta
sun. There is no other complaint & the 147. A patient with history of chronic
rash subsides when she is indoors. Biopsy liver disease is brought to the emergency
of the skin reveals IgG deposits in the with the complaint of upper GI bleeding.
rash but the surrounding skin is normal. Drug of choice in the emergency is:
Serum ANA is negative. Most probable a. Octreotide
69

b. Vitamin K infusion b. Vasomotor tone


c. Tranexamic acid Ans: B
Ans: A 155. Acute inflammation is
148. Hypoglossal i.e. CN XII injury on the a. Local response of the tissue
right side will cause b. Systemic response
a. Left tongue paralysis with atrophy Ans: A
b. Left tongue paralysis without atrophy 156. A young boy was playing in the
c. Right tongue paralysis with atrophy ground. He fell down & got abrasions on
d. Right tongue paralysis without atrophy the knee. Important local response will
Ans: C be:
149. True regarding sperms of men above a. Chemotaxis
70 years of age b. Platelet plug formation
a. are asthenospermic c. Vasoconstriction
b. many lack forward progressive motility Ans: C
c. count ">" (some figure) as they are 157. Most immediate step that occurs in
oligospermic the scenario of bleeding is:
d. reduced quantity of semen because their a. vasoconstriction
seminal vesicles atrophy b. increase tissue permeability
e. can fertilize ovum c. drop in blood pressure
Ans: A Ans: A
150. During which stage of sleep do theta 158. Triple assessment for congenital fetal
waves appear on the EEG? anomalies include one of the following:
a. Stage I a. Estradione E2
b. Stage II b. Estrione E1
c. REM c. Estriol E3
d. Stage III Ans: C
e. Stage IV & V 159. Important cause of epithelial
Ans: A metaplasia is:
151. A middle aged man came back from a. Ultraviolet rays
Cairo & developed painless hematuria… b. Gamma radiation
Likely diagnosis is? c. Chronic irritation
a. Squamous cell carcinoma of the urinary Ans: C
bladder 160. One of the following drugs is found
b. Transitional cell carcinoma of the urinary to play a beneficial role in subarachnoid
bladder hemorrhage
c. Transitional cell carcinoma of the prostate a. Propranolol
Ans: A ( Cairo is the capital of Egypt and is b. Nifedipine
endemic for schistosomiasis which causes c. Verapamil
SCC of the urinary bladder) d. Nimodipine
152. Difference between primary & e. Labetalol
secondary intention wound healing is? Ans: D
ans. Contraction of myoblast 161. Carbon laden macrophages develop
153. Feature of primary shock is in
a.hypovolemia a. Silicosis
b.bradycardia b. Pneumoconiosis
c. tachycardia c. Anthracosis
d. initial hypertension d. Asbestosis
e. lower peripheral vascular resistance Ans: C (Ref Goljan)
Ans: C 162. Characteristic of TB granuloma is:
154. Total peripheral resistance is a. non- caseating
determined by? b. caseation
a. Venous compliance
70

c. cavitation b. Flexor digitorumprofundus


Ans: B c. Median nerve &?
163. Temperature center of the body is d. Ulnar & flexor digitorumsuperficialis
located in Ans: A
a. Hippocampus 171. true about conjoint tendon:
b. Hypothalamus a. formed by fused aponeuroses of internal
Ans: B oblique &transversus muscles
164. Severe dehydration will bring about b. Forms medial most fibers of inguinal
following change: ligament
a. Total body water will decrease c. prevents indirect inguinal hernia
b. Only ECF will decrease Ans: A ( Ref RJ Last)
c. Only ICF will decrease 172. Which of these statements is correct
d. Systolic BP will rise about triglycerides?
e. Heart rate will decrease a. are carried by lipoproteins
Ans: A b. contain fatty acids attached to glucose
165. Unilateral undescended testes should Ans: A
be placed in the scrotum surgically 173. In the chest wall, neurovascular
because if not corrected & left in the bundle lies between:
abdomen, it can lead to: a. External & internal intercostal muscles
a. Infertility b. Internal & innermost intercostal muscles
b. Malignancy Ans: B
Ans: B 174. Adult hemoglobin is composed of:
166. A young girl has history of SOB & a. 2 alpha & 2 gamma chains
wheezing. Investigations reveal C1 b. 2 beta & 2 gamma chains
deficiency & she lacks an esterase c. 2 alpha & 2 beta chains
required by the body. Most likely d. 2 alpha & 2 delta chains
diagnosis is: Ans: C
a. Angioedema 175. Which of the following structures
b. Myeloperoxidase deficiency prevents flow of unwanted substances
c. SLE between cells?
167. Which of the following structures a. Tight junctions
cannot regenerate? b. Gap junctions
a. Lens c. Desmosomes
b. Skin Ans: A
c. Intestinal mucosa 176. (History of TB, ATT & some eye
Ans: A signs. Drug involved was asked)
168. Regarding Anti-tuberculosis therapy, a. Streptomycin
correct statement is: b. Ethambutol
a. . INH produces adverse effects that can be c. INH
reversed with pyridoxine d. Pyrizinamide
169. Pulse pressure is determined by e. Rifampicin
(confusing options) Ans: B
a. Mean arterial pressure – central venous 177. (Something about the most beneficial
pressure drug in the event of acute coronary
b. Stroke volume syndrome)
Ans: B a. Heparin
170. A man injured his wrist by falling b. Warfarin
with outstretched hand over pieces of c. Aspirin
broken glass. Which structures of the d. Streptokinase
wrist will be damaged that run superficial e. Atorvastatin
to flexor retinaculum? Ans: C
a. Ulnar nerve & artery
71

178. A tumor that contains all germ layers a. Laminin


is: b. Collagen
a. Teratoma c. Fibronectin
179. Radial Nerve supplies which of the d. Tyrosine kinase
following muscles: Ans>Controversial
a. Biceps brachii PAPER - I MEDICINE & ALLIED
b. Triceps
c. Flexor digitorumprofundus 10 NOVEMBER 2016 EVENING
d. Palmaris longus 1. Potassium is mainly regulated by -
Ans: B Aldosterone.
180. Prolonged use of corticosteroids lead 2. Tip of scapula at level – T7.
to: 3. Oblique fissure of the lung at level - T3 to
a. Neutrophilia T6 Costochondral.
b. Lymphopenia 4. Decussating of medial lemniscuses -
c. Eosinopenia Internal Arcuate Fibers.
d. Thrombocytopenia 5. Superficial temporal artery relation with
Ans: A which nerve - Auriculotemporal nerve.
181. What happens in a patient suffering 6. Correct about Thyroid gland - lymph
from HIV? drainage to deep cervical lymph nodes.
a. Decrease in helper T lymphocytes 7. Anemic hypoxia occur in –
182. True about the features of JVP: Methemoglobulenemia.
a. ‘a’ wave occurs during ventricular systole 8. In cerebral circulation brain arteries do
b. ‘c’ wave occurs during atrial systole not anastomise - once entered in the
c. ‘a’ waves appear as giant brain subtance
Ans: C ( Read the full topic, Very Imp) 9. Phase 1 of transformation of drug
184. Increase in bleeding time. Probable metabolism – Oxidation.
diagnosis? 10. Standard deviation shows - Variability
a. von Willibrand disease of individual observation.
b. Christmas disease 11. Counseling in patients is - To help
c. Hemophilia A themselves.
d. Vitamin K deficiency 12. A young girl who is going to die and
Ans: A asks you “Am I going to die?” Doctor
185. Pregnant lady gets pain, swelling & response should be – “What your
redness in the left leg. Which drug should parents have told you?”
be prescribed to prevent the adverse 13. In whole wheat – Thiamine.
complications? 14. Cholesterol enriched diet – Egg
a. Aspirin 15. 1g protein gives energy – 4 kcalories
b. Clopidogrel 16. Organelle where protein combines with
c. Warfarin carbohydrates, packed and released - Golgi
d. Heparin complex.
Ans: D (IV Heparin) 17. Correct about DNA – Euchromatin is
186. A young boy has history of painful transcriptionally active.
burning micturition. Urine contains pus 18. ADPKD associated with – ) Cerebral
cells& few epithelial cells. What further haemorrhage
investigation will you advise? 19. Cause of delay in healing – Infections.
a. Urine culture 20. PaO2 decreased, PCO2 increased,
b. Urinalysis hydrogen ion increased; manifestation-
c. Serum unconjugated bilirubin Hypoventilation.
Ans: B 21. PCO2 31, HCO3 19, pH increased
187. During wound healing, collagen is (Metabolic alkalosis scenario) -
laid down. The process that goes on Hyperventilation
within the cell includes:
72

22. Person with tachycardia, and heat 47. Typical feature of falciparum - Black
intolerance with low level of TSH, on giving water fever.
TRH; level of TSH and thyroid hormones 48. Alcoholic patient with deranged LFTs;
increases. Diagnosis – Hyperthyroidism on biopsy – Mallory bodies.
with thyroid problem. 49. Councilman bodies seen in – Apoptosis.
23. Origin of oxytocin and ADH – 50. Natural self-defense against tumors –
Hypothalamus. Apoptosis.
24. Difference between systemic and 51. P53 gene absent results in - cell
pulmonary circulation - Low resistance in survival.
pulmonary circulation. 52. Pain mediator - Bradykinins.
25. Mean systemic filling pressure is 53. Metaplasia – Functional change in
regulated by - Venous return. cells.
26. Systolic pressure is directly related to 54. Female with infection of HPV, comes
which one of the following – Renin. after 2 years, Pap smear shows prominent
27. ADH responds to – Osmolarity. nucleoli and increased nucleus
28. Osmoreceptors – ADH. size – Dysplasia.
29. Right border of heart on X-ray also 55. Gas exchange occur - Simple squamous
visible a part of – SVC. epithelial layer.
30. In MI sensitive cardio marker – 56. Patient with granulomatous disease,
Tropinin T. biopsy done. Microscopic finding that
31. Diabetic nephropathy investigation - suggests TB – Epitheloid cells.
Urine albumin. 57. The spindle fibers will decrease in
32. Young boy with generalized edema and discharge of impulses when - Muscle
proteinuria - Lesion of basement contracts (vs) When efferent gamma
membrane. discharge occur -muscle contracts
33. Lesion of parasympathetic system 58. Diagnosis for leprosy, initial
affects mostly - GI muscles. investigation - Nasal scrapping.
34. Stress hormone of our body - ACTH. 59. Benign neoplasm – Adenoma.
35. S2 sound heard on - Closure of aortic 60. 3 germ layers tumor – Teratoma.
and pulmonary valve. 61. When adrenalin release from medulla,
36. A patient with history hemorrhage causes vasodilation by acting on - Beta 2
(trauma) receives a bag stored for 2 weeks adrenergic receptors.
mainly contains – RBCs. 62. Increased GFR and increased plasma
37. Due to inspiration – Decreased negative flow occur due to - Dilation of afferent
intrapleural pressure. arteriole.
38. Important buffer of blood - HCO3- 63. Charateristic of cerebellar lesion -
39. Max increase in ECF due to infusion of - Dymetria
Hypertonic NaCl. 64. Emax of a drug depends on – Efficacy
40. Auscultation of tricuspid valve best 65. Study in which every person of a
heard at – Right lower end of the body of population has equal chances of being
sternum. selected – Random sampling.
41. GVE vagus nerve for preganglionic fiber 66. Amniocenthesis is done - After 14th
arises from – Dorsal nucleus. weeks.
42. Thorn prick in left lower limb caused 67. 1st response against acute inflammation
abscess - Staph aureus. in tissue – Macrophages.
43. Diabetic female after abdominal surgery; 68. In dark granules containing cells; IgE
dyspnea and cough - Pulmonary embolism. attaches to – Basophils.
44. MCC of pulmonary embolism – DVT. 69. Opsonization - C3b.
45. Typhoid fever 1st week test – Blood 70. Exudate - more than 3g of proteins.
culture 71. About active transport of drug all are
46. Typhoid fever 2nd week test – Blood true except - All drugs pass via active
culture and Widal test. transport.
73

72. Pulmonary artery supply to – Alveoli. 96. Protrusion of mandible – Lateral


73. Muscles of back innervated by - Dorsal pterygoid
rami. 97. peroxisome made from --------------->
74. In young boy dyspnea produced on lying SER
- Retrosternal goiter. 98 clot retraction is mediated by --------------
75. Most important cause of bronchogenic ---> Thrombosthenin
cancer – Smoking. PAPER II MEDICINE & ALLIED
76. Edema caused by - Increased
hydrostatic pressure. 10 NOVEMBER 2016 EVENING
77. Edema caused by - Lymphatic 1. Acute warfarin toxicity immediately
blockage. treated with –
78. BP 210/180mmHg and creatinine 8% a. FFP
damaged part – Juxtaglomerular b.Vit k
apparatus. Ans A
79. Female with blood group A, have 2 2. Patient unable to supinate forearm,
children one with O and other with AB, injury to:
blood group of father is –B A.Radial nerve
80. Genetically true hermaphrodite – B. Median Nerve,
XX/XY. C. Musculoskletal and Radial nerve
81. DNA replication occur in – Interphase. D. Ulnar Nerve
82. Glycogenolysis caused by deficiency of Ans c
which hormone - Insulin 3. Thumb plus Deltoid Plus Bicep tendon
83. Investigation for liver amoebic abscess – affected, Nerve root:
Serology. a. C4
84. Adult female with breast atrophy due to b. C5
dec. in - Estrogen and progesterone c. C5,6
85. Child with yellow sclera and coloured d. C6
urine, best investigation - Bilirubin & ALT ans c
86. Patient with all symptoms of cushing 4. Back Muscles Supplied By
with increased ACTH cause- Cushing A Dorsal primary Rami
Disease B ventral primary Rami
87.Typical symptom of food intoxication by Ans a
Clostridium Botulinum - Flaccid paralysis 5. During Inspiration, there is decerease
88. If left circumflex artery occluded - in
Infarction of left atrium and left ventricle. A arterial Plus volume ,
89. MCC of multiple fractures in adult – B. Heart rate ,
Osteoporosis. C- Negative Intrapleural pressure
90. Patient with fracture of many bones and Ans c
low BP immediate treatment - Volume 6 Woman with respiratory symptoms that
replacement. improve on her vacations,
91. Most common fracture of long bone - bronchoplumonary biopsy shows increase
Tibia. in lymphocytes, some esosinophils and
92. Collagen fibers –lightly stained with nuetrophils, also allergic alveolitis and
Eosin some hilar nodes, what will be the history
93. Gamma efferent supply to – Intrafusal findings
muscle spindles.. a. she crack cocain
94. Isotonic and isometric contraction b. She has pigeons/parrots in basement
difference is that isotonic contraction – c. tobacco smoke
consumes more phosphate bond. d. house has lead composition
95. Autonomic nervous system – ans b
parasympathetic increase salivary secretion. 7 Woman goes to Sunlight, Malar rash
appears with proteinuria, hematuria (
74

SLE case), what other finding you will see B. Sural Nerve
in this patient: C. Common peroneal Nerve
A. glomerulonephritis D. Obturator Nerve
b. Pericarditis Ans C
C. Pulmonary fibrosis 15. Thoracic duct passes through
Ans A A Aortic Opening in diaphragm
8. Creatinine Clearance when urinary B inferior vena caval opening
[]=196mg/ml , Urine flow rate = 1ml/min C esophageal opening
& Plasma [] = 1.4mg/ml Ans a
A 140 16. The slow growing Tumor:
B 1.96 A. papillary ca
C 125 B. Medullary ca
Ans a C. Follicular ca
9 MAO inhobitors cause fatal excitation Ans a
with which Opioid : 17. Pathogenesis of Diphtheria
A. morphine a Exotoxin
B. Pethidine b enterotoxin
C. fentanyl , c endotoxemia
D. sufentanyl ans a
E. Alfentanyl 18. Reinfection with what causes Fever
Ans b and Hemorrhagic manifestations:
10 About fetal Heart: A. CMV
A. Foramen ovale blood regurgitate from B. Meseals
left atrium to right atrium C. Dengue
B. Oxygenated blood comes from Arteries D. Mumps
like that Ans c
C. Majority blood through truncus arteriosus 19. Only helminth that spreads through
from pulmonary trunk to aorta Mosquito Bite
D. large Muscular and small membranous a Filariasis
interventricular septum b ascariasis
Ans d c ochronosis
11 A young Boy with dyspnea on lying ans a
down 20 .Ingestion Eggs of what Causes
A Retrosternal goiter Cystecercosis
B multinodular goiter A. Taenia Solium
C colloid goiter B. T. Saginata
Ans a Ans a
12 How to differentiate between 21 IV Drug Abuser :
obstructive lung disease and restrictive A. Infective endocarditis
lung disease B. Limb sacks Endocarditis
A. Fev1/FVC Ans a
B PFFR 22. T4 in the blood Mostly binds with
C FEv which plasma protien
Ans a A TBG
13 Anterior Intercostal arteries are B thyroglobulin
Branches of: Ans a
A. Posterior thoracic arteries 23. Merkel Diverticulum
B. internal thoracic arteries Ans 2 feet from ileocecal Valve
Ans B
14 In anesthesized patient most common 24. ALT raised with fever and malaise
nerve damage: A Viral Hepatitis
A. femoral Nerve B alcoholic hepatitis
75

Ans a c type 4
25. Bile salts Absorb in: ans a
A. Ileum 34. Acute treatment of acture severe
B. Stomach Ulcerative colitis:
C. Jejunum A. Methotrexate
D. Ascending Colon B. Sulphasalazine C.
Ans A c IV Hydrocortisone
26. Drug , 1st order reaction like that D. Azathioprine
A. Oral Ans c
B. IV 35. Trochlear Nerve damage will cause:
C. I.M A. Loss of upward gaze and adduction
Ans a B. Loss of downward gaze and abduction
27. FRC : c. Loss of downward gaze and adduction
A. ERV + RV d. loss of lateral gaze
B. Cant be measured by spirometer ans b
Ans a 36. Most effective disease modifying agent
28. Not secreted from zona Reticularis in Rheumatic Arthritis
A. cortisol A. Azathoprine
B. Aldosterone B. Methotrexate
C. testosterone C. Cyclophosphamide
D. Cortisone Ans B
E. Androtendione 37. A boy with respiratory and GIT
Ans a infections with normal lymphocytes but
29. A diagnosis of Complete hyadatiform decrease or absent plasma cells:
mole is done what further to know more: A. Isolated IgA Def
A. triploid like 2 ovum 1 sperm B. Common variable immunodeficiency
B moderate HCG c. DeGorge
C genotype. 46xx parental origin ans b
Ans b 38. Patient works in a mine with all
30. Mid-leuteal Hormone increase symptoms of TB :
A. LH A Asbestosis
B. FSH
C. estrogen B. Silicosis
D. Progesterone Ans c
E. Estrogen + progesterone 39. Brunner Glands present in
Ans d A submucosa of Duedenum
31. Hypotension due to Benzodiazepenes B ileum
with given in Ans a
A. Diabetic 40. H2 Blocker is
B. Hypovolemia A Cimetidine
C. Old age > 65 B metronidazole
D. Pregnancy C carbimazole
Ans b Ans a
32. Afferent of Carotid Sinus: 41. Rx of Nausea and Vomiting:
A. Vagus Nerve A. H1 Blockers Only
B. Glosopharyngeal Nerve B. Steroides
C. acessay Nerve C. Metochlorpomide is less effective in
Ans b cytotoxic induced emesis as compared to
33. Hypersensitivity reaction in hydrops Ondasterone
fetals Ans c
a type 2 44. Chenoxycholic acid decrease turnover
b type 3 in:
76

A. Cholesteryamine ans c
B. ileal resection 54. Zollinger Ellison Syndrome :
Ans b A. Dec. gastrin
45. Blood Transfusion : B. increase Insulin
A. Hep C C. Peptic ulcer disease
B. HIV Ans C
c Hep B 55. Truncal vagotomy will cause:
ans b A. Gastrin secretion augmented
46. Abscess of tracheal area and deep B. HCl Secretion augmented
investing layer drains into: C. Delay gastric emptying
A. Retropharyngeal space Ans C
B. In thorax cavity anterior to precordium 56. Parasympathetic Action of vagus on
C. at mid of sternal manubrium SA node due to :
Ans a A. Potassium
47. Female 16 week pregnant with Hb 8, B. Calcium
MCH & MCV Low, HbA=96% C. Sodium
HbA2=4%: D. Potassium and Sodium
A. Iron deficiency anemia Ans a
B. Sideroblastic Anemia 57. “ A” Wave
C. Sicklemia A Atrial Depolarization
D. Thalasemic Trait Ans A
Ana d 56. Postural Tone of antigravity Muscles
48 QRS complex is maintained by constant stimulus by:
A Ventricular Depolarization A. Cerebellum Purkinji fibers
B ventricular repplarization B. Cortocospinal fibers
Ans a C. Lateral vestibular fibers
49. Feeding Centre Located In D. Fibers of Red Nucleaus
: A. Temporal Ans d
B Lateral hypothalamus 57. Oral propanolol contraindicated In
Ans b : A. Asthma
50. A patient with Pleural Effusion on B. HTN
standing u will see effusion in: Ans a
A costodiaphragmatic recess 56. 2 year Boy from AZAD kashmir with
B. Cardiac Notch failure to thrive for 6 month comes with
C. Oblique fissure Hepatosplenomegaly and pallor, plus
D. Horizental Fissure pancytopenia and ESR 80, dx
Ans a A. Visceral Lesihmaniasis
51 Pernicious Anemia dx : B. Gaucher disease
A antismith antibodies C. A.L.L
B. Antinuclear antibodies Ans a
C. Anti intrinsic factor antibodies 57. Structure that connects Hipocampus
Ans c to hypothalamus :
52. Palmar Arch Innervation : A. Fornix
A. Median Nerve B. Amygdala
B. Deep branch radial c. Sria Terminalis
C. Deep branch ulnar . D. Superachiasmatic
D Superficial Ulnar Ans a
Ans c 58 Patient with HTN preoperative drug
53. Surfacant : decrease BP rapidly
A. Secreted after 36th week A. Hyralazine
B. Colesterol B. Metoprolol
C Dec. surface tension of water in alveoli C. Glyverl Trinitrate
77

D. Nicardipin 67 Patient with problem in posterior


Ans a tongue taste and in elevation of shoulders,
59. Digoxin effect increased by: these affected nerves pass through:
A. K _ Loosing diuretics A. foramen rotundum
B. Hypocalcemia B. formen ovale
Ans a C. foramen spinosum
60. V/ Q Increase D. Jugular Foramen
: A. inadequate Ventilation Ans D (hint acessary nerve is damaged)
B. Increase dead space 68. A 32 year old patient with fever 100
C. Change in composition of alveolar gas fh, dry cough, joint pain WBCs 7.5 with
Ans c 70% neutrophils, cause :
61. A obstructed labour will damage in A. Strep. Pneumoniae
Pelvic Diaphragm structure: B. Stap. Aureus
A. Lavator ani C. Mycoplasma Pneumoniae
B Sphincter urethae Ans c
C anal sphincter 69. small saphenous vein runs along with
Ans a :
62. Hb synthesis is active in A. Saphenous Nerve
A. prerthroblast B. Radial Nerve
B. early Normoblast C. Sural nerve
C. Intermediate Normoblast D. Common peroneal Nerve
D. late Normoblast Ans c
E. reticylocyte 70. Regarding Ear and Hearing:
Ans d A. High frequency waves/ sounds in apical
63. Hormone have short half life in blood portion of choclea like that
A. Aldosterone B. Stapedial foot at oval membrane
B. LH C. Pressure in ear affects sound
C. Fsh Ans b
D. Testosterone 71 Bladder carcinoma in workers exposed
E. GnRH to:
Ans e A. zinc
64. Regarding testosterone : B. Niclke
A. regulated by FSH C. Arylamine
B. has receptors on membrane D silicon
C. Decend testis inlate fetal life Ans c
D abundentaly secreted by adrenal cortex 72. Initial marker of MI
Ans c : A. CKMb
65 cortical oocyte release : B. Myoglobin
A. Lh C LDH
B. FSH Ans b
C. Progesterone 73. S2 sound heard
D. Estrogen a Closure of aortic and pulmonary valve
E. Act as lysosomal vesicle b closure of mitral and tricuspid valve
Ans e ans a
66. Cornea Innervation 74. GVE carried in vagus nerve for
: A. Optic Nerve preganglionic fibers arise from
B. Abducen Nerve A dorsal nucleus
. C. Occulmotor Nerve B solitary nucleus
D. Tochlear Nerve Ans A
E. Trigeminal Nerve 75. Old RBCs are removed in:
Ans e A. Capillaries and red pupl
B. capillaries and splenic cord
78

c. Capillaries and sinusides 86. Dosral Coulmn fibers of lower body


d. Cord and sinusides related to upper body at cervical level
ans a A. Medial to lateral
76 Release of Neurotransmittor due to B. Lateral to media
A Influx of calcium in presynaptic fibers C. Dosral to ventral
Ans a D. Rsotral to caudal
77. Half Life of Kanamycin in perilymph Ans a
A 2 hours 87. About Climbing Fiber
B 4 hours A Inferior olivary Nucelus
C 30mint, ANS A
D 12hours, 88 Rheumatoid arthritis in pregnancy
E 20hours A MR
Ans b B MS
78. Digoxin act by C pulmonary regurgitation
A block Na/Ca exchanger Ans B
B increase intracellular ca+ 90 Bile salts absorbe from
Ans a A terminal iieum
79. A patient with low hb with B colon
pancytopenia investigation C duodenum
A Bone marrow study Ans (a)
B cbc 91; Concentrated solution of quinidine
Ans a applied on tongue as compared to distild
80. Absent P wave, pacemaker is in water sensation via
A AV node A trigeminal nucleus
B purkinje fibers B VPM of thalamus
Ans a Ans B (controversia Bcql)
81. A patient with Pneumonia cell present
A Neutrophils
Ans A
82. G6Pd
A Self Limiting
B autosomal dominant
Ans a
83. Systemic Fungi treatment
A Amphotericin B
B ketoconazole
Ans a
84. A patient is on radiotherapy after
that DNA damage A gene which arrest in
G1 phase progression defective
unregulated growth occurs due to which
A. K-ras
B. n-myc
c. P53
D. Bcl-2
Ans d
85. In Nucelus
A Bar-body is inactive x chromosome in
male
B . Hematoxylin stains the nucleus pruple or
blue
Ans b
79

SURGERY PAPERS 2016


SEPTEMBER, OCTOBER, NOVEMBER
80

5TH SEPTEMBER SURGERY 20. Conduction aphasia = arcuate


fasciculus.
MORNING 21. Delayed language issue, can speak
1. 1/3 of total body water= but not propulsive speech= lesion of
extracellular fluid. angular gyrus.
2. Which hormone will be less after 22. Apoptosis show = councilman
drinking water =antidiuretic bodies.
hormone. 23. Irreversible injury sign-=
3. Phospholipase c is activated by contraction band necrosis.
which hormone = antidiuretic 24. Brain perfusion depends on =Pco2.
hormone (action on blood vessels by 25. Middle meningeal artery = foramen
Gq proteins and causes spinosum.
vasoconstriction. 26. Labyrinthine artery is branch of
4. Maximum blood pressure in =renal =anterior inferior cerebellar
arteries. artery.
5. Basic drug binds with alpha- 27. Association of adult polycystic
glycoprotein. kidney disease is with = cerebral
6. Left shift of oxygen saturation curve hemorrhage.
= HbF 28. A patient came in night 2o’clock
7. Maximum potassium shift from with chest pain from last 4 hours all
intracellular to extra cellular baseline was normal what to do
compartment = strenuous exercise. next= ckmb.
8. C nerve fibers = carry temperature. 29. Middle wall , roof formed by
9. All preganglionic fibers = carry b =tegmen tympani.
fibers. 30. Position of vocal cords after bilateral
10. Esophagus starts at level of = cricoid recurrent laryngeal nerve damage=
cartilage. Paramedian
11. Nerve supply of posterior 1/3 of 31. Muscles not supplied by bilateral
tongue = 9th cranial nerve. recurrent laryngeal nerve =
12. Thyroid lymph drainage into = deep cricothyroid muscle.
cervical lymph nodes. 32. Contents of posterior triangle
13. Dislocation of temporo mandibular =subclavian artery and 3trunks of
joint = anteriorly. brachial plexus.
14. Glucose is main and only source of 33. Muscles of anxiety and sorrow that
energy for = RBC if asked organ pulls the angle of mouth downwards.
than brain if asked cell or as a = platysma muscle.
whole glucose is main and only 34. Contents of internal auditory canal =
source of energy so its rbc and if its facial nerve and vestibulo cochlear
not in option click neuron. nerve.
15. Extra embryonic coelom forms from 35. Atropine cause =dry mouth.
=hypoblast. 36. Subarachnoid space ends at =S2.
16. Main parasympathetic supply to 37. Upper motor neuron type of lesion
=parotid gland. Babinski positive = damage to
17. Right sided tongue atrophy and medullar pyramid.
wrinkling =12th cranial nerve. 38. Clostridium botulism= Respiratory
18. Main myelinating cells in CNS are muscle failure.
=oligodendrocytes. 39. At c7 level = Cervico thoracic
19. Oxytocin and antidiuretic hormone ganglia located.
originate in =hypothalamus. 40. Anemia causes= Atrophic glossitis.
41. Hyper magnesemia causes =
Neuromuscular junction problem.
81

42. Patient with bleeding time increase 64. Bacteriostatic Tb drug =


and anemia= Platelet function Ethambutol
defect. 65. Prostaglandins inhibit by =
43. Case of achondroplasia = Corticosteroids.
Autosomal dominant. 66. Left adrenal vein drain into= Left
44. Most common type of genetic disease renal vein.
= Autosomal dominant. 67. Case of renal failure after post-
45. Long term steroid use = partum hemorrhage damage in
Osteoporosis and fractures. kidney= Proximal convoluted
46. GMP or GTP related hormone = tubules.
Atrial natriuretic peptide. 68. More in endolymph in ear =
47. Metastatic calcification of kidney = Potassium.
Hyperparathyroidism. 69. Nerve supply to infra hyoid muscles
48. Nerve supply to maxillary sinus = = Ans:a cervicalis.
Superior alveolar branch of 70. Inferior thyroid artery = Related to
maxillary division of trigeminal. recurrent laryngeal nerve.
49. Nerve supply to pinna= Greater 71. 2nd pharyngeal arch derivative =
auricular nerve. Stlyohyoid ligament.
50. Which nerve accompany superficial 72. Nerve supply to fibrous pericardium
temporal artery = Auriculotemporal = Phrenic nerve.
nerve. 73. Commonest presentation of vitamin a
51. Patient with acute asthma 1st deficiency = Night blindness.
treatment = Inhalational 74. Acrosome of sperm formed by =
salbutamol. Golgi complex.
52. Case of pneumonia after bone 75. Simian crease= Trisomy 21.
marrow trAns:plant inclusion bodies 76. Goblet cells replaced by clara cells in
in cell = Cytomegalovirus. = Terminal bronchioles.
53. Tumor that stains with vementin = 77. Case of aspergillous eosinophilia in
Sarcoma. blood and bronchial secretion =
54. Septic meningitis caused by = Noninvasive aspergillious.
Streptococcus. 78. Glycocalyx = Carbohydrate moiety.
55. 4ml of 2% lidocaine = 80 mg of 79. Resistance in inspiration = 2/3 by
lidocaine. small bronchioles and bronchi (
56. Patient on table without clothes loss intermediate size bronchioles was not
of heat by = Radiation & in option)
Conduction. 80. Supply of esophagus = Inferior
57. Mother gave birth to a child having thyroid artery.
congenital cataract = Rubella 81. Valve less structure = Superior vena
infection during pregnancy. cava.
58. Grave disease scenario = Anti 82. Trachea = little shift to right side is
thyroid stimulating antibodies. normal.
59. Animal as reservoir = Influenza. 83. Root of lung relation = Descending
60. Action of propylthiouracil stop aorta.
oxidation of= Iodide. 84. Pancreatic secretion increase and
61. Kid with edema problem in kidney = increase calcium influx = Hormone.
Basement membrane damage. 85. Enzymes needed to activate
62. H+ ions more in urine = Secretes trypsinogen = Enterokinase.
plasma secretion. 86. Maxillary sinus opens in = Middle
63. Carbon laden macrophages = meatus.
Anthracosis. 87. Case of COPD but no history of
smoking or any risk factor fev1
82

decrease neutrophils will clear which 109. type of necrosis in brain =


particles = Carbon. liquefactive necrosis
88. Heme binds in blood with = 110. function of ICAM and VCAM =
Hepatoglobin. neutrophils adhesions
89. Cardiac reserve decreases in 111. viruses effect cells by = altering
=Athletes. protein synthesis
90. Systemic sclerosis = Excessive 112. metaplasia = replacement of one
fibrosis. type of epithelium with another
91. Pleural effusion case from where to type of normal
take sample of fluid = Upper border epithelium.
of the rib at area of highest 113. papillary carcinoma of thyroid
dullness. spread by = lymphatic invasion
92. H2o2 is present in = Peroxisomes. 114. atrial fibrillation = many p waves
93. Wound healing strength given by = followed by QRS
Collagen type 1 . 115. GFR will be more when protein in
94. Commonest etiology of genetic blood is= less
disease = Multifactorial. 116. which tissue does not regenerate =
95. Difference between malignant and lens
benign tumor = Metastasis. 117. labor working in heat sodium is 125
96. On histology of malignant tumor = lost his consciousness cause =
Pleomorphisum. excessive
97. Cause of pyogenic peritonitis = sweating
Bacteriods. 118. ionizes calcium = parathormone
98. Mycobacterium is = Acid fast secretion regulated
bacilli. 119. somatostatin inhibit insulin
99. LMN type of lesion = Flaccid secretion = paracrine way
paralysis. 120. pregnant lady with jaundice = ggt
100. Caseof Tb granuloma was 121. corrosive intake = squamous cell
given. Best way to confirm the carcinoma of esophagus
diagnosis = Acid fast 122. iron stored in tissue in the
bacilli. parenchyma as = Ferritin.
101. Case of two years child with fever 123. red blood cell a and b antigens =
and anemia Hb 5 mcv 107. How to glycoproteins
confirm 124. hydrocarbons cause = bronchial
diagnosis = vit B12 carcinoma
102. Angular cheliosis in patient with 125. hyperplastic bone marrow =
renal trAns:plant cause in = candida leukemia
albicAns: 126. polycythemia vera = neoplastic
103. A 10 yr. old kid with neck swelling proliferation in bone marrow
solid mass present = lymphoma 127. trapezoid body = role in hearing
104. Beetle nut history girl 20 years
trismus = sub mucosal fibrosis Surgery paper 2016 – 5th Sep
105. site of antigen binding in antibody =
hyper variable regions on l and h 2016 EVENING
chains 1. Lymphatic from the lateral quadrant
106. in pregnant female which will be of the breast mainly drain into the
increased =TIBC A. Inferior deep cervical lymph node
107. immunity effected in = HIV B. Inter pectoral lymph node
infected helper t cells C. Pectoral (anterior) lymph node
108. posterior triangle is supplied by = c2 D. Retrosternal lymph node
c3. E. Supra clavicular lymph node
83

Ans:: cc
2. A student wanted to study
relationship between increase in
temperature and duration of surgery,
to evaluate results which statistical
test he should apply?
A. Student t test
B. Anova test
C. Mc witneys
D. Chi squared
E. Regression Analysis
Ans:: ee
3. Action of Gluteus Medius & Minimus
A. Abduct the hip joint & Rotate the
thigh medially
B. Adduct the hip joint and rotate the
thigh laterally
C. extend the hip joint
D. flex the hip joint
E. helps in sitting in squatting position
Ans:: aa
84

4. Heavy smoker complains of dyspnea c. Anterior Interventricular


chest pain cough hemoptysis since 3 d. LCX
months. Bronchoscopy reveals fungating e. Diagonal
mass obstructing medium sized bronchus. Ans:: aa
Which type of tumor? 11. Which of the following is Ectoparasite
A. squamous cell carcinoma a. Lice
B. small cell carcinoma b. Fleas
C. large cell carcinoma c. Ticks
D. Bronchiolar carcinoma d. Bed Bugs
Ans:: aaa Ans:: aa
5. Skeletal Muscles are attached to Bones 12. Before final composition of
by: Concentrated Urine, which part of
a. Anchoring filaments Tubular fluid contains least osmolar
b. Fascia solution?
c. Ligaments a. PCT
d. Tendons b. Collecting Duct
Ans:: dd c. Ascending loop of Henle
6. Sugar in Urine will be detected if its d. Descending loop of henle.
concentration in blood becomes e. Distal Convoluted Tubule.
a. 250mg/dl Ans:: eee
b. 200mg/dl 13. Best Term to describe to describe a
c.180 mg/dl condition when a cell gets extra number
d. 150 mg/dl of chromosome other than haploid
e. 375mg/dl a. Non disjunction
Ans:: bb (refrence ganong) b. Trisomy
7. Dorsal Nucleus of Vagus Nerve lies at c. Euploid
a. Pons Lower Lower Part d. Aneuploid
b. Medulla e.Polyploidy
c. Pons Upper part Ans:: bb
d. Midbrain 14. Alveolar Surface is kept dry because
Ans:: bb of
8. 2% solution of 4ml lidocaine will a. Tight Junctions
contain How much Lidocaine b. Surfactant
a. 80 mg c. Negative Interstitial Pressure
b. 160 mg d. Macrophages
C. 8 gm. Ans:: cc
D. 40 mg 15. Histoplasmosis affects which system
Ans:: aa mainly
9. 0.85% Nacl Solution contains how a. Respiratory
much NACL b. Reticuloendothelial
a. 85mg/100ml c. Cardiac System
b. 85 mg / 1000 ml d. Nervous System
c. 850 mg/100ml Ans:: bb
d. 8.5 mg/100 ml 16. A female delivered a baby and she was
Ans:: cc started on warfarin as she developed dvt.
10. A person with Chest pain having ST The next day she has a purplish patch on
Elevation in Lead V4 only. which artery her right thigh what can be the cause
involved A. lupus anticoagulant
a. LAD b. warfarin overdose
b. RCA c.protein c deficiency
d.proteins deficiency
85

Ans:: cc 25. In VIT K deficiency, which factor will


17. Grade 3 Tumor, % of undifferentiated be depleted first and foremost
cells is: a. Factor 7
a. 25 % b. Factor IX
b. 50-75% c. Factor 10
c. >75% d. Protein C
Ans:: bb e. Protein S
18. In a Pre-OP patient after inducing Ans:: aa
General Anesthesia, heat will not be if asked factor then its factor 8 and if
produced due to: asked protein then its protein c)
a. Muscle tone is lost 26. SKIN GRAFT will be most successful
b. Na/K Pump Blocked in which of the following conditions
Ans:: aa a. Excessive Burns
19. Initial symptoms of Bupivacaine b. Agammaglobenemia
Overdose c. Terminal cancer patient
a. Arrhythmia d. Liver failure
b. Unconsciousness Ans:: bb
c. ringing of ears 27. Which cancer invades large blood
Ans:: cc vessels?
20. Cytoskeleton communicates with a. Clear cell ca of Lungs
Extracellular environment through b. Clear cell ca of Kidney
a. Intermediate Filament c. Bronchogenic Ca
b. Integrins d. TrAns:itional cell Ca
c. Cadherins Ans:: bb
Ans:: bb 28. In Old what happens
21. True about Glycocalyx: A. Decrease Vital Capacity
a. Carrier Protien B. Increase Cardiac Output
b. Carbohydrate Moiety Ans:: aa
c. Lipid Membrane 29. Neutral absorption of sodium occurs
Ans:: bb at which part
22. Acoustic Neuroma is a tumor of a. Proximal convuated tubule
a. Cochlear part of CN 8 b. Thick ascending limb of loop of hence
b. Vestibular part of CN 8 c. Thick descending limb
c. Facial Nerve d. Cortical collecting duct
d. Trigeminal nerve e. Distal convulated tubule
Ans:: bb Ans:: aa
23. In young age, Limb growth will be 30. Breast Atrophy in elderly females,
affected if fracture passes through Major cause
a. Epiphysial Plate a. Estrogen & Progesterone deficiency
b. Epiphysial Line B. estrogen deficiency
d. Diaphysis Ans:: bb
d. Metaphysis 31. Major cause of fatty liver in our
Ans:; aa society
24. Cause of Death after Liver a. Hep B & Hep C
trAns:plantation due to which pathogen b. alcohol
a. CMV c. increase fat intake
b. HBV Ans:: aa
c. HDV
d. HCV 32. Unable to evert and dorsiflex, which
e. HEV structure injured
Ans::aa a. Common peroneal nerve
86

b. deep peroneal nerve c. External Iliac


Ans:: aa Ans:: aa if the option doesnot contains both
33. Scrotal Swelling after injury, contents then select Internal Iliac nodes.
of swelling extends up to abdomen but not 40. Dec in ESR due to
thigh, most common cause of this swelling a. Increase Fibrinogen
in scrotum b. Increase Albumin
a. Injury to Bulbar Urethra Ans:: bb
b. injury to penile urethra 41. Recurrent Laryngeal nerve injured on
c. membranous urethra both sides, which muscle still intact
d. neck of urethra a.thyrohyoid
Ans:: aaa ( urine extravasation rembered b.postcricio thyroid
by NMEMONIC c.cricothyroid
BUS : Bulbar urethra rupture  Scrotum d.cricoarytenoid
MUD: Memranous urethra rupture DEEp Ans:; cc
pereaneal pouch 42. Gastric Lymphoma due to
PUS : Penile urethra  superficial a. H Pylori
pereneal pouch ) b. CMV
34. Mediolateral Episiotomy, which part Ans::aa
at risk of being getting injured 43. The features of BCC
a. External Anal sphincter a. metastasis
b. Bulbospongiosis b. benign tumor
c. levator ani muscle C. borders are undermined
d. perineal body d. Locally invasive.
Ans:: bb Ans:: dd
35. Axillary Artery 44. Posterior wall of Rectus sheath b/w
A it lies post to pectoralis minor muscle Symphisis pubis & Abdomen
B lateral chord lies posterior to it a. Arcuate line
C starts at the lateral border of pectoralis b. Linea semilunaris
minor c. Linea semicircularis
D all the chords are lateral to it Ans:: aa
Ans:: aa 45. Bundle of His Blood supply
36. Ecchymosis and bleeding history in a. RCA
fisherman b. LCA
a. Vit C Deficiency c. LAD
b.Vit b12 def d. Circumflex
c. Folic acid deficiency Ans:: cc Reference : snell
d. Iron deficiency 46. Head of Femur Blood supply
Ans:: aa a. Medial & Lateral Circumflex Arteries .
37. Chromosome 9:22 abnormality B . Retinacular.
a. CML Ans:: B> A
b. CLL 47. Major Cause of Epiphyseal Tumor
c. AML a. Osteoblastoma
d. ALL b. Osteosarcoma
Ans:: aaa c. Osteoid Osteoma
38. Function of Vitamin A d. Ewing Sarcoma
a. Forms Rhodopsin Ans:: aa
b. Not required for night vision . 48. DIC is initiated by
Ans:: aa a. Tissue Thromboplastin
39. Lymph drainage in Cervical CA b. fibrinogen
a. External & Internal Illiac lymph nodes Ans:: aa
b. Internal ilaic
87

49. Seventy years old male develops 56. Pregnant lady with raised SGPT,
sudden loss of power in both right upper history of visit to Remote village, which
n lower limb with angle of mouth hepatitis
deviated to the left during talking. The a. Hep A
most likely site of lesion is in b. Hep E
a.Internal capsule c. Hep B
b.pons d .Hep C
c.medullaoblongata Ans:: bb
d.midbrain 57. Which Ca will not metastasize
Ans:: aa a. Basal Cell Ca
50. Sciatic nerve injured which structure b. Squamous cell ca
will still flex the knee Ans:: aa
a. Short head of biceps femoris 58. Isthmus of trachea lies in front of
b. long head of bicep femoris A. 2nd 3rd and 4th Tracheal Ring
c. popliteus B. Rings 3, 4
Ans:: aa C. Rings 2, 3
51. Patient died of Stroke, which type of Ans:: aa
necrosis 59. 50 years old hypertensive patient
a. Liquificative necrosis presented with severe chest pain radiating
b. Caseous necrosis to back
c. coagulative necrosis suddenly collapsed On autopsy what can
Ans:: aa be seen
52. Median Umbilical Ligament is a. Medial Necrosis of Aorta
remnant of b.ischemic necrosis
a. Urachus c..atherosclerosis
b. umbilical artery Ans:: aa
c. umbilical vein 60. Abdominal Aorta
d. allantois a. Gives Renal Arteries at L2
Ans:: aa b. bifurcates at L5
53. Which structure will be damaged in c. bifurcates at T5
Lumbar Puncture d. lies at the right side of IVC
a. Ligamentum Flavum Ans:: aa
b. Posterior longitudinal ligament 61. Loss of Supination & Flexion after
c. Anterior longitudinal ligament Stab Injury to Inner Arm
d. Pia matter a. Musculocutaneous
Ans:: aa b. ulnar
54. Scenario with Vitamen K deficiency, c. radial artery
Vitamin K not absorbed, which factor d. axillary artery
decreased Ans:: aa
a. Prothrombin 62. Regarding Ciliary body
b. Protein C A. Pigment epithelium produces aqueous
c. Protein S humor.
d. Factor VIII b. Pigment epithelium of retina is
Ans:; aa continuous With pigment epithelium of
55. Low estrogen pills will cause ciliary body
a. Breast Ca C. Non pigment epithelium produces aq.
b. Lungs Ca Humor
c. Endometrial Ca D. Supplied by short ciliary arteries
d. Deep venous thrombosis Ans:: cc
e. Hepatic Adenoma 63. Minerals are least found in
Ans:: ee a. Pulses
88

b. Cereals c.parotid
c. Tubers Ans:: cc
d. Green Vegetables. 72. Regarding anal canal
Ans:: cc a.supplied by both superior n inferior rectal
64. Patient had previously Episodes of arteries
Dizziness and Diplopia, patient presented b.inferior rectal artery part of portal system
now in Acute Comatosed state, Cause : c.external anal sphincter is involuntary
a. Carotid Artery Thrombosis d.has two lateral curves
b. Basilar Artery Thrombosis e.sup inguinal lymph nodes drain the upper
c. Arachnoid Hemorrhage half of anal canal
Ans:: bb Ans:: aa
65. Which of the following lymphoid 73. Patient started on anti coagulant how
organ receives lymph via to monitor further
SUBCAPSULAR SINUS A. Aptt
a. Thymus B. Clotting time
b. spleen C. Pt
c. Lymph node Ans:: cc
Ans:: cc 74. Trachea commences at
66. True regarding rectum is : A. C4
a. lies in front of S1. B.C5
b. is 9 inches long C. C6
c. is biconvex wall in front of s2,3,4 D. C1
d. also supplied by median sacral Artery Ans::cc ( lower border of C6)
Ans:: dd 75. A female of reproductive age
67. Infection anterior to pretracheal presented with DVT. She gave history
fascia will spread to: Of taking some kind of pill what is the
a. Anterior Mediastinum cause of her condition
b. Superior mediAns:tinum a. Thrombo embolism
c. Middle Midiastinum b. Analgesics
Ans:: aa c..OCPs
68. Regarding abdominal angina which Ans:: cc
artery is involved 76. 45 years old man presented with chest
a..inferior mesenteric artery pain not related to respiration
b..aorta a. Myocardium
c..superior mesenteric artery b. Pericarditis
Ans:: cc c. chostochondritis
69. premature baby born with Ans:: aaa,,, if th pain aggravates eith
a..PDA changing posture its pericarditis,, if it is
b.patent foramen ovale related to respiration that feels pain with
c.right atrial pressure increases breathing then its costochondritis
Ans:: aa 77. Tongue is deviated towards right side ,
70. Which hormone causes fatty acid lesion is
synthesis and protein synthesis a. Right hypoglossal nerve
a.cortisol b. Left hypoglossal nerve
b.thyroid c. Rt. Vagus
c.glucagon d. Lt. vagus
d..insulin Ans:: aa
Ans:: dd 78. Deep part of the parotid fascia forms
71.Most common salivary gland tumor a. bucco pharyngeal membrane
a.salivary b.stylomandibular ligament
b.submandibular c.sheno mandibular ligament
89

d.stylo hyoid ligament 86. Bioavailibity of minerals is lowest in


Ans:: bb a.roots
79. first Lumbrical b.green leaves
a.bipennate c cereals
b.attached to proximal phalynx d pulses
c.origin near the radial side of first meta e.Tubers
carpal bone Ans:: ee
Ans:: cc 87. Drug drug interaction by
a. Both pharmacokinetics and
80. In a terminal resection of ileum 100 pharmacodynamics
cms of ileum was resected. What will be b. only pharmackokinetics
the out come? c. only pharmacodynamics
a.less absorption of bile salts Ans: : aa,, as they havnt mentioned whether
b.less absorption of bile acids the interaction is by the drug on the body or
c.more absorption of bile salts by the body on the drug so we select both.
d.increase sodium loss 88. Regarding sweat glands
Ans:: aa a. Cuboidal epithelium
81. Regarding inguinal canal b. Columnar epithelium
A. Absent in infants. Ans:: aa
B. Extends from anterior superior iliac 89. Bioavailibility is not affected by
spine to pubic tubercle. a.age
C. Roof formed by conjoined tendon b.liverdisease
Ans:: cc c sex/gender
82. Greatest clearance is of Ans:: cc
a. PAH 90. Dimercaprol
b. insulin a.increases clotting
c. Glucose b.decreases clotting
d. Creatinine c.decreases bleeding time
Ans:: aa D. Increase bleeding time
83. Regarding cardiac plexus Ans:: dd
a.formed by both symathetic n 91. For sodium neutralization
parasympathetic nerves a. k
b.thoracic splanchnic nerves are the main b. organic ions
nerves c. Cl
c.starts at trachea bifurcation d. mg
d.present at the base of heart e. hco3
e.anterior cardiac nerves are the main nerves Ans::cc
Ans:: aa 92. Amino acid containing sulphur
84. Baroreceptors a. Cysteine
a.hypotention b. Phenylalanine
b.respond to rapidly increasing arteriolar c. Leucine.
pressure Ans:: aa
c.respond to rapidly decreasing arteriolar 93. Sodium regulation is through
pressure A osmoreceptors
Ans:: cc refrence BRS phsiology b.sodium absorption
85. Insulin is inhibited by c.Adh
a. secretin d.Reninangiotensin
b. glycogen e.Aldosteron
c. beta agonists Ans:: aa
d. beta blockers 94. Circumflex artery supplies
Ans::dd a.anterior surface of heart
90

b.left atrium n ventricle 102. Eversion of foot leads to ligamentum


c.right atrium fractured
d.left atrium a.lateral ligament
e.left ventricle b.planataris ruptured
Ans:: bb c.Deltoid ligament
95. 41%haematocrit contains Ans:: cc
a.rbcs 103. A farmer while spraying suddenly
b.rbcs,wbcs,platelets got collapsed with salivation from his
c.platelets mouth and
Ans:: bb broncho spasm. What is the specific anti
96. Venous blood as compare to arterial dote
blood has a.atropine
a.low ph b.neostigmine
b.more packed cell volume cphysostigmine
c.increasepo2 d.Pralidoxime
d.increase ph. Ans:: dd
Ans:: bb 104. A person was walking bare feet
97. During stab injury suddenly stepped upon some sharp
a. Ipsilateral lung collapse n ipsilateral chest object. He quickly
wall spring out. removed his leg due to which reflex
b. ipsilateral lung collapse n contralateral a.flexor response
chest wall spring out b.mono synaptic reflex
c. contralateral chest collapse and c.multi synaptic reflex
contralateral chest wall spring out Ans:: cc
Ans:: aa 105. Opsonization due to interaction of
98. A person got lacerated injury on his a. FC portion of igG and C3b
elbow what will be the initial affect of b. C5a
blood c. C3b
a. haemostasis d. C3a
b. vasoconstriction Ans:: aa
Ans:: bb 106. Morphine is used in
99. during a one km running the venous a. Terminal cancer illness
return to the heart is by b. Pain of gastric ulcer
a.valves in the leg veins valves c. UTI
b.pumps in the legs Ans:: aa
c.skeletal muscles contraction 107. Inferior wall MI
Ans:: cc a. Right marginal
100. right atrial pressure is increased this b. RCA
will lead to c. LAD
a.inc cardiac output d. LCA
b.heart rate inc . Ans:: aa (cpsp demo)
c.inc contraction of right ventricle 108. Most appropriate about Middle
Ans:: aa meningeal Artery
101. Regarding occupational diseases a.pterion is the surface marking
increase risk of problems with exposure b.its branches form grooves on the under
to surface of skull
a.coalmines c. Involved most commonly in Subdural
b.asbestos Hematoma
c.silicons d.enters skull thru foramen spinosum .
d.textile industry Ans:: dd
Ans:: bb 109. vagotomy done for
91

a.decrease motility of gut 117. a child presented to opd with 104


b.decrease gastric acidity fever for the last 4 days. He has been
c.decrease pepsin passing cola
d. decrease gastric acidity n pepsin motility colored urine for the last one day. He has
of gut bee n on anti malarials too. His LFTs are
Ans:: dd deranged with increase unconjugated
110. pain of Gastric ulcer is due bilirubin. What is the most probable
a. Greater splanchnic nerve diagnosis
b.Lesser Splanchnic nerve a.drug induced jaundice
Ans:: aa b.black water fever
111. medial wall of ischiorectal fossa is c.paroxysmal nocturnal haemaglobinuria
formed by dhaemolytic uremic syndrome
a. Levator ani muscles e G6pd
b. external anal sphincter Ans:: ee
Ans:: bb 118. 46XX associated most probably with
both are correct refrence rj last which of the following conditions
112. Double membrane bounded a. Adrenogenital Syndrome
Organelle b. Mixed Gonadal dysgenisis.
a. Nucleolus c. Congenital Adrenal Hyperplasia
b. RER Ans:: aa
C. Mitochondria 119. Turnor syndrome with
d. ribosomes a.Autosomal dominant
Ans:: bb ( no option of mitochondria and b.Autosomal recessive
nucleus was present) c shows secondary infertility
113. Most radio sensitive tumor is d stunted growth is the main feature
a. seminoma Ans:: dd
b. glialglioma 120. ECF is increased by infusion of
c. craniopharyngioma A hypotonic saline
Ans:: aa B norma saline
114. Man is intermediate host in C hypertonic saline
A. Hydatid cyst d dextrose water
B. Beef tapeworm infestation Ans:: cc
c. trichomoniasis 121. Peri capsular sinuses in
d. filiriasis a.lymph node
e. hook worm b thymus
Ans:: aa c palatine tonsils
115. a young man fell from two storey Ans:: aa
building and now he has urinary 122. Weight bearing line passes through
incontinence. lesion is at a. Ischial tuberosities
a. s2s3s4 b. pubic crest
b. l2 l3 l4 c. ischial spines
c. s1,s2,s3 Ans:: aa
Ans:: aa 123. A 14 years old girl presented with
116. S2 differs from S1 by right iliac fossa pain. pain associated with
a..s2 has increase duration vomiting and was radiated to peri
b.s2 has decrease pitch umbilical region. Which segment is
c. s2 has increase frequency involved/or pain radiation due to
d s1 has increase pitch a. ilioinguinal nerve
e s1 has decrease duration b.iliohypogastric nerve
Ans:: cc c.t12level
d L2level
92

e T10 level C longitudinal layer


Ans:: ee D circular layer
124. microscopic slide of liver shows E lamina propria
portal vein hepatic artery and bile duct at Ans:: aa
periphery and central vein in the centre. 131. chronic hepatitis histology on
What is this called microscope shows
a.hepatic lobule A fibrosis
b portal acinus b neutrophills
c hepatic acinus (also called liver acinus) c lymphocytes
d Simple acinus d lymphokines
Ans:: cc Ans:: aa
125. EEG of a patient shows 132. sarcoma as compare to carcinoma
A increase frequency of waves in active a. spread to bones early
mind than in deep sleep b. in the form of clusters of cells
B is symmetrical in both hemispheres c. more sensitive to radiotherapy
C small amplitude of waves in intense d. Clumps/Clusters
thinking e.Aggressive
D determine intelligence Ans:: ee
Ans:: aa 133. Systemic antifungal is
126. what is produced in muscles a.Amphoterecin b.
A. phospholipids b.Ketokonazole
B. creatinine c.Nystatin
C. urea Ans:: aa
D. uric acid 134. Heparin inhibits
E. glycogen A organization of clot
Ans:: bb B retraction of cot
127. About sertoli cells C dissolution of clot
A produce testosterons D propagation of clot
B rapidly dividing cells Ans:: dd
C release androgen inhibiting factor 135. During pregnancy in which period
D break blood testis barrier teratogen affects fetus mostly
E reinkes crystals A 1 to 2weeks
Ans:: cc B 3 to 8 weeks
128. In Chronic renal failure what is least C 22 to 36weeks
likely to be presented? D last trimester
A .low ph E 8 to 12weeks
b . hypo phosphatemia Ans:: bb
c . enlargedparathyroids 136. muscle is prevented from tearing
d. metabolicacidosis under action of which receptor
e. hypocalcemia a.muscle spindles
Ans:: bb b GTO
129. On autopsy of CRF patient which c ruffinis
organ was hypertrophied d meissners
A. Para thyroid glands e phosphate bonds
b. adrenal glands Ans:: bb
c. thyroid glands 137. maximum sodium entry in nerve
d. pituitary gland fibre takes place at
Ans::aa A during depolarization
130. Strongest layer of small bowel B during repolarization
A submucosa C at nodes of Ranvier
B musoca
93

D as the impulse propagates along length of C carbon dioxide


nerve fibre D nitrogen
Ans:: cc E oxygen
138. The valve which has more propensity Ans:: cc
toward diseases is highest diffusion capacity across respiratory
a. Mitral Valve membrane is of carbon dioxide while
b. tricuspid valve affinity of carbon monoxide is greatest
c. pulmonary valve 145. Tumor suppresser gene
Ans:: aa a. p53
139. Non bacterial endocarditis due to b. ras
A rheumatic fever c. c-myc
B SLE Ans:: aa
C streptococcal glumerulonephritis 146. upper motor neuron lesion leads to
D thrombo embolism A decrease tendon reflexes
E neoplastic diseases B fasiculations
Ans:: ee C inc muscle tone
140. 23 years old boy came with mild D flaccid paralysis
jaundice. His hbsAg is negative, anti Hbc Ans:: cc
igM is Neg, anti Hbe positive. Whats the 147. which nerve passes through
current situation cavernous sinus
a Acute resolving phase A. abducent nerve
b acute hepatitis b. trigeminal nerve
c window period c. occulomotor nerve
d chronic hepatitis Ans:: aa
Ans:: aa 148. premalignant growth
141. Difference between CSF and cerebral A sebborhic keratosis
capillary is B compound nevus
a. increase ph C intradermal nevus
b carbohydrates D dysplastic nevus syndrome.
c increase pco2 e junctional nevus
d less proteins Ans:: dd
Ans:: dd 149. 35 years old girl has been using
142. Benign tumor among following is analgesic for backache. She presented
a. Leiomyoma with platelets in normal range.wbc 7000
b. Hamartoma neutropenia. Her hb was 9 with small
c.Choristoma immature cells in peripheral smear.
Ans:: aa Diagnosis
143. Hypotensive pt admitted in ward wd a acute leukemia
h/o dysuria and high grade fever. His b ITP
blood culture is positive for pseudomonas. c aplastic anemia
The disease is due to release of d drug induced
A. endotoxin Ans:: aa
B. exotoxin 150. Gastric emptying increases by
C proteases a.secretin
D. Toxic shock syndrome toxin. b.Gastrin
Ans:: aa c.Cck
144. Highest diffusing capacity across d.ach inhibition
respiratory membrane and in body fluids Ans:: bb
is 151. local edema cause
A helium a.Allergy
B carbon monooxide b. Flare
94

c. hyperresposiveness c Mucormycosis
Ans:: aa d hyperkalemia
152. Exogenous antigens acted by e blood glucose
interacting with Ans:: cc
a. MHC class II 159. A person got an accident and there
b. APC was severe bleeding from his body. He got
Ans:: aaa multiple trAns:fusios but Ecg showed
153. A lady presented with yellow eyes Changes after trAns:fusion due to
and she has got hepato megaly. Also A hypocalcemia
xanthomas B .hyperkalemia
present. What can be the cause in her C hypokalemia
A AMA D hypermagnesemia
B ANA E PH changes
C sclerosing cholangitis Ans:: bb
D lipidemia 160. Doctor patient relationship
Ans:: aa a. Active listening
154. Regarding b. Having sound medical knowledge
PseudoHYPERparathyroidism? c. Updating skills
A. because of Hypocalcemia d. good bedside manners
B. Because of PTHrP by various tumor Ans:: aaa
C. associated with Nephritic syndrome 161. after adrenelectomy taste preference
D. because of Vit D deficiency increases for
Ans:: bb A sweets
155. Right ovarian vein drains into b.Nacl
a. IVC c sour
b.Lt. adrenal vein Ans:: bb
Ans:: aa 162. 60% of ventricular filling is because
156. Regarding biceps brachi of
a. insertion on humerus A Rapid inflow
b. Arises from supraglenoid tubercle B slow inflow
c. pronation C Atrial systole
d. extention of elbow D slow ejection phase
Ans:: bb E iso volumetric relaxation
157. A 22 years old man came to Opd Ans:: aa
with ALT raised viral markers were 163. about follicular cancer of thyroid
negative. He has got some problem in the a.vascular invasion
eye and Immunological tests were b psammoma bodies
derranged. Whats the most useful c lymphatic spread
investigation for him d hyperchromatism
A liver biopsy Ans:: aa
B HbsAg 164. protein on Rbc membrane for
C HCV virology chloride exchange
D serum copper level a.spectrin
E Sr. cerulo plasmin level b. band 3
Ans:: ee c.glycocalyx
158. A diabetic patient presented with Ans:: bb
increasing degree of progressive illness 165. Regarding cholangio carcinoma
involving respiratory system. a. Chlinorcis sinensis
Gastrointestinal. What can be the cause b. Tinea saginata.
a histoplasma Ans:: aa
b acid base balance 166. primary syphilis initial diagnosis
95

a. mouth ulcer membrane will rupture and open


b.genital sore cranially as
c.tabes dorsalis a extrophy of bladder
Ans:: bbb b imperforate anus
167. Dead space does not decrease in c ectopic anal canal opening
a.shallow breathing d hypospadias
b.deep breathing Ans:: cc
c.lung diseases 174. Diptheria toxin will affect most
d.gravity lethally on which organ
e.tracheostomy a. Larynx
Ans:: aa b. Heart
168. chemical mediator of inflammation c. Kidney
a.histamine Ans:: bb
b.serotonine 175. Sparing of the lateral part of the
c.lymphokines thenar eminence with conspicuous
Ans:: aa ,,, prostaglandin is prefered if wasting of other small muscles of the
avaialble in options hand suggests a lesion of:
169. Adverse blood trAns:fusion reaction A. Ulnar
occurs in B. Median
A o+ to A+ c. C5 and C6 spinal cord segments
B o neg to A+ D. C8 T1 spinal cord segments
C A neg to A+ e. Radial nerve
d. AB+ to A+ Ans:: aa
e Ab neg to A+ 176. Blockage of beta adrenergics will
Ans:: ddd result in
170. Regarding un coupling of Oxygen a. Decrease heart rate
that’s produced by b. heart contractility
a.thyroid c. insulin production from islet of pancreas
b.norepinephrine d. lipogenesis
c.epiephrine Ans:: aa
Ans:: bbb reference ganong physiology 177. Due to stress of surgery what will
171. If a 70kg healthy adult loses one litre increase and cause vaso constriction
of blood within 5 minutes there would be A. ACTH
a.decrease TPR B. Serotonin
bi.ncrease GFR C. Cytokines.
c.increase venous tone D. increase level of circulating
d.decreasse diastolic pressure catecholamines
e.splanchnic vasodilatation ANS:: AA
Ans:: cc 178. Intraarticular disc of tmj involved..
172. injury to pulmonary alveoli causing Muscle paralyzed.
constriction of alveoli by A. Lateral pterygoid
a.hypoxia B. Medial pterygoid
b.atherosclerosis . C.Temporalis.
c.adenosine Ans:: aa
d.prolonged oxygen therapy 179. Femoral arterial pulse.
Ans:: dd A. Mid inguinal point
173. Primitive streak mesoderm cells B. Mid inguinal ligament.
migrate Round cloacal membrane to form C. Adductor canal.
anterior abdominal wall cells. If this Ans:: aa
migration does not happen around cloacal 180. Regarding posterior Communicating
artery
96

A. Connects PCA to ICA above D. Bound to prealbumin


occulomotor nerve Ans:: bb
B.Connect Pca below occulomotor nerve 190. Parasympathetic effect on Heart
C. Connect ica to basilar artery. A. Increase PR interval
Ans:: aa Ans: : aa
181. Child has episodes of bleeding and 191. which of the following augment the
echymosis since birth hb 9.8 mg/dl And effect of immune system?
platelets count was normal 180 x 10^3 BT A. Complement System
prolonged B.opsonins
A. Hemophilia
B. itp Ans:: aa
C. platelet functional defect 192. For chemotaxis
Ans:: cc A. Lymphokines
182. Corona radiata forms from B. serotonin
A. Germ cells. Ans::aa
B.theca interna. 193. Testosterone main function
C. Theca externa a. Grows hair on scalp
d. Granulosa B. Inc muscle and bone growth
Ans:: dd Ans:: bb
183. All preganglionics neurotrAns:mitter
A. Acetylcholine 194. Sweat gland by.
Ans:: aa a.sympathetic
184. Ppd false negative Ans:: aa
A. Malnutrition 195. Primary active trAns:port
B. Immunosuppressant A. Enzymes
Ans:: bb b. Carriers
185. Most important cell of chronic c. Proteins
inflammation d. Pumps
A. Lymphocytes Ans:: dd
B. Macrophages
c. neutrophils 196. Question regarding edema---
Ans:: bb A. Lymphatic blockage
186. patient with lung abscess develops Ans: : aa
meningitis, causative organism.? 197. Primary Brain vesicle is
A. Staphylococcus a. Mesen cephalon
B. Streptococcus Ans::aa
Ans:: aa 198. Test for typhoid fever in first week
187. 2x2 table a. Blood culture
A. T test. Ans:: aa
B. Anova .
C Chi square
Ans:: cc
188. How can doctor provide better
management to patients:
a. By better communication
Ans:: aa
189. Which form of thyroxin is an active
form
A. Reverse t3
B. Unbound form
C. Bound to albumin
97

Surgery paper 2016 – 5th Sep


2016
1. Diptheria toxin will affect most lethally 11. Breast Atrophy in females, Most
on which organ common cause
a. Larynx a. Estrogen deficiency
b. Heart b. Estrogen & Progesterone deficiency
c. Kidney Ans:: aa
Ans:: bb 12. Most Common cause of fatty liver in
2. Histolplasmosis affect which system our country
a. Reticulo endothelial a. Hep B & Hep C
3. 2% of 4m Lidocaine solution, how b. HCV
much lidocaine c. Hbv
a. 80 ml Ans:: aa
b. 160 ml 13. Glycocalyx :
c. 400 ml a. Carbohydrate Moiety
d. 200 ml b. Helps in defence against microbes
Ans:: aa Ans:: aa
4. 0.85% of Nacl contains how much Nacl 14. Unable to evert and dorsiflex, which
a 850mg/100 ml structure injured
Ans:: aa a. Common Peroneal Nerve
5. Prevents muscle from tearing b. Superficial peroneal nerve.
a. GTO Ans:: aa
b. muscle spindle 15. Scortal Swelling after injury, contents
Ans:: aa of swelling extends upto abdomen but not
6. Skin Graft successful in thigh, Most common cause of this swelling
a. Excessive Burns in scrotum
b. Agammaglobenemia a. Injury to Bulbar Urethra
Ans:: bb b. Injury to Penile urethra
7. Follicular Thyroid Ca, Microscopic Ans:: aa
feature 16. Episiotomy, which part at risk of
a. Psamomma bodies being getting injured
b. Vascular Invasion a. External Anal sphincter
Ans:: bb b. Bulbospongiosis
8. Which cancer invades large blood Ans:: bb
vessels 17. Axillary Artery
a. Clear cell ca of Lungs a. Lies lateral to Biceps tendon
b. Clear cell ca of Kidney b. Present behind pectoralis minor
c. Bronchogenic Ca c. Extends to lateral border of Pectoralis
Ans:: bb minor
9. In Old what happens Ans:: bb
a. Decrease Vital Capacity 18. Anal Canal Blood supply
b. Increase Cardiac Output a. Superior & Inferior Rectal Arteries
Ans:: aa b. Internal Iliac artery
10. Neutral absorption of sodium occurs Ans:: aa
at which part 19. Ecchymosis and bleeding history in
a. PCT fisherman
b.DCT a. Vit C Deficieny
Ans:: aa b. Vit e deficincy
Ans:: aa
98

20. Which clotting factors Depletes early a. Osteoblastoma


a Factor 7 b. Osteosarcoma
b. Factor 5 c. Osteoid Osteoma
Ans:: aa d.Ewing Sarcoma
21. Chromosome 9:22 abnormality Ans:: aa ,, if osteoclastoma ( giant cell
a. CML tumor) in options prefer that.
b. ALL 33. DIC by
Ans:: aa a. Tissue Thromboplastin
22. Function of Vitamin A b. Factor 3
a . Forms Rhodopsin Ans:: aa
b. Not essenial for dark adaptation 34. Jaw deviation to Left and paralysis on
Ans::aa Right Side, Lesion is in
23. Lymph drainage in Cervical Ca a. Pons
a. External & Internal Illiac b. Internal Capsule
Ans:: aa Ans:: bb
24. Decrease in ESR due to 35. Dorsal Nucleus of Vagus
a. Increase Fibrinogen a. Medulla
b. increase Albumin b. Pyramid
Ans:: bb Ans:: aa
25. Double membrane bounded 36. Sciatic nerve injured which structure
Organelle will still flex the knee
a. Nucleolus a. Short head of biceps femoris
B RER b. Long head of biceps.
c.SER Ans:: aa
Ans:: bb ( there was no option of 37. Patient died of Stroke, which type of
mitochondria or nucleus in cpsp exam ) necrosis
26. Recurrent Laryngeal nerve injured on a. Liquificative ncrosis
both sides, Which muscle still intact b. Coagulative necrosis
a. Cricothyroid Ans:: aa
Ans::aa 38. Median Umbilical Ligament is
27. Gastric Lymphome due to remnant of
a. H Pylori a. Urachus
Ans:: aa b. Umblical arteries
28. Locally Malignant Cancer Ans:: aa
a. Basal Cell Ca 39. Which structure will be damaged in
Ans:: aa Lumbar Puncture
29. Posterior wall of Rectus sheath b/w a. Ligamentum Flavum
Symphisis pubis & Abdomen b. Posterior Longitudnal ligament.
a. Arcuate line Ans:: aa
b.Linea semilunaris 40. Scenario with Vit K def, Vitamin K
Ans:: aa not absorbed , which factor decreased
30. Bundle of His Blood supply a. Prothrombin
a. LAD b. Protein C
b. RCA c. Protein S
Ans:: aa refrence Kaplan. d. Factor VIII
31. Head of Femur Blood supply Ans:: aa. If factor 7 in options prefer that.
a.Medial & Lateral Circumflex Artries 41. Low estrogen pills will cause
b. Retinacular a. Breast Ca
Ans:: B> A b. Lungs Ca
32. Most common cause of Epiphyseal c. Endometrial Ca
Tumor d. Hepatic Adenoma
99

Ans:: dd Ans:: aaa


42. Pregnant lady with raised SGPT, 52. Radiosensitive Tumor
hiatory of visit to Remote village, which a. Seminoma
hepatitis b.Lymphoma
a. Hep A Ans:: aa
b. Hep E 53. T cell Receptor Binds with
c. Hep B a. MHC
d. Hep C b. Antigen presenting cells
Ans:: bb Ans:: aa
43. Which Ca will not metastasize 54. Breast Atrophy in adult female is due
a. Basal Cell Ca to
b. SCC a. Estrogen
Ans:: aa b. Estrogen & Progesterone
44. Which of the following in most Ans:: aa
unlikely to occur in CRF 55. Outer quadrant of Breast is drained
a. Hypophospatemia by which lymph nodes
b. Anemia a. Pectoral (Anterior Axillary)
Ans:: aa b. Central group
45. Action of Gluteus Medius & Minimus Ans:: aa
a. Abduct & Medially Rotate the thigh at 56. Loss of Supination & Flexion after
hip Stab Injury to Inner Arm
b. Adduct & Laaterally rotate a. Musculocutaneous
Ans:: aa b. Axillary
46. Isthmus of trachea lies in front of Ans:: aa
a. 2nd 3rd and 4th Tracheal Ring 57 Chemical mediator of inflammation
b. 2nd and 3rd Tracheal ring. a. Lymphokine
Ans:: aa b. Serotonin
47. ECG changes only in lead v4, due to Ans:: aa
which Artery 58 Femoral artery palpated
a. Left anterior descending a. Mid inguinal point
b. RCA b. Mid inguinal ligament
Ans:: aa Ans:: aa
48. Bund of HIS, blood supply 59 Pseudomonas causing fever etc
a. Right coronary artery a. Endotoxins
b. Left anterior descending b. Erythrotoxin
c, Left circumflex artery Ans:: aa
d, Left Main Coronary Artery 60 Cholangiocarcinoma
Ans:: bb reference Kaplan B.D churasia a. Choloronicus sineses
49. History of Hypertension & Chest pain Ans:: aa
radiating to back, patient died. Findings 61 Lateral Thenar muscles spared but
on Autopsy in such patient short muscles of the hand paralyzed ..
a. Medial Necrosis of Aorta where is the Lesion?
b. MVP a. C8, T1,
Ans:: aa b. Ulnar Nerve
50. Sertoli Cells inhibit what c. Median Nerve
a. Androgen binding protein d. Radial Nerve
b. FSH Ans:: bb
Ans:: aa 62 which of the following lymphoid organ
51. Abdominal Aorta receives lymph via SUBCAPSULAR
a. Gives Renal Arteries at L2 SINUS
b. gives Lumbar branches at T 8 a. Thymu
100

b. spleen
c. Lymph node
Ans:: cc
63 Regarding first lumbrical
a. Bipennate
b. attached to proximal phalanx of index
finger
c..attach to radial side of first phalanx
Ans:: cc
64 Patient having severe chest pain died,
Autopsy says it was dissecting aortic
aneurysm... Most likely cause:
a. Medial Necrosis of Aorta
b. Saccular Berry Aneurysm
Ans:: aa
65 heparin prevents
a. propagation
b. embolization
c. organization
Ans:: aa
66 Patient had previously Episodes of
Dizziness and Diplopia, patient presented
now in Acute Comatosed state.. Cause :
a. Carotid Artery Thrombosis
b. Basilar Artery Thrombosis
Ans:: bb
67 Regarding Ciliary body
a. Outer Epithelium Pigmented and Inner
Not
b. Pigmented layer is a continuation of
pigmented layer of Retina.
Ans:: bb ,, if there is option : non-
pigmented epithelium secretes aqueous
humour then select that …
68 Superficial Cardiac Plexus contain.
a. Both sympathetic and parasympathetic
b. Sympathetic
c. Parasympathetic
Ans:: aaa
69 In VIT K deficiency, which factor will
be depleted first and foremost
a. Factor 7
b. Factor IX
c. Factor 10
d.Protein S
Ans:: aa
101

SURGERY 6 SEP Morning


1. low frequency – meissner’s corpuscle
2 .256 HZ – pacinian corpuscle
3. pudendal block – ischial spine
4 phrenic nerve – pericardiophrenic artery
5 trigerminal nerve – temporalis
6 brain vein – drain into dural sinuses
7 ant relation of ant scalene muscle –phrenic nerve
8 sub acute endocarditis – strep viridiAns:
9 carotid sinus – decrease discharge in hypovolemia
10 CNS (CSF) – decrease glucose
11 parathyroid – post medial of thyroid
12 tongue ant 2/3 nodule – metastasis to submental?
13 prevent unwanted trAns:port- tight junctions
14 protrusions – lateral pterygoid
15 superior parathyroid artery – external laryngeal nerve
16 myxiod degeneration- mitral valve prolapse
17 medial necrosis- dissection of aorta
18 thyroid- epithelial changes with activity
19 myxedema- complete lack of thyroxine
20 smokers epithelium –squamous metaplasia
21 free nerve ending – unencapsulated
22 fracture – avascular necrosis of femur head
23 haversion lacunae- osteoclasts.
24 mid trimester hematopoises- liver
25 polycythemia vera cbc shows – lymphocytes count are normal
26 ADH inhibited by –alcohol
27 anemia in pregnancy – dilutional.
28 maxium absorption in kidney pct – glucose
29 secondary active trAns:port – amino acid in pct
30 microtubules – centrioles
31 neck pain of young girl specially during study which nerve damage – spinal accessory
32 cause of hyperthyroidism – graves
33 short muscle of hand – t1
34 levator ani – pudendal + s4
35 sigmoid colon – s234
36 pheochromocytoma – alpha 1 + beta blocker
37 meckal diverticulum - vitelline duct
38 ureter post relation – genitofemoral nerve/internal ilaic artery
39 pudendal nerve – pudendal canal
40 post duodenal ulcer- gastrodueodenal artery
41 breast- pregnancy changes alveoli secretory
42 hipjoint inferior relation – obtruator externus
43 antibiotic use, causing bleeding – give vitamin k
44 benign – hemartoma
45 hemarthorosis – factor 8 assay
46 b- thalassemia major – HBF
47 chondrocytes to osteocytes – sammatomamotrophin.(GH)
102

48 normal basal metabolic rate by – liver


49 myleoperoxidase – neutrophil
50 glycoytic loaded – mitochondria.
51 hereditary spherocytosis – membranous defect
52 metastasis - loss of e cadherin’s
53 fish tap worm diphylobotharium Latium – fish in diet of fisher man
54 antioxidant – vitamin e
55 adenocarcinoma – adenomatous polyp
57 increase bilirubin 16 alkaline phosphatase 300 carcinoembryonic antigen 200, diarrhea due to
psc and colon ca, - ca colon
58 virus – oncogenes
59 pus – dead neutrophils
60 t3 n2 m0 breast – stage 3
61 throne prick – stap. aureus
62 sulphur containing – cysteine methionine, valine (cmv)
63 most common breast ca – invasive ductal
64 cervix stratified squamous epithelium - metaplasia
65 estrogen in post menopausal women - endometrial ca
66 artery remain in pelvis – middle rectal
67 rbc nucleus absent in – reticulocytes
68 left 4th intercostal space stabe- intercostal membrane
69 accompany lad – great cardiac vein
70 ascaris – respiratory symptom
71 amoebic liver abscess – serology
72 mc Burney point – ilio hypogastric nerve – direct inguinal hernia
73 hassel bechs triangle– direct inguinal hernia
74 bulbous urethra – superficial perianal pouch
75 aortic aneurysm at opening – thoracic duct azygous vein
75 aorta – branches to esophagus, pleura pericardium
76 abducted arm drops at once – deltoid damaged
77 loss of abduction upto 30 – supraspinatus
78 thumb tingling – c6 nerve
79 young boy with fracture decrease serum calcium increase urine calcium – vitamin d deficiency
80 medial meniscus attached to – medial cruciate ligament
81 below umbilicus rectus sheath – arcuate line
82 bile salts decrease – after resection of ileum
83 pseudomonas colitis – clindamycin
84 vancomycin – drug monitoring
85 atropine – decrease acetylcholine binding
86 sweat glands – cholinergic symptoms
87 parasympathetic effect – ciliary muscle contraction
88 radial muscle contraction – alpha 1
89 not secreted of nerve terminal in ANS:- l- dopa
90 allergy – bradykinin (histamine was not in option )
91 prozone phenomena – increase antibody titer
92 Rbc antigen – secreted in saliva
93 infective endocarditis – damaged valves > bacteremia> thrombus>perforation
94 cataract+ deafness + cardiac defects – rubella
95 Sexually trAns:mitted disease leading to neonatal conjectivitis after vaginal delivery –
gonorrhea
96 most common malignancy after radiotherapy – hematopoeitic.
103

97 for trAns:plant specimen – wbc


98 old man cough with thick sputum – klebsiella
99 teratogen – alcohol
100 septic meningitis ( fatal) – streptococcus
101 smoking teratogen during – 3-8 weeks
102 isograft ( identical twins ) late rejection due to minor hla antigen
103 csf different from plasma – dec glucose
104 septum supply mainly – LAD (2/3)
105 trachoma – chlamydia
106 myocardial infarction sign on ecg – st elevation
107 after MI patient in ICU suddenly become dyspneic and cyanosis – vent perfusion mismatch
108 vein effected in portal hypertension - portal vein
109 cirrhotic patient with diated umblical veins developed edema in lower limb most effected –
inferior vena cava.
110 right heart border on x ray – superior vena cava
111 lobar pneumonia – sputum culture
112 specific of shock – tissue hypoxia
113 lidocaine side effect – dizziness and light headedness
114 difference between anaphylactic shock and hypovolemic shock – temperature.
115 barret esophagus – adenocarcinoma esophagus
116 steatorrhea most due to – pancreatic resection
117 5cm painless purpura in old lady ( senile purpura) – due to capillary wall fragility and defect
118 primary brain vesicle – mesencephalon
119 posterior cruciate ligament – prevent ant dislocation of femur
120 same in skeletal and smooth muscle – calcium influx before contraction
121 collagen atrophy
122 temporal lobe – memory
123 external carotid difference to internal carotid – gives off branches in neck
124 partially alleviate thirst – GIT distention
125 ADH regulation – osmoreceptors
126 Parathyroid surgically surgically removed – tetany + troussieurs sign
127 increase parathyroid hormone – chronic renal failure
128 residual volume – left air
129 left shift of – HBF
130 normal blood pressure 120/80 mmhg
131 second peristalsis – esophageal distention
132 anthrax – 90% skin lesion
133 negros – keloid
134 lymph nodes with stratified squamous epithelium – palatine tonsils
135 scanty barr body – turner syndrome
136 edema – lymphatic obstruction
137 clara cells – terminal bronchioles
138 pulmonary artery supply – alveoli
139 vertebral disc – ¼ of column
140 cubital fossa injection which nerve will be damaged- median nerve
141 most common fracture – tibia
142 first bone to ossify -clavicle
143 venous cut down – great saphenous
144 iron stores as – ferritin
145 aplastic anemia – fat in bone marrow
104

Surgery paper 1 & 2 Mix 7. Aphasia…damage or lesion at


A. Parietal
14th Oct 2016 B. Occipital
1. Ectoderm and endodermunite at C. Temporal
A. Cloacal membrane Ans:C
B. Coelom 8. Na is actively absorbed from
Ans:A A. Jejunum
2. Suprapubic catheterization done B. Ileum
because bladder is not covered by C. Colon
peritoneum at D. Stomach
A. Dome E. Duodenum
B. Posterior Ans:C
C. Anterior 9. Long scenario…..Bladder ca gene was
Ans:C asked
3. Corneal opacity is due to A. C-neu
A. Ethambutol B. Kras
B. Phenobarbital C. Nmyc
C. Chloroquine HRAS>KRAS>NRAS
Ans:C Ans:B
4. A difficult question regarding pons 10. Parents have 6 or 7 children. only 1
options were has retinoblastoma…which gene is
A. Facial colliculus responsible
B. Facial nuclei ventrolateral surface A. P53
facial motor nucleus is located ventrolateral B. Rb
to abducent nucleus and facial colliculous. C. Kras
The facial colliculous is an elevated area Ans:B
located on the dorsal pons in the floor of the 11. Melanoma most common site
4th ventricle. It is formed by fibers from the A. Soft palate
motor nucleus of facial nerve. So both B. Hard palate
correct C. Tongue
5. Scenario regarding lateral rectus Ans:B
muscle damage…lesion at 12. Recurrent infections…diarrhea…
Ans. Facial colliculus IgA, igG, IgE levels given..all were
Facial colliculous is facial nerve which low..cause?
loops around abducent nerve nucleus. A. Chronic variable agammaglobinemia
Patient with facial colliculous syndrome will B. Bruton’s agammaglobinemia
demonstrate an ipsilateral horizontal gaze Ans:B
palsy with an ipsilateral lower motor neuron 13. Cell to cell adhesion
pattern of facial weakness. The horizontal A. Integrins
gaze palsy is secondary to a lesion that is B. Selectins
effecting the CN 6 necleus that is C- cadherin>selectin
responsible for abduction of the ipsilateral Ans:A
eye by sending projections to the ipsislateral 14. Ejection Fraction decreases. what
lateral rectus muscle. increases
6. Prevertebral fascia encloses A. ESV
A. Sternocledomastoid and trapezius B. EDV
B. Muscle of cervical vertebrae Ans:A
C. Trachea and esophagus 15. Single dose for streptococcal infection
D. Trachea and thyroid A. Ampicillin
Ans:B B. Benzyl penicillin
C. Benzathine penicillin
105

Ans:C 18. Another difficult question regarding


16. Thyroid hormone at physiological anatomy of basal ganglia options were
condition causes A.Below tentorium cerebelli
A. Fatty acid oxidation
B. Protein synthesis B.Midbrain pretectum part of basal ganglia
Ans:A Ans( Controversial)
17. Depression and low motivation lesion 19. Standard deviation is calculated along
at mean because it shows
A. Midbrain A. Variability among individual values
B. Temporal B. Sample size
C. Basal ganglia
Ans:C
Ans:A B. Increase pco2
20. If all values fall on one point or lie Ans:B
within given range it is? 27. Final motor pathway
A. Rate A. Corticospinal tract
B. Frequency B. Alphamotor neurons emerging from
C. Prevalence anterior horn
D. Data C. Inhibiting fibers along alpha motor
Ans:B neurons
21. Glandular acini pseudostratified Ans.B
columnar epithelium with smooth muscle 28. Egg contains protein avidin… eating
calcification raw eggs will lead to deficiency which
A. Prostate causes
B. Seminal vesicle A. Increase transamination
C. Parotid B. Decrease phosphorylation
D. Breast C. Decrease carboxylation
Ans:A D. Decrease coA formation
22. Regarding Glycolysis Ans.C
A. Hexose monophosphate shunt 29. Creatinine 8mg….B.P 210/110…
B. Includes hexose sugars to pyruvate damage to which is a cause of this
Asn:A A. Glomerular cells
23. No. of somites at 30th day B. JG cells
A. 42-44 C. Macula densa
B. 30-32 D. PCT
C. 26-28 Ans.B
D. 32-36 30. Eversion done by
Ans.D A. Peroneus tertius
24. Linea aspera of femur joins upward B. Peroneus longus
and medially to C. Tibialis anterior
A. Pectineal line Ans.B
B. Intertrochanteric line 31. Loss of dorsiflexion and sensation on
C. Intratrochanteric crest foot..damage to
Ans:A A. Tibial nerve
25. Main pillars of medical ethics B. Common peroneal nerve
Ans. Autonomy, beneficence, C. Femoral nerve
nonmaleficence, justice (correct) Ans.B
26. School students play game of 32. Axillary vein formation
breathing in a bag…boy starts A. Basilica vein and vena communicante of
hyperventilating after few mins, reason? brachial artery
A. Decrease p02 ANS:a
106

33. Patient during surgery transfused A. Plasma cells (correct)


with blood… she develops high grade Remember Antibodies formed IN spleen
fever, hypotension and shock in post formed BY plasma cells.
operative period…cause? 43. 1.5L blood loss…low GFR and
A. Graft versus host reaction urinary output…part of nephron
B. Febrile non hemolytic reaction responsible in bringing back GFR and
C. Anaphylactic shock Urinary output
D. Bacterial contaminated blood A. PCT
ANS.D B. DCT
34. Pyruvate between C. Thick ascending limb
A. Glucose and acetyl coA (correct) D. Thin ascending limb
35. Long scenario…cotton wool E. Collecting tubule
sensation.. MCV raised…which Ans.C
deficiency 44. Type of Ig in Myasthenia gravis
A. Vit b6 A. IgA
B. Vit c B. IgM
C. Vitb12 C. IgG
ANS .C D. IgE
36. A question regarding coronary Ans.C
supply..options dnt remember..difficult 45. 2L sweating..drinking 2L pure water
one causes
37. Stroke volume increases…what A. Increase ICF volume
decreases B. Increase ECF volume
A. ESV C. Decrease ECF osmolarity
B. EDV D. Decrease ICF osmolarity
Ans.A Ans.A
38. A patient of sickle cell anemia 46. Loss of flexion and sensation of little
develops hemolysis acutely hb 4… cause? finger. damage to
A. Hemolytic crisis A. Radial nerve
B. Sequestration crisis B. Ulnar nerve
Ans.A C. Median nerve
39. Increase bleeding time and appt cause Ans.B
A. Low platelet 47. Superior thyroid artery ligation.
B. Hemophilia damage to
C. Vit k deficiency A. External laryngeal nerve
D. Vwd B. Recurrent laryngeal nerve
ANS.D Ans.A
40. Patient on ATT develops 48. Regarding infrahyoid muscles
hyperuricemia..which drug A. Cause upward movement of larynx
A. Isoniazid B. Between sternum and hyoid
B. Pyrazinamide C. Supplied by ansa cervicalis
C. Rifampin ANS.C
D. Streptomycin 49. Regarding Trachea
Ans.B A. C4-t3
41. Lymphoid follicles are present in B. C6-T4
A. Thymus C. C6-t7
B. Cortex of lymph node Ans.B
C. Spleen 50. Anterior suture closes at
Ans.B A. 18-24mnths
42. Antibodies formed by B. 12-18
C. 6-12
107

Ans.B ANS.B( discussed thousands time in group


51. Medullary branch of vertebral artery ans was b)
occlusion causes damage to 61. During bronchoscopy first structure
Ans:pyramids visible is
52. Vibration sensed by A. Upper lobe
A. Pacinian B. Upper basal lobe
B. Meckel C. Lower basal lobe
C. Meissner D. Middle lobe
D. Ruffini Ans.C
Ans.A 62. Angular cheilitis caused by
53. Regarding virus infection A. Chlamydia
A. Proto oncogenes B. Cryptosporidium
B. Oncogenes C. Candida
Ans.B ANS.C
54. DNA virus 63. CD4 cells are
A. RSV A. Cytotoxic
B. Picorna virus B. Helper
C. Adeno virus ANS.B
ANS.C 64. Subacute endocarditis caused by
55. Yellowish discharge from lesion A. Staph aureus
A. Staph aureus B. Strep viridans
B. Actinomyces C. Strep pyogenes
C. Pseudomonas Ans.B
Ans.B 65. Cysticercosis caused by ingestion of
56. 3yrs old boy having swollen right knee A. Ova of tenia solium
with pain and redness… B. Eating raw beef
A. Streptococcus fecalis C. Eating raw mutton
B. Pseudomonas Ans.A
C. Staph aureus 66. HIV confirmation test in baby of HIV
D. Strep viridans pos mother
Ans.C A. Western blot
57. Greenish discharge from lesion B. Elisa
A. Staph aureus C. HIV PCR
B. Pseudomonas ANS.C
C. Actinomyces 67. Light on retina causes
Ans.B A. Increase cGMP
58. Long scenario…parasite having B. Increase neurotransmitter release
narrow anterior end C. Increase conversion of cis retinal to all-
A. Tenia solium trans retinal
B. Trichuris trichura ANS .C
C. Ascaris 68. Vit D3 is associated with
Ans.B A. Storage form
59. Type of hypersensitivity in TB B. Animal source
A. Type 1 C. Plant source
B. Type 2 D. Active form
C. Type 3 Ans.B
D. Type 4 69. Which of the following is essential
ANS.D aminoacid
60. Most important investigation in TB A. Tyrosine
A. Caseous granuloma on biopsy B. Cysteine
B. AFB staining C. Phenylalanine
108

ANS.C ANS .C
70. Most common inherited defect in 80. LVF occurs due to malfunction or
hemostasis damage of
A. Factor 2 deficiency A. Mitral stenosis
B. Factor 5 deficiency B. Aortic
C. Factor 1 deficiency C. Tricuspid
D. Factor 9 deficiency Ans.B
Ans.B 81. Artificial pacemaker leads should be
71. Pseudomembranous colitis caused by in
A. C. botulinum A. SA node
B. C. difficle B. AV node
C. C. tetani C. Bundle of his
Ans.B Ans.B( ideal ans is right ventricle which is
72. Regarding Vibrio cholera CPSP key,but it was not in our paper)
A. Grows at 42 C 82. Regarding esophagus
B. Grows in alkaline medium A. It lies on right side throughout its course
C. Grows in acidic medium B. Supplied by thoracic aorta only .
Ans.B C. Joins stomach at 10th costochondral
73. Surfactant contains compound junction
Ans. D. Has thick serosa on lower end
Dipalmitoylphosphatidylcholine(correct) Ans.C
74. Regarding surfactant 83. Shortest pre –erythrocytic phase of
A. Decreases compliance which plasmodium
B. Increases surface tension A. Malariae
C. Decreases surface tension B. Falciparum
ANS.C C. Vivax
75. Lesion on D. Ovale
nose…proteinuria..vasculitis.. Ans.B
A. Leprosy 84. History of pharyngitis..enlarged
B. Wegeners granulomatosis cervical lymph nodes..cause
ANS.B A. Lymphoma
76. Most imp test for gonococci B. Burkitts lymphoma
A. Blood culture C. Infectious mononucleosis
B. Gram stain ANS.C
C. ZN stain 85. Most common cause of
D. AFB nasopharyngeal Ca
Ans.B A. CMV
77. Baby having cataract cause B. EBV
A. Toxoplasmosis C. RSV
B. Rubella Ans.B
C. Syphilis 86. RCA supplies
Ans.B Ans. was all SA,AV node and
78. Right gastric branch of bundle(correct)
A. Gastroduodenal 87. Femoral hernia strangulation due
B. Splenic A. Inguinal ligament compression
C. Hepatic B. Tight boundaries
ANS.C C. Narrow canal
79. Free edge of lesser omentum contains ANS .C
A. CBD 88. Inferior rectal artery branch of
B. portal vein A. Internal carotid
C. hepatic artery B. Internal pudendal
109

C. External carotid Ans. Erythrogenic toxin(correct)


D. Inferior mesenteric 100. dry mouth ..dry eyes
Ans.B Ans. Anti ssa..ssb(correct)
89. Flexion of knee and extension of hip 101. protein not filtered due to
A. Sartorius Ans. Pore size and neg charge(correct)
B. Semitendinosus 102. long scenario…brain necrosis was
C. Biceps femoris asked
Ans.B Ans. Liquefactive(correct)
90. Regarding medulla 103. long scenario regarding anti
Ans. Decussation of pyramids cancerous drugs… tumor regresses due to
91. Superior mesenteric supplies A. Apoptosis
Ans. Till proximal 2/3rd of transverse B. Necrosis
colon Ans.A
92. Regarding defecation reflex 104. blood supply of head of humerus
Ans. Opening of internal anal sphincter A. Anterior circumflex
by parasympathetic B. Posterior circumflex
93. Regarding autonomic nervous system C. Arcuate
A. All sympathetic nerves are adrenergic Ans.A
B. All sympathetic nerve are cholinergic 105. cardiac muscle cannot b tetanized
C. All parasympathetic nerves are due to
cholinergic Ans. Due to long refractory period(correct)
ANS.C 106. blood vessels innervated by
94. Sympathetic stimulation causes A. Parasympathetic
A. Salivation B. Somatic
B. Mydriasis C. Sympathetic
C. Miosis ANS.C
Ans.B 107. high catecholamines and blue cell
95. Parasympathetic nucleus of vagus tumor
A. Nucleus ambiguous A. Pheochromocytoma
B. Solitary nucleus B. Neuroblastoma
C. Dorsal nucleus C. Pituitary adenoma
ANS.C Ans.B
96. 25yrs old female has weakness, 108. ACTH secreted through
malaise and pallor…occult blood in A. Hypophyseal portal system
stools. What is the cause? B. Arterial supply of ant. Pituitary
A. Iron deficiency anemia C. Venous supply of ant. Pituitary
B. Hemolytic anemia ANS.C
C. Sideroblastic anemia 109. suprachiasmatic nucleus function
Ans.A A. ADH production
97. Cervical Epithelium Stratified B. Circadian rhythm
Squamous C. Temperature regulation
A. Metaplasia Ans.B
B. Anaplasia 110. testis lymphatic drainage
C. Dysplasia A. Bilateral para aortic
Ans: A B. Unilateral para aortic
C. Inguinal nodes
98. Regarding collagen Ans.A
A. Most abundant protein in body 111. regarding internal capsule
B. Is acidophilic Ans. Medial to globus pallidus
Ans.A 112. 4th Arch derivartives
99. Scarlet fever Ans:Cricothyroid Muscle
110

113. Small cell Ca produces B. Lateral pterygoid


A. ACTH C. Temporalis
B. PTHrp Ans.B
C. B-HCG 124. regarding popliteus
Ans.A A. Medially rotated femur on tibia
114. sterilization( CPSP mcq) B. Attached to lateral ligament
Ans. Autoclave for 3min option correct.. C. laterally rotates femur on tibia
115. parotid gland supplied by ANS.C
A. GVA 125. patient lying on ot table at 25 c…80
B. SVE % humidity..heat loss due to
C. GVE A. Conduction
D. SVA B. Radiation + conduction
Ans.C C. Evaporation
116. cold intolerance, constipation, weight Ans.B
gain 126. selective inhibiter of COX-2
A. Hypothyroidism A. Aspirin
B. Hyperthyroidism B. Celecoxib
Ans.A C. Ibuprofen
117. which of the following is related to Ans.B
cancer 127. GFR and renal blood flow increases
A. Silicosis by
B. Asbestosis A. Efferent dilatation
ANS.B B. Efferent contraction
118. Hydrocarbons cause C. Afferent dilatation
A. Small cell ca ANS.C
B. Mesothelioma 128. cervical and I think supra clavicular
C. Bronchogenic ca nodes palpable were given..stage was
ANS.C asked
119. hypocalcemia and A. 2a
hyperphosphatemia B. 2b
A. PTH C. 3
B. CRF ANS.C
C. Vit D 129. phrenic nerve accompanied with
Ans.B A. Musculophrenic artery
120. structure pierced during lumber B. Pericardiophrenic artery
puncture ANS.B
Ans:Ligamentum flavum(correct) 130. movement of glucose from higher to
121. patient has fever..chest pain..not lower conc… this type is called
related to respiration A. Na glucose cotransport
A. Costochondritis B. Simple diffusion
B. Myocardium C. Facilitated diffusion
ANS.B Ans.B
122. medial lymphatic drainage of breast 131. RER involved in
A. Pectoral A. Toxin removal
B. Supraclavicular B. Protein synthesis
C. Internal thoracic ANS.B
D. Inferior phrenic 132. tricuspid valve auscultation
Ans.C Ans. Lower right border of
123. patient cannot lower his jaw muscle sternum(correct)
involved 133. dyspnea on lying down
A. Medial pterygoid Ans. Retrosternal goitre(correct)
111

134. therapeutic and adverse affect of B. Celiac


loop diuretics C. Splenic
A. Alkalosis D. Gastroduodenal
B. Low blood volume ANS.C
C. Acidosis 146. left gastroepiploic artery branch of
Ans.B A. Gastroduodenal
135. cause of menstruation B. Splenic
A. LH..FSH C.Hepatic
B. Estrogen ANS.B
C. Hormone withdrawl 147.Pancreatic fluid leakage accumulates
D. Progesterone in
ANS.C A. Subhepatic space
136. in pheochromocytoma drugs given B. Lesser sac
A. B-blockers C. Paracolic gutter
B. A-blockers ANS.B
C. Ca channel blockers 148. optic chiasma lesion causes
D. Both alpha and beta blockers Ans. Bitemporal heminopia
ANS .D 149. pubic symphysis
137. investigation in pregnant patient A. Primary cartilaginous
A.GGT B. Secondary cartilaginous
B. ALT ANS.B
ANS.A 150. malformation of diaphragm due to
138. claw hand due to A. Failure of fusion of lateral buds with
Ans.Unopposed action of extensor pleuroperitoneal membrane
digitorum and flexor digitorum A. act on genes
profundus(correct) B. act on cytoplasm
139. MELAS syndrome histological C. act on receptors in cell membrane
finding ANS.A
Ans. Red ragged fibers(correct) 151. feco oral route
140. Cardiac fibrillation can occur if A. Hep b
stimulus is given at B. Hep c
A. Peak of action potential C. Hep e
B. Start of action potential D. Hiv
C. End of action potential ANS.C
ANS.C 152. aortic opening in diaphragm
141. Regarding asthma Ans. Azygous vein and thoracic duct
Ans. FEV1 153. drug-drug interaction due to
142. Regarding nitroglycerine except A. Pharmacokinetics
A. Blood vessel dilatation B. Pharmacodynamics
B. Decrease pre load C. Both
C. Decrease HR ANS.C
ANS.C 154. dislocation of shoulder which nerve
143. ischial spine inverted damaged
Ans. Android pelvis Ans. Axillary(correct)
144. Temporal lesion causes loss in 155. regarding thyroid gland
A. Olfaction A. Its lobes are pyramidal in shape
B. Memory B. Lobes are symmetrical
C. Vision C. Lies infront of tracheal ring 1,2,3
ANS.B ANS.A
145. Short gastric artery branch of 156. maximum contribution in power of
A. Hepatic eye
112

A. Anterior surface of cornea 168. HCO3 reabsorption in


B. Lens A. PCT+DCT
C. Iris B. PCT+CT
ANS.A C. DCT
157. organ least likely to get infarction D. CT
A. Lungs ANS.B
B. Kidneys 169. Functional residual capacity
C. Liver A. RV+ERV
D. Spleen B. ERV+tidal
ANS.C (cpsp key although lungs is also C. RV+ERV+tidal
correct) ANS.A
158. ascent of horse shoe kidney 170. patient in CCU
A. Superior mesenteric A. Anxiety
B. Inferior mesenteric B. sense of loss
ANS.B ANS.A
159. phenol as compared to alcohol(poor 171. edema in nephrotic syndrome due to
recalled bcq) A. Increase blood volume
Ans. Less painful analgesia B. Hypoalbuminemia
160. tachyphylaxis C. Lymphatic obstruction
Ans. Diminishing action of drug ANS.B
developing rapidly 172. shock by injection…Ig involved
161. stress hormone of body A. IgA
A. GH B. IgG
B. Testosterone C. IgE
C. ACTH D. IgM
D. Cortisol was not in options ANS.C
ANS.C 173. question regarding parts of
162. heat intolerance palpitations duodenum
Ans. Hyperthyroidism A. Retroperitoneal
163. start of 5th week B. 1st2nd intraperitoneal
A. Heart tube ANS.B
B. Stomach rotates 174.most common fractured carpal bone
C. Limb buds A. Lunate
ANS.B B. Scaphoid
164. starling law C. Pisiform
Ans. Increase cardiac output with ANS.B
increase venous return(correct) 175. immediate reflex after acute blood
165. anatomical question loss
Ans. tail of pancreas in splenorenal A. Baroreceptor
ligament B. CNS ischemic
166. patient on thyroxine what should b C. Renin angiotensin
monitored D. ADH
A. T3 ANS.A
B. TSH 176. regarding staging
C. T4 Ans. Used to detect spread of tumor
ANS.B 177.difference btw interstitial and plasma
167. trachea bifurcates at fluid
A. T4-t5 A. Protein
B. T5-T6 B. Electrolytes
C. T6-T7 C. Osmolarity
ANS.A ANS.A
113

178. open wound healing 181.free nerve ending


A. Contraction of myofibroblasts Ans. Un encapsulated(correct)
179. highest mitochondria at apex 182. Raised ICP which drug cannot be
Ans. Cilia(correct) given
180. calcitonin secreted by Ans. Ketamine
Ans. Thyroid ca
• D)Nose
7TH NOVEMBER MORNING • E)Tongue
Ans:: C
SURGERY 10)Maximum reabsorbed from renal tubule
1)Cervical branch facial nerve supply. to plasma?
A)Plytsma A)chloride sodium potassium glucose
b)Masseter Ans:: A
Ans:: a 11)Sensory supply of uterus travels in
2)Lateral half of thenar spared, root injury A)Round Ligament
involved. B)Uterosacral Ligament
a) ULNAR C)Tranverse Cervical Ligament
Ans:: A D)Broad Ligament
3)Retrograde mamory loss.. Ans:: D
a)Brain stem lesion 12) 62 years old female with change in
b)Precentral frontal lobatomy_ bowel habits. DRE negative and
Ans:: B sigmoidoscopy shows 6cm mass near
4.Pendular knee jerk?? caecum. Diagnosed by pathologist as
Ans:: Cerebellar lesion Adenocarcinoma. Most likely
5)Chorda tympanea nerve.. a) -K-RAS with neoplastic cells
a)autonomous function b) I CAM VCAM
b).innervate ant 2 3rd of tongue .. Ans:: A
.C)branch of mandibular nerve 13)Subcostal incision. Rectus abdominis not
Ans:: B necrosed even after damage of superior
6)Supinator muscle paralysis...function epigastric artery because it anastamoses
replace by which muscle with ??
( A)bicep brachii A) deep circumflex iliac :
B)brachialis B) inf Epigastric artery
C)brachiradialis C)subcostal artery
Ans:: A external pudendal artery
7)I/V drug abuser most likely develop Ans:: B
A)Marantic Endocarditis 14) 256Hz tuning fork Wala.
B)Libman Sac Endocarditis A) Pacinian
C)Rheumatic Disease B) Meissonier
D)Infective endocarditis Ans: A
Ans:: D) 16.Which of the following receive max
8)Wat causes inc aldosteron secretion drainage from arachnoid granulations.
A)Hyperkalemia A) Straight sinus
b)FLUID OVERLOAD B) Sup sagital sinus
ANS:A C) cavernous sinus
9)-The Characteristic Lesion Of Prmiary Ans: B
Heridator Telangectesia Is On 17. 8 years old boy with difficulty in
• A) Palate swallowing and mass on left side of
• B)gingivae esophagus. Desmin negative and Vimentin
• C)Lips + . He had a history of fetal alcohol
114

syndrome and not doing well in school. b)appendicitis


Most likely c) minor surgery
A)-Wilms tumour Ans:: A
B)-Rhabdomyosarcoma 26) Trichnomus vaginalis??
C)-Metaplastic Lymphoma a)STD
D)-Retinoblastoma b)blood borne
Ans: B Ans: A
18.Rejection of trAns:plantion WITIHIN 27) Lateral rectus muscle ?question
MINUTES? A) abducent nerve
A) cell mediated b) trochler
b) preformed antibody c) optc
c) immune complex Ans: A
d ) blood group 28.Which of following is tetrogenic?
Ans: B a) coffee
19..Endothelum histolgy? b) alcohol
simple squamous cells Ans: B
Ans:: A 29.Grl wth type 1 diabetes hve mostly
20..Feco oral route which ? hyperglycemia in morning .whixh is
a)hep E appropriate option?
b)hep C a)regular insulin before meal shrt acting
c) hep b insulin
Ans:: A b)twice b4 meal long acting insulin twice a
21..In Emergency deprtmnt ? day
a)Doctr relation patient options given? c)b4 meal long acting nd intermediate insulin
b)Social skills twice a day
c)Family shld b outside ER d) intermediate insulin twice a day
d)Active listening Ans: B
Ans:: c 30..3 yr old child had fever ,flushy face nd
22.Pt wth pulmonary t.b case ...change in tachycardia aftr eye drops instillation
urine nd one symptom given..which of ...which drug cause above characteristics?
following drug is responsible ? a) atropine )
a) RIFAMPICIN b)phenlnephrine related wth othr
b) ethambutol c)tropine trponide
c) pyrazynamide Ans: ;A
d) inh 31)Fatty acid oxidation by which Vitamin
Ans: A a)Biotin
23.Pt hve 1 week typhoid ? Which b)vit d
investigation wil do? c) riboflavin
a) urine Ans:; A
b)stool 32..mechanism of action blockage of
c) blood cultue steroids ?
d)widal A;Cyclocygenase
Ans: C B)Phospholipase A2
24.Neurons in Spinothalamic are not Ans: b )
involved in ? 33.urine coming out of umblical vein
a)temp A)Patent vitalline duct
b) pain, b) PDA
c) cordination c}malrotation
Ans: C d) bladder extrophy
25.Morphine used ? Ans: ;A
a)In terminal stage of cancer pt
115

34.There was a fistula after 12 day of c sec a)Exercise


between bladder and vagina ? b)in stress
A)vv fustula due to obstructed labour c) severe pain
Ans:;a Ans: 'A
35..women with edema till knees, lungs had 44.Superficial inguinal ring..
pleural effusion,heart border visible,liver b ..Ans:..in fascia of ext oblique
kharab tha 45) Microscopic characteristic feature of
a) recurrent thrombophlebitis metastatic carcinoma???
b) itp a)Invasion
c) copartment syndome b)Pleomorphism
Ans: A c) shape
36..Tumour of unrinary baldder cause by.. Ans: ; A
A ) schistosomiasis 46..Post op wound infection aftr 72 hrs
b) trichomonas what will help neutrophlis in process of
c) h pylori inflamation?
Ans:; A a)C3b
37)Which hormone is responsible to change b)C5a
proto oncogene to oncogene.? c)Complements c5 to c9
------------controversial bcq---------------------- d)Fc portion of IgG
------------ Ans:;C
38( A scenario was like a man need blood 47..Heparin prevents..
trAns:fusion but no cross match with any a)propagation of thrmbus
type of blood. Which antigen is present in?? b)clot formation
A) DUFFY c) coagulation
B)kelly Ans:;A
C) RH 48..Hormone therapy given size of tumor
Ans: ; A reduced by the process of
39) Standard deviation,,, a) Atrophy
A) Individual observation b)Apoptosis
b)Variation among individual Ans: B
Ans: b ) 49..Myxoid degeneration in?
40) high cardiac output failure a)Mitral valve prolapse
A) Thiamine deficiency b) aortic regurgitation
b) vit c excess c) tricuspid stenosis
c) b12 def Ans: ;A
D).iron overload 50..After hepatic reaection which helps in
Ans: ;A regeneration ?
41)Area of gaseous exchange .... A)Hepatocyte growth factor
A)respiratory bronchiole till alveoli b)TGF
B ) bronchiole Ans:A
c} trachea 52)Ek tha jis ka Ans: was
Ans: ; A a}caseating granuloma
42,,Which moves potassium out of cell... Ans: a
A) insulin 52) Glycolysis ..
b)salbutamol a)hmp shunt ......reversible
c)exercise Ans: A
Ans: ;c 53)Colon carcinoma metastasis first to
43..Boy was busy in a well ventilated room a) mesocolic lymph node
temp 26 degree C , sweating.,bp was 150 b)liver
/95,pulse 120. he was Ans:; B
54)Achondroplasia scenario
116

a) autosomal dominant b) posterior surface of lower end has right


b)autsomal recessasive vagus
c) not heridatary c) upper end has smooth muscle
Ans: ; A nonkeratinized
55) Cancer causing d) stratified squamous epi
a)benzidine (Ans: d ref kaplan)
b) cooking oil 66)Wilson disease???)
Ans: A Ans:>Ceruloplasmin level
56)Accomudation lost scenerio??? 67)Which Pleura supplied by phrenic only
A) mid brain ???
Ans: A A)Diaphragmatic parietal pluera
57)Left leg UMNL, LEFT face LMNL,left B) viscreal
homonymous hemianopia,,lesion at Ans: A
a}Lat pons (aica) 68)Disease vs non disease
b)cerebellum A) nominal
Ans: A .. ref FA Ans: A
58)Spleen feature 69) Elastic fibres with interlinked collagen
A) accesory spleen in 25% fibers A A)dermis
b)12 cm normal size b)epidermis reticular layer
c)lax ligament cause wandering spleen Ans: B
Ans: B 70)Severe trAns:fusion reaction
59..Steroid like mechanism hormone a) A+ to O+
a) glucagon b) o neg to b pos
b)insulin Ans: A
c) THYROXIN 71)Good analgesic but not a good anesthetic
Ans: C A)nitrous oxide
60)Uterus major support to b)ketamine
A)PELVIC DIAPHRGM Ans: A
Ans: a 72)true about heart..
61)Unilateral orchidectomy Lymph will A)Great cardiac vein starts at interventricular
drain to sulcus
A)bilateral paraaortic b)small veins drain to rt atrium
B)ipsilateral para aortic and c)Other options not remembered
C)ipsilateral pelvic Ans: B
Ans: A 73)One was about some peripheral smear
62..Basilar artery thrombosis jis ma spherocytes n polychromasia ,
fainting comatose patient tha aik wo bhi A)G6pd
A. locked in syndrome b) IDA
B . cerebellum Ans: A
Ans: A 75..Blue cells catecholamine high
63,,Adult female breast atrophy abdominal
A ;estrogen estrogen mass<
b)progesterone A) nephroblastoma
Ans: A) b)Neuroblastoma
64) Breast lies in front of Ans: A
A)pect minor
B)pect major fascia 76)Ca head of pancreas compress??
.Ans: B a)Portal Vein
65) Regarding esophagus b) duodenum
A) t12 Ans: A
77)4th part of dudonum
117

A)Retroperitoneum 86)ulceratd mass in cecum with tubular


b) intraperitoneal glads what is relevantMembr of family has
Ans: ..A adenomatus polip and aur option
78)Caecum cancer feature A) FAP
A) keratin b)crohns
Ans: a c) uc
79)Qrs Ans: A
A) before ventricular systole 87)Friction rub on auscultation,creatitine 9
B ) during sytole ,urea bhi raise tha kafi pericarditis ??
c) after systole a)Uremic pericarditis> fibrinous
Ans: A Ans: ;A
80)wound on Toe lympyh nodes involved 88.Night secretion increases and inhibition
A)Vertcal inguinal group of gnrh >a)tsh
B )horizontal inguinal group b)melatonin
Ans: ;A Ans: B
81)Ak tha diagnostic test tb stem aur tha 89)Pericarditis type in arf?
but matlab yhi tha😎 A)Fibrinous
A) Afb (Ans: .uremic coverts to fibrinous)
b) blood culture 90)Extra embryonic coelome
c ) x ray A)Hypoblast
Ans: A Ans: A
82)A typical senario About 91)Physiologic herniation >
pulmnry artry athetosclerosis. A) clockwise
((( An old aged pt having increased b) 4 6 week
trAns:luency of lungs in upper fileds c) around celiac trunk
flattened diaphgrams no evidenece of Ans: A
infiltration prominent both pulmonary 92)Peristalsis of pharynx slow as compared
arteries and prominent right border of to small intestine
heart no hx of fever what is likely findings a) troposin vs calmodulin
in pulmonary arteries? b) k vs ca
a)granulomatous vasculitis Ans: A
b)atherosclerosis 93)Level of thyroxin altered by >
c)aneurysm A) TRH
d)medial dissection B) GH
Ans: B )) Ans: A
83)Some defect with vision ct done 94)Thyroid epithelium >
occulsion of what artry ? A)change with change in secretory function
A) central retinal artery b) constant lifetime
b) optic Ans: A
Ans: A 95)Farmer with cushing syndrome
84)Sub dural hematoma a)adrenal adenoma
a).briding vein b) copd
b)Superior cv Ans: A
c)Inf cv 96)Virus alters >
Ans: A a)Dna
85)EFFECT ON CARDIAC CYCLE? b) RNA
A)HEART RATE CHANGE CHANGES C) gene
LENGTH OF CYCLE Ans: A
Ans: A 97)Kidney activates>
A)cholicalciferol
b) ACE
118

c) riboflavin 109)Difference BT interstitium n cell tha


Ans: ;a kuch
98)Meningitis after lung abscess > A) I marked 1 mosm
a)staphylococcus aures Ans:
b) s pneumona 110)One was about cecum adenocarcinoma?
Ans: A A)K-RAS Mutation
99)Severe dehydration Ans: A
A) nacl 0.9 % 111)Black water fever wala scenario >
B) 5 % dextrose A)falciparum
c) hemacell b) ovale
Ans: ref Ganong Bcq c) plasmodium malaria
100)Nasopharyngeal CA Ans: 'A
a)rhinovirus 112)G6pd
b)adenovirus A) Heinz body
c) ebv Ans: A
Ans: C 113)Hco3 calculation >
101)Receptor drug effect > A)urine flow × hco3
A)bradycardia causing atenolol Ans: A
B) hyperglycemia due to insulin 114)TrAns:fer of radicals>
Ans: A A)trAns:ferase
102)Abt parasympathetic b) mutases
A) All parasympthtc post all muscurinic Ans: A
b)all sympathetic post all muscurinic 115)Yolk sac tumor >>
Ans: A A) AFP raised
103}Sarcomere b) b hcg raised
A) Z band Ans: A
b) i band 116)Portal hepatis tumor >
Ans: a Ans:>portal vein obstruction
• 104)Intraorbital 117)Icam vcam >
• a) br of maxillary a)leucocyte adhesion
• Ans: A b) RBC adhesion
• 105)Atonic bladder > Ans: A
• a) parasympathetic efferent damage 118) Child coronary aneurysm >
• b) symp out flow A)kawasaki
• Ans: A b) wegner
106)Filiform > Ans: A
A)cover whole dorsum of tongue 119) Median atlanto axial joint?
Ans: A a)pivot
107)rubella in pregnancy?? Ans: a
A) cataract 120)Colon carcinoma >
b) river blindness A) kras
Ans: A b) afp
108)Patient with acute hepatitis trAns:fused Ans: A
with 1 pint cross matched,,after week 121)Inversion>
trAns:fused again A)tibialis ant n post
with 1 pint cross matched Ans: A
Week later present with HB 7,coombs 122)Gaba wala substantial nigra damage
positive,reticulocyte 10% >A)chorea
A) drug induced hemolysis b) rigidity
B ) delayed hemolytic trAns:fusion reaction Ans: A
Ans: A 123)Chromatid allign<
119

a) metaphase b)serratous pos


b) anaphase Ans: A
c)telophase 135.Cervical facial >
Ans: A a) platysma
124)One was hx of family bleeding bt raisd b)stylohyoid
and aptt raisd Ans: A
A) .vwd 136.Left side fall >
b) vit k def a)left cerebellum
c)thalasemia b) right hemisphere
Ans: A Ans: A
125).nerve danage in tonsilectomy 137.Medullary pyramid lesion
a ) 8th A) propioception
b) 11 B ) taste
c) 9 nerve Ans:. a
Ans: C 138.. 51 years old female having iron def
126) Anorectal reflex > anemia and esophageal carcinoma... wheres
a)defecation lymph drainage ??
b) urination A)celiac nodes..
Ans: a B)deep cervical
127)Thiamine cause Ans: B{upper 1/3 esophagus}
A)Subacute degeneration of neurons 139..old man admitted in oncology ward
b) choreoathetosis has ear ache and fever pata nae doosra b
Ans: a kuch infection hua tha head n neck region
128)Defecation reflex< ka.. culture revealed pseudomonas
a)rectoanal aeruginosa and cbc neutropenia.. this is
Ans: A because of.
129) Asthma . a IL5,
a)fev/fvc more than 75 b.TNF
b)fev/fvc less than 75 c. granulocyte stimulating factor
Ans: B d. granulocyte macrophage factor
130)oral ulcers > Ans: B-
a)candidiasis 140)Microscopic feature of compact bones
b) ebv a)Haversian canals are arranged obliquely
c) kissing disease b) lacunae have osteoblasts
Ans: A c)Haversian canals are not interconnected
131)Kidney difference collagen imparts colour
a)structure in hilum d).lamellae are regularly arranged
b) arteries Ans: d
Ans: a 142.. Food entring in larynx prevented by
132) Lateral thenar spared, short muscles A)Pharyngeal muscle contraction
of hand paralysed b)Approximation of vocal cords
a) radial Ans: B
b)long thoracic 143..study on sexually trAns:mitted
c) ulner n damaged disease,one group of boys had penile ulcers
Ans: C and the other group had no ulcers,non
133) Overhead abduction > ulcerating leisons were d/t?
a)subscapular nerve A.treponema paliidum
b) infraspinatous B..chlamydia
Ans: A C.HPV
134)LONG THORACIC NERVE supplies D.ducreyi
A)SERRATUS ANTERIOR E.Gonorhea
120

Ans:; B 154)Anterior compartment syndrome of lower


144..Regarding physiological herniation limb nerve involved
A)On IMA axis A) common perronal
B)Clockwise b)femorral
C)Herniates in extra embryonic celoem c) sciatic
DMidgut connected to allantois Ans: A
Ans: B 155) Spleen
• A)12 cm
145.Infra orbital artery is terminal branch • b)accessory spleen in 25 %
of • Ans: A
a)Facial 156) Left leg UMNL,left face LMNL,left
b),Maxillary homonymous hemianopia area damaged ??
c)TrAns:verse facial • a)cerebrum
Ans: B • b).lateral pons
146.Anterior surface of duodenum • c)medulla
perforate, fluid will accumulated in? • Ans: b
a) Right iliac fossa 157) Steroid acts on??
b)Right posterior subphrenic space Ans:> via Gene
Ans: A 158) Post triangle biopsy>
147.Testicular ca drains into a)spinal acessry
a)Ipsilateral pevic and contralateral p.a nodes b) long thoracic
b).lateral pelvis c) 12th
c)Ipsilateral p.a nodes Ans: A
d)Bilateral paraaortic nodes 159)Carcinogenic>
Ans: D a)benzidine
148)Femoral pulsation cab be feel at b) alcohol
a)Mid inguinal point Ans: A
b).Mid point of inguinal ligament 160)uterus main support >
Ans: A a)trAns:verse cervical
149)Regarding Sephanous vein b)broad lig
A)Ascend in front of medial malleolus Ans: A
b)Pierce s.f faci a 161)River blindness
c)Connect with deep veins of calf A)Oncoceriosis
d)Start from medial venous arch b) schistosomiasis
Ans: A Ans: a
151)One q characteristics feture of well def 162)Microscopic feature Tb lymph node >
sq cell ca A)caseous necrosis
A)Keratin perls Ans: A
b)Pleomorphism 163)Factor 8 >
c)Polarity a)cryoprecipitate
Ans: A b) ffp
Ans: A
152.Regarding heart supply 164.Immunologic test related question
A)great cardiac vein start from apex run in A)Amobic liver disease
interventricular grove b)Hydatid cyst
Ans: A Ans: a
153..Which harmone released at night cause 165.Human is intermediate host of.
block of gnrh A) hydatid
A)Melatonin b) malaria
B) tsh Ans: A
Ans: A
121

166.Increase hydrostatic pressure related c) Deposited first in wound healing... Collagen


question...... III
a).Constriction afferent arteriole d)Collagen I takes 3 weeks or more
b)Constriction efferent arteriole 176 Right border of heart??
Ans: B Ans:> Right atrium
167)Which is pivot joint 177.l-dopa inhibit ..
A)Atlantoaxial joint a)prolectin
b) pubic b)oxytocin
Ans: A Ans: A
168)C7 vertebra specific feature 179..Primary perstalsis different from
A)Longest Spine secondry in
b) body a)Orophargeal phase
Ans: A Ans: a
169)Decrease osmotic pressure . 180..Virus in cell cause alter.......
......infusion A)DNA
a)Isotonic saline b)Oncogen
b)20% albumin solution Ans: A
c) .Distilwater 181.60 year gardner present with caushing
d)10% glucose syndrom
Ans: c[ distalled water is istonic bec no a.renal cell carcinoma
salute) b.adrenal adenoma
170.Person come to pakistan and develop c.Thyroid carcinoma
fever from 2 days and become unconcious Ans:wer B
a)P .falciperum 183.Heanz body presnt in
b) p ovale • a)hereditry spherocytosis
Ans: A • b)sickle cell
171.athlete has infection of big toe..there is • c).G6pd
tenderness in groin.. which nodes involve. • Ans: C
a. seperficial inguinal horizental. 184.Methicilin resistance staph aureus
b..Ans: superficial inguinal vertical.. sensitive to .a).vancomycin
.c. deep inguinal b) gentamycin
d.. node of cloquet c)lincomycin
Ans: B Ans: A
172.Acute renal failure related question 185..Mineralocorticoid....Adlosteron
which part of tubule involve..... secretion stimulate raise ..related question
a) PCT tha
b) Dct A)Potassium
c) Loop of Henle b)Renin
Ans: A Ans: A
173)Wound strengthing collegen 186..Staff nurse and hospital acquired
Type infection related question..
a) type 1 .A. Left Gloves on floor after blood sample
b.type 2 B.) Not hand washing after glove removed
c.type3 Ans: B
d.type4 187..superior parathyroid lies superficial to
Ans: A .A).pretracheal fascia
175.Imprtnt regarding wound... Ans: A
aCollagen points 188..CSF related question
a)Inc wound tensile strength.... Collagen I a) Drain arachnoid villi
b)Imprtnt in wound healing III b) Absorb choroid plexus
Ans: A
122

189,filiform papillae of tongue 7) Histology slide shows rete ridges with


a.. mushroom like nuclear atypia and nuclear pleomorphism
b imparts red colour to tongue with infiltrating malignat epithelial cells
c.. mostly present on tip a-Verroceous Carcinoma
Ans: a b-Squamous Carcinoma
8TH NOVEMBER 2016 c-Transitional Cell carcinom
Ans;B ref robbins
SURGERY PAPER -1 AND 2 8) Three days after delivery a women died
because of pulmonary thrombo -
MORNING embolism. There was blockage at
1) Pretracheal fascia completely covers pulmonary artery which result in
a-Thyroid a-V/Q ratio achieve infinity
b-Trachea b-V/Q ratio becomes zero(shunt)
c-Esophagus c-V/Q ratio is unchanged
d-Laynx d-Shunting of the alveoli occurs
e-Pharynx Ans:B ref brs
Ans:A(ref snell) 9)A substance freely filtered and actively
2) Long scenerio of fibroadenoma reabsorbd by the nephron
showing features of firm fibrous mass. a-Clearance will be increased in prescence
Gross appearance of fibroadenoma of metabolic poison
a-Metaplasia -b-Clearance will be decreasd in presence of
b-Dysplasia metabolic poison
c-Desmoplasia c-It's concentration in urine is more than
d-Aplasia plasma
-Anaplasia (It is a feature of d-It's concentration in urine is less than
Carcinoma) plasma
Ans:C Ans:D
3) Proto oncogenes converted to 10) Low grade fever with cervical
oncogenes lymphadenopathy. Histology shows
a-Gene amplification diffuse infiltrating macrophages with
b-Point mutation spaces and increased mitotic index
(ans:B ref robbins) a-Tuberculosis
4) Tumour Metastasis: b-Burkitt lymphoma
A. migration c-Lymphocytic Lymphoma
B. degradation of extracellular matrix d-Sarcoidosis
C.degradation of "E-cadherin" Ans:B ref first aid
D. loss of attachment of tumor cells 11) 24 years old male presents with fever
with each other and weakness. Non-lactose fermenting
Ans:c organism isolated with greenish metallic
5) Sertoli cells-remove blood testis barrier colonies
(exact words) -a-Pseudomonas Aeuroginosa(answer)first
a-produce androgen binding protein aid
b-secrete testosterone b-Staphlococcus Aeureus
Ans:A ref first aid c-Streptococcus
6) Regarding parotid gland d-Klebsiella
a-contain serous acini Ans:A ref first aid
B- duct opens in the second molar 12) Histolgy slide shows circular and
tooth(oposite upper 2nd molar tooth) longitudional arrangement with skeletal
C-pierces masseter(pass over messater chaned into smooth muscle fibres
pierces buccinator) a-Upper esophagus
Ans:A ref first aid+klm b-Middle esophagus
123

c-Pharynx a-Mitochondria
d-Palatine tonsil b-Golgi Bodies
e-Lower esophagus c-Endoplasmic Reticulum
Ans:B d-Nucleolus
12) Patient presents with signs of low Ans:A ref guyton
grade fever, non- healing ulcer on tongue 19)Which of the following is absent from
and cervical lymphadenopathy. Biopsy the axon
shows epitheliod cells with casseous a-Mitochondria
necrosis b-Nissil Bodies
a-Tuberculosis c-Microfilament
b-Sarcoidosis d-Microvilli
c-Burkit Lymphoma Ans:B ref kaplan
d-Lymphocytic Lymphoma 20) ECF and ICF
Ans:A ref robbins and first aid a-Na and Ca inc in ECF as compared to ICF
13) The functional residual capacity ??? b-ECF more than ICF
a is the sum of residual volume and c-ECF has high K and Na concentration
inspiratory reserve volume Ans:A ref guyton+brs
b. Volume of air in the lungs at the end of 21)Blood Group A+ receipient cross
tidal expiration matched blood group with every other
c. Vital Capacity + Expiratory Reserve blood group.
Volume Cross matching failed due to the presence
d. Expiratory capacity +tidal volume of antivodies
e. Can be measured by spirometry a-Anti-M
Ans:B ref brs b-Anti e
14) Scenerio of Hemorrhage , patient was c-Anti gG
transfused with 150mmol/L Normal d-Kell
Saline. Most likely change occurs e-Duffy
a-Inc in ECF Na level Ans:E
b-Inc in intracellular volume 22)Nerve damage resulting in loss of
c-Inc Urinary osmolarity sensations in lateral three and a half
d-Inc ICF Osmolalite finger.
Ans:B a-Ulnar
15)28 years lady presented with vaginal b-Median
bleeding and show signs of cervical c-Musculocutaneous
hyperplasia d-Accessory
a-Inc estrogenic stimulation e-Axillary
b-Dec estrogenic stimulation Ans:B ref snell
c-Inc leutinizing Hormone Note:If dorsum of hand and lat 3 and ½
Ans:A fingers than choose superficial branch of
16)Potassium secretion decreases in radial nerve.
-Inc dietary potassium(low k diet) 23) Greater tuberosity of humerus
-Inc aldosterone secretion(true) a-Medial side of shoulder
-Decrease Na absorption b-Short head of bicep arise from it
Ans:B c-Attachment of Sub-scapularis
17)Sodium is primarily regulated via d- Embedded beneath Teres major
a-Osmoreceptors e-tere minor attached at inferior facet of
b-Aldosterone(reabsorption) greater tubercle
c-Renin Ans:E ref snell
Ans:A 24)Girl with type 1 diabetes hve mostly
18)Which of the following has the hyperglycemia in morning .which is
capacity of regeneration appropriate option?
124

a-regular insulin before meal D-4


B- shrt acting insulin twice b4 meal E-8
c- long acting insulin twice a day b4 meal Ans:C
D-long acting and intermediate insulin twice 31)35 year old man bilateral crepts, on
a day xray chest heart fills completely ,mass in
E-intermediate insulin twice a day RT ventricle , What will be the main
Ans:D cause
25) Breast Carcinoma undergo surgery a.mesothelioma
with axillary clearance done during b.rhabdomyosarcoma
surgery. Winging of scapula scenario due c.angiosarcoma
to surgery complication. Nerve damage d.myxoma
a-Thoracodorsal Ans:D ref cmdt
b-Long Thoracic 32) Organism associated with bladder
c-Dorsal Scapular squamous cell carcinoma
d-Sub-scapular A-Schistosoma hematobium
Ans: B ref Snell B-Schistosoma Mansoni
26)Sciatic nerve severed, dorsum of foot Ans:A ref robbins +cmdt
supplied by which nerve?? 33)26 years old boy presents with pallor
a.saphenous nerve and weakness. Fever of 101°F. Echymosis
b.Sural nerve and Bruises were there. Patechial leision
c.Common peroneal nerve was mentioned. Definitive diagnosis via
Ans:A ref snell investigation ? Options were
27)After hemorrhage which of the A-Serology
following is the last to return to normal B-CBC
a-RBC in peripheral smear C-Bone marrow aspiration
b-Hemoglobin D-Molecular biology
c-Reticulocytes Ans:B
Ans:A 34) Infratemporal fossa is also known as
28) Circle of Willis is formed by Ans> #Parapharyngeal #Space
A-Vertebral artery and Middle cerebral 35) Hematopoetic function of
B-2 internal carotid and Basilar endothelium??
artery(answer) A-machanical barrier
c-Basilar artery with external Carotid B) synthesis of fibronectin
D-Anterior cerbellar with Posterior C) synthesis of anti plasminogen inhibitor
Communicating artery factor
E-Between 2 vetebral arteries and internal D) synthesis of prostacycline
carotid arteries Ans:D
Ans:B ref kaplan 36) 62 years old lady presents with
==>I check again and again Posterior backache. 30% monoclonal plasma cells
cerebral was not in the option. with features of joint pain. Most likely
29)Vitamin C deficiency diagnosis
A-Defective Collagen Formation A-Multiple Myeloma
B-Decreased Collagen Formation B-Waldernstorm
C-Fibrillin/Collagen ratio deranged -C-Monoclonal gammopathy
Ans:A ref lippncott D-Plasmacytome
30)Fetal movement detected by mother in Ans:A ref robbins and cmdt
which month 37) Extensive Lesions in the cerebrum
A-6 with involvement of brocas and wernicke
B-7 area
C-5 -A-Global Aphasia
-B-Transcortical Aphasia
125

C-Sensory-Motor aphasia left is crescent. Supplied by three arteries


ANS:A ref kaplan and one vein. Only suitable option is D.
38)A patient presents with type 2 diabetes 43) A patient presents in E.R with a
mellitus. Most likely complication in limb hemorrhage. Blood loss (500m/l).
will be Compensatory mechanisms were
A-Dry Gangrene activated. Which will decrease after
B-Wet Gangrene compensation
C-Raynaud's Phenomenon A-HeartRate
Ans:B B-Venous Capacitance
Diabetes=Dry C-Urine Output
Diabetes with complication=Wet Ans:A
39)Absolute contraindication of propofol What inc and what decreases( Decided on
:- group) :-
-A-Hypersensitivity reaction Hemorrhage During compensation
-B- Cardiac problems 1)venous capacitance decrease 2)urine
- c-Porphyria output decreases 3) h.rate increases due to
D-Pregnancy sympathetics.
Ans:A ref katzan After compensation 1)h.rate decreases and
39)Xylocaine given to a patient with returms to normal. 2)urinary output
multiple drug allergy. Sign of increase increases and returns to normal
sensitivity? 3)v.capacitance increases and returms to
a Rash. normal.
b Inc blood pressure 44) A patient had difficulty in abducting
c Rise in temp and adducting his fingers and has lost
d Syncope abduction of his thumb. The nerve most
e Tacchycardia likely involve
Ans:D ref lippncott A)Sup. Branch of Ulnar Nerve
40)Regarding neutrophil, correct B)Deep. Branch of Ulnar Nerve
statement is C)Dorsal Branch of Ulnar Nerve
A. Migrate into and out of the blood D)Median Nerve
B. Phagocytic in bloodstream E)Anterior Cutaneous branch
C. Decrease in infective conditions Ans:B
D. Decrease when corticosteroids were Rabia ali Page 30 MCQ No 12 with little
given changed option.
Ans:B ref guyton 45)Non-Pregnant , 26 years lady
41)Rt kidney differentiated from lft presented with FBS:122mg/dl, After one
kidney by..? hour 198mg/dl, After two hours 194mg/dl.
A. Relation of structure within hilum Most likely
B. Surface and pole A-Normal Glucose Toleramce
Ans:A ref snell B-Overt Diabetes
Rest of the Options were changed but A is C-Secondary Diabetes
the most appropriate one ! D-Gestational Diabetes
42)Adrenal gland E-Lag storage curve
A -has same size in adult n infants Ans:B
B- arterial supply from 2 arteries 46) No of somite at 30th day
C -pyramidal in shape A’25-30
d -has seperated from kidney by perinephric B-32-34
fat. C-34-35
Ans:D ref snell D-42-44
Question was only about adrenal gland. E-44-46
Right adrenal gland is pyramidal in shape,,
126

Ans:A (21-29 somites on 30th day ref keet ANS:A ref snell
moor embryo) Rabia ali mcq no 23 page 31
47)Which one is essential amino acid? 54) Increased bleeding time scenerio.
{A} serine Von willebrand disease.
B} tyrosine 55)Most common cause of inc bleeding
C} Phenylalanine time
D} proline -Aspirin ingestion(past paper bcq)
Ans:C ref lippncott 56)Bundle of HIS. Blood supply
48)Best example of pharmacodynamics A-Right marginal branch of LCA
drug drug interaction: B-LCA
a: tetracycline with calcium C-Interventricular branch of LCA
b: toxicity of lithium with thiazide diuretics D-Right Marginal branch of RCA
c:reverse the action of aspirin by NAHCO3 E-RCA
d:reverse the action of morphine by Ans:E ref snell
naloxone 57) Patient on ATT. Develop narrow
Ans:D ref lippncott vision amd difficulty in night vision. Drug
49)Regarding the cardiac reserve responsible
A. Decreases in atheletes A-Ethambutol
B. Increases in vitamins deficiency B-INH
C. Is increased in ischemic heart disease C-Rifampicin
D. Is the maximum percentage by which D-Diazepam
cardiac output can be increased above Ans:A ref cmdt
normal. 58) 3 years old child presents with rash,
Ans:D facial flushing and tacchycardia after eye
50)Individuals have the independent and drop instillation. Fever of 101°C . Drug
same chance of being selected: responsible
Asimple random sampling A-Atropine
B.stratified sampling B-Homoatropine
C.systemic sampling C-Tropicamide
Ans:A past paper bcq D-Pilocarpine
51)Fatty acid oxidation is helped by? E-Hyoscine
-A-Biotin Ans:C
B-vit c Tropicamide eye Drops causes rash, facial
C-vit D flushing and tachycardia.
D-vit B12 Atropine eye drops causes blurred vision,
Ans:A ref lipncott eye pain and stinging amd hypotension.
52)Most likely about PONS 59)A nursing mother presents with herpes
-A-Facial colliculus formed by facial nerve labialis. Drug of choice
nucleus A-Acyclovir
B-nuclei of pons in on ventrolateral area B-Triflouridine
C-spinal leminiscus originates from C-Ranitidine
tegmemtum D-Amantadine
_Poor ReCALL Ans:A
53) If the whole movement of abduction 60)Most likely feature present in
of arm is lost, the most likely damage part spinothalamic tract but absent in dorsal
of brachial plexus will be column medial leminiscus
A-Upper trunk and posterior cord -Long receptive field
B-Upper trunk and Medial Cord A- sensory rapidly adatping or slow
C-Lower trunk and medial cord adapting but never both
D-Lower Trunk and Posterior cord B-cam supply 1 or more than 1 limbs
It’s a poor recall.
127

61)Resection of ileum absorption effected D-Aspartate Aminotransferase


A-Zinc E-Alkaline Phosphatase
B-Iron Ans:E
-C-Vit b12 70)Most likely in Sarcoidosis
Ans:C past paper bc1 A-Erythema Nodosum
62)At the start of fifth week B-Hypercalcemia
A-Stomach rotates C-Hepatic granuloma
B-Limb formation occurs D-Pleural effusion
C-Definitive heart is formed Ans:B ref cmdt
D-Ventral mesentry ka koi chakar tha :-P Erythema nodosum only 7%
Ans:A 71)Bronchogenic Crcinoma. Most likely
63)Left sixth aortic arch forms cause
A-Ligamentum venosum -Benzidine
B-Ligamentum arteriosum -Tobacco(robbins+cmdt)
C-Subclavin artery -Asbestosis
-D#None of the above 71)Sympathetics via alpha adrenergic
Ans:B ref first aid effect
64) Drug absorption in gut is increased by A-Vasodilation
-A-Inc gut motility B-Bronchodilation
B-ionized state of drug C-Pupillary dilation
C-Lipid solubility(lippncott) D-lipolysis
Ans:C Ans:C ref brs n kaplan
65) Ear ossicles are the example of 72)A six year old girl with history of fever
A-Flat bone and sorethroat. Cultures reveal tiny
B-Irregular bones colonies with complete hemolysis on blood
C-Sesamoid Bones sheap agar plate. Organism responsible
D-small bones A-Beta hemolytic group A
Ans:D ref snell Streptococcus(strept.pyogens:bacitracin
66) Infant's joint between two mandible is sensitive)
A-Gomphosis B-Staphloccus Aureus
B-syndosmosis C-Bordetella
C-synchondrosis D-Klebsiella
D-symphysis E-Corynebacterium diphtheria
Ans:D ref snell Ans:A ref first aid
67) 42 years old women with metaplasia 73)Normal angle is 125° with shaft in
cervix. Organism responsible lower limb. In Cox Vera
A-HSV-2 A-Head Neck Shaft Angle increases
B-HPV B-Head Neck Shaft Angle decreases
C-Trichomonas Vaginalis C-Head Neck Shaft remain same
D-Gonorrhea D-Head of femur displaced laterally
Ans:B ref snell Ans:B
68) Most rapidly adapting receptors Cox vera:angle dec 《120
A-Touch 74) Patient unable to turn eye laterally.
B-Alph-1 Most likely cause
Ans:A A-Infection of right cavernous sinus
69) 45 years old female with biliary colic, B-Tumour of left cavernous sinus
rash and cholelithiasis. Enzyme most C-tumour of pituitary gland
likely raised D-tumour of anterior cranial fossa
AAlanine aminotransferase Ans:A
B-Glutamate Dehydrogenase 75)The characteristic lesion in primary
CLDH hereditary telengiectasia is on
128

A-Palate D-atonic bladder result from filling


B-gingivae sensation
C-Lips Ans:both A and B correct
D-Nose 82) Subcostal incision. Rectus abdominis
E-Tongue not necrosed even after damage of
Ans: C (ref cmdt ref taken from a dentistry superior epigastric artery because it
mcq book. Key was LIPS there). anastamoses with
76) I/V drug abuser most likely develop A-deep circumflex iliac
A-Infective endocarditis B-inf. Epigastric artery
B-Marantic Endocarditis C-subcostal artery
C-Libman Sac Endocarditis D-external pudendal artery
-D-Rheumatic Disease Ans:B ref snell
Ans:A ref cmdt as marantic occurs in 83)Which hormone is responsible for
cancer patients and limbman in sle brain maturation of fetus??
77)Most common nerve injury in lower A-Thyroxine
limb in anesthesized patient B-Growth Hormone
A-Ant. Tibial C-Cortisol
B-Saphenousnerve Ans:A
C-Common Peroneal Nerve 84) Which hormone stores carbohydrates
D-Femoral and protein??
E-Obturator A-Insulin
Ans:C ref snell B-Cortisol
78)Sensory supply of uterus travels in Ans:A
A-Broad Ligament 85) Biopsy of endocervix shows squamous
B-Round Ligament epithelium. this is called.
C-Uterosacral Ligament A Dysplasia
D-Tranverse Cervical Ligament b. Metaplasia
Ans:A ref snell c Anaplasis
79)62 years old female with change in Ans:B ref robbins
bowel habits. DRE negative and 86) Maximal level of urinary 17
sigmoidoscopy shows 6cm mass near ketosteroids is presnt in???
caecum. Diagnosed by pathologist as -normal male(ans)
Adenocarcinoma. Most likely 87)Ovarian cancer will first metastas to
-K-RAS with neoplastic cells(cmdt 2016) whch lymph node??
This is true as K-RAS and B-raf associated Para-aortic Lymph Node(snell+past paper
with CA Colon. bcq)
H-RAS with Bladder Cancer. 88)Shoulder pain referred due to??
80) Slow growing and longstanding c3 c4 c5(ans:snell)
tumour of parotid gland 89)A patient with T4N1M0 with a very
A-Pleomorphic adenoma long stem(long stem was to distract) they
B-Warthin wanna ask wat else could be seen...
C-Mucoepidermoid tumour A. Cardiac murmur
D-Mucous Cystic adenocarcinoma B. Cachexia
Ans:cmdt Ans:B past paper bcq
81)Regarding urinary bladder 90) Uterine artery branch of illiac artery
A-Sympathetic causes relaxation of bladder crosses which structure
muscle wall(brs +lippncot) A-Ureter
B-Parasympathetic causes relaxation of B-Kidney
internal sphincter(lippmcott+brs) Ans:A past paper bcq
C-Pundendal nerve relaxes external sphinter 91) Head of pancreas lie anteriorly to
A-Inferior Vena Cava
129

B-Aorta Supplied by cystic artery, branch of right


C-Sup. Mesenteric Artery hepatic artery(snell)
D-Inferior Mesenteric Artery 98) regarding small intestine
Ans:A ref snell A. jejunum thin wall n is longest
92)Regarding Cardiac Cycle B drain by both celiac n sup mesenteric
A- P wave is before ventricular c. Small intestine is suplied by both sma and
contraction(p for atrial depolarization:ref ima
first aid) d. none of it is retroperitoneal
B- Second heart sound corresponds to t Ans:B ref snell
wave 99)Myocardial Ischemia
C- first heart is during iso-volumetric A-contraction band(12-24 hours)first aid
contraction(ref brs) B-Clumping of nuclear chromatin
Ans:Both A and C appears to be true. Ans:A》B
93) 30 years old female, with fever 101 F , 100)RBC have
with pus cells in urine, ct kidney showed A-Biconvex
parenchymal changes , diagnosis B-Glycolytic activity
A. Acute tubular necrosis Ans:B guyton
B. Necrotizing 101)peroxidation of lipids
C. Chronic pyelonrphritis A amyloid
D. Acute on chronic b lipofuscin
Ans:C Ans: B
94) 30 years old female, with fever 101 F, 102)Recurrent laryngeal nerve passes
fever unresponsiveness for weeks with between
pus cells in urine, ct kidney showed A-Cricoid and thyroid
parenchymal changes, , diagnosis B-Thyroid and hyoid
A. Acute tubular necrosis Ans: B
B. Necrotizing nephritis 103) Relation of common peroneal nerve
C. Chronic nephritis with biceps femoris in politeal fossa
D. Chronic acute nepritis A.anteroir
Ans:D B-posterior
95)Aldosterone secretion increasd by : C- Medial
A-hypokalemia D- Lateral(to peroneus longus)
B-hyperiosmolality Ans:C
C-dehydration 104)study presented in 2×2 table
D-Ace inhibitors Chi test(past paper bcq)
E. Dec plasma volume 105) Facial nerve enters into mastoid part
Ans:E of temporal bone via
96)Anterior Abdominal Wall between A-Stylomastoid foramen
subcostal and umbilicus B-carotid canal
A-External oblique and anterior layer of C-petrous temoral bone
internal obliquE D-foramen spinosum
B-Anterior layer of internal oblique with E-Internal acoustic meatus
transversalis muscle Ans:E
C-External oblique aponeurosis 106)Most appropriate about hyperacute
D-Outer fibres of external oblique rejection
Ans:A ref keet moor A-occurs within minutes
97)Regarding gall bladder B-by preformed cytotoxic T cells
Has squamous epithelium C-treated with immunoglobulin
Fundus lies at edge of 6th costal cartilage Ans:A
Completely covered by peritoneum 107)Enzyme used in break down of non
epinephrine and serotonin secretion
130

1.MAO anastamoses with


2.COMT A-deep circumflex iliac
3 dopamine B-inf. Epigastric artery
Ans:A ref lipncott C-subcostal artery
108) Right Coronary artery blockage D-external pudendal artery
after giving marginal artery. Which is Ans:B
affected 114) Iritant receptor in airway
A-AV node A-cause coughing and broncho-constriction
B-SA node B-among epiyhelium
C-Purkinjee fibres -rapidly adapting
D-IV septum C-Only response to chemical irritants
Ans:A past paper bcq Ans:A
109) sub mucosal glands in. 115)Abdominal angina likely cause of
ileum. narrowing
A-duedonum Superior Mesenteric artery(snell:past paper
B-jejenum. bcq)
C-Stomach 116)Removal of gastric antrum, what
Ans:A past paper bcq+snell happens
110)8 years old boy with difficulty in A-Decrease emptying time for solids
swallowing and mass on left side of B-Dec gastric compliance
esophagus. Desmin negative and Ans:B
Vimentin + . He had a history of fetal Discussed on group
alcohol syndrome and not doing well in Remove fundus- Decrease receptive
school. Most likely Relaxation
A-Wilms tumour Remove Gastric Antrum- Dec gastrin
B--Rhabdomyosarcoma release and decrease gcompliance
C-Metaplastic Lymphoma Remove Pylorus- Inc emptying time for
D-Retinoblastoma solids
Ans:B 117) intercostal muscle supplied by
Rhabdomyosarcoma is desmin and vimentin :ventral rami.(snell)
positive, associated with fetal alcohol 118)Trichmonas vaginalis :is sexually
syndrome but mass in the neck is transmitted.(first aid)
controversial. 119)Maximum reabsorbed from renal
Wilms tumour is desmin negative and tubule to plasma
vimentin positive but it is common in 2-5 A-Glucose
years of age and is a renal tumour. B-Sodium
Nothing wrong in the stem and options. C-Potassium
111)Rearding seminal vesicles Ans:A ref brs
A- Storage of sperms 120) Free nerve ending
B-opens with ejaculatory duct into prostatic A- encapsulated
urethra B-un-encapsulated
C-Anterolateral to ureter Ans:B ref kaplan)
Ans:A snell 121) fecal oral route sy spreading.
112)Property of cardiac muscle that hep E* (Hep A not in options)
prevent genesis of 122)Chorda tympani nerve
A-tetanisation a- AutoNomic function(preganglionic
B-Refractory Period parasympathetic secretomotor fibers for
Ans:A its automaticity submandibular and salivary gland)
113) Subcostal incision. Rectus abdominis b- post 1/3 rd of tongue(ant 2/3rd)
not necrosed even after damage of c- pain fibers from middle ear
superior epigastric artery because it d- supplies stapedius
131

Ans:A ref snell B-Internal Oblique and Transversalis Fascia


123) Medical meniscus of knee C-Peritoneum
attachement Ans:C ref :(snell:it is expanded part of
A-Medial collateral ligament process vaginalos which is detived from
B-Anterior cruciate Ligament peritoneal folds)
C- Posterior Cruciate Ligament 134) Aldosterone is increasd by
Ans:A ref snell A:Hypokalemia
124)Supinator alternative or secondary B: Increasd plasma osmolarity
role C.low blood volume
A-Bicp Brachii Ans:C
B-Brachialis 135)Left recurrent laryngeal nerve
c-Extensor Carpi Ulnaris Ans:-Branch of Vagus Nerve(snell)
(Brachioradialis was not in the options) 136)Diarrhea
Ans:A ref snell -Alkalosis(it causes acidosis)
125)Widdrawal of cerebrospinal fluid -increased Sodium(cmdt)
needle moves in through 137)DELTA ANTIGEN...
Epidural-dura-sub dural-arachnoid(ans) a -acute resolvinng hep
126) About polycythemia rubra vera B- previous infection of hep
A-Person goes to himalyas C- chronic hep
B- Cancer of lung. Ans:C ref first aid)
C- myloproliferative disorders 138) 36 Years old woman with high grade
Ans: C ref cmdt fever and chills. Prescribed anti-malarial
127)Septic meningitis presents with cola coloured urine. Hb
A-chlymidia. 6mg/dl. A single test was done and was
B-streptococcus diagnosed as
Ans:B ref first aid) A-G6PD
128)Posterior inferior cerebellar artery B-Malaria Falciparum
blockage. Part of brain affected Ans:A past paper bcq
A-Dorsolateral part of medulla 139)45 yr old basiines man travelng far
B-Lateral area of Pons east complaing of abdominal discomfort.
Ans:A ref kaplan Tip of spleen palpable hilar
129)Retrograde amnesia lymphadenopathy and caseating
A:Trauma or blow to head granuloma in lungs
B:Damage to limbic system Ans-Wegners Granulomatosis
C:Can be reduced by dopamine agonist 140) A young boy presents in OPD with
Ans: B? Not sure cholesterol deposits (something like that)
130)Surgically lobes of breast on extensor surfaces of arms.
Ans:Separated by interlobar septa/stroma Serum cholesterol was 800. Family history
131) type of collegn in dermis of skin. of same problem was also present.
A-1 Mutations in gene of which protein involved
B-2 in cholesterol METABOLISM
C-3 A-HDL
D-4 B-LDL
Ans:A ref first aid C-Chylomicron
132) most imp buffer of body. Ans:B ref first aid type 2 familial
A-phosphate. hypercholestrolemia
B-protein 141)Correct sequence of events ?
C-hco3 A. Damaged valve, thrombus , perforation ,
Ans:C ref cmdt emboli
133)Tunica Vaginalis is derived from B. Damaged valve , perforation , thrombus ,
A-External Oblique aponeurosis bacteremia
132

C. Damaged valve , bacteremia , thrombus , 149) Temperature regulatory centre is


perforation present in
D. Bacteremia , thrombus , perforation , A-Thalamus
emboli B-Hypothalamus
Ans:A C-Basal Ganglia
142) LCX blocked. M.I in affected area Ans:B ref kaplan
A- ant surface of left venticl 150- Regarding thyroid
B- ant surface of right ventrical a)parathyroid lies posterior to anterior
C- posterior surface of left ventricle b)epithelium changes shape acording to
d-anterior 2/3 of IV septum function
Ans:C ref snell lat and post wall of left c)collecting ducts are present in gland
ventricle epithelium
143) perinephric fat and kidney is Ans:B past paper bcq
enclosed by. 151) Anemia in pregnancy is due to:
a.triangular ligament. hemodilution(past paper bcq)
b.gillsons fascia. 152)In Emergency
c.gerotas fascia A-Only patient should stay, family outside
d.peritoneum. B-Physchosocial support to the patient and
e.coronary ligament. family plays an imp role
Ans:C ref snell gerotas fascia is called renal Ans:B
fascia 152)Urachus---Patent Allantois(first aid)
144)350 plasma osmolality, urine 1250 153)Cerebral Blood Flow regulation
A-SIADH A-Low systemic pCO2
B-Water Deprivation B-Low pH
C-Diabetes Insipidus C-High Systemic PO2
D-Central Diabetes Insipidus Ans:A
E-dehydration 154)open wound healing=>Myofibroblast
Ans:A ref cmdt 155)Clostridium Tetani
145)Surfactant secreted from A-Lethal exotoxin
A-Type 2 pnemocytes B-Chronic reaction
B-Type 1 pneumocytes Ans:A past paper bcq
Ans:A ref guyton+brs+first aid 156) Eating Raw or undercooked Meat
146) Left 6th Aortic arch derivative. -Tapeworm ingestion(first aif:taenia solium)
A-hemizygous vein 157)Regarding thirst
B-Ductus venousum A-Inc plasma volume and dec osmolarity
C-Ductus arteriosum B-Dec Plasma volume
D-coronary sinus Ans:B past paper bcq)
E-none 158)Change in nuclear size, shape and
Ans:C polarity
147)A+ blood group. Transfusion -Dysplasia(past paper bcq)
reaction will occur with which blood 159) Open woumd healing
group Contraction of Myofibroblast
A-Ab- 160) Antigen-Antibody complex. T-cells
B-A+ reacts with
C-A- A-Antigen presenting cells
D-O+ B-Major Histocompatibility Complex
E-O- Ans:B
Ans:A past paper bcq 161Patient wth renal stones of
148)Skin disinfection A-furosamide,
-Wash hands with alcohol plus B-Hydrochlorohiazide
chlorhexidine(past paper bcq) C-Acetazolamide.
133

Ans-B past paper bcq Ans:A ref first aid+past paper bcq
161) Most common cause of HCC 170-Patient Doctor Relation strengthen's by
-Both hep B and C "Active Listening"(rabia bcq)
Africa and asia hep b》hep c for hepatoma 171-Urine dribbling from anterior
In usa and western countries hep c》hep b abdominal wall defect
for hepatoma ref cmdt 2016 page 1595 A-Patent urachus
162)Hep B most common transmission is B-Patent Allantois
via C-Median Umbilical Ligament
A-Through Contaminated Needles Ans:B
B-Orofecal route 172)Teratogenic among following
Ans:A ref cmdt A-Coffee
163) Medial Leminiscus B-Alcohol
-Nucleas Cuneatus and Gracilis(kaplan) Ans:B ref first aid fetal alcohol syndrome
164-A patient comes in ER in shock. 173-Morphine is used in "Terminal Cancer
Initial feature Patient Pain"
A-Bradycardia 174-Patient presented with weakness,
B-Rapid Pulse lethargy. Lab shows HB 5.6mg/dl, WBC
C-Increase Respiration 3.5, Platelets 12000. Diagnosis
Ans:B past paper bcq -A-Aplastic anemia
165-Pt with cancer T4N1M1 , weak, B-Iron Deficiency Anemia
lethargic and pallor +, such that these pts C-Thalassema Major
have low survival rate. What will be Ans: A ref first aid n cmdt
common to such pts ? 175)Lysosomes Contain "Hydrolyic
A. Cachexia Enzymes"(past paper bcq
B. HCC 176-A player injured his foot. First step to
C. cervical CA stop bleeding
Ans:A past paper bcq Ans-Vasoconstriction
166-Peroxidation of lipids 177)Pt has pain between upper lip and
A-Amyloid lower eyelid. Nerve involved
b-Lipofuscin Ans-Maxillary branch of trigeminal
C-Hemosiderosis SURGERY 7.11.16 EVENING
D-Melanin or Melatonin 1. Osteocyte
Ans:B a. periosteum for osteocyte for injury in
167)A businessman travels to fareast, bone
develops abdominal discomfort, jaundice, Ans:::: A
spleen just palpable? 2. Microscopic feature of compact bone
A-Tropical Spleenomegaly Syndrome A haversian canal arranged obliquely
B-Tropical Sprue B lacunae have osteoblast
C-Infectious Mononucleosis C haversian canal are not interconnected
D-Ulcerative Colitis collagen
Ans:A D lamellae are regularly arranged
168-Surgically Lobes of breast are? Ans::: D
A-Glandular tissue 3.which one is pivot joint
B-Stroma containing Gland a.median atlantoaxial joint
C-Separated by interlobar stroma/septa b.sternoclavicular
Ans:C ANS::: A
169-Discharging Sinus , yellow colour pus 4.Difference b/w pharyngeal and smooth
-A-Actinomycosis muscle of esophagus.
B-Staph Aureues a. ca bind to troponin C in pharyngeal
C-E.Coli muscle
134

b.smilar Ans::: b[c8 t1 is same as ulnar]


ANS::: A 15.Tumor in superior lobe of lung may
5.Mylenation in CNS involve which bronchopulmonary
a.Oligodendrocyte segment?
b.kuffer cells a. apical superior
ANS::: A b. superior basal
6.Postganglionic parasympathetic nerve (Ans::: a)
receptor are 16.Phrenic nerve supply
a. Cholinergic a. mediastinal pleura
b. adrenergic Ans::: A
ANS::: A 17.SA Node located at
7. During surgical removal of axillary A. upper part of sulcus terminalis [koch
lymph node a nerve damage, pt is unable triangle]
to flex and supinate at elbow? b. entrance of ivc
A. Radial nerve [Ans::: A]
b. axillary nerve 18.Superficial inguinal ring formed by
ANS::: B a. aponeurosis of external oblique
8.During surgical removal of lymph node b. internal oblique
in post triangle f neck, nerve injured? Ans: A
a. spinal Accessory nerve 19. In anterior duodenal perforation the
b.12th contents will go to
ANS::: . A A. right iliac fossa
9.After tonsillectomy , pt can’t taste B. left iliac fossa
senatation of post 1/3 of tongue which C. anterior subhepatic space
nerve damage D. posterior subhepatic space
a. glossopharyngeal E. small bursae
b.7th Ans::: A[if posterior lesser sac ]
ANS::: A 20.4th part of duodenum
10.After fall pt can’t abduct above the a. completely retroperitoneal
head which muscle paralysed b. partial retroperitoneal
a. supraspinatus muscle Ans::: A. ref FA
b .infraspinatus 21.Appendicular artery
Ans::: .A a. lies posterior to cecum
11.Dermtome of medial 1/3 of hand b. enter in mesoappendix
a.C8 ANS::: b
b.T1 22.right gastroepiploic artery branch of
C.C6 a. Gastrodudenal artery
D.C7 b. proper hepatic artery
(Ans::: a) Ans::: A
12.C8 contribute to 23.In porta hepatic which structure
a. lateral cord damage by compression of hepatoma
b. posterior cord/ a. Portal vien
(Ans::: b.posterior and medial mainly) b. cbd
13.Breast lie on Ans::: A
A Pectoralis minor 24.Tumor at head of pancrease
b. superficial pectoral major fascia compress??
(Ans::: b] a.portal vein
14.Sparing of thenar eminence all muscle b.. CBD
paralysed nerve damage? ANS::: b
a. radial nerve 25.Oblique fissure at lung
b. c8-t1 a.T3-T6
135

b.. t2..t4 ANS::: A[also known as trAns:::vers


ANS::: A cervical ligament ]
26.How to differentiate between right and 35.Aftr surgical procedure at uteus
left kidney female develops the urinary incontinence
a. by arrangement of structure most likely due to
b. no of vessel a. urinary bladder and vaginal fistula
ANS::: A[or by the length of renal vein b.fecovaginal
..left is longer ] ANS::: A
27.spleen true statement 36.injury at back pt unable to void , nerve
a. Length is 12 cm damage may be
ANS::: A a.S234
28.Regarding esophagus appropriate b. L5s1
stem ANS::: A
a. contain rt vagal trunk posteriorly 37.Atonic bladder due to
ANS::: a a. loss of inhibitory signal from midbrain
29.Regarding CSF true b. damage of pelvic splanchnic nerve
a. Drain into subarchnoid villi (Ans::: b)
b. absorbed by ependymal cells 38.rt testicular ca dain into
ANS::: A a. b/l paraaotic lymph node
30.Regarding the Filliform papillae: b. ipsilateral inguinal and ipsilateral aortic
a. These have no fibrous core ANS::: A
b. These are present at dorsum of tongue up 39.yolk sac tumor marker
to the tip[upper anterior 2/3] a. HCG
c. These are covered by soft epithelium b. afp
d. These are mushroom like in shape c. ldh
e. These are present on posterior 1/3 of ANS::: B
tongue 40.sympathetic function
ANS::: .b a. bronchodilation
31.During procedure at the b. bradycardia
submandibular nerve damage ANS::: a
a. Marginal branch of facial nerve 41.facial nerve cervical branch supply to
b. mandibular a. platysma
ANS::: A b .sternocleidomastoid
32. During swallowing food prevented ANS::: a
from entering to trachea by 42.infraorbital artery is the terminal
1) Epiglottis elevation over naso pharynx branch of
2) Close /-tight approximation of vocal cord a. facial artery
3) Movement of uvulva b. maxillary artery
4) Contraction of pharyngeal muscles c. sup temporal artery
5) Position of tongue (Ans::: b)
ANS::: B 43.sarcomere
33. true about superior parathyroid gland a. b/w two z lines
A. is at mid of posterior border of thyroid at b. A band
first tracheal ring ANS::: A
B. its position changing 44.fiber for fast pain
C. is not enclosed in pretracheal fascia a. A delta
D. all of above b.c fiber
ANS::: A ANS::: A
34.Major support of uterus 45.tuninng fork 256 hz frequency
a. cardinal ligament vibration perceive by
b. broad ligament a. pacinian corpuscle
136

b. miessner corpuscle ANS::: A


ANS::: A 55.true about great saphenous nerve
46.Most important response in shock as a a. passes in front of medial malleolus
whole b. passes in front of lat malleolus
a. CNS ischemic response ANS::: A
b. carotid body 56.Nerve supply of anterior scalneus
ANS::: A muscle
47.Carotid sinus a. Ans:::a cervicalis
a. sense rapidly decreasing arterial pressure b. brachial plexus
b. dec in volume c. C6,C7,C8
ANS::: A ANS::: [They are innervated by the fourth,
48.In action potential , depolarization by fifth, and sixth cervical spinal nerves (C4-
a. k efflux C6).]
b.Rapid Na influx 57.nerve damage in ant tibial
ANS::: B syndrome???
49.In cardiac cycle QRS complex prior to a. common perroneal
onset of b. sciatic
a. Isovolumetric relaxation Ans::: A
b. Atrial systole 58.Lymphatic drainage of toe
c. ventricular systole a. verticle group of sup inguinal
ANS::: c b. medial group of sup inguinal
50.Most important body mechanism in (ANS:::)a
maintaining te,prature in cold 59.which molecule trAns:::ported by
a. tachycardia secondry active trAns:::port
b. sweating a. glucose
c. Shivering b. protien
ANS::: c [Ans::: A..protein is a part of trAns:::porter
51. During treadmill exercise if heart ]
rate is increased to double the normal 60.If a man loss 1 liter blood in 5 min
rate, then cardiac cycle time will be: most important body response
A. 0.2 second a. inc vasomotor tone
B. 0.4 second b. inc heart rate
C. 0.8 second Ans::: is a
D. 1.2 second 61.sound produce by ventricular
E. 1.6 second vibration during rapid inflow
(ANS::: b will be half ] a. s4
52.parasympathetc action of heart b. s3
a. increase PR interval Ans::: b
b. increase systole time more than diastole 62.Most important stimulant for release
ANS::: A of aldosterone
53.fracture at greater trochanter after 7 a. Renin
days developed lower leg swelling ,pain b. k
and edematous most likely next ANS::: b
complication 63.substance dec the plasma osmolarty
a. fever a. dec vasopressin
b.pulmonary thromboembolism b. aldosterone
ANS::: B ANS::: A.inc plasma water content
54.patient is unable to invert damage may 64.hormone act ia steroid mechanism
occur a. gh
a. posterior and anterior tibial muscle b. t4
b.gastrocenamius Ans::: b
137

65.tidal volume 500ml r-r 10 calculate constriction


alveolar ventilation? b, increased by efferent arteriolar
a.3200 constriction
b.3500 ANS::: b..[ gfr inc and plasma flow dec ]
c.4200 75.41% ht emeAns:::
ANS::: b a. 41% rbc ,wbc and platelet
66.in bronchial asthma b.59 prcnt RBC
a. fev1/fvc<75 ANS::: A
b. residual volume decreased 76.In severe dehydration volume would
ANS::: A be replaced by
67.O2 –hb dissociation curve shift to left a.0.9 % n/s
a. acidosis b.ringer lactate
b. inc temp c.5 % glucose
c. alkalosis ANS::: C ganong bcq key is c there
ANS::: C 77.19 y ear boy sitting in well ventilated
68.main factor that shift k from room at temp 26degree he has sweating
intracellular and extracellular ,increase heart rate , main factor for these
a. sternous exercise changes
b. insulin a.exercise
ANS::: A b heat stroke
69.major intracellular buffer c.high bp
a. hb ANS::: A
b. protein 78.complement help in phagocytoss
c. po4 a.c3b
Ans::: B protein …. … The most important b.c5
buffer in the ECF and the body is HCO3- ANS::: A
(bicarbonate) which combines with excess 79.pt receiving chemotherapy for cancer
H+ ions to form carbonic acid. most imp mechanism to reduce the size of
70.pt died due to CRF , autopsy show tumor
which hormone function is increase a. apoptosis
a. pth b. atrophy
b. erythropoiten ANS::: A
ANS::: A 80.cushing syndrome by
71.kidney important in activation of? a. adrenal adenoma
a.b12 b. lung cancer
b. cholecalciferol ANS::: A..ref FA
ANS::: b 81.ant pituitary tumor
72.most important part of nephron a. are chromphobes
affected in shock ANS::: a
a.pct 82.dopamine given in parkinsonism will
b.dct inhibit
c.loop of henle a. prolactin
ANS::: A[ pct is the most susceptible to b. oxytocin
ischemic and toxic injury] ANS::: a
73.60% na reabsorbed in 83.after partial hepatetomy important
a.dct early hisytologic statement
b.pct a.inc GH alpha
ANS::: b…. majority of all stuff is b.EGF
absorbed in pct even water… ANS::: ..B
74.GFR 84.During defecation imp reflux is
a. increased by afferent arteriolar A. rectoanal
138

b. mass movement Ans::: b


ANS::: A 94.young girl with parasites , rashes
85.major diff in b/w primary and which type of allergy
secondary peristalsis a. TYPE 2
a. in primary oropharyngeal phase occur b. type 1
b. speed ANS::: B
ANS::: A 95. a pt undergoes
86.icam and vcam are kidney trAns:::plant on first day he was
a. leukocyte adhesion unable to produce urine there is also
b. antigen tenderness at trAns:::plant site
ANS::: a A} Graft versus host rejection
87.in heamophilia if cryoprecipitate is not {b} hyper acute graft rejection
available , alternative source of factor viii {c} blockage of ureter
a.FFP {d} renal artery stenosis)
b. stored blood {E} thrombus at the side of trAns:::plant
ANS::: a ANS::: b
88.a mother brought a boy for 96. young girl with ambiguous genitalia
circumsission he has history of bleeding having karyotype 46 xx , diagnosis
during umblical cord cut in both male a. androgen insensitivity syndrome
and female , on investigation PLT normal b. adrenocortical hypersensitivity
, bt is normal , only APTT increased , c. mixed gonadal dysgenesis
diagnosis Ans::: b… while in androgen insensitivity
a. vwb karyotype will b XY)
b. Heamophilia 97. chemical carcinogen
ANS::: b a. propyl alcohol
89.heparin actionas an anticoagulant it b. methyl alcohol
inhibits clot c. benzidine
a. propagation Ans::: c
b.formation 98.nasopharyngeal ca caused by
ANS::: A a. rhinovirus
90.a young boy short limbs ,normal trunk b.EBV
,normal intellectual what would be the ANS::: b
pattern of inheritance 99.A young boy complain of abdominal
a.autosomal dominat pain , investigation show haing mass with
ANS::: A blue cells and inc level of
91.mediator of allergy catecholamines,dianosis
a. 5ht a. neuroblastoma
b. lymphokines .b.adrenal adenoma
c. serotonin Ans::: a
d. bradykinin 100.ca colon 1 metastasize to
ANS::: A a.liver
92.A pt admitted in oncology ward with b.mesentric lymph node
hypotention with investigation on he has Ans::: a
positive blood cultutre of pseudomonas 101.virus act by oncogenesis by acting on
which is mediator for his condition A.rNA
a. tnf alpha B.DNA
ANS::: A C.protooncogen
93.most important prerequisite for renal Ans::: b
trAns:::plant 102.54 year old male came with dyspnea
a. HLA compatibility with h/o cogh ,trAns:::lucency of upper
b. ABO comptabilty lobe , no infiltration prominent
139

pulmonary arterial marking a.cataract


a.arteriosclerosis b.imperforate anus
Ans::: A Ans:::a A
103.10 year old boy while playing 115. Data is collected for some clinical
suddenly collapsed and died what will be trial based on presence or absence of
diagnosis Vomiting. What type of data it is:
a.kawasaki disease A. Nominal
b.gian cell arteritis B. Ordinal
ANS::: a C. Parametric
104.ECG changes on lead v4 ,which Ans::: A
artery will infarct 116.type of collagen in reticularis
a. LAD a.type 2
b.rca b. type 3
Ans::: A… septal infarct ANS::: b
105.Case of infective endocarditis, 117.down syndrome trisomy
definitive diagnosis by a.21
a. blood culture ANS::: A
ANS::: a 118.Thiamine deficiency cause
106.myxometous degeneration seen in a. Wernicke korsakoff syndrome
A. aortic valve b. polyateritis
B .Mitral valve prolapse ANS::: a
ANS::: b 119.high cardiac output failure
107.2 year boy raised mc 109 , lethargic , a. thiamine deficiency
fatigue , diagnosed by b. b12 def
a. serum b12 ANS::: a
b. antiintrinsic antibody 120.Glycolysis
Ans::: A a.in aerobic glycolysis pyruvate is an
108.multiple drug therapy in tb due to intermediate b/w gglycolysis and TCA
a. dec side effect b. energy isnt produced
b.decreas resistant Ans::: A
Ans::: b 121.daily requirement of helathy man of
109.pt with deranged lfts and ring in eye 70 kg
a. Wilson disease a.25-30kcal
Ans::: A ..keyer fishcher ring.. b.15-20 kcal
110.a sexulay active male complainof Ans::: a
burning micturition , no ulcer on penis 122.1 gm of hb bind to
urine culture negative for n gonorrhea a.1.34 ml of o2
a. candida ANS::: a
b. chlaymdia 123.clostridia is
Ans::: b a. spore forming
111.hormone inhibit release of gnrh b. easy to destory
release in darkness’ Ans::: A
a. melatonin 124.pt live at river and got eyes problem
Ans::: a ,which nematode involve
112.sarcoma feature a. oncocerca volvulus
a. vimentin b. leshmania
Ans::: A ANS::: A… o for optic
113.sqamous cell ca feature 125.regarding MRSA
a. keratin a. sensitive to vancomycin
114.pregnant lady infected with rubella , b. not treatable
baby born will have risk of Ans::: a
140

126.regarding aminoglycoside ANS::: a


A .monitor in elderly pt with renal problem 131.Melanocyte develop from
b. high safety profile a. neural crest cell
Ans::: A b. endothelium
127.NSAIDS ANS::: a
A. Inhibits cox pathway 132.left sided lower limb paralysis with
b. inhibit phospholipase left face and left sided homonomys
Ans::: a hemianopia lesion at
128.Drug receptor interaction a.midbran
A. heparin with protamine calcium b.pons
b. manitol diuresis c.medulla
ANS::: b heparin antidote is protamine ANS::: b
sulphate 133.tremor at end of movement in left
129.male siting with eyes closed which hand , hypotonia of left hand with ataxia
waves dominant on EEG Lesion at
a. alpha wave a. left cerebellar hemisphere
b. beta Ans::: A
c.theta 134.loss of GABA b/w G.pellidus and
ANS::: A..ref FA sustantia nigra
130.cholinergic fiber a.athetosis
a. all postganglionic parsympathic b.hemibalismus
b.. all preganglionic Ans: a
141

GYNAE/OBS

2016 PAPERS
SEPTEMBER, OCTOBER, NOVEMBER

Gynae and Obs 7th September c.Acetate


d.protien
2016 (evening) e.phosphate
1.Lateral attachment of urogenital Ans: : bb
diaphragm; 4.Content of fat in male is?
a. Sarum n coccyx a.10%
b. Obturator internus membrane b.15%
c. Inferior ramus of the pubis and ischial c.40%
ramus d.60%
d.ischial tuberosities e.17%
e. tip of coccyx Ans: : bb
Ans: : cc 5. Erythropoetin is secreted by?
2. The most appropriate therapy for a. endothelial cells of the peritubular
endometriosis is capillarie
a.Danazol b. jg cells
b.Norgesterol c. adrenal cortex
c.Oral androgens d. glomerulus
d.Oral contraceptive e. loop of henle
e.Raloxifine aAns: : aa
Ans: : aa 6. 1.The richest source of vitamin ...?
3. important buffer of blood? a. whole milk
a. Hb b. butter
b.hco3 c. margarine
142

d. liver cord oil d.nausea , vomiting


Ans: : dd Ans: : aa
7. concentration of which of the following 14.How does body adapt to cold
hormone doesnt increase during temperature in long term.
pregnancy: a .Internal pyrogens
a) PTH b .Chemical thermogenesis
b) cortisol c. Formation of brown fat
c) GH d. peripheral vasodilation
d) prolactin e.pheripheral vasoconstriction
Ans: : cc Ans: : c
8. Size of ureter is 15.which of the following factor deficiency
a. 20-25 cm cause coagulation?
b.25-30 cm a.5
c.30-35cm b.7
d.35-40 cm c.8
e. 15-20cm d.10
Ans: : bb ( 25 cm is correct ) e.12
9.Blood supply in midline of flap of Ans: : aa
forehead ?? 16. hyperbaric oxygen treatment is most
a.frontal branch of frontal artery useful in?
b. paired supra trochlear artery a.CO poisoning
c. optahlmic branch of trigeminal artery b.Anemia
d.facial artery c.Polycythemia
e. paired Supra orbital artery d.decompression sickness
Ans: : Bb e.Gas gangrene
10.A fetus was aborted .it was sent to Ans: : aa
forensic lab .how can the viability be 17. 2 feet away from ileocolic junction at
checked .that for how long the fetus was antimesentric border
viable.? a.urachal fistula
a.Fetus weight 200g b.meckel diverticulum
b.Eyes laterally Ans: : bb
c.Fetus CRL >250 mm 18. tunica albuginea
d.Absence of wrinkles a.Contain testes only
e.Absent toe nails b.Derivative of parietal peritoneum
Ans: : cc c.derivative of visceral peritoneum
11.what is absent in C1? d.covers ductus deferens
a.Spinous process Ans:: a & b both correct
b.Archc.trAns:verse process 19. Costo diaphragmatic recess
d.Lateral mass a.seen only on left side
Ans: : aa b.Wide open space in centre
12. difference between roots of right lung Option missing
from left lung? 20. deep ring of inguinal canal attaches
a.No of primary bronchi a.Fascia tranvesalis
b.No of arteries b.Tranversua abdominus
c.No of veins c.External oblique
d.No of nerves plexus d.internal oblique
Ans: : aa Ans: : aa
13. cisplatin has the following side effect 21.An anesthetist wants to give his patient
a.Nephrotoxicity most potent muscle relaxant who is about
b.Neurotoxicity to undergo cesearation so that she may
c.hepatotoxicity relax completly.he would give
143

a.Bupivacaine 29.Virus acts on cells by:


b.Lidocaine a:Damaging cell membrane,
Ans: : aa b:Damaging nuclear membrane,
22. Root value of femoral nerve c:Invloving protien synthesis,
a.L2 ,L3, L4 d:Changing protien structures.
b.L2,L3,L4,L5,S1,S2 e:Changing protien metabolism
c.L1 ,L2 Ans: : c
Ans: : aa 30. Aspirin and other NSAIDS work by
23. A patient complains of arthritis and acting on
discoid rashes. Which of the following a.Lipoxygenase
would be specific? b.Cyclo-oxygenase A2
a. Ds DNA c.HMG Co A reuctase inhibitor
b. ANA Ans: : b
c. AMA 31. Which of the following is a spore
d.Anti ccp forming bacteria?
Ans: : aa a. clostridium
24. At the end of marathon race , a person b.cornybeacterium
has ? c. Chlamydia
a. high insulin low glucagon Ans: : a
b. high glucagon & low insulin 32. microscopic feature of malignancy
c. high cortisol a.Invasion
d. high insulin and glucagon b.Pleomorphism
e.low insulin and glucagon c.N/C ratio
Ans: : bb Ans: : a
25. A 25 years old married female came to 33. A patient with pulmonary embolism
you with history of amnoerhea for 3 and extensive tissue injury. What is the
weeks her beta hcg shows that her most probable cause of his dx.
pergnanacy test is negative. What could a. Dvt
be the cause? b. Thromboembolism
a. ectopic pregnancy c. Atril myxoma
b. anovulation Ans:: bb
c. primary hypothalamic disorder 34. Aldosterone acts on?
d.Anorexia Nervosa/ normal ? a. Collecting duct
Ans:: dd b. Dct
26.test for difference of two meAns: c. Cortical collecting tubule
A. T – test Ans: : aa
B. SNOVA 35.Plasma proteins return to normal after
c. ANOVA a.2 -3 days
Ans:: aa b..7-8 days
27. In Tuberculosis, antibodies are c.10-12 days
a. cell bound d.12-14 days
b. inside cell Ans: : c
c. absent 36. An abnormally tall boy of 14 year age
Ans: : a consults his physician for his abnormal
28. Toxoplasmosis is height and secretions from his mammary
a. parasite gland. X ray skull shows enlarged
b helminth pituitary gland. Histological examination
c.bacteria of pituitary is likely to show:
d. virus a. Decreased number of pituicytes
e.Fungi b. Hessing bodies seen in karge number.
Ans: : a c. Increased no of basophill cells.
144

d. Increased no of chrimophobes Ans: : a


e. Increased no of eosinophills 43. Platelets
Ans: : e a.half life of 10 days
37. prolactin raised with infertility what b. Increase after spleenectomy
will be cause c.granules contain serotonin
a.microprolactinoma c. Stimulated by growth hormones
b.macroprolactinoma Ans: : b( remember half life is 4to 5 days
c.pituitary tumor life span is 10 days a big differnce)
Ans: : bb 44.The ascent of kidney is prevented by
38. Diagnostic test for typhoid in 2nd A.external iliac artery
week is : B.internal iliac artery
a.Blood culture and widal test . C.superior mesenteric art
b. blood culture D.ivc
c. stool culture E.inferior mesentric artery
d.widal test Ans: : e
e. urine culture 45.which of the following structure has
Ans: :aa both intra & extra pelvic extension
39. Substance which act as both hormone A.broad ligament
and nurotrAns:mittor? B.ovarian ligament
a. adrenaline C.trAns:verse cervical ligament
b.cortisole D.round lig of ovary
c . acetylcholine E.round ligament of uterus
d. thyroid Ans: : e
e. Aldosterone 46.most potent chemotactic factor
Ans: : aa A.c3a
40.Stimulation of parasympathetic nerves B.c5a
to the heart decreases heart rate by C viruses
a. Increase in permeability of K thru D.arachidonic acid metabolites
ventricles Ans:: b
b. Incraese in permeability of k through sa 47.Rbcs are
node A.biconvex
c. prolonging of AV nodal delay B.nonfragile
Ans: : cc C.largest of all cells
41..Starling's law of the heart: D.have glycolytic enzyme activity
a.Does not operate during exercise Ans: :d
b.Is not obeyed by the failing heart 48.lysosome contains
c.Explains the increase in heart rate during A.glucose
exercise B.Na+
d.Explains the increase in cardiac output C.ca +
when venous return increases D.acid phosphatase
e.Explains the increase in cardiac output due E. Water
to sympathetic stimulation Ans: : d
Ans: : d 49.gram -ve infection causes septic shock
42.A lady presented with per vaginal in which of the following tracts
bleeding,history of bleeding disorder and A.biliary tract
having a dead fetus ..first step of B.colon
management C.small bowel
A.blood TrAns:fusion D.urinary tract
B. Evacuation of uterus E.genital tract
C.platelet trAns:fusion Ans: :d
d.iv heparin
145

50.flexor reflex by B.ischemia


A.Muscle spindle C.vit c deficiency
B.Monosynaptic D.cortisol
C.messner corpuscle Ans: : a
D.noxious stimulus 57.tricuspid auscultation area
E. By Tapping with hammer A.2nd right intercostal space
Ans: : d B.4th left intercostal space
51.study on sexually trAns:mitted C.mitral area
disease,one group of boys had penile D.lower half of right border of sternum
ulcers and the other group had no Ans: :d
ulcers,non ulcerating leisons were d/t? 58.aortic hiatus aneurysm compresses
A.treponema paliidum A.thoracic duct and vagus nerve
B.chlamydia B.ayzgous vein
C.HPV C.phrenic nerve
D.ducreyi Ans: : b
E.Gonorhea 59.pulmonary wedge pressure
Ans: : ee A.measure of pulm artery pressure
52..barr body test in saliva is diagnostic B.measure of rght atrial pressure
for C.measure of left atrial pressure
A.klinfelter syndrome Ans: :c
B.gonadal dysgenesis 60.about csf
C.turner syndrome A.It is an ultra filtrate of plasma
Ans: : c B.provides nutrition to cns
53..You are testing efficacy of unknown C.formed by arachnoid granulation
unnamed drug A and drug B in an D.separated from interstitial fluid by blood
infection...then randomnly allocated to brain barrier
pts recieving both of them..it is an Ans: :d
example of 61.growth hormone stimulation
A.double blind/placebo A.rem sleep
B.single blind B.wakeful condition
C.triple blind. C.hypoglycemia
D.cross sectional study D.free fatty acids
Ans: : a ( RCT is selected in the case when Ans: : c
a new drug is being tested for its efficacy ) 62.which one of the following is
54.anemia of maturation mesenchymal in origin
failure..immature cells in periphery A.choristoma
A.iron def anemia B.hemartoma
B.normocytic C.sarcoma
C.pernicious anemia D.carcinoma
D.microcytic anemia E. Choriocarcinoma
Ans: : c Ans: : c
55.the difference of osmolarity b/w 63.in the first step of thyroid hormone
interstitial fluid and plasma is synthesis iodine attach with
A.1mosm/L A.albumin
B.1.5Mosm/L B.thyroxin
C.3 Mosm/L C.thyroglobulin
D.5 Mosm/L D.tyrosine
E.no difference E.hb
Ans: : a Ans: : d
56local cause of delayed wound healing 64.tumor with all three germ layers
A.infection A.teratoma
146

B.adenocarcinoma 71.measured by spirometry


C.hamartoma A.residual volume
D.sarcoma B.Total lung capacity
E.seminoma C.residual volume
Ans: : a D.vital capacity
65.female child,breast fed developed Ans: : d
jaundice on 2nd day hb 9 bilirubin 8 72.the obsteteric nerve block cause
unconjugated urinary incontinence,anaesthetized
A.mother is RH Postive muscle is located in
B.breast fed jaundice A.pelvic diaphragm
C.physiological jaundice B.urogenital diaphragm
Ans:: c C.superficial perineal pouch
66..bifurcation of common carotid artery D.none of the above
is felt at Ans: : b
A.anterior border of sternocleidomastoid at 73 true about superior parathyroid gland
the superior border of thyroid cartilage A.is at mid of posterior border of thyroid at
B. post border of sternocleidomastoid in first tracheal ring
posterior traingle B.its position changing
C.at level of trapezius C.is not enclosed in pretracheal fascia
D. post border of sternocleidomastoid at the D.all of above
level of hyoid Ans: :a
Ans: : a 74.which structure has 3 layers of muscels
67.an adult in sitting position,resonance A.fundus of stomach
can be auscultated from back of the chest B.esophagus
till C.one of the above
A.6th rib D.all of the above
B.8th rib Ans: : a
C.10th rib 75.which of the following is an example of
D.11th rib primary active trAns:port
E.12th rib A.H+K pump
Ans: :bb ( clinical methods: Bedside) B.Na glucose co trAns:port in intestine
68.rbcs C.Na-Amino acid co trAns:port in intestines
a.lack mitochondria D.none of the above
b.half life of 180days Ans: : a
c.released from bone marrow as mature 76.alpha receptors effects
erythrocyte A.contraction of radial muscle of iris
d. Present on mhc 2 B.vasodilation in skeletal muscle
e.are the largest cells of the body C.increase in activity of gut
Ans: :a D.decreased heart rate
69.scenario of increase serum Ans: : aa
calcium,irregular bone,also bone 77.regarding parasympathetic supply of
pain...which of the following causing it heart
A.PTH A.Left superficial cardiac plexus
B.GH B.vagus nerve
C.vit D deficiency C.sympathetic system
Ans: : a D.Left deep cardiac plexus
70.lysosomal storage disease with mental Ans: : b
retardation and blindness 78.which of the following supresses
A.taysachs disease growth hormone release
B.granulomatous disease A.Puberty
Ans: : a B.sleep
147

C.somatomedin b.O+ To B+
D.starvation c.A- To A+
E.stress Ans: : a
Ans: : c 86. Pregnant lady with mitral stenosis,
79.A female with regular cycle of 21 to 23 pt.developed dysponea.orthopnea and
Days will most likely be ovulated on difficulty to breath. Most probable cause
which of the following days would be?
A.12-14 a.Right heart failure
B.14-18 b.Pulmonary htn
C.7-9 Ans: : Bb
D.10-12 87.on a study about diabetes between 2
Ans: : c groups it was found
80.trypsinogen is activated by Women with mean 200±50
A.enterokinase Men with mean 205±10, this meAns:’
B.peptidase a. Women with low mean and high standard
C.amylase deviation
D.acid phosphatise b.Man with high mean and low standard
E. HCL deviation
Ans: : aa c.Both have no link
81.most early sign of vit A deficiency is d. Case control
A.keratomalacia e. Cross Sectional study
B.cheilosis Ans:: aa
C.hyperkeratosis 88. which is obsevatinal study
D.diminshed dark adaptation/night blindness a.Case control
Ans: : d b.Cohort
82.a female presents with jaundice her lab c.cross sectional study
reports show Alt 17,alk phosp.75 and Ans: : b
retics 10.cause of jaundice 89. breast outer quadrant lymph drainage
A.hemolytic anemia a.Ant./pectoral
B.pernicious anemia b.central
C.iron def anemia c. apical
D.all of the above d. posterior
Ans:: a Ans: :a
83.true regarding cavernous sinus 90. testicular lymph node drainage
A.occulomotor trochlear and opthalmic of a. Paraaortic
trigeminal present in lateral wall b. superficial inguinal
B.connected posteriorly to facial vein c. deep inguinal
C.drain directly into straight sinus d. para umbilical
D.has abducent nerve in medial wall Ans: : a
E.Related Laterally to sphenoid Sinus 91. superficial perineal poch content
Ans:: aa a.Bulbourethral gland
84.A man with bee sting,develop swelling b.Perineal nerve
and acute infalmaation.mediator for it? c.Sup.trAns:verse perineal muscle
a.Histamine d. internal pedundal vessels
b.Bradykinin Ans: : c
c.prostaglandin 92. branchial cleft
d.leukotrine a.Open laterally
e. Esinophills b.Open laterally ant.to sternocleido mastoid
Ans: : a Ans: : b
85. severe blood reaction occur 93. draining sinus and granuloma is the
a.A+ To 0+ feauture of
148

a. aspergillus Ans: : b
b. nocardia 102. Person wid bloating abdominal pain
c. actinomycosis very foul smelling stools .First
Ans: : c Investgation would be
94. Which of the following helps in a.Cumplete stool d/r
formation of the bones? b.Stool cs
a. osteoblast c.Lipid profile
b.osteoclast Ans: : a
c.osteoprogenitor cells 103.most Comon mediator of fever
d.fibroblast a.1L-1
Ans: : a b.Tnf.
95. more central value of no. In series c.. IL 6
a.Mean Ans: : a
b.Median 104. increase in ECF osmolarity Lead to
c.Mode a. stimulate plasma vol and osmoreceptors
Ans: : b And stimulate adh
96. A child since he was born was having b. Inc adh and also nhibit osmoreceptors nd
problem of urine leaking from ventral vol
surface it may be due to? c.inhibits adh and stimulate vol and
a. Epispadiasis osmoreceptors
b. Failure of urogenial folds to close d.stimulate adh and inhibit vol and
Ans: : b osmoreceptors
97. Lacunar ligamnt forming medial Ans: : a
boundary , fibres enrolled from 105. Person having history of trauma on
a.Inguinal ligament the left upper arm he develop swelling in
b.TrAns:verse ligament arm that was painful for 3months it was
c.Relected ligament well circumscribed 4 cm and on xray
Ans: : a appeared bright then it gradually
98. Malignacy is caused by which of the deceased in one year most Likely ....
following a.Metaplasia ...
a.Silicosis b. Hyperplasia ....
b. Asbestosis c,.Neoplasia .....
c.Smoking d.Dysplasia ...
Ans: : b e.Hypertrophy
99. ADH is inhibited by Ans: : e
A.Alchol 106. Irregular growth of bone is due to
B.increase Body water the effect of which of the following?
Ans: : a a.leydig cells ....
100. Sympathetic autonomic system true b.Ant pituary hormone effct
is.... c.Pth
a.Adrenergic preganglionic d.GH
b. Thoracolumbar outflow Ans: : d
c.Short postgang fibres
d.pre and post ganglionic short fibres
PAPER B GYNAE
Ans: : b 1.Drug via oral route having aqueous
101. Most common bacteria involved in absorption ....
STDs is a.Digoxin
a.hpv b.ephidrine
bChylmydia c.Phenobarbital
c.Gonnorhea d.diazepam
d.Hsv e.carbamizapine
149

Ans: :a a.condyloma
2.Cerebellum .... b.Papilloma
a.Iniates movemnt .... c.Herpes
b.stabalizes movement and coordination Ans: : b
Ans: : b 10.Long tubular structure having mucosa
3.Most common cause of bronchogenic and trAns:itional epithelium
carcinom a.ureter
A Tobacco b.kidney
B Asbestos c.urethra
Ans: : a Ans: : a
4.Female pale lethargic .didnot take any 11.During sexual act secretion from vagin
antenatal care .... During delivery she are effect of
bleeded heavily .... Which nutrient a.Bombesin
deficiency would be present in both baby b.Relaxin
And mother ? c. Progsterone
a .Vit k d.gnrh
b.Iron e. Parasympathetic stimulatiom
c.calcium Ans: : e
Ans: : b 12.Measurement of amount of lead in
5. permeation is defined as body
a.tumor invasion to blood vessels a.absorbed from git
b.tumor invasion to lymphatics b.absorbed from respiratory tract lower
c.tumor spread to adjacent tissues lobes.
d.distant metastasis c.absorbed from skin
Ans: : b d.absorbed from nasal passages.
6.Pt presented wid leg ulcer and severe Ans: : a
pain And on blood examination howell 13.About zygote implantation
jolly bodies present wats the diagnosis ? a.28cell stage.
a.Sickle cell b.3-4days after fertilization
b.Hereditory spherocytosis c.invasion of endometrium by
c.beta thalasemia syncitiotrophoblast
d.alpha thalasemia Ans: :c
Ans: : a 14.trigone of bladder
7.Longst acting insulin ? a.ureters entering as oblique slits
a.Porcine lente b.devoid of peritoneum.
b.Human lente .... Ans: : a
c.bovine Ultra lente 15. Carbonic anhydrase act on which
d. glargine part of kidney
Ans: : d ( if glargine not in option select a.pct.
Ultralente) b.early dct
8. 7yrs child anemic for two years with c.cortical collecting ducts
blood investigations shows decreased hb d.calyces.
6.5 also Hbf 97% , Hba2 4% no history of Ans: : a
blood trAns:fusion . spleen palpable 16. person given NaCl one litre infusion
a.Thalasemia major wat will happen
b.thalasemia trait a.osmolality increases in plasma
c.herditory sphero b.osmolality decreases in urine
d. hereditary persistence of Hbf c.na increases in plasma
Ans: : a d.na increases in urine
9.Female wid Warty lesion on vulva. Ans: : D
150

17. isonizid not metabolised in people b.arginine


with deficiency of c.ammonia
a.glucoronyl tranferase Ans: :c
b.hyloroinidase 26. fracture of clavicle inner side and
c.pseudocholinesterase medial half , upward displacement takes
d.xanthine oxidase place it is due to which muscle
e. amine oxidase a.sternocleidomastoid
Ans: : a b.subclavicular
18. cells having oxident stress as oxygen c.trapizius
radicals were produced which one of the d.perctoralis major
following can decrease this effect e.deltoid
a.nadph oxidase Ans: : a
b.glutathione oxidase 27. inner medial lower quadrant of breast
Ans: : b doesnt drain into
19. In end stage renal failure whats a.anterior axillary
affected on prolong basis b.inf phrenic
a.erythropoeitin decreased c.supraclavicular
b.thyroid hormone decreased Ans: : a
Ans: : a 28. which nerve in urogenital triangle is
20. propofol contraindicated in damaged
a.hypersensitivity a.post cutaneous nerve of thigh
b.renal disease b.ilioinguinal
c.cardiac disease c.genitofemoral
Ans: : a d.femoral
21. Toxic effect of lignocaine? e.Pudental nerve
a. Prolong seizures for 9-7 hours Ans: E
b.hypotension 29. protamine side effect
c.Perioral anesthesia a.hypotension
d.rash b.bleeding tendency
Ans::aa Ans: : b
22. data from smaller study on one group 30. after abdominal hysterectomy on 3rd
of people is combined with data from day post op there is serosanguinous
larger study what type of data is this amount of fluid from the wound wats the
a.case control cause
b.randomised control trial a.wound infection
c.cohort b.wound dehiscence
d.meta analysis c.wound hematoma
Ans: :d Ans: : b
23. cerbral metabolic rate of oxygen 31. 7yrs old child with coughing and
a.15-20ml/min/kg mucus inc eisoniophills plug on xray it
b.2-3 ml/min/kg shows hyperinflation and infiltrates whats
c.3-4ml/min/kg the diagnosis
Ans: :c a.cystic fibrosis
24. hyaline disease of new born b.bronchial asthma
a.maternal diabetes c.gastric aspiration
b.maternal eclampsia d. hypersensitivity pnumonitis
c.prematurity aAns: : b
Ans: :c 32. normal menstrual blood flow is less
25. urea is formed by liver in the body to than
get rid of a.80ml
a.ornithine b.120ml
151

c.40ml 41. Superior and inferior mesenteric


d.60ml arteries anastomose on
Ans: :a a. marginal artery
33. women during menstruation uses b. colic
tampons presents .she develops vomiting c. left ileocolic
diarrhoeahypotension high grade d. sigmoidal arteries
fever.which organism is involved in it? Ans: : a
a.staph aureus 42. after amoebic infection , rectal biopsy
b.e colli was done now to confirm which test
c.gardnerela vaginalis would be done
d.chlaymidia a. pas
e.gonnorhea b. electron microscope
Ans: : a c. perl reaction
34. remnant of epiphiseal plate d. antibodies against amoeba
a.cortical bone Ans: : d
b.trabecular bone 43. lethal hepatitis in pregnancy.
c. spongy bone a. Hep E
Ans:: c 44. Thickish white curdy discharge.
35. locally malignant cancer a. Candidasis
a.melanoma 45. Erythroblastosis. ........
b.adrenoblastoma a. Type 2 Reaction
c.pleomorphic adenoma of parotid 46. HCG secreted by
d. Adenoid cystic Carcinoma a. Syncytotrophoblast
e. BCC b. Cytotrophoblast
Ans:: ee Ans:: aa
36. blood pumped by heart per min 47. trachea ?
a.750ml/min a. related ant to thyroid.
b.6lit b.fibrocartilagenous ring
c.5lit c. Ends by dividing at the lower border of
Ans: :c T4
37. esmolol is given in anesthesia because Ans: : c
a.its selective beta blocker effect 48. Blood supply to rectum is from....
b.decreses pain’ a. superior and inferior rectal arteries.
c. doenot increase icp 49. Captopril... mechanism of action?
d. doesnot cause rebound hypertension a. Inhibits angiotensin II
Ans: : a b. ACEI
38. bladder ca Ans:: bb
a.schistosomia hematobium 50.Tetanus
b.japonicum a.exotoxin has lethal effect
c.mAns:oni b.exotoxin afect on cerebral
Ans:: a c.no role of active immunity
39.Cotisol decreases.... d.trismus is rare.
a...glucose utilization in tissues Ans:: a
Ans:: aa 51 Which one is precancerous condition
40. before Ovulation maturation of a..myelodisplastic syndrome
follicles occur by action of b.cervical erosion
a.Fsh c.osteoporosis
b.Estradiol d.psoriasis
c.Gnrh Ans: : a
d.Lh 52.Parasympathetic effect on lungs
Ans:: a a.inc compliance
152

b.inc restrictive work 61. a 7 years old boy had pharyngitis with
c.inc anatmicl dead space halitosis and peritonsillar abcess
d.inc elastic work a. cornybacterium diphtheria
Ans:: b b. streptococcus
53. cell responsible for innate immunity c.staphylococcus
are activated most commonly by Ans::b
a carbohydrate seq in bacterial cell wall 62.Spinal cord with dura arachnoid and
b cytopasmic protein of bacteria pia mater in adults terminates at the level
c eosinophil of lowr border of: L1
d glucocorticoid 63. coccygeal ligament at the level of?
e pollen a.L1
Ans:: a b.L2
54.regarding cilia: c .S1
a) stereo cilia are a type of cilia d.coccyx
b) can be seen on light microscopy Ans::a
c) can be seen on electron micscopy 64. u did a study and it rejected null
d) has absorptive function hypothesis it mean
e) cannot be seen under light microscopy a.There z difference
Ans:: d b. there z no difference.
53.Tb is closely associated with .... c. study of research is not valid.
a. .hypersensitivity 4 d. ur research is not powerful enough
56. Alpha adrenergic antagonist... Ans::a
a. doxazocin 65.Abducent arise from
b. clonidine a.midbrain
Ans:: a b.pons
57. intermediate mesoderm give rise to c.medulla
a. Somites Ans::b
b.Muscle 66.Which structure connects skeletal
c.Heart tube muscle to bone ?
d. genitourinary system a.Tendon
Ans::d b.ligament
58.Paramyxovirus; in control group c.joints
causes for which aso titre was being done Ans::a
a.Glomerulopnephritis 67.true about spinal nerve ?
b.Sydenham chorea a. derived entirely from nueral crest
c.Arthritis b. pas trough intervertebral formina
d.Carditis c. dorsal root contain both sensory n motor
e.erythema marginatum supplies
Ans:: cc Ans: :b
59. New anti hypertensive drug want to 68.Notochord forms:
check potency. a.Nucleus pulposis
a.RCT/ Metaanalysis b.Annulus fibrosis
b. Double blind Ans:: aa
Ans:: aa 69. In females neck of urinary bladder
60.Stroke volume increases? a.lies above urogenital diapgrahm
a.Venous dilation b.lies above urogenital triangle
b.Inc in heart rate at constant cardiac output Ans:: aa
c.venous compliance 70 nerve damage in surgical neck of
d.inc in blood volume humerous fracture
Ans::dd a. Axillary
b. Radial
153

Ans:: aa a. perineal body


71 Regarding carcinoma tissue used for 84 Ip3 mechanism is example of
staining and for diagnosis a. Adh
a. Cyto keratin Ans:: aa
b. Vimentin 85 Content of superficial perineal pouch
Ans:: aa a. perineal body
72.Mensturation occurs when b. bulbourethral glands
a. progesterone decreases c. membraaneous urethra
b. progesterone increases d. internal pudendal nerve
Ans:: aa
73 Lateral relation of lower vagina e.perineal body
a. ureter Ans: E
b.uterin tube
Ans:: aa
74 most common cause of bronchogenic
carcinoma GYNAE OBS 14TH OCT
a. tobacco MORNING
b. Asbestos
Ans:: aa Q1.During surgery ovary removed from
75 Regarding lignocaine body in which lymph nodes ovary drain?
a. causes Na channel blockage A)External iliac
b. causes Cl channel blockage B)Internal iliac
Ans:: aa C)Para aortic
76 osmotic diuretic acts on D)Sup.inguinal nodes
a. proximal convulated tubules ANS..C
b. DCT Q2.UroRectal septum divides into?
Ans:: aa A) Rectum and urogenital sinus
77 NO derived from b) Anus and urogenital sinus
a. ornithine , arginine, citruline ANS…A
b. leucine,isoleucine, lysine Q3.Levator ani supplied by?
Ans:: aa A) Hypogastric plexus
78 Reactive hyperemia B)Pudendal nerves
a. red infarct ANS..B
b. white infarct Q4.Bladder supplied by?
Ans:: aa A) Superior hypogastric plexus
79 lysosomal storage disease B) Inferior hypogastric plexus
a.tay sach disease ANS…B
Ans:: aa Q5.Detrusor muscle control by?
80 regarding superficial parathyroid A) Pudendal nerve
a. located at mid of thyroid posteriorly at 1st B) parasympathetic and pelvic nerve
tracheal ring C) ilioinguinal nerve
Ans:: aa ANS….B
81. S1 = isovolumetric contraction Q6.Toxoplasmosis is?
82 man with progressive dyspnea, A) Parasite
bilateral pitting edema upto knee , B) Bacteria
bilateral crypts, bilateral pleural effusion ANS….A
on x ray what will be the lab finding Q7.Pseudomembranous colitis caused by?
a. increase Na A) Cl.difficile
b. decrease Na B) Cl.botulism
Ans:: aa ANS….A
83 Superficial perianal pouch Q8.Detrusor supplied by?
154

A) Sup inguinal B) Eosinophil not change


B) Pudendal nerve C) Steroids cause no change in eosinophils
ANS….B D) Lymphocytes decrease
Q9.In a female during during labour ANS…..A
mediolateral episiotomy performed which Q16.Regarding immunological response?
muscle cut? A)Palatine tonsils covered by simple
A) Bulbospongiosis squamous epithelium
B) Levator ani B)Antibodies produce and release by plasma
C) Sup transverse perineal muscle cells
ANS…..A ANS….B
Q10.Structure damage close to phrenic Q17.Hymen epithelium?
nerve? A) Stratified squamous epithelium
A) Pericardiophrenic artery B) Simple squamous epithelium
B) Musculocutaneous nerve ANS…..A
ANS….A Q18.A boy on examination a structure
Q11.An 11 year old boy present with which is fail tu develop on ventral surface
fever for 5 days.He also c/o malaise and of penis.In girls which is similar to this
nausea for 2 weeks.He recently visit to structure develop?
village few days back where sanitation not A)Labia majora
proper.which causative factor? B)Labia minora
A)Hep A C)Clitoris
B)Hep B ANS..B
C)Hep C Q19.A female has low
D)Hep D MCH,MCV,MCHC.What type if anemia
E)Hep E is?
ANS….A A)Iron def anemia
Q12.A female at 38 weeks of gestation B)Megaloblastic anemia
with fever nausea ANS…A
ALP=275,SGPT=178,ALK=258.She visit Q20.Regarding B-thalassemia?
a remote village 6 weeks back where A)Defect in globin chain
sanitation not proper.what’s the causative B)Defect in globin ratio
factor? ANS….A
A)Hep A Q21.In 60 year old man chronic diarrhea
B)Hep B for 6 weeks causes?
C)Hep C A) Alkalosis rather then acidosis
D)Hep D B)K is predominant increase
E)Hep E C)Osmolality increase
ANS….E ANS…A
Q13.Which of the following hepatitis Q22.Vitamin D3?
leads to chronic liver disease? A)Animal vitamin
A)Hep B B)1,25 cholecalciferol
B)Hep C C)Plant vitamin
ANS….B D)25.hydroxy cholecalciferol
Q14.Most initial sign of bupivacaine ANS….B
toxicity? Q23.Rough endoplasmic reticulum
A) Nausea involved in?
B) Ringing in ears Ans)Protein synthesis
ANS….B Q24.In a boy B.P270/180 & creatinine
Q15.Regarding acute inflammation which 8%.which structure is damaged?
is true? A)Macula
A) WBC increase upto 40,000 B) JG cells
155

ANS….B ANS….. C
Q25. GFR dec due to? Q35.Corticostriate fibers contain
A) Afferent arteriolar constriction A) Dopamine
B)Efferent arteriolar dilatation B ) GABA
ANS….A C) Glycine
Q26. Which metabolic effect caused by D) Epinephrine
thyroid? E.Glutamate
A) Dec gluconeogenesis ANS….E
B) Protein synthesis Q36.In a pregnant female with sugar level
C) Dec fatty acid oxidation 340mg/dl.what is the treatment?
D) Dec fatty acid synthesis Ans) Insulin
ANS….C Q37. Due to hydrocarbon which occur?
Q27. Which effect caused by reduction of A)Aplastic anemia
p450 mechanism? B)Bronchogenic carcinoma
A) Acetylation of isoniazid ANS…..B
B) Oxidation of phenytoin Q38. A man working in mine with hilar
ANS….A lymphadenopathy.
Q28.Glucocorticoid causes? A) Silicosis
A)Dec peripheral glucose utilization B) Asbestosis
Q29. Increase respiratory effect in ANS….A
pregnant women due to? Q39.Man with pleural plaques show
A) Estrogen malignancy.
B) Progesterone A) Asbestosis
C) LH Q40. Principle of ethics?
ANS….B A)Autonomy,beneficence,non-malificence
Q30.A man with subfertility has Anti and justice
FSH antibody against FSH Q41.A female pt presented in ER in state
receptors.which of the following increase of shock.she had transfused 10 pints of
in plasma? blood 2 yrs back.B.P80/50,pulse 60/min.
A) Anti mullerian A)Sheehan’s syndrome
B) Inhibin B)Addison’s disease
C) Testosterone ANS….A
ANS….B Q42. A female present with fever and
Q31. Increase protein synthesis in muscle chills.No h/o burning micturition.On
caused by? staining gram –ve organism seen.
A) Testosterone A)E.Coli
B) Progesterone B)N.gonorrhea
ANS……A ANS…..B
Q32.Uterus is prolapsed but membrane Q43.In a female major breast duct
intact.which is due to? terminate into terminal ducts from
A) Pelvic diaphragm terminal ducts it forms?
B) Broad ligament A) Lactiferous duct
ANS….A B) Lactiferous sinus
Q33.Cervix drain by? ANS….B
A) Internal iliac nodes Q44.A female with vasculitis and fever.on
B) External iliac nodes examination gram –ve endotoxin
ANS…..A present.which is most likely causes ths
Q34. Bladder drainage due to? condition.
A) Hypogastric plexus A) TNF
B) Inf.phrenic nodes B) TGF-B
C) Internal iliac nodes C) IL-1
156

ANS…..C A) PCT
Q45. Number of observation fall on a set B) DCT
point or on given scale known as? C)Loop of henle
A) Data ANS….A
B) Distribution Q56.A person lose 2L water in sweat then
C) Rate he takes 2L of tap water.what occur?
D) Frequency A) Inc ICF
ANS…..D B) Inc ECF
Q46.In a child diphtheria vaccine is C) Inc osmolality
given.what type of hypersensitivity ANS….A
present? Q57.A pt with CO2=44,pH=
A) Type 1 7.33,HCO3=19,What acid base
B) Type 2 abnormality?
C) Type 3 A)Metabolic acidosis
ANS….C B)Acute resp acidosis
Q47.Tranfusion reaction to A+ve if we C)Metabolic alkalosis
transfuse with ANS…. A
A) AB –ve Q58.Most characteristics features of TB?
B)O+ve A)Caseating granuloma
ANS….A B)Multinucleated giant cells on microscopy
Q48. Pregnant lady with gall stones? ANS…..A
A) GGT Q59.Which is DNA respiratory virus?
Q49.Most likely feature in HELLP A)Resp syncitial virus
Syndrome? B)Adenovirus
A)Hemolysis,elevated liver enzyme and low C)Herpes virus
platelet count ANS….B
Q50.Amino acid which is not produced in Q60.Most closely related to lymphocytes?
body but essential in diet? A) Only arise from bone marrow
A)Phenylalanine B) Play major role against cancer in body
Q51.Resection of pons causes? ANS….B
A)Ceasation of respiration Q61.Regarding heart:
B)Dec rate of respiration A) RCA supply SA node,AV node,bundle of
C)Loss of respiration HIS,purkingi fibers and its branches
ANS….A B) LCA divide into Left circumflex artery
Q52.Corticostriate fibers decussate at? and anterior interventricular artery
A) Pyramid ANS…B
B) Internal capsule Q62.In a patient dec calcium,inc
ANS….A phosphate,inc PTH and normal alkaline
Q53. Cushing syndrome in old age due phosphatase?
to? A)Osteomalacia
A) Thyroid adenoma B) CRF
B) Adrenal adenoma C)Vit D deficiency
ANS…..A ANS…..B
Q54.Most salient feature regarding ANS: Q63.During surgery of colon CA first site
A)All parasympathetic supplied by of metastasis into?
cholinergic A) Liver
B)All sympathetic by adrenergic B) Lung
C)All parasympathetic by vagus nerves ANS…..A
ANS….A Q64. A female with narrow subpubic arch
Q55.A female with PPH now present with and inverted ischial spine.what type of
Acute renal failure.Defect present in? pelvis?
157

A) Anthropoid A) Right lower end of sternum


B) Android Q77.GVE fiber of tenth nerve contain
C) Gynecoid which nucleus?
ANS…..A A) Nucleus ambigus
Q65.Nasopharangeal CA associated with? B) Dorsal nucleus
A) EBV ANS….B
B) CMV Q78.A female with infection of HPV
C)HPV comes after 2 years.Cell shows prominent
ANS…..A nucleoli and in size?
Q66.Cervical CA due to? A) Dysplasia
A)HPV B) Metaplasia
B)HSV ANS….A
ANS….A Q79.Virus which cuases metaplasia then
Q67.HIV female pt with associated with dysplasia?
malignancy? A) HPV in cervix
A)Cervical CA Q80.Benign neoplasm:
Q68.Major stress hormone during A) Adenoma
surgery? B)Hepatoma
A)ACTH ANS….A
Q69.A female met an accident had lost Q81.In dark granules containing cell IgE
her son and husband.Afterward she had attaches to?
amenorrhea due to? A) Basophils
A)Inc ACTH Q82. Pulmonary artery supply to?
B)Low CRH A) Alveoli
C)Low dopamine Q83. In young boy dyspnea on lying
ANS…..C down?
Q70.Breast atrophy in adult male due to? A) Retrosternal goiter
A) Estrogen Q84.DNA replication occur in?
B) Estrogen and progesterone A)Interphase
ANS….A Q86.Collagen fibers?
Q71. Correct about thyroid gland? A)Acidophils
A) Drain into deep cervical lymph nodes B)Basophils
Q72. Longest pro-erythrocyte stage? C)Major elastic tissue in body
A)P.Malaria ANS….C
B)P.Falciparum Q87. Protrusion of mandible by?
C)P.ovale A)Lateral pterygoid
ANS….A Q88.Isotonic and isometric difference is
Q73.Shortest pro-erythrocyte stage? that?
A)P.Falciparum A) Isotonic contain more phosphate bond
B)P.Malaria Q89.A boy with lesion on nose.He also has
C)P.ovale granuloma with vasulitis.what is the
ANS….A cause?
Q74. A girl with fever and black water A) Lepsrosy
urine? B) Wegener’s granulomatosis
A) G6pD deficiency ANS…..B
B) Plasmodium falciparum Q90. Pt with intensional tremors and
ANS….B facial paralysis lesion at?
Q75. Origin of oxytocin and vasopressin? A)Pons
A)Post.hypothalamus B)Cerebello pontine angle
Q76. Auscultation of tricuspid valve best ANS…..B
at? Q91.Major best investigation in ITP?
158

A)APTT Q103.Sub acute endocarditis cuased by?


B)CBC A)Strep.viridans
C)Platelet count Q104.At day 30 fetal life,number of
D)BT somites?
ANS…..B A)40-44
Q92.A female with CRF.she has been Q105.Boy after RTA,diplopia and
advised renal transplant from twin proptosis?
sister.The most important pre requisite A)Basilar artery thrombosis
for matching is? Q106.High cardiac output in?
A)HLA compatibility A)Just after delivery
B)ABO compatibility B)In labour
ANS….B C)In first trimester
Q93.Steroids cause its action on cell by? ANS….A
A)Activation of genes Q107.Thyroid gland
Q94.Class 3 Anti arrhythmic? A)Both lobes symmetrical
A)Amiodarone B)Tracheal ring lie at level of cricoid
Q95.Histology of cornea? cartilage
A)Stratified squamous non-keratinized ANS..A
epithelium Q108.Delayed type hypersensitivity seen
Q96.Women after delivery,platelet in?
70,000.APTT normal PT normal.Diagnosis? A)TB
A)DIC Q109.Method detecting Gonococcus
B)ITP infection?
ANS…A A)Gram staining
Q97.Mechanism of DIC? B)ZN staining
A)Degradation of fibrinolytic products ANS…A
B)Dec platelet function Q110.10% formaldehyde is used for
C)Vit K deficiency sending biopsy specimens why?
ANS….A A)To prevent autolysis
Q98.Pyruvic acid is used in intermediate B)Sterilization
in which step of cell cycle? C)To denature protein
A)Carbohydrate to Acetyl co A ANS…..C
B)Glucose to Acetyl co A Q111.In female thick whitish curd like
ANS….B discharge present.causative organism?
Q99.A female diabetic and hypertensive A)Candida albicans
best drug of Rx? Q112. Cause of edema?
A)Captopril A) Lymphatic obstruction
Q100.Cataract in newborn due to which Q113. Tachyphylaxis?
maternal infection? A)Diminshed response developing quickly
A)Rubella Q114. A diabetic pt with organ
Q101.Aphasia due to lesion in? involvement cause of death?
A)Frontal lobe A)Mucor
B)Parietal lobe 115.During surgery management of acute
C)Hippocampus high BP?
ANS….A A)Hydralazine
Q102.Virus cuase cancer by mutation in? Q116.Epidural anesthesia given to
A)DNA increase its duration?
B)RNA A)Inc dose
C)Proto oncogene B)Inc conc.
D)Oncogene C)Give with 1:20000 epinephrine
ANS….C ANS….C
159

Q117.Ureter most narrow at? Q132.Patient with lower backache and


A)Ischial spine pain at posterior thigh.DR tells him that
B)Ischial tuberosity longest nerve of the body is involved?
C)Sacral promontory A)Lumbosacral plexus
ANS…A B)Sacral nerve
Q118.Artery remain in true pelvis? ANS….A
A)Middle rectal artery Q133.Heart prone to fibrillation?
Q119.Symphysis pubis: A)At the end of action potential
A)Sec.cartilagenous joint Q134.Donut shaped organism in HIV?
120.Female with low MCV,MCH,MCHC A)Cryptococcus
and sr.ferritin.what the next best 135.Person taking normal saline?
investigation? A)Inc urine sodium
A)Sr.iron & TIBC B)Dec urine osmolarity
Q121.Heinz bodies present in? ANS….B
A)G6PD deficiency Q137.A female on ATT taking
Q122.Ovary held in position by? contraceptive but still got pregnant due
A) Broad ligament to?
Q123. Hallmark of HIV? A)Rifampicin
A) Immunodeficiency Q138.Corneal opacity in a person due to?
B) Proliferation of virus inside T A)Chloroquine
lymphocytes Q139.Glucose transport across membrane
ANS…..B due to its conc.difference is called?
Q124.A baby born with erythroblastosis A)Simple diffusion
fetalis with blood group A+ve.He should B)Facilitated dissusion
treat with? ANS…A
A)A+ 140.Cradiac muscle can’t tetanized becoz
B)AB+ it has?
C)A- A)Long refractory period
ANS…..A Q141.Artery arises on substance of
Q125.Starling law of heart? parotid gland and lies at posterior body of
A)Explain the inc in cardiac output when digastrics?
venous return increases A)Lingual artery
Q126.In inferior wall MI artery involved? Q142. Breast upper and outer quadrant
A)Right marginal artery drainage?
Q127.On x-ray right border of heart A) Anterior axillary nodes
formed by? B) Pectoral nodes
A)SVC ANS….A
Q128.Drug to drug interaction? Q143.Radiation causes cancer?
A)Both pharmacokinetic and A)5-10 years
pharmacodynamic Q144.Hematocrit of 41%means?
Q129.In diabetic nephropathy best A)RBC
investigation? B)WBC
A)Urine albumin C)RBC,WBC,Platelets
Q130.Highest diffusion capacity across ANS…C
across respiratory membrane?
A)CO2 Q145.Drug not causes gynecomastia?
Q131.Plasma protein return to normal A) Griseofulvin
in? B) Androgen
A)10-12 days ANS….B
Q146.In female offspring clear cell
carcinoma of vagina due to?
160

A) DES Q158.A 60 year pt suffering from


Q147. Bladder Carcinoma due to which generalized lymphadenopathy for last 8
organism? months.Hb=6g/dl,platelet and wbc low.
A)Schistosoma hematobium A) CLL
Q148. Simple columnar partly ciliated B) Hairy leukemia
and non ciliated epithelium? C)Hodgkin lymphoma
A) Fallopian tube ANS….C
Q149.48 Years old lady has CA cervix Q159.Trachea terminate at?
that involved cervix and vagina not lower A)C6
third,lymph nodes palpable and some B)C4
suspicion of metastasis.what stage? C)T4
A) T2N1MO ANS….C
B) T2NIM1 Q160.Anterior fontanelle closes at?
ANS…B A)12-18 months
Q150.A pt in CCU: Q161.During fetal life kindney infected
A)feel terror and discomfort due to which drugs?
B)Behave vigorously and shout on everyone A)ACE inhibitors
C)Keep calm and cooperate with family Q162.In fetus carotid artery gets blood
members and staff from?
ANS..C A)Aorta
Q151.Mesonephric duct functional B)Ductus arteriosus
remnant? ANS….B
A)Ductus deference Q163.In female after some antibiotic use
B)Uterus develop bad taste,thin watery discharge
ANS…..A and extreme itching.which antibiotic was
Q152.At the beginning of 5th week? cause of itching and bad mouth taste?
A)Limb bud appers A)Metronidazole
B)Stomach rotates Q164.After breast surgery best
ANS…..B chemotherapeutic drug given to pt?
Q153.Which of the following not receive A)Tamoxifen
innervations from pudendal nerve? Q165.Most common organ affected by
A)Posterior fornix of vagina SLE?
Q154.Inferior rectal artery branch of? A)Kidney
A)Internal pudendal artery B)Lung
Q155.Needle inserted at 4th intercostals C)Eyes
space it pierces? D)Joints
A)Intercostal muscle ANS…..A
B)Intercostal membrane Q166.Most common 2nd messenger
ANS…..B pathway??
Q156. A young female with secondary A)AMP
infertilitycome to you on 18th day of his B)ADP
LMP for D & C.In which phase? C)ATP
A)Secretory ANS…A
B)Luteal Q167.Cervix drain by?
ANS…..A A)Internal iliac nodes
Q157.Vibrio cholera Q168.A farmer working in field become
A)Grow best at 25 C unconscious frothing from mouth.which
B)Grow best in acidic medium drug u give?
C)Resistant to alkaline medium A) Atropine
ANS…C Q169. A child with severe diarrhea and
bleeding.no family history.Cause?
161

A)Vit K deficiency b.3 to 6 feet


Gynae and obs 14 oct ANS b
9.maximum ventricle is filled during
2016(evening) A) diastasis
1.a female having two children ,after B)atrial systole
cholecystectomy she developed pallor her C)rapid inflow
peripheral blood shows microcytosis type D)slow inflow
of anemia Ans>C
a blood loss 10. The neural crest cells originate at the
b iron deficiency dorsal most region of the???
Ans B Ans> neural tube.
2.lowest O2 content in umblical 11.uterus main support ??
circulation is in Ans>pelvic diaphragm.
a.pulm trunk 12.sesamoid cartilage found in
b.right atrium a. epiglotis
c.right ventricle b .nasal septum
d ivc c .larynx
e.svc d. ear pinna
Ans |e Ans>C
3.The principal difference between 13.left renal vein drain into ???
cytotoxic type 2 n immune complex type 3 Ans>ivc.
hypersensitivity is 14.left renal vein anrerior to???
a. Delayed hypersensitive reaction Ans> aorta.
b .The class of immunoglobulin involved 15.max renal clearance of ???
c. the participation of T cells Ans>PAH.
d. The participation of complement
e. The site where antigen antibody 16. A boy collapsed 2 hr after
complexes are found tonsillectomy .what is the reason behind
Ans E this
4. A female diabetic from last 4 years, A. Hypovolumic shock
anemic, admitted with pseudomonas B. Septic schock
infection, blood sugar was normal but C. Neurogenic shock
ater 6 days she developed shock, how D. Cardiogenix shock
a.electrolyte imbalance E. Toxic shock
b.acid base imbalance Ans>A
c.inc plasma osmolarity 17. A patient had to undergo amputation
d.Increased vascular permeability of right leg. At the site of amputation ha
Ans D present with a mass, biopsy shows
5.cryptorchoidism should be treated as haphazard pattern.
there is increase risk of?? a- neuroma
ANs> malignancy. b- schwanomma
6.temporomandibular joint dislocation c- lymphoma
will lead to which muscle paralysis?? Ans>A
Ans>lat pterygoid 18.post ganglionic fibres found in
7.unable to open mouth which muscle a - all spinal nerve
paralyse? b. mid thoracic
Ans>lat pterygoid Ans>A
8. Airborne droplet infection can spread 19.diaphragmatic surface of heart form
upto by???
a.1 to 2 feet Ans> right ventricle.
162

20. In a study 40% women recived HRT e.corticosterone.testosterone.cholestryl


20% women have not recived HRT. the sulphate
best way to represent such study is??? Ans>A
Bar chart 31.nipple plus lat part breast drain Into
pie chart ???
chi square Ans>anterior axillary lymph nodes.
Ans>B 32.lat part breast drain into ??
21.scenario was of case control Ans>anterior pectoral nodes.
22. regarding free water diuresis.. 33. boil on labia majora will drain Into?
A.NEGATIVE WATER CLEARANCE Ans> medial group of superficial inguinal
B,positive water clearance, (ans) lymph nodes.
c.plasma osmolarity 34.star shaped lumen
Ans>B a. ureter
23. Pt had severe dehydration, what is the b.fallopian tube
best treatment to increase body water Ans>A
a.5% dextrose
b.10% dextrose
c.Albumin 35.sensitive to radiation = lymph nodes
d.Normal saline (germ cell , git epi also v. sensitive to
Ans>A radiation).
36.tumour removed from ovarian fossa
24.female presented in oncology depart which structure prone to damge ??
hypotensive fever what is the cause of this Ans>internal illaic artery.
condition? 37. 45 yrs old woman has invasive ductal
Ans >TNF (IL1 was not in options) carcinoma of left breast with metastasis in
25.true hermaphrodite fur ipsilateral axillary lymph nodes.
a.xx Tumor measures 6.5cm in its max
b.xxy diameter. Acc to Manchester's staging.
Ans>B Stage of this tumor would be
26. The antimotabolite which is used for A. Stage 1
nonneoplatic diseasea and is also an B. Stage 2B
immunosuppresant??? C. Stage 2A
A.Azathioprime D. Stage 3
B.cytarabine E. Stage 4
C.Methotrexate Ans>D
Ans>C 38.structure posterior to ureter ???
27. Methotrexate is?? ans>genitofemoral nerve.
Ans> a folate antagonist 39.change of one epithelium to other is ??
28. cyclophosphamide side effect ?? Ans>metaplasia.
Ans>hematuria. 40.an anesthetic drug given to pregnant
29.eye blunt trauma which structure will lady which conc inc in mother blood as
be more prone to damage=?? compared to fetus name of drug?
Ans> limbus?? a. lidocaine
30. progesterone is a precursor of.?? b. bupivaciane
a.de oxycortisone,Aldosterone,cortisol and Ans>B
corticosteriods 41. histological section of tissue sample
b.hydrocortisone,pregnenolone;estriol shows normal mitoses,pleomorphism and
c.cholestrol :cortisone,cholic acid hyperplastic cells are observed in
d.aldosterone .deoxycholic acid different stages of cell cycle with no
.pregnenlone invasion of basement membrane it is most
likely due to
163

A.metaplasia a. uric acid


B.dysplasia b.xray
C.anaplasia c.ANA
D.malignancy d.anti Ds Dna
Ans>B e.rheumatoid factor
42.an anesthetic given to a patient Ans>A
thorugh which route it given will cause
highest conc in blood 54.dont remember exact words
a.spinal ( The greater, lesser, and least splanchnic
b. epidural nerves arise from the thoracic part of the
Ans>B sympathetic trunk and carry much of the
43.ovarian artery is the branch of ??? sympathetic and sensory supply of the
Ans>abdominal aorta. abdominal viscera. The nerves pierce the
44.Ant wall of bladder devoid of?? diaphragm and reach the celiac and
Ans> peritonium. aorticorenal ganglia)
45.medial boundry of ischiorectal fossa by 55. which hernia lies medial to inferior
a.ana canal epigastric artery??
b.levator ani indirect inguinal
Ans>B Direct inguinal correct this one is answer
46.SLE scenario investigation of choice ?? femoral
Ans>anti Ds DNA. umbilical
47.women developed cogulation defect on Ans>A
ultrasound having IUD baby 56.cause of epispadasis is urethral fold
management? fusion defect. faulty position of genital
Ans>evacuation of fetus tubercle
48.illiacus muscle supplied by ?? 57.Hernial sac which is ramanant of
Ans>femoral nerve. processus vaginalis and carry all layer
49. ischiorectal fossa abscess drainage will alonwith is which type of hernia??
result in damage to a. direct inguinal hernia
a.inferior rectal nerve b indirect inguinal hernia
b.sacrotuberous ligament Ans>B
c.internal pudendal artery 58.vagina histology shows abundant
Ans A a.collagen plus elastic
50.female having complain of fever b.elastic
productive cough diagnosed case of staph Ans A
aureus pneumonia now fever perisist but 59. on microscopy how carcinoma
gradually increased dyspnea xray chest diagnosed = ??
showed air fluid level what complication Ans> local invasion
she have developed? 60.cells present in epidermis which give
a.granuloma colour pigment =???
b.fibrosis ANs> melanocytes
c.abscess formation 61. in hypoxemia which will respond
ans>C a.carotid body
51. sphincter urethra supplied by? b carotid sinus
a. sacral splanchnic nerves c.central chemoreceptors
b.pudendal nerve Ans>A
Ans>B 62. A patient on hypertensive drugs
53.49 year female developed pain in right presents postural hypotension. in heart
big toe with fever she had same episode of rate
pain 6months back in knee joint what while standing
investigation will u do is mediated by:
164

A. decrease fire rate of baroreceptors Asn>full body irradiation ( poor recall of


B. decrese sympathtic stimulation statement but ans was full body radiation)
C. Inc vagal tone 73.girl diagnosed with rubella what ishe
D. Inc vasomotor tone indication of terminating pregnancy ??
Asn>A Ans: presence of IGM
63.lady having carcinoma developed 74.most abundant antibody ??
generalized edema cause is ?? Ans:IgG
Ans lymphatic obstruction 75.Established autosomal dominant disease
64.Edema due to renal disease cause ?? a. heridatory tubular adenoma familial
Ans> salt retention plus albuminurea adenomatous
65.common cause of DVT b. hemochromatosis
a.post operatively Asn:A
b.prolong bed rest 76.automatic bladder when lesion is
Ans>B a.above sacral segment
66.old male patient case of atherosclerosis b.below sacral spine segment
was under your treatment with blockage Asn:A
of coronary artery suddenly brain 77.bladder detrussor muscle contract due
ischemia occured cause = to ??
a. thrombus Ans:pelvic splanchnic plus hypogastric
b embolus plexus
Ans>B 78. 1 gm of normal Hb can bind a
67. a lady, known diabetic for 5 yrs. she is maximum of??
on Glibenclamide. she had an acute Ans: 1.34 ml O2
asthma attack for which she is still taking 79.scenario of hemolytic anemia
medication. this morning she felt dizzy 80. A patient has hb 6 .platelets 450000
her blood sugar is 40. choose the drug e.hypercellular
which interacted with glibenclamide Bone marrow.diagnosis is
a. b-blocking agents A. acute blood loss?
b. steriods B.leukemia
Ans>B Ans:B
68.cynosis due to ?? 81.vitamin c required for which step in
Ans> deoxyhemoglobin collagen synthesis ??
69.Ineffective erythropoiesis in Ans: hydroxylation of proline and lysine
a. thalsemia residues.
b.sickle cell anemia 82. Renal failure feature ??
Ans>A Ans: oligourea
70.when volume of water decrease all of 83.Pulmonary artery supply??
following will increase except Asn: alveoli
a. ADH 84.How to differentiate b/w iron def and
b. aldosterone thalassemia . what lab investigation will u
c estrogen ask
Ans>C a. hb F
71.ventilation perfusion ratio zero in b. serum ferritin
a. Pulmonary embolism c. hb electrophoresis
b.pleural effusion d. mcv
c fibrosis Ans:C
Ans>A 85.Single best investigation for iron
deficiency anemia ???
72.worst from to do immunosuppression Ans: serum ferritin
?? 86. pregnant lady with jaundice n bile
duct stone , investigation??
165

Ans:GGT b.catecholamines (scenario of


87.Bleomycin ?? pheochromocytoma)
Ans: pulmonary fibrosis Ans:B
88.insulin secretion increase ?? 96.self replicative
a. secretin a. mitochondria (ans)
b. somatostatin b.smooth ER
C.GIP c.nucleus
Asn:C d RER
89.vit k plus c deficiency causes ?? 97.essential amino acid
Ans: bleeding disorder a. tyrosine
90.A 14 yr child with bleeding and also b. threonine
diarrhea no family hostry of any bleeding Ans:B
disorder cause of bleeding 98.PTH normal in
a. Liver faliure a. osteoprosis
b. Hemophilia b pseudohyperparathyroidism
c. Vwf Ans:A
d. vit k deficiency 99.Total body water
Ans:D a. 60% of body weight
91. Fetal brain development is caused by b. ECF +ICF+ transcellular fluid
a. Growth hormone Ans:B
b. Thyroid hormone 100. Tetanus vaccination ??
c. ADH Ans:Toxoid is used.
d. Prolactin 101.Hypervitaminosis ??
____________________________________ Ans:Scally dermatitis
____ 102.Widest space in vertebral coulm??
Ans. b. Maturation of CNS of the fetus Ans:L2( dis question has come many times
requires the presence of thyroid hormone in in exam)
the perinatal period. Thyroid hormone 103.child ribs
deficiency causes irreversible mental a. vertical
retardation. Because there is only a brief b. on cross section almost circular
prenatal period when thyroid hormone Ans:b
replacement therapy is helpful, screening for 104. Micturation reflex ??
neonatal hypothyroidism is mandatory. Ans: self regenrative
92. sertoli cell?? 105. Hypothyroidism ??
Ans: androgen binding protein Ans: Inc cholesterol level
93. Mean arterial pressure ?? 106.malaria in which stage inactive in
Ans: diastolic pressure plus 1/3 rd pulse body
pressure. a.sporozite
b.merozite
94.Hormone which produce smooth c .shizont
muscle of git relaxation and also increase d.hypnozoite
git secretion Ans:D
a. substance p 107.mother o negative blood group father
b.VIP o postive child devlope erythroblastosis
Ans:B fetalis child blood group?
95.Femlae presented with complain of Ans:B positive
sever headache , uncontrolled 108.Fibroblasts ??
hypertension lab shows k 3.4 most Ans: amorphous extracellular matrix
probably cause of hypertension 109. Scenario of malaria, black water
a. aldosterone fever??
Ans: falciparum
166

110. Post fontanalle?? 128. Bacterial vaginosis lead to ??


Ans: smaller than anterior Ans:preterm labour.
111.Trachea divide at c4-c5 (rest all 129.Deep inguinal ring ??
irrelevant options). Ans: facia transversalis
112. Chronic liver disease on microscopy 130.true about pituitary??
a. fibrosis Ans:Pituitary drain in dural venous sinuses.
b. lymphocytic infiltration 131. other was about pitutary ans was
asn:B Post sup relation of pituitary is sphenoid.
113.In complete heart block ?? 132. pain on defecation mediated by
Ans:atria and ventricle contract pudendal nerve. (inf rectal was not in
independently. options)'
114. pain mediator ?? 133. fast pain A delta fibres
Ans:bradykinin 134. maximum inc cardiac ouput in
115.Megaloblastic anemia scenario was pregnant lady is after delivery.
there ans was megaloblastic anemia. 135. ocp prevent ovarian ca.
116. 23 year old woman regular 136. adrnelectomy done which
menstrual cycle of 21 to 23 days replacement necessary for life
.ovulation is expected to occure between a. aldosterone
cycle days: b. corticosteriod
a. 10,12 c. epinephrine
b.7,9 Ans:B>A
Ans:B 137.How to check staph aureus virulence
117. Mesonephric duct develop from ?? a. coagulase positive
Ans:intermediate mesoderm. b. hemolysis
118. Alpha 1 agonist cause ?? Ans:A
Anscontraction of radial muscle of iris. 138. GTO prevent
119.Non healing diabetic ulcer due to?? Ans:tearing in overstretching.
Ans: angiopathy and neuropathy. 139. Tension detect??
120.Cimitidine ?? Ans: by golgi tendon.
Ans:Hepatic enzyme inhibitor 140.Diabetic nephropathy confirm ??
121. Most common non bacterial infection Ans:by urine albumin
developed in burn patient?? 141. most common brain tumor in ??
Ans: candida.(ref Goljan patho) Ans:adult glioblastoma multiform
122. Post ventricular branches 2 in 142. uterus absent because of
number and supply ?? malformation of ??
Ans:diaphragmatic surface heart. Ans:paramesonphric duct.
123.How u differentiate chest lesion being 143.paramesonephric duct male remnant
or malignant?? a?
Ans = recurrence after excision As:ppendix testis.
124. prolactin raised with infertility what 144. second heart sound ??
will be cause Ans:closure of Aortic and Pulmonary
a.microprolactinoma valve.
b.macroprolactinoma 145.excessive hemolysis bilirubin bind
c.pituitary tumor with albumin and go to liver.
Ans:b 146.EBV associated with ??
125. plasmodium malarie ?? Ans:nasopharyngeal carcinoma.
Ans= quartan fever 147. Antibodies againt ACH receptors = ?
126. Highest clearance ?? Ans:mysthenia gravis
Ans: PAH 148. first pass effect avoided by giving
127. Gynecoid pelvis outlet ?? drug
Ans= AP > transverse dia a. IM (ans)
167

b. rectal 7-..Increase in respiration during luteal


c oral phase of menstrual cycle is due to
d oral suspension form A. Fsh
iv was not in options B.lh
149.Endometriosis treatment danazole. C. Estrogen
150. Highest bioavailability D.Testosterone
metachlopromide. E.Progesterone
GYNE OBS 8TH NOVEMBER Ans E
8.Regarding lesser pelvis
2016 A.has a cavity whose post wall is much
1- Tumor suppressor gene shorter than ant
A.N-myc B.Outlet is bounded by sacrotuberousand
B.C-myc sacrospinous
C.P 53 C. has relatively longer antposterior
D. C- erb diameter at the inlet
E.ras D.has a small sub pubic angle
Ans:C E. Is generally circular in cross section
2-Shortest acting local anesthetic Ans:B
A.Procaine 9-Effect of estrogen on cervix
B.Lignocaine A.Cervicaladenosis
C.Ropivacaine B.Sqamous cell metaplasia.
Ans:A3-Risk measurements, exposed Ans:A10-Pt on ot table heat loss through
group vs non exposed A.Conduction and radiation
A.Cohort study b.Conduction
B.Case control c.Radiation
C.Prospective d.Evaportion
D.None of above e.Convenction.
Ans:A Ans:A
4-Data from smaller study on one group of 11-Pt having butterfly rash n multiple
people iscombined with data from larger miscarriage
study.what type of data is this a..sle
A.Case control b.rehmatoid arthritis
B.Cohort c. scleroderma
C. Randomized control trial d. rheumatic fever
D.Metaanalysis e.sjogren syndrome
Ans:D Ans:A
5-Chi square is used to measure 12-True about implantation of zygote
A.Statistical analysis A. Occurs 3-5 days after fertilization
B.Confidennce interval B. Occur when 128 cells structure
C.Sensitive something formed
D.Specificity C.invasion of endometrium by
E.Valadity syncitiotrophoblast
Ans: A D.progesterone inhibit implantation by
6-Automatic blader cause acting on corpus luteumAns:c (asim bcq).
A.lesion above sacral segment 13-Pregnat female at 20 weeks gestation
B.lesion below sacral segment withhyperthyroidism.best treatment
C.Frequentmictuiration A.propylthiouracil
D.loss of sensory supply B.thyrodectomy
Ans:A C.no treatment till delivery
Ans:A
14-Nerve supply of levatorani
168

A.hypogastric C. Fascia transverselis.


B.illioinguinal D.skin
C.pudendal nerve E. Sup Fascia
Ans:C15-Contents of superficial Ans:C
perinealpoch 22-Pt given harmonal therapy for
A. Perineal body. metatstatic cancer.
B. Pudendal vessels A .Apoptosis
C. Bulbo urethral glands B. Atrophy
Ans:A C.hypertrophy
16-Pt with pallor... mcv raised cause D. necrosis
A.folic acid deficiency . Ans:A
B. Sideroblastic anemia 23-Virus causing alteration in
C.IDA A.DNA
Ans:A B. RNA
17-On histopathology partially celiated, C. Proto oncogene
partially non ciliated D. Oncogene
A.Uterus Ans: C
B.Cervix 24-Diaphragmatic hernia is associated
C.Uterine tubes with
Ans:C A.lung aplasia
18-Breast milk secretions contain B.surfactant deficiency
A.IgA Ans: A25-Trachoma..
B.Igg A.chlymadia
C.IgM B.crcinoid
D.Igd C.condyloma
E.IgE D.lymph granuloma venerum,
Ans:A Ans:A
19-At what age the head of fetus become 26-Pt with bicornuate uterus. Associated
half of CRL length anomalies are
A.3rd week A.git
B.strt 3rd month B.cvs
C.strt 4th month C.resp
D.6th month D.utI
E. 7 month E.Renal
Ans:c Ans:E
20-Inflamation of the portal triads which 27-Pt in labour for 2 hours.on pv inverted
spills into adjacent lobules bands of spines,narrowsubpubic arch and sacral
inflammation extending between portal promontory not reachabletype of pelvis
areas and terminal hepatic veins and foci A.Gynaecoid
of liver cell necrosis throughout the lobule B. Android
is most likely pic of. C.platypalloid
A.chronic acute hepatitis D.anthrapoid..
B.cirrohsis Ans:b
C.chronic passive congestion Explanation :(Traditional obstetrics
D.chronic persistent hepatitis characterizes four types of pelvises:
E. fulminant hepatits Gynecoid: Ideal shape, with round to
AnsA: slightly oval (obstetrical inlet slightly less
21-Stucture between tranverselismuscle transverse) inlet: best chances for normal
and extraperitoneal fat vaginal delivery. Android: triangular inlet,
A.internal oblique and prominent ischial spines, more
B.ext oblique angulated pubic arch. Anthropoid: the
169

widest transverse diameter is less than the A.thymus


anteroposterior (obstetrical) diameter. B.spleen
Platypelloid: Flat inlet with shortened C.lymph node
obstetrical diameter). Ans: A
28-Niacin deficiency... 37-most common cause of malignancy in
A.Plegrra workers
B.Dermatisis A,tobacco industry
C.Chelosis b.silicone
D.Periperal neuropathy C.asbestosis
Ans: A Ans: A38-maximum bioavailability ?
29-Plataeu is formed due to A.I\M
A. Na and CaChAnnels B.sublingual
B.Voltage gated Na channel. C.per rectal
c.k channels. .D.Orall
Ans:A(ca chaneels wasn’t in options)30- Ans:B(i/v wasn’t in option)
which part of blood test should be sent for 39-most common cause of fatty liver in
matching histocompatibility of the our country
recipient and donar A, hep b and c
A.Wbc B.alcohol
B.Rbc C.fatty food
C.Sserum D.protein malnutrition
D.Bone marrow Ans:D
E.Plateles 40-Lacunar is extension of inguinal
Ans:a ligament..notreflectd
31-cortisol req for ? 41-poliomyelietis .
A. Thyroxine A.anterior horn cell injury
B. Epinephrine B.Virustraveles along peripheral nerve.
C.Glucagon and Epinephrine Ans:A
Ans: c 42.Ocp
32-Increase glucose, fatty acid and A.inhibiting LH
protein in serum B. inhibition of ovulation
A.Insulin . C.thickening cervical muous
B.Growth hormone. D.hepatic adenoma
C.Cortisol Ans:b...
Ans:c 43-In pregnancy
33-Well differentiated sq cell tumor A.muscle bulk increase cz hypertrophy
a.Loss of polarity. andhyperplasia
b. Keratin or something. B. uterus remain relaxed till labour
Ans.B C.breast size inccoz of prolactin
34-Maximally filling ventricle with blood D.BMR increases by 10%
during which phase Ans: A
A. Diastasis 44-Infection from pretacheal region can
B.Rapid inflow travel to...
C.Atrial systole A.superior mediastinum
D.isovolumetriccontraction B. anterior.mediastinium.
Ans:B C. Middle mediastinum
35-Atropine same action. D. posterior mediastium
A.physostigmine Ans:B
B.Scopolamine 45-step which is same in glycogeneolysis
Ans: B and glycogenesis??
36-lack of lymphoid nodules A. glucose6 phosphate
170

B.fructose 6 to 1,6 bi phosphate... 54-Platelets ridges/lines of zahn are


Ans:B present inA.Propagated clot
46-Pelvic outlet most appropriate B.Coraline thrombus..
A. AP diameter more than transverse Ans: B
B.Tranverse diameter more than AP 55-platelets .
diameter A.fragments of megakarocytes
Ans:a Ans:A
47-Stratified squamous epithelium 56-Prostacyclin formed from
A. renal tubular cells A.edothelial cells
B.duct of salivary gland B.plateles
C.corneal epithelium C.peyerpathes
D.bulbular conjunctiva D. mast cell
E.ovarian follicle E. fibroblast
Ans: B Ans: A57-cardiac output increase by
48-sympathetic stimulation causes?? A. venous compliance
A. bronchodilation.. B.Inrease Blood vol...
B. vasodilation of skin C.Increase interstitial fluid
Ans:A D. increase oncotic pressure.
49-Which is the content of superficial E.increase capillary permeability
perineal pouch Ans: B
A. perineal body 58-Length of adult ureter.
B. membranous part of urethra A.25-30cm
C.bulbuourethralsphincture B.20-25cm
D.internalpudenal vessels C.30-35cm
Ans:A D.35-40 cm
50-Radiation effect. E.15-20 cm
A.Protooncogenes Ans:A (kLM)
B.Oncogenes 59-Herniation of meninges from skull
C.DNA A.craniomeningeocele
D.RNA B.anencephaly
Ans: A51-Uterine artey arises from C.cranioschisis
internal ilac artery cross above? D.craniosynostosis
A.ureter.. E. encephalocele
B.fallapion tube Ans A
C.ligament of ovary 60-Oxygen hemoglobin curve shift to
D. urinary bladder right side?
Ans:A A.alkalosis
52 most commonuti... B.acidosis
A.staphlococci Ans:B
B.Echoli 61-Oxygen hemoglobin curve shift to left
C.streptococci in
Ans:B A. Hyperthermia
53-structure posterior to ureter? B. CO poisining
A.genitofemoral nerve.. C. Ans.B
B. gonadal vessels 62-Radiation late effect.....
C.illeocoelicvessesls A.endarteritis obliteration.
D.left colic vessels B.Damaged of epithelium
E.root of mesentry Ans:A
Ans:A 63-Endocervix epithelium change.
A. Metaplasia
171

B. dysplasia D.0.5 to 1 L
C.hpv Ans:A
D.papilloma 71-Left supra renal vain drain into
E.carcinoma A.left renal vein
Ans:A B.Abdominal aorta
64-Radio sensitive C. Iva
A.Ovary D. Azygous vein
B.Cervix Ans:A
C.Endometrium 72-Regarding CSF..
D.Lymph node. A.Provide nutrition
Ans:D B.prevent brain injury by cushion effect
65.secondary center of ossification in long Ans:B
bones develop in. 73-Internal oblique apponeurosis forms?
A.epiphysis A.Conjoint tendon
B.metaphysis B.Internal spermatic fasica
C.epiphysis plate C. Transcersalis fascia
D.diaphysis Ans:A
E.synovial joints 74-Drug similar in physiological functions
Ans:A like Atropine
66-Structure developing from neural A.scopolamine
crest?? B.physostigmine
A. Autonomic ganglion Ans:A
B. adrenal gland 75-Coccygeal ligament most superior
C. Neural plate border where all three meningeal layers
D.sensory ganglion present A.L1
Ans:A upper border.
67-Inc mean systemic filling pressure with B. lower border of L1
A.decreased venous resistance Ans: B76-Structure not Pierced
B.exercise. A.tunica albuginia
C. Hypothermia B.skin
D. temperature C.cremasteric fascia
Ans: A D.internal muscle and colles fascia
68-Mostly growth factor receptors E. internal spermatic fascia
present on Ans:A
A.plasma membrane(via tyrosin kinase) 77-In hypothyroidism...
B.Neuleus A. TSH
C. DNA B. T3
D.Roughenoplasmic reticulum C. T4
E.mitochondial D.T3 & T4
Ans:A Ans:A
69-Highest PO2 is in 78-In pregnancy iron deficiency
A.arterial blood anemiadiagnostic investigation
B.venous blood A.serum iron
C.right atrium B. serum ferritin
D.left ventricle
E.pulmonary capillaries C. TIBc
Ans:D D. Ferritin n TIBC
70-Oliguria... Ans: D
A.less than 500ml 79-Structure between peritonial
B.less than 50 ml connective tissue and transversus muscle
C.less than 100 ml
172

is C.cancer
A. abdominis fascia D. metaplasia
B.transversalis fascia E.dysplasia
C. TRAnverses muscle Ans:A
Ans:B 87-Pt with rigors n fever suprapubic pain,
80-Ectopic thyroid is secreted in which gram negative rods on stainA.E.coli
carcinoma B.streptococci
A.sqauamous cell ca larynx C.H. influenza
B. small cell carcinoma of lung D.Nesseriamenigitidies
C.Small cell carcinoma of larynx Ans:A
D. adenocarcinoma of colon 88-Circle of willis
Ans:A A.Post. Communicating artery connect
81-Milk not produced in pregnancy basilar to internal carotid
despite increased Ans:A
prolactin hormone inhibiting it 89- Cerbral cortex is supplied by which
isA..estrogen artery
b.progesterone A. anterior and middle cerebral arteries
c.oxytocin B.ant and post cerebral artery
Ans:B C.basillar artery.
82-Triglycerides high in D.vertebral artery
A.chylomicrons E.middle and posterior cerebral
B. HDL Ans:A
C .LDL 90-Autosomal dominant=
D. vLDL A. both parent of affected child have disease
Ans:A B.only 1 parent of affected child have
83-Acidosis disease
A. Increases ventilation C.only males are affected
B.decreases ventilation D. only females are affected
Ans: A Ans:B
84-Nissel granules absent in 91-diaphragmatic hernia is associated
A. axon with
B. hillock A. pulmonary agenesis
C. dendrite B.Surfactant deficiency
D.cell bodies C.Bronchial obstruction
Ans:B Ans:A
85-Hypokalemia causes 92-Primigravida 2nd trimister, gelt
A.Increased h ion in plasma. sudden lower abd severe cramp followed
B. decreased ph of plasma. by watery foul smelling discharge, after 2
C. increased negative base in plasma. days deliverd a still born child. On ex
D .increased bicarbonate in plasma. chorion and placenta have neutrophilic
Ans: D infiltration, cause by which organism?
86-Cervical malignacny. A. Mycobac
A.hpv B. Hsv
B.hsv C,Toxoplasmosis
Trponema
E. E.coli
173

Ans: E Ans:E100- Severe dehydration causes..


93-vulva is supplied by A.total body water loss
A. Pudendal nerve B.loss of ecf volume
B.illioinguinal nerve c.Loss of icf volume
Ans:A D.acidosis Ans:A
94-potassium depletion is associated 101-Abnormal over growth and irregular
with?? thickening of bone caused by
A. bakers dystrophy A.chief cells of parathyroid
B. mysthenia gravis B. Cells of adenohypophysis
C.duschene muscular dystrophy C.cells of parsintermedia
D. Familial periodic paralysis D. Leydig cells
Ans: D AnsB102-Foramen Ceacum is embryonic
95-blood pic shows macrocytosis derivative of?
hypochromic pic? dx is? A. Thyriod.
A. folate deficiency B.Thymus
B.irondeficiency C. Ant pituitary
C.sideroblastic anemia D. Post pitutry
D.megaloblastic anemia Ans:A
Ans:D 103-lady, known diabetic. she is on
96-Pt with blood grp A has... Glibenclamide. she had an acute asthma
A.Anti A antibodies attack for which she is still taking
B.Anti AB antibodies medication. this morning she felt dizzy
C.anti B antibodies her blood sugar is 40. choose the drug
D.no antibodies which interacted with glibenclamide
Ans:c A.-blocking agents
97-messeter is supplied by which nerve.. B. steriods
.A. mandibular v3 C.insulin
B. maxillary nerve D.beta blockers
C. facial nerve Ans:B
D. trigerminal nerve 104-Tb vaccination….
Ans: A98-.which one has transient A. hypersenstivity type 4
adhesive property. B. cell mediated
A. cytokines C.antibody mediated
B. imunoglobins D.type 3 Ans:A
C. c5-9 105-Tubular structure with star shaped
D. integrins lumen lined by transitional epithelium..
E. selectin A. ureter
Ans:E B.fallopian tube
99- 2 liter fluid loss in sweat and person C.oesophagus
takes 2 liter distilled water what will D.stomach
happen..A.decicv Ans:A
B.dec intracellular osmolarity 106-call exner bodies present in
C.inc intracellular osmolarity a.sertoli cell tumor
D.inc ecfvoloume b. ledying cell tumor
E.dececfosmolarity c.granulosa cell tumor
d.dysgerminoma
174

Ans:C Ans: A113-Uterine tube epithelium.


107-12yr old grl passes pieces in .A. ciliated columnar ans.A
menstrual flow...worried about tiny piece 114.secreti0n of prostate contains
of tissues she passed in her menstrual a. Acid ph0sphatase
flow.she is reassured and explain that B. Alkaline ph0sphatase
thsese piece are likely to be C.Fructose Asc0rbic acid
A.endometrial sheds Ans: A
B. Unfertilized ovum 115-HCG has alpha and beta units.each
C.small clots unit has amino acids
D.cervicalepithjelium A.Alpha subunit 92 and beta 145
E. pieces of spiral artery B.alpha 102 and beta subunit 210
Ans: A108-Collagen type in skin C. 100 alpha and 160 beta
A. 1 Ans:A
B.2 116- Tenth intrc0stal nerve enter in rectus
C.3 sheath through
D.4 A. Xiph0id pr0ces
Ans.A B.Pubictubercle
109-Free nerve endings ... C.Pyramidalismuscle
A.non encapsulated receptors.... D.umbilicus
B. detect pain only Ans:D
C. type of special visceral enteroceptor 117-male narrowest part of urethra
D.found in volountry muscle A. externa meatus
E. not present in epidermis of skin B.prostatic part
Ans: A C.membranous part
110-Lymphatic drainage around anus. D.pelvic brim
A.Medial group of horizontal superficial Ans:A
nodes 118-One molecule of glucose how many
B. lateral group of horizontal superficial atp
group A.38
C. vertical group of node B.30
D.internal iliac nodes C.10
E.external iliac nodes D.20
Ans: A111-Which hormone regulate E.4
prolactin ..... Ans:a
A.dopamine(Prolactin inhibitory hormone) 119-Avidin
B.cortisol A.Biotin
C.growth hormone Ans:A
Ans: A 120-Esr increase in
112-Pregnent lady 34weeks with vaginal A. anemia
bleeding and coagiulation problem . frist B,polycythemia,
step of managment was C.infection
A.. evacuation of uterus D.ccf
B.blood transfusion Ans:C
C.heparin therapy 121-Structure arches over Root of left
D.platelates transfusion lung
E.streptokinase A. Aorta
175

B.azygous vein C.tropinin T


C.phernic nerve D.tropnin I
D.vagus nerve E. alkal;aine phosphate
Ans:A Ans: A129-Muscle of mastication
122-2% lidocaine A. ant and post belly of diagastric
A. 4ml B.stylohoid
B.8ml C Lateral Ptergoid
C.20ml D.stapedus
D .10ml E.tensor tympani
Ans:B Ans:C
123-48 hrs child meningitis ? 130-Milk has excessive.
A e.coli A. igA
B.Gp A strep B..igg
Ans A GBS was not in option so ECOLi C.ig m
here D. ig e
It is ASIM SHOAIB Mcq Ans:A
124-Oestrogen increase risk of 131-True abt testes
A. thromboembolism A. para aortic node
B.dvt B.superficial inguinal node
C.hepatic carcinoma C.internal iliac node
D. adenoma D.celiac node
Ans:A Ans:A
125-Beta thalasemia trait 132-Chemotectic factor Is
A.5%... A.nitrous oxide
B.10% B. C3b
C.1% C. C5-9 complex
D.15% D. leukotrine B4
Ansa:A126-Purpra on whole body E.histamine
A.Defect in capillary permeability Ans: D133-Abt doctor patient relation
B. Dec. Platelet count LESS THAN 90000 and how good a doctor is known by=
C.dec platelets less than 20000 A.his degree
D. itp B.his records
Ans:C C.good bedside manners and polite behavior
127-Baby born..apnoicspells.no breath D.punctuality
sounds on left sideimprove on upright Ans:C
postureA.pDA B.VSD 134-Severe mg deficiency causes..
C.Diaphragmatic hernia A. hypocalcmia
D.ASD B.Hypophosphate
E.TOF C.Hypokalemia
Ans:E Ans: A135-Regarding thyroid drain into
128-1st enzyme to appear after 2 hr after A.deep cervical lymph nodes
mi. B.superficial cervical lymph node
A. ckmb C.submental lymph node
B. LAD Ans:A
136-Upper lateral quadrant of breast is
drained by which grp of nodesA.anterior
176

axillary lymph node d.hepatoma


B.post group of axillary lymph node Ans:B
C.internal mammary lymph node 143-Double peritoneal ligament is
D.postintercostals lymph node A.round ligament
Ans:A137-Symphysis pubis B.BroadLiagamnet
A.secondry cartilaginous joint C.ovarian ligament
B.primary cartilaginous joint D.tranverse cervical (cardinal ) liagament .
C.fibrocartilagenous Ans:B
D.syndesmosis 144-During hypoxia changes will
E.synarthrosis occur???
Ans:A A. Increase oxidative phosphorylation
138-Costochondral joint B. Reduced pH due to acidosis
A.primary cartilaginous joint C. all the processes will be blocked due to
B.fibrocartilagenous joint lack of ATP or due to hypoxia
C.secondarycartilahenous joint D. anaerobic glycolysis forming lactic acid.
Ans:A Ans:D
139-Sensations of urinary bladder before 145-Tumour regression by
emptying is carried by which tract... A.necrosis
A.Anteriorspinothalamic track B.Infraction
B.Postspinothalamic track C.apoptosis
C.Dorsal col medial D.ischemia
D.leminicus lateral Ans:C
E.lateral sphinothalamic tract . 146-maternal mortality calculation 50
Ans:E mother died n 10000 thousand live births
140-Superficial temporal artery is A.MMR=5/1000 live births
accompanied by which nerve= ans:A (asim bcq)
A.auriculotemporal nerve 147-which has anabolic action and
B.glossopharngeal nerve increases lipolysis and gluconeogenesis
C.deeppetrosal nerve A.thyroxine
D.vagus nerve B.Insulin
Ans:A C.GH
141-Pt having chills n fever 2 days later Ans:A
she complained of weakness and passage 148-neurogenic shock
of black colouredurine.Amobile health A.dec vasomotor tone
team was in the area .did a sample test B.occur in comatose pt
and diagnosis it as the most likely cause C.tachycardia
A. falciparum malaria Ans:A
B.renal stone 149-BP 80/50 ,Inc LDH, DEC CVP
C. G6ph deficiency A.Hypovolemic shock
D.paroxymal nocturnal hamoglobinuria B.Septic shock
E.poststrtetococcalglomerculonepharitis C.Pericardialtamponade
Ans:A C.CCF
142-Which is benign D.cardiogenic shock
a.Lymphoma Ans:A
b. Leiomyoma
c.Melanoma
177

150- Patient with some haematological B. toxoplasma


disorder developed dyspnea after pronged Ans:Aq
hospital stay 158-destroyed in substantianigra
On choincpulm gradient something The A.Dopamine
primary cause is B.Acetycholine
A.MI C.epinephrine and norepinephrine
B.Primaryhaematological disorder ANS:A
C.Pulm.HTN 159-actinomycin D mechanism of action
Ans:C A.Binds DNA with hydrogen bond
151-increase ICP B.Inhibits mRNA and RNA Polymerase
A.Halothane C.Inhibits t RNA
B.Isoflurane D.Inhibits ribosomal RNA
C.Sevoflurane Ans:A
ANS:A 160-Gonorhea easy method of detection
152- follicle with oocyte surrounded by A.gram stain
zonapellucida and multiple granulosa B.z N stain
layers plus antrum containing fluid C. biochemical method
A.Primordial D.il6
B.Primary follicle Ans:A
C.Sec follicle 161-sepsis diagnosis Criteria
D.Tertiary follicle A.Wbc<3000 or >15000
Ans:C B.Blood culture postive
153-how to prevent congenital rubella C.Heart>100
A. avoiding contact D.Heart<100
B. vaccination in young females Ans:b
C.vaccine to babies 162-preganglionic fibres
D. family education A.A alpha fiber
Ans:B B.Abeta fiber
154- Gfrdec in C.B fiber
A.afferentartiole constriction D.C fiber
B.afferentartiole dilatation E.A gamma fiber
C.efferent arteriole constriction Ans:c
D.dec angiotensin 163-Dec plasma osmolarity by infusion of
Ans:A A.NS
155-Lysosomes B.VASOPRESSIN
A.Hydrolytic enzymes C.aldosterone
B. oxidase enzyme Ans:B
C.produce from golgi 164-MOST IMP SOURCE OF
D. formed from ribosome CREATININE
Ans:A A.SKELETAL MUSCLE
156-regarding fetal heart B.Kidney
A.Endocardial cushions divide it into rt and C.Heart
left atriovencular chambers D.liver
Ans:A Ans:A
157-chorioamniotis cause
A. E.COLI
178

165-B12 Absorption in B. Dec synthesis of type 3 collagen


A. Terminal Ileum C. Dec tensile strength of collagen
B.colon Ans:c
C.jejunum 172- of 20 women with neonates getting
Ans:A tetanus 19 had not taken a tetanus toxiod
166-Twave appear vaccine and of 40 whose babies did not
A.ventricular repolarization have tetanus.30 had taken two tetanus
B.ventricle repolarization toxiod shots during pregnancy .the odd
C.atrial systole ratio will be??
D.ventricle systole A.10
Ans:A B.11
167-milk is notoriously deficient in C. 7
A.pantothenic acid D. 8
B. vitamin c E.9
C. vitamin a Ans:e
D.iron 173-highest resistance in
E.riboflavin A.Arteries
ANS:a (cpsp key) B.Veins
168- There is variation in incidence of pre C.Arterioles
eclampsia .most significant change in fall D.Capillaries
of dis is d/t A. Dec Ans:c
induction of labour 174- thymus
B.dec edema of preg women A. regress after birth
C.fall in maternal and perinatal mortality B.formed by 4th pharyngeal arches
D.inc c/s rate C.produced T cell
E. unanimous definition of disease D.none of above
Ans:c E. B cell maturation occur in thymus
169-c AMP mediated mechanism in Ans:c
membrane 182- Sub arachnid space contain
A.Receptors A. s2 s3
B.Pumps B.S1-S2-
C.Carrier proteins C.S3-4
D.Enzymes Ans.A
Ans:a 175.pregnant lady in her late trimester
170-erytropoitein synthesis came to opd …pale…noprevious
A.cells in peritubular capillary antenatal checkups done..to find out type
B.maculadensa of anemia which ix. Is best
C.juxtaglomuler cells a.ferritin
Ans:a b.tibc
171- 30 years old lady after c.hb electrophoresis
cholcystectomy..she complains that her Ans. B
wound is not healing. Her diet contains 176.least role in healing
increase protein intake and lacks fruits A.proteins
and vegetables. .factors responsible for b.vit c
delayed wound healing c.ascorbic acid
A. Dec formation of granulation tissue d.factor 7,9Ans.A
179

177.side effect of antihyperlipemic drugs A.inc.Na2co3


A.gi effects Ans.A(asim bcq)
b.peripheral neuropathy 196.pacinian corpuscle
178.50 yr old para7 hving 3rd degree A.pressure
prolapsed ..weakness of which ligament B.high frequency vibration
A.transverse cervical ligament C.touch
B.pelvic diaphragm Ans.B
C.uterosacral ligament 188.roof of ant.horn of lateral ventricle
Ans: B A.body of corpus callosum
179.longest pro erythrocytic time Ans.A
A.vivax 189.volume of distribution not affected by
B.malaria A.age
C.falciparam B.sex
Ans.B Ans.B
180.decrease venous return 190.pregnant lady using antihypertensive
A.leg muscles paralysed drugs..fetal kidney affected..drug used
Ans.A A.captopril
181.muscles derived from Ans.A
A.mesoderm 191.incidence
B.,ectoderm A.noof new cases in a total population at
C.endoderm risk
Ans.A Ans.A
182.sriatum 192- Pseudomonas toxin
A.putamen plus caudate A.endotoxin
B.putamen plus globus pallidus B.exotoxin
C.caudate plus globus pallidus C.erythrogenic toxin
Ans.A d.toxic shock syndrome
183.dynamic support of uterus Ans. B
A.pelvic diaphragm, GYNAE 9 NOVEMBER
B.transverse cervical ligament
C.uterosacral ligament MORNING PAPER
Ans.A Q1. Irritant receptors in the airways
184.female with vaginal discharge given a.carries by myelinated fibres
treatmentnow presented with bad mouth b.rapidly adapting
taste and thick vaginal discharge with c.stimulate cough and bronchoconstriction
itching drug given was? ANS: C
A.metronidazole Q2.a 45 years man, smoker working in
B.clindamycin tyre factory developed transitional cell ca.
Ans.A cause
185.respiratory symptoms A.smoking
A.ascariasis b.aromatic amines
Ans.A ANS:.A
186.Good analgesic bad anaesthetic Q3.cimitidine main side effect
A.NO a.effect liver mtabolism (Ans:)
Ans.A Q4.ovary lymph drainage
187.depletion of potassium in body a.paraaortic lymph nodes
180

b.iliac nodes c.0.8


c.inguinal nodes Ans:>A
ANS:.A Q13.L1-L2 forms
Q5.epithelium in intestine a.inferios hypogastric plexus
a.simple columnar non ciliated epi b.mesenteric plexus
b.ciliated columnar epi c.lumbar plexus
c.stratified columnar ANS:.A
ANS:.A Q14.detrousor muscle contraction is
Q6.True hermaphrrodite controlled by??????
a.xxy a.parasympthetic from pelvic sphlanchic
b.xx/xy b.pudendal s1 s2 s3
c.xx ANS:.A
d.yy Q15.In RTA due to trauma to sacrum
ANS:. A CPSP KEY and ischeal spine it can rupture which
Q7.Increase carbohydrate consumption viscera???
requires a.urethra
a.thiamine b.urinary bladder
b.riboflavin c.anal canal
c.biotin d.rectum
ANS:.A ANS:.A
Q8.Man had lipid profile after having Q16.no of functional ova after 2nd
burger and fatty food.. his blood shows meiotic division
whitish frothy appearence a.1
a.chylomicron b.2
b.vldl c.3
c.Hdlp d.4
ANS:.A e.8
ANS:.A
Q9.Cervix lymph drainage Q17.structure to ryt side of trachea in sup
a.internal iliac mediastinum????
b.paraaortic a.Right vagus
ANS:.A b.aorta
Q10.max effect of 5% lignocaine in spinal c.lymph nodes
anaesthesia can b achieved by ANS:.A
a.injecting while pt is sitting Q18.location of submandibular gland in
b.coughing neck
c.giving adrenaline a.bounded by two bellies ofdiagastric and
ANS:.C mandible
Q11.amount of o2 binding to 1g of hb b.other options were anterior n posterior
a.1.34gm triangle boundries
b.2g ANS:.A
c.2.34g Q19.after emerging from jugular
ANS:.A foramina immediate relation of internal
Q12.safe dose of fio2 in pregnant lady jugular vein
a.0.5 a.accesory nerve
b.0.65 b.carotid sheath
181

ANS:.A Q28.period btw remission and


Q20. granulosa cell tumor contain reappearence of symptms
Ans:.call exener cells a.recovery
Q21.safest site of pleural effusion b.incubation
drainage c.latent
a.lower border of intercostal space in mid ANS:.C
axilary line Q29.malignant tertian malaria
b.lower border of intercostal space in ant a.falciparum
axilary line b.malarae
c.upper border of intercostal space in mid c.vivax
axilary line ANS:.A
d.lower border of intercostal space in mid Q30.etiologic agent of disease best
clavicular line determined by
ANS:.A a.incidence
Q22.pudendal nerve root valu b.prevalence
a.s1 s2 s3 c.disease index
b.s2 s3 s4 Ans:>C
c.l1 l2 l3 Q31.systemic hemosiderosis due to
ANS:.B a.excessive iron intake
Q23.pregnant lady e- fever chills rigors b.decrease iron utilization
and black urine cause c.hemolytic anemia
a.plasmodium falciparum ANS:.C
b.G6pd dficiency Q32.thelesemia trait investigation
ANS:.A a.dec MCH MCV
Q24.nitrous oxide when given wid o2 in B.INC HBF
50:50 best acts as C.normal HBA2
a.good analgesics D.decrease ferritin
b.good anasthetic AND.A
------------------------------------------------------ Q33.Hormone given as adjuvant in
----------------------- depression therapy????
c.relaxes uterus well a.ACTH
ANS:.A B.Cortisol
Q25.in theory ur null hypothesis is c,.thyroxin
rejected it meAns: d.prolactin
a.there is difference btw observed values ANS:.C
b.there is no difference btw observed values Q34.about left kidney
ANS:.A a.separated from psoas by quadratus
Q26.relation of left kiidney posteriorly lumborum
a.body of pancreas b.PCT lined y cuboidal epithelium
b.Diaphram An>A
Ans:>b Q35.oocyte surrounded by cell layer
Q27.relationship of supra renal gland called
a.supplied by thoracis sympthetic/ a.zona pellucida
b.left lies on left central tendon b.comulus oophorus
c.right lies on right central tendon ANS:.A
Ans:>A Q36.a long scnerio asking at the end b/w
182

trAns:versus muscle n parietal a.secretory phase


peritonium structure b.proliferative phase
a.trAns:versus fascia ANS:.A
b.internl oblique Q48.mid luteal phase surge
c.visceral peritonium a.LH
ANS:.A b.progeterone
Q37.specific side effect of ANS:.A
cyclophosphamide Q49.scnerio young girl ulcers on body
ANS:.hematuria ,howell jolly bodies/
Q38.ramnant of urachus a.sickle ceell anemia
ANS:.allantois b.thalasemia
Q39.birth canal is supplied by ANS:.A
a.pudendal nerve Q50.patient lost2 litres of blood what will
b.parasympthetic decrease
c.obturator nerve a.stroke volume
ANS:.A b.vascular resistance
Q40.urogenital triangle is supplied by ANS:.A
a.ilioimguinal nerve Q60. What prevents lactation during
b.pudendal nerve pregnacy
c.obturato a.high prolactin level
ANS:.A b.high estrogen and progestrone
Q41.inferior rectal artery is branch of c.high level oh HPL
a.internal pudendal D.low growth harmone
b.internal iliac ANS:.B
.femoral Q61.HCG is
ANS:.A a.91 ,145 aminoacids
Q42. anterior interventricular artery is b.44,67
accompanied by ANS:.A
a.greater cardiac vein Q62.what effects the positive predictive
b.lesser cardiac vein value
ANS:.A a. incidence
Q43.ca ovary ..surgically ovary removed b.prevalence
contains epithiliod cells and lymphocytes c.specifity
a. granuloma pyogenes d.sensitivity
b.granulomata formation ANS:.B
Ans:>B Q63.2+2 (2 into 2)is
Q44.non caseating necrosis.cause a.chi-square
ANS:.Sarcoidosis b.t table
q.45.tractus solitarios carries c.student test
ANS:.taste fibres ANS:.A
Q46.bp falls belo 50 mmhg .main control Q64.histolgy of tissue shows partly
by ciliated and some non ciliated columnar
Ans:>Cns ischmic response epithelium the tissue is
Q47.Proliferative epithelium containing a.uterine tubes
tortous glands thickened epithelium b.uterine cervix
which phase c.cervix
183

d.intestine c.not dangerous


ANS:.A ANS:.B
Q65. A test which help in diagnosis n Q73.cause of edema in renal dsease
eliminating ths disease a.dec albumin and salt retention
a.sensitive b.inc hydrostatic pressure
b.specific c.dec hydrostatic pressure
c.positive predictive ANS:.A
prevalence Q74 .cause of NA retention
ANS:.A a.kidney failure
Q66.lymph drainage of upper outer b .heart failure
quardrant of breast c.liver failure
a.pectoral lymph nodes ANS:.B
b.anterior axillary Q75 .long scnerio asking both obese and
c.internal thoracis non obese has the equal risk of developing
d.clavicular which disease
ANS:.B a.cirrhosis
Q67.medial quadrant of breast drains b.osteoarthritis
into c.hypertension
a.internal thoracis nodes d.diabetes
b.anterior axillary Ans:>C
c.infraclavicular nodes Q76.in dehydration which will have low
ANS:.A concentration in body
Q68.necrosis in brain a.ADH
a.coagulative b.water concentration
b.liquefective c.ANP
c.fat necrosis d.aldosterone
ANS:.B ANS:.C
Q69.most common cause of pulmonay Q77.locally malignant tumor
embolism a.adenoid cystic carcinoma
a.DVT b.pleomorphic adenoma
b.mural thrombus ANS:.A
c.MI Q78.Toxin produce in scarlet fever
d.surgery a.erythrogenic toxin
ANS:.A b.exotoxin
Q70.in chronic hmolysis c.endotoxin
ANS:..decrease heptoglobin d.invasive
Q71.the antihypertensive best to use in ANS: A
diabetics Q79.lypmhocytes provide protection
a.captopril against
b.b blockers a.bacteria
c.ca channel blockers b.cancer cells
ANS:.A ANS:.B
Q72.about vibrio cholera Q80.structure present above pirformis
a.vaccination provides specific protection muscle
b.trAns:mitted by contaminated water and a.inferior gluteat
food(Ans:) b.superior gluteal
184

c.pudendal nerve Q88.pseudomona infection (stem not


d.femoral nerve remembered exactly)
ANS:.B a.TNF
Q81.lymphocytes provide protction Q89.thiopentene action remains for only
against 15 mintues
a.exogenous factors a.bcz of renal excretion
b.endogenous factors b.bcz it is rapidly distribued throughout the
ANS:.A body
Q82.what is absent in 1st cervical c.bc zof liver metabolism
vertebrae ANS:.B
a.spine Q90.in elderly dose of gentamycin is
b.foramina trAns:verserum reduced bcz
c.antereior arch a.because of decrease renal clearence
d.posterior arch b.because of decrease distribution of action
ANS:.A c.bcz of rapid clearence froom liver
Q83.deep inguninal ring is formed by metabolism
a.internal oblique muscle ANS:.A
b.trAns:versus muscle Q91.sceondary centre of ossification is
c.fascia trAns:versalis present in
d.externl oblique a.metaphysis
Ans:>C b.epiphysis
Q84.hairy leukoplakia is characteristic of c,diaphysis
a.HBV d.cotrex
B.HCV ANS:.B
C.HIV Q92.cardiac muscle doesnotdevelop
D.squamos cell ca tetany because
ANS:.C a.long duration of action potential
Q85.about tendineous insertion of linea b.long pleateu phase
alba c.increase amount of calcium in cardiac cells
a.one is present at xiphoid process d.decrease amount of calcium in cadiac cells
b.total in no ANS:.B
c.present at symphysis pubis Q93.about compact bone
ANS:.A a.lamellae are regularly arranged
Q86why atropine is given before b.haversian canal are present in medullary
anasthesia rays
a.to dec heart rate c.rich blood supply
b.to decrease tone of muscle ANS:.A
c.to provide good analgesia Q94.breastr milk is rich in
d.to decrease the secretion of GIT tract a. immunoglobins
e.to decrease the tone of git sphincter b.calcium
ANS:.D c.vit c
Q87.MMR definition d.vit k
a.no of materna deaths/1000 population ANS:.A
b.no of maternl deaths /100000 Q95.tumor causing homonymous
c.no of maternal deaths /100 hemianopia is present in
ANS:.B a.middle of chiasma
185

b.lower part of optic chiasma .b.pelvic diaphragm


c.upper part of optic chiasma c.perineal body
ANS:.C d.levator ani
Q96.etilogy of cervical carcinoma ANS:.B
a.HPV Q103.minimum caloric requirment in
B.herpes virus adult male is
c.chronic irritation a.25-30 cal
d.cervical metaplasia b.30-60cal
ANS:.A c.40-80 cal
Q97.metaplasia is d.80 -100 cal
a.functional change in one normal tissue to Ans:>A
other Q104.which of following acts on both
b.increase in nuclear cytoplasmic ratio central and peripheral chemoreceptors
c.[pleomorphism a.PH
d.irreversible B.oxygen
ANS:.A c.co2
Q98.histological examination shows d.hypoxia
squamous cells is ANS:.C
a.dysplasia Q105.blood flow through brain is
b.ca in situ cervix regulated by
c.metaplasia a.oxygen
d.ca cervix b.PH
ANS:.C c.co2
Q99.a lady lost his family in an earth d.acidosis
quake she would be having e.hypoxia
A.situational crisis ANS:.C
b .depression Q106.a long scnerio with woman with
c.anxiety hyperpigmentation moon face central
ANS:.A obesity. What will be increased in this
Q100.about hyaline cartilage women
a.present in knee meniscus a.ACTH
b.present in pinnae b.cortisol
c .not present in nose c.aldosterone
d.develop from enchondral ossification d.renin
e.larynx ANS:.A
Ans:>E Q107 about active trAns:port
Q101.most potent stimulus for a.NA is always involved
erythropoitin secretion b. is uphill movement
A.hypoxia c.requires co trAns:porte
b.anemia d.counter trAns:port
c.acidosis ANS:.B
d.erythrocytosis Q108.Erythropoitin is released by
ANS:.A a.peritubular capillary cells
Q102.uterus is prolapsed but anus is in its b.juxtaglomerular cells
positiion due to intact c.cuboidal cells
a.urogenital diaphargm d.mesangial cells
186

ANS:.A c.fungal
Q109. Inferior vena cava commences at d.protozoal
a.l5 ANS:.A
b.l4 Q117increase ejection fraction what Will
c.s5 decrease???
d.l1 a.EDV
ANS:.A B.ESV
Q110 ) epithelium of vagina and uterus is ANS:.B
derived from Q118 best example of drug
a. endoderm pharmakokinetic
b. ectoderm ANS:..antagonising naloxone on morphine
c.endo plus mesodem Q119.scnerio of young male child
d.ecto plus mesoderm presented with dark colour urine
ANS:.C anemia,no hx of fever cause
Q111.cause of necrosis in most of tissues a.GG6PD Deficincy
a. atheroma b.malaria
b. thrombosis ANS:.A
c. mechanical Q120chromosome are arranged of
ANS:.B equator in which phase
Q112.protrusion of jaw by a.prophase
a.lateral pterygoid b.anaphase
b. medial pterygid c. metaphase
c. temporalis d.telophase
d.stylomastoid e.prometaphase
ANS:.A ANS:.C
Q113 a person sustained a temporal blow q121 thiamine deficiency will cause
with rupture of artery, artery damage is .a.subacute combined degnration of spinal
a.middle meningeal artery cord
b. anteriorr cerebral artery b.peipheral neuropathy
c.posterior cerebral artery c.mallory bodies
d.lateral cerebral ANS:.B
ANS:.A Q122. Regarding platelets what is true
Q114.most imp factor causing delayed a. fragmentation of megaaakaryocyte
wound healing b eosinophils has half life of 1 month
a.ischemia c. platelets are increased in bacterial
b.infecton infections
c.thrombosis ANS:.A
d.metabolic Q123 biguanides side effects
ANS:.B a. flaulance and bloating stomach
Q115.location of SA node b. hypoglycemia
a.upper part of crista terminalis c.metabolic alkalosis
b.lower part of crista terminalis ANS:.A
ANS:.A Q124.NeurotrAns:mitter present at
Q116.pyogenic meningitis cause presynaptic cleft??
a.bacterial I a. calcium
b.viral b.acetycholine
187

c.epinephrine ANS:.A
d.dopamine Q133scnerio of molar pregnancy what
ANS:.B will increased thn normal pregnancy
Q125.which surface of bladder not a. b-HCG
covered wid peritonium b.alpha-HCG
a.anterior c.alpha fetoprotein
b.superior ANS:.A
c. posterior Q134 scnerio of acute panctreatitis ,wht
ANS:.A will be increase
Q126 paramesonephric duct forms a. lipases
a.uterus uper part of vagina and uterine Q135 vasoconstrictor
tubes a. cocaine
b.lower part of vagina b.lignocaine
ANS:.A ANS:.A
Q127.Part of segmental bronchi with its Q136. Local anasthesia given which fibrs
artery is called will be anasthesized first
a.broncho pulmonary segment a. A-DELTA
b.tertiary bronchi b. A-beta
c.secondary bronchi c. c fibres
ANS:.A ANS:.A
Q128.Regardind micturation reeflex??? Q137 vit c causes
a.intergrated in sacral segment of spinal cord ANS:.hydroxylation reaction in collagen
b. once begin is self regenrative formation
c.control by pudendal nerve Q138.1st bone to ossify
ANS:.B a.clavicle
b.femur
Q129.Flight and fight response will c.tibia
cause??? ANS:.A
a.brochoconstrion Q139. Cause of septic meningitis
b.pupilary constriction a.Streptococcus
c.bronchodilation b.h influenza
ANS:.C c.crypococcus
Q130.lymphoid follicle is present in ANS:.A
a.cortex of lymph nodes Q140 fats proteins and lipid deposit
b.spleen harmone involve is
c.medulla pf lymph node a. insulin
thymus b.glucagon
ANS:.A c.corticosteroid
Q131. regarding mast cellls ANS:.A
a.it release histamine Q141 CNS myelination is caused bby
b.increase in acute in flammatory response a. oligodendrocytes
ANS:.A b. schwan cells
Q132.mid gut supply c, astrocytes
a.t12-l2 ANS:.A
b.l2-l4 Q142.Pus contains
c.t8-t10 a.dead bacteria
188

b.dead neutrophils c.iGM


ANS:.B d.parasite
Q143 smoking plus asbestos has highest ANS:.C
risk of Q152counselling
a.mesothelioma a.help patient themselves
b. bronchogenic carcinoma b. help family
ANS:.B c.help doctor to treat pt
Q144 microscopy of malignant tumor ANS:.A
a. invasion of othr tissue Q153most commn cause of hep b and c
b.pleomorphism a. needles
ANS:.A b.blood
Q145 characteristic of malignant tumor c. mother to fetus
a.metastasis ANS:.A
b.pleomorphism Q154.Immunological test is done fr
c.inc nucler cytoplasmic ratio a.amebic cyst
ANS:.A b. hyadatid cyst
Q146 skeletal muscle cannot sythesize ANS:.B
glucose bcz it lack which enzyme Q155. Glucose appear in urine of pt at
a.glucose 6 phosphorylase blood sugar level of 100mg/dl, cause
b.glucose phosphatase a. dec renal threshold fr glucose
ANS:.A b.decrease reabsorption of glucose
Q147 old lady wid ca endometrium cause c. diabets mellitus
a.exogenous estrogen d.impaired glucose tolerane
b.HPV ANS:.A
C.HSV Q156.Pt presented with ammenorhea and
d.irradiation weight gain cause???
ANS:.A a.hypperthyroidism
Q148 .LP done containing blood cause b.hypothroidism
a.subarachnoid hemmorhage c.hypoparathroidism
b.extradural hemmorhage ANS:.B
c. mengitis Q157 leprosy initial investigation sample
ANS:.A is taken from
Q149 counterpart of labia major a.blood
a.scrotum b.urine culture
b. testis c.nasal scrapings
c.penis ANS:.C
ANS:.A Q158 which closely covers the viscera
Q150.scnerio of iron deficiency anemia, a.visceral peritoneum
cause b.parietal peritonium
A.chronic blood loss c.fascia
b.folic acid deficiency ANS:.A
C.vit B12 def Q159. Scnerio od emphysema etilogy
ANS:.A ANS:..dec alph 1antitrypsin
Q151 eosinphils are raiseed in all except 7TH NOVEMBER MORNING
a.igE RADIOLOGY
b.asthma
189

1. infraorbital artery is a branch of infarction artery damage is


A) Maxillary artery A) Anterior coronary artery
B) Superficial temporal artery B) posterior coronary artery
C) Mandibullar artery C) circumflex artery
ANS A D) left anterior descending artery
2. Sacral hiatus is located at ANS C.(both RCA n circumflex can cause )
A) In S5 and S1 9. Most important chemical mediator in
B) Between S1 and S2 allergy is
C) Between S5 to coccyx A) Interleukins
ANS C B) Bradykinin
The sacral hiatus corresponds to the C) Monocytes
posterior caudal opening at the end of the D.Histamine
sacral canal, which usually ANS D
occurs at the fifth sacral vertebra (S5), at the 10. Oblique fissure of lung is located at
posterior surface of the sacrum. A) T3 spine to T5 costochondral junction
3. Superior parathyroid gland location B) T3 spine to T6 costochondral junction
A) Anteior to thyroid gland ANS B
B) posterio lateral to thyroid gland 11. patient with flu like symptoms with
c) posterio inferior to thyroid gland fever, test for infectious mononucleosis is
ANS B positive likely ??
4. function of baroreceptors in blood ANS monospot test
pressure is 12. most potent chemical carcinogen is
A) To decrease arterial pressure A) Methyl alcohol
B) to increase arterial pressure B) ethyl alcohol
C) hypertension C) xylol
D) hypotension ANS B
ANS A 13. patient with low grade fever, night
5. In brain, major blood vessels are located sweats , weight loss shows
in A) Caseous granuloma
A) Subdural space B) macrophages
B) sub arachnoid space C) epithelial cells
C) within pia ANS A
D) extradural space 14. Structure passing through aortic opening
ANS B of diaphragm is
6. Melanocytes in skin are derived from A) vagus nerve
A) ectoderm B) thoracic duct
B) emdoderm C) Recurrent laryngeal nerve
C) mesenchyme ANS B
ANS A 15. important feature of 7 cervical vertebrae
7. In anterior wall MI artery damage is is
A) Right coronary artery A) massive body
B) left coronary artery B) heart shaped body
C) Left Anterior descending artery C) short cervical spine
D) circumflex artery ANS All incorrect
ANS C 16. Lymphocytes Have
8. In inferior wall septum myocardial A) important role to prevent body against
190

cancer point artery likely to be damage is


B) convert to monocytes A) iliiohypogastric nerve
ANS A? (poor recall of dis bcq) B) inferior epigastric artery
17. Pendular knee jerk is present in C) inferior mesentric artery
A) upper motor neuron lesion D) deep circumflex iliac artery
B) lower motor neuron lesion ANS D
C) cerebellar dysfunction 25. Patient is suffering from meningitis
ANS C organism involved after causing pygenic
18. function of ICAM and VCAM is lung abscess and then
A) direct endthelial injury spread to brain is
B) margination A) Ecoli
C Adhesion/tight binding B) Streptococcus
Ans :C C) staph aureus
19. In CNS myelin sheath is formed by D) pnumococcus
A) schwan cells ANS C
B) glial cells 26. Virus that causes cancer are likely to
C) oligodentrocytes cause damage to
ANS C A) RNA
20. the potent intracellular buffer is B) protooncogenes
A) Phosphate C) Oncogenes
B) bicarbonates D) DNA
C) CO2 ANS C
D) Hb 27. Respiratory zone is situated
ANS A A) Between terminal bronchioles and
21. Contents present at free border of lesser alveoli
omentum B) Between bronchioles and alveoli
A) CBD, cystic duct, hepatic artery C) Between alveolar ducts and alveoli
B) CBD, cystic duct , hepatic artery ANS B
C) CBD, hepatic artery, portal vein 28. Patient with markedly decrease TSH
ANS C increase T3 What is the diagnosis
22. Patient with whole body patechea, A) Hypothyroidism
platelets decrease, rest of labs normal with B) Hyperthyroidism
no previous history ??? C) hyperparathyroidism
A) thrombocytopenia D) hyperparathyroidism
B) hemophilia ANS B
C) thalessemia 29. Patient with occasional diplopia
D) ITP dizziness suddenly collapsed.
ANS A A) hemianopia
23. patient who got injury and his arm hangs B) basillary artery thrombosis
on the side winging of scapula nerve C) pontine hemorrhage
damage is ANS C
A) Suprascapular nerve 30.Location of SA node in right atrium
B) axillary nerve A) Subepicardium
C) long thoracic nerve B) Subendocardium
ANS C C) endocardium
24. Appendiectomy done, at MC burneys ANS A
191

31. IN which lobe of the lung has 3 A) pain an temperature is lost on the same
beonchopulmonary segments side vibration and position sensation is lost
A) right upper lobe on the opposite
B) Left lower lobe side
C) right lower lobe B) Only vibration ans position sense is lost
D) right medial lobe on the same side
ANS A C) Pain and temperature is lost on opposite
32.Hemochromotosis diagnosed by side and vibration and position sense is lost
A) Ceruloplasmin level on same side.
B) antimitochondrial antibodies ANS C
C Serum Ferritin+transferrin saturation 40. one gram of Hb carry how much O2
ANS C A) 7.34 ml
33. WILSON disease is diagnosed by B) 5.34 ml
A) Urinary ceruloplasmin level C) 3.34 ml
B) serum ceruloplasmin level D) 1.34 ml
ANS B ANS D
34. Worst reaction presents by transfusion of 41. Penetrating wound at left side of margin
A) A- TO A+ of sternum, structure damage is
B) A+ TO AB- A) Right atrium
C) O+ TO A+ B) right ventricle
D) O+ TO B+ C) intercostals membrane
ANS A D) Intercostals muscle
35. Caput medussa is formed by ANS C
A) abdominal vein to paraumblical veins 42 IN anterior duodenal ulcer perforation
B) portal vein to greater splancnic vein fluid will accumulate in
ANS A A) Right illiac fossa
36. vascular segment of kidney is B) pouch of dougles
A) 5 C) hepatoduodenal pouch
B) 7 ANS A
C) 4 43. Apoptosis
D) 6 A) Councilman bodies
ANS A ANS A
37. In which disease splenectomy is best 44. ambigious genitalia with 46 XX
responded karyotype diagnosis is
A) hemophilia A) klinefelter syndrome
B) thalessemia B) turner syndrome
C) G-6PD deficiency C) mixed gonadal dysgenesis
D) Hereditary spherocytois ANS c?
ANS D 45. A 50 years is short stature due to short
38. Beta -thalassemia limbs but intellegence is normal this
A) deficient synthesis of alpha chain disorder is
B) decrease synthesis of beta chain A) autosomal dominant
C) increase synthesis of beta chain B) autosomal recessive
D) deficient synthesis of beta chain C) X linked
ANS B ANS A
39. In brown sequard syndrome 46. lesion at optic chiasma its effect will be
192

A) bitemporal hemianopia B) iliohypogastric lymph node


B) binasal hemianopia C) Para aortic
ANS A ANS C
47. Trauma at medial epicondyle results in 55. In young males mobile cilia are present
ulnar nerve damaged, loss of function is in???
A) medail 1/3 of palmer surface of hand A) ductus difference
B) medial 1/3 of palmer and dorsal surface B) epidydmis
of hand C) testis
C) lateral 2/3 of dorsal surface of hand D) seminal vesicle
ANS B? ANS B
48. Location of sternal angle 56. IN males narrowest part of urethra is
A) at meaniburo sternum A) bulbous
B) in accordance with costocondral joint B) penile
ANS A C) memberanous
49. A 40 year old female has prominent D) prostatic
neck swelling, hoarseness of voice ANS C
A) Throid scan 57. A 6 year old child while playing
B) T4 stimulation suddenly falls down and died on the spot on
C) biopsy autopsy there is fresh
ANS A thrombosis in coronary artery, all cardiac
50. Male with sterility due to azospermia chambers are normal
what is dignostic test A) subacute endocarditis
A) FSH and LH B) Kawasaki disease
B) Prolactin ANS B
C) Testosterone 58. Location of cruciate ligament of knees
D) progesterone A) subcapsular
ANS A ANS A
51.On Xray right Border of heart?? 59. IN adult female breast atrophy is caused
A..SVC by
B..right atrium A) prolactin
Ans A B) estrogen and progesterone
52. blood supply of left atrium C) estrogen
A) Left circumflex artery D) progesterone
B) left anterior descending artery ANS C
C) right anterior descending artery 60.in RTA there is fracture of left 8TH rib,
ANS A the fracture most likely to damage
53. A 55 Year old female with cervical A) left kidney
cancer, lymph node involved. B) spleen
A) external iliac lymph node C) Pancreas
B) inguinal lymph node D) Stomach
C) internal iliac lymph node ANS B
D) external and internal lymph node 61. In splenectomy the surgeon seperates
ANS D which structure to reach spleen
54. A 60 year old man with testicular tumor A) transverse mesocolon
lymph node first involved are ??? ANS A
A) testicular lymoh node 62. secondary active transport which
193

substance is transported B) ventricular depolarization


A) glucose ANS B
B) Sodium 71. Highest caloric value is of
C) Co2 A) proteins
D) bicarbonates B) carbohydrates
ANS B C) Fats
63. Immunological test done for diagnosis of ANS C
A) Liver abscess 72. Kidney activates which substance
B) hydated cyst of liver A) erythropoietin
C) hemoochromatosis B) renin
ANS B C) Carbohydrates
64. 41% hematocrit means there are ANS B
A) 41% of rbc is present 73. Most common carpal bone fracture
B) 41% of platelets and Rbc are present A) Scaphoid
C) 41% of rbc wbc and platelets are present B) lunate
ANS C C) Trapezium
65. abccess in first web space drainage is D) Pisiform
A) supra clavicular Lymph node ANS A
B) infraclavicular luymph node 74. Which antigen is common in featus and
C) Axillary Lymh Node adult with colonic cancer ????
D) supratrochlear lymph node A) CEA
ANS C B) alpha fetoprotein
66. feature of malignant Tumor ANS A
A) Pleomorphism 75. in terminal ileum resection there is loss
B) metastasis of ??
ANS B A) iron
67. IN a severely dehydrated patient fluid B) B 12
replacement is done by C) Calcium
A) 5% glucose solution ANS B
B) 10% glucose solution 76. in a 70 kg man minimun caloric
C) Isotonic saline solution requirement per day is
D) distilled water A) 25-30 g/Kg
ANS C B) 35-50 g/Kg
68. During wound healing strength to wound C) 50-60 g/Kg
is given by D) 70 g/ Kg
A) type 3 collagen ANS A
B) type 1 collagen 77. example of piviol joint is
C) type 2 collagen A) shoulder joint
ANS B B) axial joint
69. part of pleura supplied by phrenic nerve ANS B
A) Visceral pleura 78 how will you break bad news?
B) parietal pleura Ans:Separatel session with close relative or
C) Diaphragmatic pleura close friend
ANS C 79. diabetic patient on a diuretic presented
70. QRS complex occurs prior to with hyperkalemia and coma Cause
A) atrial depolarization A) diuretic intake
194

B) insulin and alveolar arch of one side is gone nerve


ANS B.(lack of insulin can present with involved is
DKA and hyperkalemia) A) inferior alveolar nerve
80. At cold exposure bodys first reaction to ANS A
heat production is by 91.Narrowest part of constriction of ureter
A) piloerection A Ishcial spine
B) Increase release of catecholamines B Ishcial tuberosity
C) shivering C. Sacroiliac joint|
ANS C Ans C
81. boy presents in room temperature 28c 92.Fastest conduction fibres of heart
pulses 120/min, BP 150/ 95 mmhg with Ans: purkinje
hyperventilation ?? 93.Scenerio of Cervical CA,lymphatic
A) hyperthyroidism drainade first to which node..???
B) Excercise A.Paracervical nodes
ANS A B.Intrnal iliac nodes
82. lymphatic drainage of greater curvature Ans>B
of stomach is 94.Bare area of liver not having ??
A. celiac node ans)peritonium
ANS A 95.Regarding lesser omentum?
83. inguinal canal posterior wall reinforced Double layered
by 96.Thoracic wall formed by which muscles
A) conjoint tendon ??
ANS A A internal intercostal
84.Phrenic nerve origin B diaphargm
A) c4 Ans>A
ANS A 97.Weight of liver
85.Carcinoma which filaments involve Ans>1.5 kg
A) desmin 98.Right bronchus measurement
B) keratin Ans>2.5cm
ANS B 99.Structure between celiac trunk n superior
86. svc and ivc drain into mesenteric ..
A) right ventricle Ans>Pancreas
B) left atrium 100,Celiac trunk arteries Right
C) right atrium gastroepiploic is branch of???
ANS C Ans> gastroduodenal
87.Two ureter and two calyses are due to 101.Select true statement regarding blood
A) premature divison of mesonehric duct supply of stomach:
ANS A Ans>right gastroepipoloic artery is branch
88. Structure pierced lateral to sternum of gastroduodenal artery
A) intercostal membrane 102.First branch of abdominal aorta ???
B) intercostal muscle Ans> inferior phrenic
ANS A 103.Surgeon recongnize the left kidney???
89. Nasopharygeal carcinoma caused by A.brown colour
A) EBV B.anterior relation with
ANS A stomach,pancrease,descending
90. in surgery supply to to molar lower lip colon,spleen,jejunum
195

Ans >B Lmn


104.Thoracodorsal supplies? Umn
Ans>Latismus dorsi Lmn wd ant cells
105.Rhomboid supply? Ans:C
Ans> Dorsal sapular 116:In hypertension .. Sensory loss ta
106.12 year old boy with obesity shayad .. Which area affected repeat q
hyperphagia short stature mental retardation Ant part of post limb wd genu
(some other things) Post part of ant limb wd genu
A.Turner Ans:A
b.Klinefelter 117:Total No of Lower pulmonary airways?
c.Prader villi Ans: 23
d.Angleman 118.Lunate fractured,nerve demaged is??
Ans C Ans: ulnar
107.Relation of structures in femoral 119.Footballer was hit by ball on Right side
triangle from medial to lateral?? of head on x-ray parietal bone fracture and
Ans>Femoral vein, femoral artery femoral hematoma at temporal region.?????
nerve. Structure damaged
108.Structures below inguinal ligament , Asn:Middle meningeal artery
from medial to lateral ? 120.Rapidly adapting sensory receptora?
Ans.Femoral Vein artery and nerve A. Baroreceptors
109.Lymphatic drainage of upper outer B.touch receptors.
quadrant of breast..??? C.pain receptora
Ans>Ant axillary Ans:A
110.Hdl which lipoproteins??? 121.Embryo se...neural tube is formed in ???
Ans.B100 Ans:4th to 5th week
111.Reticulocytes decreased in ? 122.Radial nerve damage?
A.Chronic renal disease Asn: loss of extension of wrist joint
B.Bleeding from gut 123.Child with edema of lower limbs. best
Ans A initial investigation
112.Which tumor invades nerve sheath ? A.Urine for proteins
A.Adenocarcinoma B.Serum album i
B.Expiliomorfic carcinoma Ans:A
c.Adinoid cyctic carcinoma 124.Axon hillock related..?
Ans:B Asn:absence of ribosomes
113.About highest level of iliac Crest. 125:Regardin Optic canal ,whic structure
Asn:Lies at the level of l4 passing???
114.Triceps reflax lost wd index and large Ans:Opthalmic artery n nerve
fingees parestesia 126.Maxillary nerve exit from rotandum
C6. 7 then enter into
C7. T1 A.Temporal fosa
C5. 6 B.Infra temporal
AnsA C.Pterygopalatin fosa
115.Stop bleeding of internal carotid at??? Ans :C
Ans: C6 127.Which fatty acids have coronary
116.Lateral calf muscle weaknes wd los of risk...something like that was asked?
reflaxes A.Omega f.a,
196

B. polyunsaturated, a.Cricoid
C momunsaturated, b.Nostrils
D.transaturated c.Vocal cord
Ans:C Ans:A
128.Regarding post cord of brachial plexus 141.Microscopic featire of malignant tumor
branches suppy ? was asked?
Ans:Extensors AnsInvasion.
129.Zinc def leading ??? 141.Transmission of herpea simplex...
Ans:scaly dermatitis Ans.droplets
130.S.A node..located ?? 142.Bitemporal hemianopia aneurism is
Ans:in upr part of R.A at???
Which structure recieves input from cerebral Ans:anterior communicating artey
cortex but don't give input to cerebral cortex 143.Hormone release in stress ?
in return(something like that) A.Gh
Ans:Caudate nucleus B.Catcolamn
131.Steroid has anti inflammatory action C cortisol
mechanism?? Ans:C
Ans:Inhibit phospholipase 144.Strongest ligament??...
132:Regarding Lesser sac ? Ans:illeofemoral
Asn:Hep artery and portal vein 145.Middle thyroid vein drains into
133:Surgeon operting lower part of ureter , A.Internal jugular
surface marking like that B.External jugular
Asn:common iliac artery bifurcation C.Anterior jugular
134:Inferior surface of central portion of Ans:A
diaphragm is supplied by. 145.Genital fold ??
Ans:Phrenic nerve Ans:clitoris in female
135.Urethral Sphincter is located at 146:Irreversible cell injury??
a.above prostate Ans:Massive influx of calcium.
b.sup perineal pouch 147.Medial arcuate ligament formed by?
c.deep perineal pouch A.Lumbar facia
Ans:C B.Rectus abdom muscle
136:Hypotension amd vasodilation in which C.Diaphragm
shock? Ans:C
Ans:Septic 147.Edema in CCF??...
137.Transitional cell Ca in a smoker Ans:inc hydrostatic pressure
working in tyre factory due to 148.One regarding spleen
a.Tobacco a.length of spleen 12cm
b.Aromatic Amines b.more echogenic than liver on UsG
ans:a (ref goljan patho) Asn:A
138.Regarding abdominal aorta 149.Cornary sinus drains ?
Ansinf phrenic is 1st branch Ans:Rite atrium
139.Highest transverse point? 150.Ek cervical lymphadenopathy, aur on
A.Pubic tubercle biopsy kuch follicles lkha tha, aur mutation
B.Poster sup iliac spine poochhee thee, I did 8:14
Ans:B 151.Adh is stimulated by ...??
140.Narrowest part of airway in child Ans:inc plasma osmolarity amd dec plasma
197

vol Ans: Tib ant and Tib post


152.HEAT STROKE..CONTRIBUTING 163.Co2 passes more readily actoss cell
FACTOR membrane thn o2 because
A.EXCESS SWEATING I did diffusion co efficient
B.DECREASED SWEATING 164.Sartorias supply by ??
C.POSTERIOR HYPOTHALAMUS Ans:femoral nerve
UNRESPONSIVE 165.Superficial Dorsal vein of penis drains
Ans:c into
153.Fungal sinusitis in nose most common?? ASantorini's plexus
Ans: aspergillus.. b(Superficial or great) saphenous vein
154.A baby boy..recurrent resp inf...dec b Ans:b
cells and dec immunoglobulins? 166.Atrial systole ?
a. x linked agamaglobinemia Ans:”a’ wave on JVP
b. brutons agaamaglobulinemia 167.Thoracolumbar nerves ...
Ans:B Ans.Sympathetic NS
155.Ligamentum teres...?? 168.Ek pregnant lady wid obstructive
Ans:umbilical vein jaundice best test ??
156.Viral infection? Ans :GGT
Ans:Inc lymphocytes 168.Parasympathetic outflow frm segents
157.During intrauterine life testosterone a.L1 L2
secretion by stimulated by b.S2 S3 S4
A.FSH c.S1 S2 S3
B.LH Ans:b
C.Placental estrogen 169.One mcq of horner syndrome chooses
D.Androstenedione one
E.Placental Hcg A.Anhydrosis
ans:B b.Anhydrosis and Ptosis
158.Arterial supply to cbd ? ANS:b
Asn:gastroduodenal and hepatic 170.Sensation of distention from rectum "
159.Letral upper breast drain ? 1. Parasympathetic afferents.
A.Anterior axilary 2. Superior rectal nerve
b.Lateral axilary 3. Inferior rectal nerve
Ans:A 4. Middle rectal nerve
160.Infront of lower part of head of Ans.3
pancreas? 171.Achpndroplasia ( autosomal dominant
A.Sup mess art scenerio)
b.Left colic 172.One question on cause of hydrocele in 1
C.Ivc year old child :
Ans A A. Filariasis
161.Theophylline B Testicular inflammation
most commonly causes C. Scrotal wall something
a.nausea Ans:A
b. seizures 173.Electrolyte whose variability in Ecf
c. apnea affects heart the most???
Ans:A a.Ca
162.Foot turns inward? b.K+
198

Ans:B
174.Diff b/w primary and sec intention
healing
a.keloid
b.Granulation tissue
c.Wound contraction
Ans:C
175.Cingulate gyrus is supplied by
Ans anterior cerebral artery
176. touch n vibration by
AVentral spinothalamic
bSpinothalamic
CMedial laminiscus
Ans:C
177.Unlocking of knee joint by???
Ans: popliteus
199

RADIOLOGY
2016 PAPERS
SEPTEMBER, OCTOBER, NOVEMBER
b. increasedAchrelease
c. tetany
d. arrhythmia
Ans:: aa
RADIOLOGY5SEP 4. Originof thyroidfollicular cell is
a.Endoderm
MORNING2016 b.Mesoderm
1. Earliestsignof VitaminA deficiencyis c.Ectoderm
a.Night Blindness. Ans:: aa
b.Keratitis 5. Epithelium
c. Bitot’ spots. ofUterusandVaginaisderivedfrom
Ans:: aa a.Endoderm andMesoderm.
2. Young boywithpainlessswelling b.Mesoderm alone
ofrightorbit, other eyeis normal , what is c. Endoderm alone
the Dx ? Ans:: aa
a. Optic nerve glioma. 6. A
b. Retinoblastoma. womanhadfallopiantubeligature,sheisfoun
c. Cataract dtobeinshockafter6hours,Laparotomy is
d.Syphilis doneandintraperitonealhemorrhageisfoun
Ans:: aa d. bleeding ismostlikelyfrom branches of
3. Hypermagnesemia causes a. uterineartery
a. decreasedAchrelease
200

b. commoniliacvessels e. thalamocortical tract.


c. external iliacvessels Ans:: ee
d. aorta 14.Manduring running developsaching
Ans:: aa paininankle,nextdayswhenhewakes
7. A uphefounds
manwalks100yardsandfeltspainincalf,reli ecchymosedaroundanklejoint,hecanstand
evedafterstopping,arteryinvolvedis ontiptoes butithurts.
a. Popliteal artery. a.Ruptured Plantaris tendon.
b.Peroneal artery. b. Ruptured Achiles tendon.
c. Posterior tibial artery. Ans:: aa
Ans:: aa 15.structureanterior topancreas,
8. Leftrenal veinisposterior to A. hepatoduodenal ligament b. leftcolic
a. headofpancreas c. portal vein
b. 3rdpart ofduodenum d. superiormesentericvessels
c. aorta d. IVC e. splenicvessels
e. Portal vein Ans:: dd.
Ans:: bb 16.vessel dilatedmostlikelyinportal HTN
9. Nick during surgery onrightside a. Leftcolicvein
ofhepatoduodenal ligamentwill cause b. inf. epigastricvein
damageto Ans:: aa
a. Cysticduct 17.Neuronsofvestibularnucleiendsat a.
b. IVC cortex
c. hepaticveins b. dentate nucleus
d. hepaticartery c. granularlayer of cerebellum
e. portal Vein. d. cochlearnucleus
Ans: : E Ans:: cc
10.Massatportal hepatiswill compress 18.IrreversiblechangeinMI is
a.Cystic duct. a. Contractionbands
b.IVC. b. appearance of neutrophils
c.Hepatic vein c. Mac callum patch formation.
d.Hepatic artery. Ans:: aa
e.Portal vein, 19.Featheryappearancein
Ans:: ee a.Jejunum .
11.Regarding Cephalicvein b.Duodnum
a. startsfrommedial dorsal surface ofhand b. c. Ileum
connectswithbasilicveininelbow. Ans:: aa
c. Runsindeltopectoral groove 20.Childof2years age
d. piercesClavipectoral fascia presentsinopdwithFever,diarrheaand
Ans:: cc pallorfor2days. Hob 5.6%,MCV
12.whichcranial nerve nucleilieslateral 107MCHC inc. Inv of choiceis a.
torhomboidfossa/sulcuslimitAns: a. dorsal trAns:cobalaminlevels
vagal nucleus b. intrinsicfactorantibodiesassay
b. CN6 c. CN3 c. serum b12level
d. CN11 e. CN9 d. rbcfolate.
Ans:: aa Ans:: bb ,
13.Posteriormostfibersinposteriorpartofth ( if it were an adult than Ans:wer would be
erim ofinternal capsulecontain a. ccc).
opticradiations 21.RighttesticularCA 1stmetastasizes to
b. corticospinal fibers ofupperlimb c. A. Aortocavolumbarnodes b. renal
CSfibers oflowerlimb hilarnodes
d. frontopontinefibers c. sup. inguinal d. commoniliac
201

e. paraortic Ans:: aa
Ans:: ee 29.Amino glycoside
22.Extraembryoniccoelom isderivedfrom halflifeisincreasedinOldage patientsdueto
a. Epiblast a. hepaticmetabolismisdecreased
b.Hypoblast b. decrenal function
Ans:: bb ( langmann embryo). c. decVol ofdistribution
23.TrueaboutTrigeminal ganglion Ans:: bb
A. liesinAnt. cranial fossanear cristagalli 30.A patientcomeswithswelling ofbig toe
B. liesinapex ofpetrouspartoftemporal onrightsideoffoot. nopainwae there,, what
boneinmiddlecranial fossa investigation would you do ??
c. coveredinganglion d. bathesinCSF a. serum uricacid
Ans:: bb ( if bb option not present prefer b. serum ureaanduricacid c. microscopy
dd) ofjointfluid d. urine uricacid
24.Young e.Synovial fluid for polarized light.
adultinvolvedinRTA,gotinjurytomiddlecr Ans:: ee
anial fossa,bleedingfrominternal ear.After 31.Clara cellsarepresentin a.terminal
recovery presentswithlossof Bronchioles
tearformation. Injurytowhichstructure b. alveoli.
during RTA causedthis c. alvelar duct.
a. Greatersuperficialpetrosal nerve Ans:: aa
b. deeppetrosal nerve 32.Foreheadisformedfrom A. FrontoNasal
c. interaortocaval plexus d. cilliaryganglion Process B. maxillary process
e. carotidplexus c. palatonasal process
Ans:: aa Ans:: aa
25.A manpresentswithgradual 33.A footballer getsinjuredduring
lossofvision, match,presentsinERwithlimbinstability.
radiologicalstudiesshowgrowing O/Ethereisswelling inpre-
clotinavessel,the mostlikelyinvolvedvessel patellarregionandhis
is TibiaisdrawnforwardonFemur.
a. vertebralartery b. basilarartery Thestructuremostlikelyinjured is
c. MCA A. AnteriorCruciateLigament
d. PCA b.Posterior cruciate ligament tear.
e. posterior communicating artery Ans:: aa
Ans:: dd 34.8patientswerereportedinacasestudy,the
26.A womanpresentswithbleeding iragewas20,25,25,30,40,20,30.
withsuperficial cuts. O/Inv. Themedianof valuesis
onlyBTtimeisprolonged. defectis dueto a.27.5
a. thrombocytopenia b.25
b. factor8def c. factor9def c.20
Ans:: aa Ans:: aa
27.Regarding medianis calculatedwithvalues
mostcorrectstatementaboutbundle ofHIS putinascending orderand ifoddno.
a. only pathwayconducting thenthecentral oneis median. Ifeven no.
impulsesfromatria toventricles thenmiddletwovaluesaverage,inthis
b. situatedinmembranouspartofIVseptum case(25+30)/2isAns:
nearatrioventricularjunction 35.Regarding anatomy ofInguinal Canal
Ans:: bb a. liesfromASIS to pubictubercle
28.mostprominentstructure during IVU b.
ofkidneyis Superiorboundaryformedfromconjointtendon
a.Calyces Ans:: bb
b.Renal pelvis.
202

36.A b.Liver
studyinwhichdiseasedpeoplearecompared Ans:: aa
tonondiseasedhealthy people, looks for 44.P. Malaria trAns:mits tohumaninform
prior exposure for a risk faactor is called of
a. crosssectional a.Sporozoite
b. cohort b.Merozoite
c. casecontrol d. RCT Ans:: aa
Ans:: cc 45.A
37.Ingrowing malefromBaluchistanpresentswithletharg
embryo,themesotheliumcovering theLung y,jointpainsandleg ulcers. Diagnosiswill
forms most likely
a. pleuroperitoneal membrane A. sicklecell
b. pleuropericardial membrane c. parietal b. thalassemiamajor c. HbC
pleura Ans:: bb
d. visceral layer ofpleura 46.A
e. fibrous pericardium childpresentswithpallor,lethargyanddecac
Ans:: dd tivity. O/I shows decHbdecMCVandBone
38.FoodIntake/reflex changesareprominentonFace
a. ispromotedifhungercenterisdamaged b. a. thalassemiamajor b. sicklecell disease c.
isinhibitedifsatietycenterisdamaged thalassemiaalpha
c. inhibitedbyleptinssecretedfromadipocytes Ans:: aa
d. inhibitedbyleptins secretedfrommyocytes 47.Regarding RBC antigenonsurface
Ans:: cc A. glycolipids
39.A patientpresentswithcough. B. Glycoprotein
CXRshowsB/L peri-hilarinfiltrates. c.Glycosphingolipids
Biopsy of skin lesionsshows Ans:: cc
granulomatousinflammation. 48.Regarding anklejoint
Diagnosismostlikelyis a. surfacecoveredfromfibrocartilage
a. silicosis b. TB b.
c. sarcoidosis deltoidligamentonMALLEOLARSIDE(medi
d. asbestosis al)wasnotwritten c. superficial peroneal
e. histoplasmosis nerveisonlysupplytoit
Ans:: cc d. talofibularligamentisonmedial side
40.Extensormuscles ofarmsaresuppliedby Ans:: bb
a. PosteriorCord 49.Youareallowed
b. Medial cord tobreachpatient’sconfidentialitywhen
Ans:: aa a. patientallowstodoso b. insuranceclaim
41.Infracture ofhamate,whichisinvolved? Ans:: bb
a. ulnarartery b.ulnarnerve 50.A patientpresentswithcentral
Ans:: bb chestpainfor4-6hours.
42.Manwithlymphomawas being Painisradiatingtoarmandjaw. Best
treatedwithanti cancer drugs,biopsy markertoconfirmMI atthisstageis
ofalymphnodesshows clumping a. CPK
ofnuclearmaterial andfragmentation. b. CKMB
Themostlikely mechanismis c. LDH
A.Apoptosis Ans:: bb( Trop T was not in options).
b.necrosis 51.A patienthadheart-lung
Ans:: aa trAns:plantationandwas
43.Ininfectiousmononucleosisorganatrisk onimmunosuppressive drugsnow presents
ofruptureis withhighgradefever,neckrigidity+vekerni
a.Spleen.
203

g.CSFstudiesshow anorganismwithclear 57.Lymphnode ofinner


cytoplasmwithahaloaroundit. breastdrainsintoall except
a. CRYPTOCOCCUS a. infphrenicnodes
b. EBV b. supraclavicularnodes
c. staphaureus d. klebsiella c. internal thoracicnodes d. pectoral nodes
e. CMV Ans:: dd
Ans:: aa 58.Mostcommonfracture offemuratage
52.InShockwhichis of60is
themostimportantmechanismwhichconser A. femurheadfracture
vesthe organismas b. intra capsularfracture
awhole a. baroreceptor c. neck offemurfracture
b. sympathoadrenergicsystem d. shaftfracture
c.CNSischemicresponse Ans:: cc
Ans:: bb,,,,,, isbestbczitconserves 59.Epidural SpaceEndsat a. S2
perfusionto vital Ans:: aa
orgAns:whileCNSischemicresponsecomesat 60.Leftgastricarteryisabranchof
endstagewhenperfusiontobrainislow,itcauses a. CeliacArtery
widespreadvasoconstriction,whichcauses b.Hepatic artery
organfailure. Ans:: aa
53.RemnantofUmbilical Veinis 61.Structureaccompanying
a. falciformligament esophagusindiaphragmis
b. ligamentof teres a. leftvagus
Ans::bb b. rightvagus/vagal trunk
54.Patienthadhypovolemicshock,presents c. phrenicnerves
withsignsofAcuteRenal failure. Mostlikely d. azygousvein
damage towhichpart ofthe Ans:: bb
nephronisgreatest 62.InhibitorsofProstaglandinsare
a. PCT A. Corticosteroids
b. descending c. ascending b. aspirin
d. DCT Ans:: aa
e. Collecting duct 63.Hall markofAdrenocortical
Ans:: aa insufficiencyis
55.A school goingboy a. Hyponatremia
developsRightsidedflankswelling b. hypokalemia
andlumbarpain,USshows c. metabolicalkalosis d. hypocalcemia
Hydronephrosisof therightkidneywhilethe Ans:: aa
otherisnormal.Whichof the 64.FastPain travelsby
belowismostlikely thecause ofit A. A deltafibers
a. Urethral stricture b.C fibers
b. ureteral stricture Ans:: aa
c. posteriorureteral valves 65.Sympatheticnervesending
d. vesicouretericreflux atInferiorhypogastricplexushaverootvalue
Ans:: cc of
56.A A. L1-L2 .
patienthadanAccidentwithfractureofsurgi B. L3-L4
cal neck Ans:: aa
ofhumerus.Mostlikelyinvolvednerve is 66.Detrusormuscle ofbladdersuppliedby
a. axillary A. pelvicSplanchnicnerves
b. radial Ans:: aa
Ans:: aa 67.Pectoral groupofaxillary nodes drain
a. Upper halfof trunk
204

b. Upper halfofupperlimb Ans:: aa Refence Snell


c. Majorpart ofthe breast 78.Ulcer 2ndpart
d. Lowerhalfof trunk ofduodenumatposteriorsidearteryerodedi
Ans:: cc s a. Gastroduodenal artery
68.Regarding VagusNerve Ans:: aa
a. Exits throughpassing middle partofjugular 79.Slowestgrowing tumorof thyroid
canal A. Papillarycarcinoma
b. Exitsthorax by passing withAorta Ans:: aa
Ans:: aa 80.Commonestdislocationof TMJ is
69.FragileXsyndrome is A. Anterior
a. Trinucleotiderepeatdisorder Ans:: aa
Ans: ; aa 81.A
70.Mostcommoncause ofcongenital manhasshorttrunkandlongerarms,thisdise
inheriteddefects aseis ?
a. Multifactorial a. Autosomal dominantdisorder
b. Autosomal dominant Ans:: aa
Ans:: aa 82.Type3reactionis
71.A a. Arthusreaction
patientpresentsinOPDwithvesicularerupti Ans:: aa
verashwhichisstarting fromlefttemporal 83.MyelinformationinCNSisfrom
side andendsnearmedial side oflefteye. a. Oligodendrocytes
Rashispainful,diagnosisofShinglesismade. b.Schwann cells
Whichof the following nervemay Ans:: aa
beinvolved 84.Mandibular branchoftrigeminal
a. Ophthalmicnerve nerveexits cranial cavitythrough a.
b. Superiororbital nerve Formanovale
c. Supra trochlearnerve d. Maxillary Ans:: aa
e. Mandibularnerve 85.Hemeinbloodbindswith
Ans:: dd a. Haptoglobulin
72.Acrosomeisformedby a. Golgi bodies b. Hemopexin
b. Endoplasmicreticulum. Ans:: aa
Ans:: aa 86.Cerebral bloodflowisregulatedby
73.External urethral sphincterrelated a. PCO2invenousblood
toprostatic:
a. Apex b. Base b. pH ofCSF
c. Posterior Ans:: aa
d. anteriorside 87.Eparterial bronchusis a. Rtsuperior.
Ans::bb Ans:: aa
74.Small particlesare clearedby 88.Epidural sinusinspinal canal
A. Ciliainterminal bronchioles. a. Longitudinal venousplexus
B. Macrophages b. CSF
Ans:: bb c. Isbetween Duraandarachnoidsspace
75.HighestPCO2isinwhichvessel Ans:: aa
89.Mostcommoncongenital anomaly
a. Pulmonaryartery ofheadis a. Cleftlip&palate .
b. IVC Ans:: aa
c. SVC 90.A womanheadRadical
Ans:: aa mastectomyafterrecovery
76.Notisotonictoplasmais a. 5% unabletoraisearmabove her head,
dextrosewater structure damageis
77.Lumbar triangleanteriorly boundby a. Long thoracicnerve
a. Posteriorborder of External Oblique
205

Ans:: aa 102.Intrinsicfactorissecretedfrom
91.Anteriorrelationofleftkidneywhichisdir a.Gastricfundus
ectlyincontactwithkidneywithoutanymem Ans:: aa
brane a. Lessersac 103.Virus causes
b. Pancreas a.Alterationof proteinsynthesis.
c. Duodenum Ans:: aa
d. Leftcolicflexure. 104.Moral
Ans:: bb supporttopatientinoursocietycomesfrom
92.Structure passing a.Family
abovelowerheadofPancreas a. SMA . b.Eidhi.
Ans:: aa Ans:: aa
93.ICAMandVCAMare. 105.A patientisbinge drinkerofAlcohol
a. adhesionmolecules. presents toERwithC/ORTside
Ans:: aa hypochondrial pain. US
94.Derivative of2ndarchis a. Stylohyoid showsfattyliver,Biopsyshowscouncilmanbod
Ans:: aa iesinhepatocytes. A. Apoptosis
95.Adverseeffectinlong term use Ans:: aa
ofglucocorticoidsis a. Osteoporosis 106.Feature ofAtlasis
Ans:: aa a.AbsentBody
96.Branches ofbrachiocephalictrunkare. Ans:: aa
a. RtcommoncarotidandRtSubclavian 107.A patientisbeing evaluatedfor TB.
97.If LCXisoccludedwhatwill betheeffect The bestinvestigationwhichwill
a. Posteriorofleftventricle bemoreaffected diagnoseTBis
b. SA nodal blockwill occur a. CaseousnecrosisonBiopsy
c. Postinterventricularseptum1/3will b. EpitheloidCells
beinfarcted c. DemonstrationofAcidfastbacilli
Ans:: aa onZNstain
98.SA nodeispacemaker ofheartbecause Ans:: cc
A. Itliesatthe bottom 108.Middlemeningeal artery
ofatrioventricularjunction b. Itliesatthetopof Enterscranial
RTatrium CavitythroughforamenSpinosum
c. Ithasaplateau
d. Itsfibers havefastestconduction. 109.WhatparameterisincreasedinPregnan
e.it has automaticity n generates impulses at twomen
a faster rate. a. TIBC
Ans:: e b. Serumferritin c. PCV
99.Heinzbodiesarepresentin d. Hb
a.G6PD Ans:: aa
100. 2yearoldchildwas playing 110.Nervesupplytopericardiumisfrom
withtoyswhenhesuddenly a.phrenicNerve
becomesuffocatedandcyanosedfor Ans:: aa
fewseconds. Mothertook himahospital 111.Highestenergycompoundis
onCXR he hadateasmall coin,wherewill a. Fats
itbelodged mostlikely 112.Pubicbonesjoinanteriorly by
A. Rtlowerbasal segment. a. Secondarycartilaginous joints
Ans:: aa 113.Veinhaving novalvesis
Ans:: aa a.SVC
101. A young 114.Following veinisa tributary ofPortal
childinhaledapeanutwhereitwill vein
belodgedmostlikely a. Superiorrectal vein
Ans:. Lowerlobe ofrightlung
206

b. Hemiazygousvein c. Infepigastricvein d. 125.Whichhormonelevelwill fall after


Leftrenal vein drinking water
e. Lumbarveins a. ADH
Ans:: aa b. Angiotensin
115.A ladyhasdifficultyinstanding Ans:: aa
fromasitting 126.Maximum bloodpressureisat a. Renal
positionbutcanwalknormally. artery
Muscleinvolvedis Ans:: aa
a. Gluteus Maximus 127.Mostcommoncause ofKshiftfromICF
Ans:: aa toECFA. Vigorous exercise
116.A detachedembolusfrom Ans:: aa
deepveinsofleg will gofirstto 128.Glucoseismainsourcefor a.Neurons
a. IVC b. Liver
b. RtAtrium c. Ltventricle Ans:: aa
d. Pulmonaryveins 129.OxytocinandADH originatein a.
e. Pulmonaryartery. Hypothalamus
Ans:: ee 130.MostcommonassociationofADPKDis
117.DefectofRBCsinHereditarySpherocyt a. Renal failure
osis b. Cerebralhemorrhage
a.Cytoskeletonabnormality Ans:: bb ,, cause of death is renal failure.
b.Defect in spectrin and ankyrin 131.Musclethat causessadness
Ans:: aa a. Platysma
118.Mixedvenousbloodsamplecanbe 132.Clostridium botulinumcauses
obtainedfrom a. Flaccidparalysis
a. SVC 133.Nerveaccompanyingsuperficialtempo
b. IVC ral arteryis
c. Rtatrium Auriculo-Temporal nerve.
d. Rtventricle 134.SarcomadiffersfromCarcinomain
e. Pulmonary arteries IncreasedVascularity
Ans:: ee. 135.Patientontable heatlossmainly by
119.Long term glucosecontrolcanbe a. Conduction
bestdetectedby b. Radiation c. conduction and radiation.
Ans:. HbA1C Ans:: cc
120.Glycocalyxis 136.AcidicurineinTubuleswill cause a.
a. Polysaccharide moiety onbacterial cell increasedsecretionofCL-
wall
b. Carbohydratechainoncell wall b. DecreasedHCO3-
Ans:: bb. c. DecK+secretion
121.Basicdrug bindswith Ans::
Alphaglycoprotein d. ReducedH+reabsorption
122.Thyroiddrains to e. Aldosteroneinc
a. deepcervical nodes. Ans::dd
Ans:: aa 137.Baby bornwithcongenitalcataract,
123.Pregnantwomenwithviral hepatitis. mother
Bestinv. todetectis hadmostlikelywhichintrauterineinfection
A. GGT A. Rubella
Ans:: aa 138.Leftadrenal veindrains to leftrenal
vein
124.1/3rdofTBWis 139.Lower Esophageal Veinsdrainto
A. ECF A. Portal Vein
Ans:: aa
207

140.Leftlung hasgroovefor a. Ascending b. LowK&HCO3incomparisontoplasma c.


aorta Hypotonicity ismorethanplasma.
b. Descending aorta c. Archofaorta Ans:: cc
d. Azygousvein 152.Osmoticpressure dependsupon
Ans:: cc a. Noofparticles
141.Deepveinthrombus1stlodgesin b. Diffusionpotential
a. Branches ofpulmonaryartery c. Charge onmembrane oneitherside
b. Rtatrium Ans:: aa
c. Rt. Ventricle d. IVC 153.Diffusionpotential dependsupon
Ans:: aa a. Surfaceareaofmembrane
142.PatienthadTB, developedpleural b. Charge onmembrane.
effusion. Needleinsertedwill bealong c. Charge oneitherside
upper border oflowerrib d. Fromlow conc. tohigh conc.
143.Cause ofpyogenicperitonitisis Ans:: aa
a. E. Coli 154.From
b. Bacteriodes c. Klebsiella
d. Staphaureus 3rdto9thmonthofpregnancy,source
Ans:: aa ofestrogenis
144.Metaplasiais a. Corpusluteum
onecell typereplaces otherwithfunctional b. Granulosa cells c. Placenta
change Ans:: cc
145.A patienthadacoronary 155.Strengthprovided towoundduring
bypassdevelopedA-fibnow healing isby
presentsinERwithsignsofstroke. Brains a. Collagen1
showswhichtype ofnecrosis b. Collagen3.
a. Liquefactive Ans:: bb ,, if only mentioned tensile strength
146.HIVaffects than it is type 1
CD-4CellsT 156.Granulosa cellsarefoundin
147.Laborworking inhotweathercollapses. a. Corpusluteum
Mostlikelyabnormalityfoundinhimwill be b. Groundsubstance
a. Hyponatremia c. Graffianfollicles
b. Hypervolemia c. Hyperkalemia d. d. Follicularepithelium.
Hypercalcemia Ans:: cc
Ans:: aa due to excessive sweating. 157.Esophageal constrictionisby
148.Carcinoma compared toadenomahas a. Leftatrium
a. Capsule 158.Regarding RtAtrium
b. Invasion a. Formslowerborder ofsternocostal surface
c. Nolossofdifferentiation d. Vascularity b. Is the posteriormoststructure
Ans:: bb c. IVC Ans:SVC
149.TBresistsimmunity. Inthis junctionisvisibleatoutersurface d.
caseAntibodiesarefound IssuppliedbyLCX
a. Attached tocell Ans:: aa
b. Boundtoorganism. 159.Regarding boneformation
c.No ntibodies. A. Newlylaidboneisintheform ofosteon
Ans:: cc 160.Vibrationsduring
150.Ironisstoredinparenchymaunder RapidVentricularfilling produces
normal conditionsis A. 3rdheartsounds
Ferritin b. 4thheartsound
151.Regarding saliva Ans:: aa
a. HasincNa as compared toplasma
208

161.TPRisdirectlyrelated to a. 171.Incase
Capacitance ofposteriorperforationof1stpart
b. HR ofduodenum,fluidwill becollectedinto
c. Vasomotor tone a. Lessersac
Ans:: cc b. Rtsubphrenicspace
162.Exposuretohydro-carbonscause c. RtSubhepaticspace
a. Acute nonlymphocyticleukemia b. CML d. Rtparacolicgutter
c. AML Ans:: bb
d. Livercancer 172.Antibodiesareformedby
e.Bronchogenic carcinoma. Ans:. Plasma cells
Ans:: ee 173.Glucocorticoids cause
163.Cause ofmesotheliomais a. Increasedproductionofprotein b.
Asbestos Hepaticgluconeogenesis
164.Subacutebacterial endocarditisis c. Decaminoacidmetabolism
causedby Ans:: bb
A. StrepviridiAns: 174.Whichhormoneactsthroughphospholi
165.Cornayae diphtheriamode of paseC
trAns:missionis a. Oro-fecal a. Hormonecausing
b. Respiratory droplets increasedbloodflowinpenis b. Causing
c. Skincontact d. Fromspores waterreabsorptioninkidneysAdhV2
Ans:: bb c. Constrictionofvasculature byADH V1
166.Inhibinisproducedby a. Sertoli cells d. ANP
167.Lymphaticnodulesabsentin e. Angiotensincausing vasoconstriction
A. Thymus Ans:: cc
B. Spleen 175.Sensorysupplytoskinoverlowerhalfofd
c. Payer’spatches eltoidissuppliedby
Ans:: aa A. Axillary nerve
168.Rapidlyadopting fibersare. a. 176.Preganglionicautonomicfibersare
Pacinian A. Bfibers
b. Meissners c. Merkel 177.Cellswhichareenlargedinadenomaofp
d. Ruffini arathyroid a. Chiefcells
e. Free nerveendings b. Oxyphil cells c. Both
Ans:: aa Ans:: aa
169.Regarding vagusnerveinGIT 178.Genital branchofgenito-femoral nerve
a. Stimulationcausesrelaxationofano-rectal a. Passesthroughdeepinguinal ring
sphincter b. Gives branchestoanteriorscrotal wall.
b. Contractionoflongitudinal Ans:: aa
musclegroupforperistalsis 179.CharacteristicofIgM
c. Decreasedacidsecretion A. Isalarge molecule (pentamer)
d. Constrictionofoddi sphincter 180.Highesttriglycerides%isin
Ans:: bb A. Chylomicrons
170.Incase ofAnteriorduodenal 181.Efficiency ofadrug is
perforation,fluidwill goto a. Efficacy
a. Leftparacolicgutter b. Potency
b. Rtsubhepaticspace Ans::a
c. Rtsubphrenicspace 182.Tensioninmuscleissensedby
d. RtPara colicgutter A. Golgi tendon
Ans:: dd,, if this is not in option than rt. Iliac 183.A patientpresents to youinemergency.
fossa. HispH is 6.95HCO34,glucose500mg/dl.
You administer insulin
209

A. Regularinsulin 6)Blood trAns:fusion reaction will occur


184.Isthmusofuterinetubesis if we trAns:fuse?
A. Belowthefundus a) O+ to A+
b. Is the narrowestpart of tube. b) A+to AB+
Ans:: bb c) B- to AB+
185.Genioglossussuppliedby d) AB+ to A+ ANS:
A. C1fibersthroughhypoglossal nerve 7)Pain of ovary on medial side of thigh is
186.Chondorcyteschangeintoosteocytes felt by?
A. Thyroxin a) Femoral nerve
b. Somatotrophs b) Illiohypogastric nerve
Ans:; bb c) Obturator nerve ANS:
187.Patientwithlowerbackacheandpainra d) Sural nerve
diatingtoposterior thigh. 8)Lady with hepatoma and dilated
Themostlikelycauseis abdominal veins?
A. DischerniationatL4-L5 a) IVC obstruction ANS:
b. DischerniationatL5-S1 b) Internal iliac vein obstruction
Ans::bb c) Superior mesenteric vein obstruction
Radiology Paper 5 September 9)Structure connects ECM to
Cytoskeleton?
2016 Evening Session a) Inermediate filament
1)Structure in right free margin of lesser b) Integrin ANS:
omentum? c) ProteoglycAns:
Bile duct,hepatic vein,portal vein 10)Lymph drainage of prostate?
Bile duct,hepatic artery,hepatic vein a) Internal iliac ANS:
Bile duct, hepatic artery,portal veinANS: b) External iliac
2)Parts of bile duct? c) Internal iliac+External iliac
Supraduodenal,infraduodenal,retroduodenal, 11)Which of the following structure is
intraduodenal ANS: drained to inguinal nodes?
Supraduodenal,retroduodenal,intraduodenal a) Cervix
Supraduodenal,infraduodenal,retroduodenal b) Ovary
3)Lymph drainage to superior mesenteric c) Part of body of uterus ANS:
nodes? d) Urinary bladder
Stomach 12)Patient can stand on toes with severe
Duodenum pain which of the following is ruptured?
Jejunum ANS: a) Achillis tendon
Liver b) Plantaris ruptured ANS:
4)Regarding lymph drainage of testes? c) Fracture of medial melleolus
Intrtnal iliac nodes 13)Hormone required for conversion of
Preaortic nodes bHcg to estradiol?
External iliac nodes a) FSH ANS:
Paraaortic nodes ANS: b) Gonadotropins
5)Regarding posterior communicating c) LH
artery what is correct? 14)Blood supply of shaft of humerus by?
a) Connects ICA to basilar artery Radial artery
b) Connects ICA to posterior cerebellar a) Profunda brachii artery ANS:
artery above occulomotor nerve ANS: b) Ulnar artery
c) Connects ICA to posterior cerebral 15)Main supply of head of femur by?
artery below occulomotor nerve Femoral artery
d) Connects ICA to posterior a) Medial and lateral circumflex artery
communicating artery b) Retinacular artery Ans:
c) Obturator artery
210

16)Loss of suppination and flexion of b) Facial nerve


arm..nerve damaged? c) Buccal nerve
a) Axillary nerve d) Auriculotemporal nerve ANS:
b) Median nerve 27)Submandibular gland surgery
c) Musculocutaneous nerve ANS: done,which nerve is damaged?
d) Ulner nerve a) Cervical nerve
17)Loss of sensation on medial half of b) Facial nerve ANS:
hand in front and behind.the structure c) Maxillary nerve
damaged is? d) Submandibular nerve
a) Supracondylar fracture 28)In axilla which part of brachial plexus
b) Fracture of medial epicondyle ANS: is present?
c) Fracture of shaft of humerus a) Trunks
d) Fracture of scaphoid bone b) Devisions
18)Physiological fuction of vit A? c) Cords ANS:
RhodopsinANS: d) Roots
a) Other options were weired dnt recall e) All of above
those. 29)Spastic paralysis occurs in?
19)Dorsal nucleus of vagus is located in? a) Upper motor neuron lesion ANS:
Midbrain b) Lower motor neuron lesion
a) Pons c) Cerebellar lesion
b) Medulla oblongata ANS: d) Cerebral lesion
c) Cerebellum 30)Pendular knee jerk is present in?
20)Remanant of urachus? a) Cerebellar lesion ANS:
a) Medial umbilical ligament b) Basal ganglia disorder
b) Mickels diverticulum c) Upper motor neuron lesion
c) Median umbilical ligament ANS: d) Cerebrum lesion
d) Medial arcuate ligament 31)Counter part of rete testes cords is?
21)Child with eating disorder,represents a) Mullerian ductsANS:
with weight gain and hypogonadism,the b) Mesonephric ducts
most likely due to? c) Ductus deference
a) Craniopharyngioma ANS: 32)Femoral triangle does not contain?
b) Basal ganglia disorder a) Femoral nerve
c) Cerebral disease b) Obturator nerve ANS:
22)arch of aorta compresses? c) Lymphatics
Azygous vein d) Adductor magnus
a) Esophagus 33)Muscle which flexes knee and extends
b) Left bronchus ANS: hip?
c) Trachea a) Sartorius
24)Abdominal aorta at diaphragmatic b) Tensor fascia lata
opening compresses? c) Semitendinous ANS:
a) Phrenic nerve and Thoracic duct 34)Vessel which runs obliquely upward in
b) Azygous vein and Thoracic duct ANS: parotid gland?
c) Inferior vena cava and azygous vein a) Facial artery
25)Inferior vena cavastarts at the level of? b) Maxillary artery
a) L3 c) Lingual arteryANS:
b) L4 d) Occipital artery
c) L5 ANS: 35)Optic canal is present in which part of
d) L6 fascial bone?
26)Superficial temporal artery a) Lesser wing of sphenoid
accompanies in parotid gland? b) Greater wing of sphenoid ANS:
a) Lesser occipital nerve c) Vomer
211

d) At junction of sphenoid and ethmoid d) Normal kidney ANS:


36)Germ cells incorporate at? 46)About esophagus what is correct?
a) 3rd week a) Cardiac orifce at 10th costal cartilage
b) 6th week ANS: b) Lies on right side throughout its course
c) 3rd month c) Serosa is thickened at lower endANS:
d) 6th month d) Supplied by thoracic aorta only
37)Drainage of bare area of liver into? 47)About intercostals nerves what is
a) Celiac nodes correct?
b) Posterior mediastinal nodes ANS: a) Supply skin of whole body wall
c) Lymph nodes in porta hepatis b) 12 in number ANS:
38)In tetralogy of fallot what is present? c) Supply abdominal muscles only
a) Overriding of right ventricle d) Part of sympathetic chain
b) Ventricular septal defects ANS: 48)Patient with sciatica compression on
c) Left ventricular hypertrophy 5th lumber segment causes?
d) Pulmonary hypertension a) Absent ankle jerk
39)Behind head of pancrease which b) Absent babinski
structure is present? c) Brisk ankle
a) Superior mesenteric artery d) Weak dorsiflexion ANS:
b) Lesser omentum e) Weak plantar flexion
c) Inferior vena cava ANS: f) Confirm it by yourself.
d) Aorta 49)About spleen what is correct?
40)In front of uncinate process which a) Develops in ventral mesogasrium ANS:
structure passes? b) Tail of pancrease is related to its lower
a) Aorta border
b) Superior mesenteric artery ANS: c) Cant recall other options,bohat ajeeb aur
c) Splenic artery confusing options thay.
d) Left renal vein 50)Another question about spleen?
41)Fracture of neck of fibula which a) On CT concave surface on medial side I
structure damages? did
a) Ulner nerve b) Can’t recall other options bohat he
b) Brachial nerve confusing aur fazul options then.bus aik
c) Common peroneal nerve ANS: isi ki sense ban rahe the…
d) Radial nerve 51)What is correct about bile duct?
42)Biceps Brachii attaches to? a) 6 inches long
a) Medial epicondyle of humerus b) Present on right side of hepatic artery in
b) Lateral epicondyle hepatogastric ligament ANS:
c) Supraglenoid tubercle of scapula ANS: c) Present on left side of portal vein
43)Aortic area is present in? 52)Arterial supply of jejunum is by?
a) Left 2nd intercostals space a) Celiac artery
b) Right 2nd intercostals space ANS: b) Splenic artery
c) At lower border of sternum c) Superior mesenteric artery ANS:
d) Right 3rd intercostals space d) Inferior mesenteric artery
44)What causes hypercalcemia in renal 53)Sciatic nerve passes through?
failure? a) Lesser sciatic foramen
a) Hypervitaminosis D b) Greater sciatic foramen ANS:
b) Parathyroid adenoma ANS: c) Obturator foramrn Behind inguinal
c) Thyroid cancer ligament
45)Metastatic calcification occurs in? 54)Number of segments of kidney are?
a) Follicular adenoma thyroid a) 5 ANS:
b) Tuberculosis b) 6
c) Malarial parasite c) 7
212

d) 8 a) Lies close to 3rd part of duodenum I DID


55)Sequence of renal artery? b) Crosses by illeal mesentry
a) Lobar artery-Arcuate artery-Interlobar c) Related to left side of aorta
artery 64)What lies close to subepicardium?cant
b) Segmental artery-Lobar artery- recall rest of scenario…
Interlobar artery-Arcuate artery- a) SA node ANS:
Interlobular artery ANS: b) AV node
c) Lobar artery-Interlobular artery-Arcuate c) Bundle of HIS
artery-Interlobar artery d) Purkinje fibers
56)Length of thoracic duct? 65)Breast atrophy in adult female is due
a) 38cm to?
b) 40cm ANS: a) Estrogen ANS:
c) 45cm b) Progesteron
57)Patient unable to turn foot medially c) Estrogen and Progesteron
but can turn laterally…muscles 66)Cause of fatty liver in Pakistan?
paralyzed? a) Hep B and C ANS:
a) Tibialis anterior and extensor hallusis b) Fatty diet Malnutrition
longus 67)A patient is taking fatty diet…which
b) Tibialis anterior and tibialis posterior organ is most likely effected?
ANS: a) Kidney
c) Tibialis posterior and flexor hallusis b) Skin
longus c) Brain
58)Pancreatic fluid accumulates in? d) Liver ANS:
a) Paracolic gutter 68)What lies posterior to periequiductal
b) Subphrenic space grey area?
c) Lesser sac ANS: a) Substantia nigra
d) Subdiaphragmatic space b) Red nucleus
59)Question about hemophilia? c) Tectum ANS:
a) X chromosome ANS: d) Pineal glnd
b) Y chromosome e) Colliculus
c) Chromosome 6 69)At what level glucose begins to appear
60)A patient has history of myocardial in urine?
infarction one year back…which artery a) 180
supplies the inferior portion of posterior b) 200
interventricular septum? c) 250 ANS:
a) LCA 70)Maximum number of sodium channels
b) LAD are present in?
c) RCA ANS: a) Initial segment of axon
61)Which segment of kidney has more b) Node of ranvier ANS:
diluted urine? c) Collecting tubules
a) Proximal convoluted tubule d) Distal convoluted tubules
b) Distal convoluted tubule ANS: 71)Most important cells of chronic
c) Thick ascending limb of loop of henle inflammation?
d) Collecting ducts a) Lymphocytes
62)Lymph drainage from nipple of breast b) Macrophages ANS:
is to? c) Eiosinophils
a) Posterior axillary nodes d) Neutrophils
b) Apical nodes 72)Surgery of parotid gland is done,which
c) Anterior axillary nodes ANS: muscle will be paralysed?
63)What is correct about inferior vena a) Medial pterygoid
cava relations? b) Lateral pterygoid
213

c) Buccinator ANS: 81)A structure at L1 level having coils


d) Temporalis Maseter with grey colour at outer side and black
73)Content of anterior mediastinum? colour in middle?
a) Trachea a) Colon
b) Vagus nerve b) PYLORUS of Stomach ANS:
c) Phrenic nerve c) Jejunum
d) ThymusANS: d) Ieium
74)Bundle Of His is supplied by? 82)The following organism is resistant to
a) LCA ANS: stomach acid?
b) RCA a) Vibrio cholera
c) Carotid artery b) Pneumococcus
75)A very very long scenario ended up by c) Mycobacterium Tuberculosis
asking parasympathetic supply of d) h.pylori ANS:
descending and sigmoid colon? 83)Multifactorial inheritance?
a) L4 L5 a) Sickel cell anemia
b) S1 S2 S3 b) Cystic fibrosis
c) S3 S4 S5 c) Omphalocele ANS:
d) S2 S3 S4 ANS: d) Alkaptonuria
76)Lymphatics of gall bladder will first 84)a patient while walking…pelvis sink to
drained into? left side is due to?
a) Pre aortic nodes a) Right side of gluteus maximus and
b) Para aortic nodes medius
c) Porta hepatis nodes ANS: b) Right sided gluteus maximus medius
77)Inferior epigastric artery lies lateral and minimus ANS:
to? c) Left sided gluteus medius and minimus
a) Direct inguinal hernia ANS: d) No separate option of right sided gluteus
b) Inguinal hernia medius and minimus is present there..
c) Indirect inguinal hernia 85)A patient having strangulated
78)Patient with sensory loss of left side of hernia,strangulation occurs due to?
face and left side of body…lesion is at? a) Excessive movement of lower limb
a) Thalamus ANS: b) Pressure by inguinal ligament
b) Pons c) Less space of femoral canal ANS:
c) Internal capsule d) Tight boundaries of femoral ring
d) Medulla 86)About right brachiocephalic vein,what
79)Motor loss of right side of is correct?
body,something like that?Lesion is at? a) Begins behind right sternoclavicular
a) Anterior limb of internal capsule joint
b) Genu+Posterior part of posterior limb of b) No valves
internal capsule c) Shorter than left
c) Genu d) Vertical ANS:
d) Anterior part of posterior limb of e) More than one options are correct.
internal capsule 87)Descending thoracic aorta passes
e) Genu+Anterior part of posterior limb of behind?
internal capsule ANS: a) Median arcuate ligament
80)Radial artery is accompanied by? b) Lateral arcuate ligament
a) Median nerve c) Medial arcuate ligament ANS:
b) Musculocutaneous nerve ANS: 88)A very very long confusing scenario is
c) Deep branch of radial nerve given,only hint of 9:21 chromosome is
d) Superficial branch of radial nerve given…
a) CML ANS:
b) Mylodysplastic syndrome
214

c) Muscular dystrophy 6) Tail of pancrease


d) 9:21 Ans) Lie in splenorenal ligament
89)A patient with history of RTA,CT scan 7) Corneal opacities caused by
shows a mass in petrous part of temporal A)Ethambutol
bone..where such mass is present? B)Chloroquine
a) Base C)Quinidine
b) Inferior surface ANS B
c) Apex 8) RTA scenario presented with proptosis
d) Anterior surface of eye rushing sound in ear .episcleral
e) Roof ANS: vessels dilated
90)Superficial dorsal vein of penis drains A)Retro opthalmic hemorhage
into? B)Floor of or bit damage
a) Great sephanous vein ANS: C)Roof of orbit damage
b) Internal iliac vein D)Carotid cavernous fistula
c) Femoral vein ANS D
d) Santorini plexus 9) Paraneoplastic syndrome caused by
RADIOLOGY OCT 2016 A) Small cell ca of lung
B) Adenoca stomach
1) Fibrillation commonly occur during C)Renal ca
A) start of action potential D)Thyroid ca
B) End of action potential E)Parathyroid ca
C) Peak of action potential ANS A
D) Refractory period 10) Most potent stress hormone
ANS B A) Acth
2) Right testes ca drans to B)Epinephrine
A) Bilateral paraortic C)Norepinephrine
B) Ipsilateral pelvic ANS A
C) Bilateral pelvic 11) Raw egg ingestion avidin which
D) Bilateral paraortic and ipsilateral pelvic biochemical reaction hampered
E) Ipsilateral pelvic and ipsilateral paraortic A)Decarboxylation
ANS A B)Transamination
3) a docter prescribe vit d3 to pt what it ANS A
actually is 12) Glycolysis
A) Inactive A)Reversible
B) 1,25 dihydrocholecalciferol B)Hexose phosphate converted to lactate
C) 25 hydroxycholecalciferol and pyruvate
D) Animal source ANS B
E) Plant source 13) Sarcoidosis associated with
ANS B A)Pulmonary embolism
4) Difference in plasma and interstitium B)Increased calcium
A) Proteins C)Erythema nodosuum
B) Calcium D)Hepatic granuloma
C) Sodium ANS D
ANS A 14) Posterior mediastinum desending
5) Multigravida women presented with aorta
thigh swelling painless expansile and Ans) Gives 9 post intercostal arteries
pulsatile below inguinal ligament
A) Femoral hernia 15) Extensor and lateral rotation of hip
B) Femoral artery aneurysm A)Gluteus maximus
C) Saphena varix B)Tensor fascia lata
ANS B ANS A
215

16) Lesser omentum B)Gap junction


A) Attach to duodenum C)Desmosomes
B) Forms falciform ligament D)Nerve fiber
C) Related to right renal gland ANS. A
D) Left renal gland and left kidney 26) Prostate weight
ANS D A)18 gram
Note …post wall related to left adrenal B) 50 gm
and upper pole of left kidney… C)40 gm
17) Extension of epidural space D)30 gm
A)Foramen magnum to sacrococcygeal E)10 gm
membrane ANS .A
B)Foramen magnum to s3 27) Four pillar medical ethics
C) C1 TO s4 A) Beneficence autonomy non maleficence
D) foraman megnum to l5 justice
ANS A ANS. A
18) Within kidney renal vessels run in 28) Symphysis pubis
A)Colum of drummond A)Secondary cartilag joint
B)Columns of bertin ANS. A
ANS. B 29) Breast lies on
19) 10 th rib fracture scenario A)Pect major
A) Spleen damage B)Serratus anterior
ANS. A ANS. A
20) Posterior boundary of epiploic 30) Solider recieved shrapnel wound in
foramen neck after 3 yrs presented with swelling in
ANS. Ivc neck bulges on coughing and sneezing
21) nystagmus loss of pain and general A)Damage to supra plueral membrane
sensation on left side of face left sided B)Unhealed parts of first rib
weakness damage to C)Damage to deep facsia in roots of neck
A)Pica ANS. A
B)Superior cerebellar 31) Wound greenish pus discharge
C)Basilar A) Pseudomonas
D)Post communicating artery ANS. A
ANS .A 32) Primary sign of shock
22) Number of renal segments A) Dec o2 perfusion
A). 5 B) Inc pulse
23)Ureter narrowest area C) Bradycardia
A)Sacral promontary Ans. B
B)Sacroiliac joint 33) Important feature of malignant tumor
C)Ischial tuberosity A) Metastasis
D)Ischial spine B) Pleomorphism
E)Just below kidney C) Invasion
ANS. D D) Inc n/c ratio
24) Neural crest derivative ANS. A
A)Ventral horn motor neurons 34) Inc mitochondrial activity occurs in
B)Accessory nerve apical region of
C)Retina A) Ciliated cells
D)Vestibular and cochlear nerve nucleus B) Hepatocytes
E)Cerebellar nuclei ANS. A
ANS. B 35) In coronary careunit patient expected
25) Anatomical conduction of heart to show
A)Intercalated disc
216

B)Behave calmly without psychiatric A) Baroreceptors


problem B) Cns ischemic response
ANS I marked b ANS A
36) Values coming on point or range of 46) Easy way to diagnose n.gonorhea in
measurement scale is called exuadate
A) Rate A) Gram stain
B) Data B) Blood culture
C) Distribution C) Zn stain
D) Frequency ANS A
E) Prevalance 47) Vibrio cholera
ANS. D A) Grows best at slight acidic ph
37) Thirst partially inhibited by B) Nutritionally fastidious
A) Gastric distension C) Tolerance to alkaline ph
B) Adh D) 24 degree
C) Inc blood volumes ANS A
ANS C( see renal chapter for details) 48) Achalsia scenario
38) Tachphylaxis A) LES tone dec
A) Unresponsiveness develop rapidly B) Absent myenteric plexus
B) Unresponsiveness develop gradually ANS B
ANS A 49) Ca cervix drains first into
39) Free nerve endings A) Parametrial lymph nodes
A) Non.encapsulated B) Paracervical lymph nodes
ANS A C) Internal iliac nodes
40) Hogkin lymphoma D) Superficial
A) Contagious spread Lymph nodes E) Paraortic
B) Extranodal stage common ANS C
C) Lymphocytes dec in young age 50) Bitemporal hemianopia due to
ANS A compression of optic chaisma b/c of
41) Lesion in temporal lobe anuerysm of
A) Memory loss A) Ant communicating artery
ANS A B) Post communicating artery
42) Dec calcium.dec phosphate normal C) Basilar artery
pth ANS A
A) Crf 51) Right cornary artery supplies
B) Inc prarthyroid A) SA node
C) Vit d dec B) Infundibulum of right ventricle
D) Metastatic calcification C) I/v septum
ANS C ANS A
43) Lesion in right parietal lobe 52) Anterior fontanella closes at
A) Memory loss A) 12- 18 months
B) Weakness on B) 18-24
C) Left right disorientation ANS A
D) Visual defects 53) Child presented with sudden onset of
E) Olfactory generalized purpura
ANS C A) Plt 80000
44) Franks Starling law B) Plt 20000
A) Inc venous return leads to inc cardiac C) Capillary damage
output D) Factor 8 deficiency
B) Inc Venous return leads to inc heart rate E) Vwf deficiency
ANS A ANS B
45) Most immediate response to shock
217

54) During swallowing food prevented 64) regarding Ovary


from entering to trachea by A) supplied by Ovarian artery
A) Epiglottis elevation over naso pharynx B) Drain into paraortic nodes
B) Close /-tight approximation of vocal cord C) Covered by squamous epithelium
C) Movement of uvulva D) Attached to lateral margin of uterus by
D) Contraction of pharyngeal muscles mesoovarium
E) Position of tongue E) Both a and b
ANS B ANS E
55) Cephalic vein 65) Infection in pretracheal fascia spread
A) Pierces clavipectoral fascia to
B) Lie between deltoid and pect major A) M.sterni
C) Arise from medial side of dorsal venous B) Retropharyngeal space
arch C) Anterior to pericardium in thoracic cavity
ANS B ANS C
56) Nasopharyngeal ca 66) Failure of fusion of median nasal and
A) Ebv maxillary process
ANS A A) Cleft lip
57) Meningitis with rash B) Cleft palate
A) Meningiococemia ANS B
ANS A 67) Which nerve arises from trunk of
58) Pancreatic fluid drains into brachial plexus
A) Lesser sac A) Thoracodorsal
ANS A B) Supraclavicular
59) Esophageal opening in diaphragm C) Long thoracic nerve
A) T10 D) Medial cutaneous nerve
ANS A E) Musculocutaneous nerve
60) Metabolic function of thyroid at ANS B
physiological state 68) Rotator cuff formed by
A) Dec fatty acid synthesis A) SITS
B) Inc fatty acid oxidation ANS A
C) Dec gluconeogenesis 68) Esinophilic infiltrates characteristics
D) Inc synthesis of protein of
ANS B Ans) Parasitic infestation
61) Orofecal transmission 69) Bladder
A) Hep e A) Devoid of peritonium
B) Hep b B) Drains to superficial inguinal lymph
C) Hep c nodes
D) Hep d C) Drain to internal iliac
ANS A ANS C
62) Peudo hyperparathyroidism 70) Cause of edema in nephrotic
A) Pthrp syndrome is
B) Inc secretions from parathyroid A) Hypoalbuminemia
C) Other condition secreting paratharmone B) Inc hydrostatic pressure
secretions ANS A
ANS C 71) Superficial petrosal sinus lie in
63) Middle age female with epigastric A) Falx cerebelli
pain radiating to back after fatty meal B) Tentorium cerebella
and to right shoulder ANS B
A) Gall bladder 72) Disorganized cell size shape. Strucure
B) Pancrease A)Metaplasia
ANS A B) Dysplasia
218

ANS B A) ivc obstruction


73) Tb confirm by ANS A
A) Acid fast 85) somites number AT 30 DAY Of
ANS A development
74) Cd4+ cells A) 42 to 44
A) Helper t lymphos B) 32-36
B) Cytotoxic lymphos ANS B
ANS A 86) vit b12 deficency scenario person
75) Dec ach function, ptosis lethargy feels that she is walking on wool coton
weakness floor, posterior cruciate
A) M.gravis A) ligament tibia moves forward
B) Lambert eaton syndrome ANS A
ANS A 87) nucleus of vagus,
76) Acute inflamation characterized A) Dorsal nucleus
by??? 88) Obstruction in inferior vena cava
A.Lecocytosis superior to azygous vein leads to
ANS A diversion of blood to
77) Detrosor muscle contraction A) Hepatic vein
controlled by B) Ivc
A) Inf hypogastric plexus+parasym pelvic C) azygous vein
ANS A ANS C
78) Dna virus causing respiratory. 89) Example of delayed type 4 reaction
Infection A) Tb
A) Adenovirus B) Leprosy
ANS A ANS A
79) bare area of liver is limited by 90) Virus causes neoplastic changes by
A) coronary ligament alteration in Dna Rna
ANS A A) Protooncogenes
80) pt can stand on feet but hurting B) Oncogenes
A) plantaris tendon rupture ANS B
ANS A(see my post in details for dis) 91) Metastatic calcification
81) area of tricuspid auscultation A) Ghon focus
Ans) right side of lower body of sternum B) Peudocyst pancrease
82) trachea bifurcation level, C) Atherosclerosis calcification
A) c5_ c6 D) Nephrocalcinosis
ANS A ANS D
83) trachea extension level,
A) t5 to c6
ANS A
84) dependent edema scenario

Potrebbero piacerti anche